0% found this document useful (0 votes)
551 views560 pages

Respiratory

Respiratory MCQ

Uploaded by

Kenny Josef
Copyright
© © All Rights Reserved
We take content rights seriously. If you suspect this is your content, claim it here.
Available Formats
Download as PDF or read online on Scribd
0% found this document useful (0 votes)
551 views560 pages

Respiratory

Respiratory MCQ

Uploaded by

Kenny Josef
Copyright
© © All Rights Reserved
We take content rights seriously. If you suspect this is your content, claim it here.
Available Formats
Download as PDF or read online on Scribd
You are on page 1/ 560
2i25/2014 10:16:08 AM ‘Mark this question => Question Td : 29024 Question 1 of 30 A 16-year-old boy presented with wheezing when playing football and noctumal cough. ‘Which of the following is the best test to confirm the underlying condition? a) A trial of oral corticosteroids and inhaled salburatnol b) Serial peak expiratory flow rate measurements c) A trial of inhaled conicosteroids ) A trial of inhaled salbutamol €) Spirometry and oral corticosteroids Question Explanation: ‘Demonstration of vanable obstruction of the airways prowdes good evidence for asthma, with its characteristic morning dips. Faiure to respond to bronchodilator therapy does not exclude asthma as response may be small in children and in adults with persistent or more severe asthma, Those who ful to respond to inhaled bronchodlator requre a steroid trial (cither 4 weeks of high dose ithaled steroids or 2 weeks of oral Prednisolone). 2i25/2014 10:16:08 AM ‘Mark this question => Question Td : 29024 Question 1 of 30 A 16-year-old boy presented with wheezing when playing football and noctumal cough. ‘Which of the following is the best test to confirm the underlying condition? a) A trial of oral corticosteroids and inhaled salburatnol Y © b) Setial peak expiratory flow rate measurements c) A trial of inhaled conicosteroids ) A trial of inhaled salbutamol €) Spirometry and oral corticosteroids Question Explanation: ‘Demonstration of vanable obstruction of the airways prowdes good evidence for asthma, with its characteristic morning dips. Faiure to respond to bronchodilator therapy does not exclude asthma as response may be small in children and in adults with persistent or more severe asthma, Those who ful to respond to inhaled bronchodlator requre a steroid trial (cither 4 weeks of high dose ithaled steroids or 2 weeks of oral Prednisolone). 2/25/2014 10:16:22 AM ‘Mark this question q => Question 2 of 30 Recent-onset dyspnea and noisy breathing are complains ofa 61 year old female. Her chest x-ray showed right deviation ofthe trachea due to a Retrosternal goiter. Which of the following tests is most usefil in the assessment of airflow obstruction due to the goiter? a) Forced expiratory flow volume in one second b) Forced vital capacity ©) Flow voume curve 4) Peale expiratory flow rate ©) Residual voturae Answer | Gxplanation Other User's Explanation Report An Error Question Explanation: Tnspection of the maximal expiratory and inspiratory flow-volume curve is currently the simplest method to establish the presence of upper airway obstruction associated with retrostemal goiter. This may be present in up to 40% of patients with retrosternal goiter and gensrally requires at least 50% obstruction of the airways before symptoms arise. 2/25/2014 10:16:22 AM ‘Mark this question q => Question 2 of 30 Revent-onset dyspnea and noisy breathing are complains of a 61 year cld female. Her chest x-ray showed right deviation of the trachea due to a Retrosternal goiter. Which of the following tests is most usefil in the assessment of airflow obstruction due to the goiter? a) Forced expiratory flow volume in one second b) Forced vital capacity Y © 0) Flow volume curve d) Peak expiratory Dow rate ©) Residual voturae Answer | Gxplanation Other User's Explanation Report An Error Question Explanation: Tnspection of the maximal expiratory and inspiratory flow-volume curve is currently the simplest method to establish the presence of upper airway obstruction associated with retrostemal goiter. This may be present in up to 40% of patients with retrosternal goiter and gensrally requires at least 50% obstruction of the airways before symptoms arise. 2i25/2014 10:16:37 AM ‘Mark this question & => Question Td : 29563 Question 3 of 30 A 66-year-old chronic smoker has lang cancer. Chest X-ray revealed elevation of left hemidiaphragm and left phrenic nerve palsy wae suspected. On dueroscopy, which of the following will confirm the ciagnosie? a) No movement ofthe left hemidiaphragm +) Paradoxical movement of the left hemidiaphragm c) No movement of the right hemidiaphraam <) Nonmal movement of both hemidiaphragm ©) Paradoxical movement of the right hemidiaphragm Answer | Bevionation | Other User's Explanation Report An Error Question Explanation ‘The diagnosis of phrenic nerve palsy is suspected when on the chest radio graph: the claphrazmatic leaflet is elevated and is confirmend fuoroscopically by observing paradoxical diqphragmatic motion on sniff and cough. In patients with normal kngs, unilateral paralysis is usualy asymptomatic and rarely requires treatment, 2i25/2014 10:16:37 AM ‘Mark this question & => Question Td : 29563 Question 3 of 30 A 66-year-old chronic smoker has lang cancer. Chest X-ray revealed elevation of left hemidiaphragm and left phrenic nerve palsy wae suspected. On dueroscopy, which of the following will confirm the ciagnosie? a) No movement ofthe left hemidiaphragm WV © b) Paradoxical movement of the left hemidiaphragm c) No movement of the right hemidiaphraam <) Nonmal movement of both hemidiaphragm ©) Paradoxical movement of the right hemidiaphragm Answer | Bevionation | Other User's Explanation Report An Error Question Explanation ‘The diagnosis of phrenic nerve palsy is suspected when on the chest radio graph: the claphrazmatic leaflet is elevated and is confirmend fuoroscopically by observing paradoxical diqphragmatic motion on sniff and cough. In patients with normal kngs, unilateral paralysis is usualy asymptomatic and rarely requires treatment, 2/25/2014 10:16:50 AM ‘Mark this question & => Question Ti : 29753 Question 4 of 30 Sleep apnea syndrome is best diagnosed by the following: a) Blood gases during apneic episodes +) EEG ©) Presence of HLA-DR2 and DQw1 4) Polygraphic sleep studies 8) Spirometry Question Explanation: Sleep apneais characterized by cessation of breathing sleep. This causes extreme restlessness with frequent respiratory pauses daring night sleep. Consequertly patients reoort dastime drowsiness and irritability. Itis associeted with snoring The diagnosis is established bby polygraphic recording of sleap which show periods (at least 30 of 10 or more second’s duration in seven hours of sleep) of apnea associabed with fallin arterial oxygen saturation, 2/25/2014 10:16:50 AM ‘Mark this question & => Question Ti : 29753 Question 4 of 30 Sleep apnea smekome is best diagnosed by the following: a) Blood gases during apneic episodes +) EEG c) Presence of HLA-DR? and DQw1 Y © 4) Polygraphic sleep studies ©) Spirometry Question Explanation: Sleep apneais characterized by cessation of breathing sleep. This causes extreme restlessness with frequent respiratory pauses daring night sleep. Consequertly patients reoort dastime drowsiness and irritability. Itis associeted with snoring The diagnosis is established bby polygraphic recording of sleap which show periods (at least 30 of 10 or more second’s duration in seven hours of sleep) of apnea associabed with fallin arterial oxygen saturation, 2i25/2014 10:17:01 AM ‘Mark this question & => Question Td : 48923 Question 5 of 30 Extrinsic allergic alveolitis may be acquired by breathing air contaminated hy which substance? a) Fine silicaceous dust ») Degreating solvent c) Urea formaldehyde foam insulation a) Fungal spores 2) Corine gas Answer | Explanation Other User's Explanation Report An Error (Question Explanation: Extrinsic alleraic alveolitis chypersenstivity pneumonitis) is a type of inflammetion in end around the tiny air sacs (alveol) and smallest airways (bronchioles) of the lnng caused by an allergic reaction to inhaled organic dusts or, less commonly, chemicals. Tt can be caused by sensitization to many organic dusts mainly fungal spores. Many types of dust can cause allergic reactions in the fangs. Organic dusts that contain rrictoorganisms or proteins and chemicals, such as isocyanates, may cause hypersensitivity pneumonitis. Farmer's ling, which results from repeated inhalation of heat-lovirg (thermophilic) bacteria in moldy hay, is a well- known example of hypersensitivity preumonilis 2i25/2014 10:17:01 AM ‘Mark this question & => Question Td : 48923 Question 5 of 30 Extrinsic allergic alveolitis may be acquired by breathing air contaminated hy which substance? a) Fine silicaceous dust ») Degreating solvent c) Urea formaldehyde foam insulation Y © A) Fungal spores 2) Corine gas Answer | Explanation Other User's Explanation Report An Error (Question Explanation: Extrinsic alleraic alveolitis chypersenstivity pneumonitis) is a type of inflammetion in end around the tiny air sacs (alveol) and smallest airways (bronchioles) of the lnng caused by an allergic reaction to inhaled organic dusts or, less commonly, chemicals. Tt can be caused by sensitization to many organic dusts mainly fungal spores. Many types of dust can cause allergic reactions in the fangs. Organic dusts that contain rrictoorganisms or proteins and chemicals, such as isocyanates, may cause hypersensitivity pneumonitis. Farmer's ling, which results from repeated inhalation of heat-lovirg (thermophilic) bacteria in moldy hay, is a well- known example of hypersensitivity preumonilis 2i25/2014 10:17:13 AM Mark this question & => Question Td : 51327 Question 6 of 30 Pulmonary function tests are being performed on an 80 years old male with persistent cough. Which one ofthe following would occur normally with age? 2) Fall in vital capacity ) Fallin FEVIFVC ) Fallin arterial pO 8) Increases in arterial pCOy ©) Decreased in arterial pe Question Explanation: Pulmonary function tests PFT's) provide measures of low rates. lung volumes. gas exchange, and respiratory muscle function. Basic pulmonary finction tesis avoilable in the ambulatory setting include spirometry and pulse oximetry, these tests provide physiologic measures of pulmonary function and can be used to quickly arrow a differential diagnosis and suggest a subsequent sirategy of additional testing or therapy. FEV1, FVC, FEF25-75% and PEF increase and FEVIFVC% decrease with age until about 20 years old in femsles and 25 years in males. ABer this, all incices gracually fll, though the precise rate of decline is probably masked due to the complex: intesrelationship between age and height. The fall in FEVIFVC% with age in adults is due to the greater decline in FEV1 than FV, n Report An Error 2i25/2014 10:17:13 AM Mark this question & => Question Td : 51327 Question 6 of 30 Pulmonary function tests are being performed on an 80 years old male with persistent cough. Which one ofthe following would occur normally with age? 2) Fallin vital capacity JV © b) Fallin FEVIEVC ©) Fallin arterial pO <) Increases in arterial pCO, ©) Decreased in arterial pe Question Explanation: Pulmonary function tests PFT's) provide measures of low rates. lung volumes. gas exchange, and respiratory muscle function. Basic pulmonary finction tesis avoilable in the ambulatory setting include spirometry and pulse oximetry, these tests provide physiologic measures of pulmonary function and can be used to quickly arrow a differential diagnosis and suggest a subsequent sirategy of additional testing or therapy. FEV1, FVC, FEF25-75% and PEF increase and FEVIFVC% decrease with age until about 20 years old in femsles and 25 years in males. ABer this, all incices gracually fll, though the precise rate of decline is probably masked due to the complex: intesrelationship between age and height. The fall in FEVIFVC% with age in adults is due to the greater decline in FEV1 than FV, n Report An Error Mark this question —& => Question Tid : 51629 Question 7 of 30 A 32 year old man with asthma complains of daily wheezing and occasional waking at night with cough and chest tightness for three week. His usual medication is salbutamel two pull tid-gid. What isthe nest step in management? a) Add long term Theophyline +) Increase salbutamol te twro puff gid ©) Ada ipratropium bromide two puffé gid 6) Add Beclomethasone two pulfe gid ¢) Discontinue salbutamol and begin prednisone 50 mg and taper over 2 weeks power ERIN) 0 ersslanaton epee ance Question Explanation: Stage one of asthma is when a patient has symptoms 2 or less times per week. For these patients the use of a short acting beta agonist (eg albuterol, salbutamol) on an as needed basis is recommended, Stage two of asthma is when a patienthas symptoms greater than twice a week. Atthis point you must add as inhaled steroid (eg fluticasone, beclomethasone) to the patient's teatment. Some physicians will dso add a night time dose of Montelukast. Mark this question —& => Question Tid : 51629 Question 7 of 30 A 32 year old man with asthma complains of daily wheezing and occasional waking at night with cough and chest tightness for three week. His usual medication is salbutamel two pull tid-gid. What isthe nest step in management? a) Add long term Theophyline +) Increase salbutamol te twro puff gid ©) Ada ipratropium bromide two puffé gid V © & Add Beclomethasone two puffs gid ¢) Discontinue salbutamol and begin prednisone 50 mg and taper over 2 weeks power ERIN) 0 ersslanaton epee ance Question Explanation: Stage one of asthma is when a patient has symptoms 2 or less times per week. For these patients the use of a short acting beta agonist (eg albuterol, salbutamol) on an as needed basis is recommended, Stage two of asthma is when a patienthas symptoms greater than twice a week. Atthis point you must add as inhaled steroid (eg fluticasone, beclomethasone) to the patient's teatment. Some physicians will dso add a night time dose of Montelukast. ‘Mark this question & => Question Td : 51679 Question § of 30 A 42-year-old female has cough for last 3 months that started with a "bad chest cold”. She was prescribed albuterol inhaler 6 weeks ago for reactive airway disease, Now she reports dyspnea on exertion, She takes nitrofarantoin for chronic UIT She is tachypneic PTs show FVC of LAL (avg 3.3) and FEV1 of 1.6L (avg 2.8). Inhaled bronchodilators do not improve these numbers. Likely cause of her problem is @) Chronic asthma +) Persistent coughing resuting from a viral respiratory iliness ©) Cardiac diseases 6) Intersitial lung diseases Question Explanation: This patient as a merkedly reduced FVC with an FEV /FVC ratio is 1.1426, This is consistent with moderately severe pulmonary restriction, Most likely the patent has chronic interstital restrictive lung disease, Nitrofuwranton can cause this picture, usually after continuous treatment for 6 or more months, and pulmonary finction may be. impaired permanently. A wide variety of additional causes have been described including noxious gases, puimonary hypersensitiities, neoplasia, and systemic diseases (e.g, sarcoidosis), Management includes avoidance of the offending agent or treatment of the underlying condition. ‘Mark this question & => Question Td : 51679 Question § of 30 A 42-year-old female has cough for last 3 months that started with a "bad chest cold”. She was prescribed albuterol inhaler 6 weeks ago for reactive airway disease, Now she reports dyspnea on exertion, She takes nitrofarantoin for chronic UIT She is tachypneic PTs show FVC of LAL (avg 3.3) and FEV1 of 1.6L (avg 2.8). Inhaled bronchodilators do not improve these numbers. Likely cause of her problem is @) Chronic asthma +b) Persistent coughing resuiting from a viral respiratory ilness 6) Cardiac diseases V © 4) Interstitial lung diseases Question Explanation: This patient as a merkedly reduced FVC with an FEV /FVC ratio is 1.1426, This is consistent with moderately severe pulmonary restriction, Most likely the patent has chronic interstital restrictive lung disease, Nitrofuwranton can cause this picture, usually after continuous treatment for 6 or more months, and pulmonary finction may be. impaired permanently. A wide variety of additional causes have been described including noxious gases, puimonary hypersensitiities, neoplasia, and systemic diseases (e.g, sarcoidosis), Management includes avoidance of the offending agent or treatment of the underlying condition. 2/25/2014 10:17:53 AM Mark this question & => Question Td : 54601 Question 9 of 30 An 80 year old male has a long history of COPD. His resting oxygen saturation is 86% on room air. Treatment includes oral bronchodilators, mhaled corticosteroids, iahaled Beta-agonist, inhaled choinergics, and home oxygen. Which one of hic treatments has been shown to prolong survival in such cases? 2) Oral bronchodilators ) Inhaled corticostercids ©) Tnhaled Beta-agonists 4) Home oxygen 6) Inhaled cholinergies Question Explanation: ‘Treatment of bypoxemia is criteal in the management of COPD and trials have shown a reduction in mortality with the use of oxygen for 15 or more hours daily, Inhaled Beta-adrenergic agonists and cholinergic agents, cither alone or in combination, provide syinptomatic relief but do not prolong survival. Theophylline can be used for symptoms inadequately relieved by bronchodilators Tnhaled corticosteroids do not appear to alter the rate of decline in hung function in COPD. However, some evidence shows that these agents alleviate symptoms and reduce disease exacerbation. Pulmonary rehabilitation improves quality of li and reduces, hospitalizations 2/25/2014 10:17:53 AM Mark this question & => Question Td : 54601 Question 9 of 30 An 80 year old male has a long history of COPD. His resting oxygen saturation is 86% on room air. Treatment includes oral bronchodilators, mhaled corticosteroids, iahaled Beta-agonist, inhaled choinergics, and home oxygen. Which one of hic treatments has been shown to prolong survival in such cases? a) Oral bronchodilators ) Inhaled corticostercids ©) Tnhaled Beta-agonists Y © 4) Home oxygen 6) Inhaled cholinergies Question Explanation: ‘Treatment of bypoxemia is criteal in the management of COPD and trials have shown a reduction in mortality with the use of oxygen for 15 or more hours daily, Inhaled Beta-adrenergic agonists and cholinergic agents, cither alone or in combination, provide syinptomatic relief but do not prolong survival. Theophylline can be used for symptoms inadequately relieved by bronchodilators Tnhaled corticosteroids do not appear to alter the rate of decline in hung function in COPD. However, some evidence shows that these agents alleviate symptoms and reduce disease exacerbation. Pulmonary rehabilitation improves quality of li and reduces, hospitalizations 2i25°2014 10:18:08 AM Mark this question & => Question Td : 55532 Question 10 of 30 A.27 year old patient who has a poorly controlled major seimre disorder and a 6 week history of recurrent fever, anorexia, and persistent, productive coughing visits your office. On physical examination he is noted to hawe a temperature of 383°C (101.0°F), a respirctory rate of 16/min, gngval hyperplasia, and a fetid odor to his breath. Lungs auscutation reveals rales in the mid-portion of the right lang posteriorly. The most lcely fincing on his chest radiograph would be which of the following? 8) Sarcoidosis, ) A lung abscess ©) Miliary calodications 6) A nght hilar mass 6) A right pleural effision Anewer [UEVIRISARY) omer users Explanation Report An Error Question Explanation: “Anaerobic fang abscesses are most often found in a person predisposed to aspiration who complains of a productive cough associated with fever, anorexia, and weakness. Physical examination usualty reveals poor dental hygiene, a fetid odor to the breath and sputum, rales, and pulmonary findings consistent with consolidation. Patients who have sercoidosis usually do not have a productive cough and have bilateral physical findinas A persistent productive cough is not a striking finding in disseminated tuberculosis, which would be suggested by miliary calcifications on a chest film. The clinical presentation and physical findings are not consistent with a simple mass in the right hilum or with 2 right pleural effusion. 2i25°2014 10:18:08 AM Mark this question & => Question Td : 55532 Question 10 of 30 A.27 year old patient who has a poorly controlled major seimre disorder and a 6 week history of recurrent fever, anorexia, and persistent, productive coughing visits your office. On physical examination he is noted to hawe a temperature of 383°C (101.0°F), a respiratory rate of 16/min, gngval hyperplasia, and a fetid odor to his bresth. Lange auscultation reveals rales in the maid portion of the sight lang posteriorly. The most licely fincing on his chest radiograph would be which of the following? 2) Sarcoidosis ¥ © b) A lung abscess ) Miliary caleifcations 6) Aight hilar mass ©) A right pleural effusion Anewer [UEVIRISARY) omer users Explanation Report An Error Question Explanation: “Anaerobic fang abscesses are most often found in a person predisposed to aspiration who complains of a productive cough associated with fever, anorexia, and weakness. Physical examination usualty reveals poor dental hygiene, a fetid odor to the breath and sputum, rales, and pulmonary findings consistent with consolidation. Patients who have sercoidosis usually do not have a productive cough and have bilateral physical findinas A persistent productive cough is not a striking finding in disseminated tuberculosis, which would be suggested by miliary calcifications on a chest film. The clinical presentation and physical findings are not consistent with a simple mass in the right hilum or with 2 right pleural effusion. Mark this question & => Question Td : 57038 Question 11 of 30 ‘An 86-year-old fom an elderiy care home has sudden onset of dyspnea and palpitations. A ventilation-perfusion scan shows a high probability for a perfusion defect involving a pulmonary arterial branch, Which of the following is the most important favering her complaint? 2) Neutrophilia ‘b) Cirrhosis of the liver €) Generalized atherosclerosis 4) Poor mutrition €) An increasing platelet count Question Explanation: ‘A neutrophilia would suggest infection leading to ventilation defect, and not a perfusion defect. ‘This would lead to a prothrombotic state, increasing the risk of pulmonary embolism, Citthosis and possibly poor murition, would lead to decreased production of coagulation factors thus prolonging the intemational normalized ratio (INR). “Atherosclerosis would pre-dispose to arterial thrombo-embtus, Mark this question & => Question Td : 57038 Question 11 of 30 An 86-year-old ffom an elderly care home has sudden onset of dyspnea and palpitations. A ventilation-perfusion scan shows a high probability for a perfusion defect involving a pulmonary arterial branch. Which of the following is the most important favoring her complaint? 2) Neutroptilia +) Cishocie of the liver ©) Generalized atherosclerosis 4) Poor nutrition Y © ©) An increasing platelet count Question Explanation: ‘A neutrophilia would suggest infection leading to ventilation defect, and not a perfusion defect. ‘This would lead to a prothrombotic state, increasing the risk of pulmonary embolism, Citthosis and possibly poor murition, would lead to decreased production of coagulation factors thus prolonging the intemational normalized ratio (INR). “Atherosclerosis would pre-dispose to arterial thrombo-embtus, 2:25/2014 10: AM “Mark this question => Question Id: 60340 Question 12 of 30 A patient presents with the manifestations of early-stage sarcoidosis, There are currently no symptoms. Management at this time should consist of 8) Nonsteroidal anti-inflarametory (NSAIDs) medivations +) Prednisclone. 6) Mo treatment 4) Methotrexate 6) Cyclosporine. Question Explanatio No treatment is necessary in the early stages of Sarcoidosis, as most of the patients will go on to spontaneous remission. NSAIDs are usefil for the cysternio manifectatione of the disease. Corticostercide are the treatment of choice for cevere and Me threatening disease. Immunosuppressive drugs may also be hefpfil in the treatment of severe sarcoidosis, 2:25/2014 10: AM “Mark this question => Question Id: 60340 Question 12 of 30 A patient presents with the manifestations of early-stage sarcoidosis, There are currently no symptoms. Management at this time should consist of 8) Nonsteroidal anti-inflarametory (NSAIDs) medivations +) Prednisclone. YM © ©) No treatment. 4) Methotrexate 6) Cyclosporine. Question Explanatio No treatment is necessary in the early stages of Sarcoidosis, as most of the patients will go on to spontaneous remission. NSAIDs are usefil for the cysternio manifectatione of the disease. Corticostercide are the treatment of choice for cevere and Me threatening disease. Immunosuppressive drugs may also be hefpfil in the treatment of severe sarcoidosis, ‘Mark this question & => Question Td : 61329 Question 13 of 30 A. 60-year-old female is noted to have labored breathing six days after admission for low hack pain which required complet= bed rest, A pulmonary embolism is suspected. ‘The most appropriate initial intervention would be a) Asterial blood gas, b) Duplex ultrasound of the lower extremities ¢) Ventlaton-perfision scan 4) Pulmonary angiography, ©) IV heparin tral Question Explanation: ‘An attetial blood gas is the most appropriate intial sudy. A decrease in oxygen saturation and pO would suggest a high possibilty of pulmonary embolism. & history.physical exam, and electrocerdogram ate all part of the initial work-up of a patient with a suspected pulmonary embolism A duplex ultrasound of the lower extremities (B) may establish a diagnosis of DVT, but this does not help establish the diagnosis of pulmonary embolism. While itis thought thet pulmonary embolistns onginate from D'VTs, most patients with DVT do not go on to have pulmonary embolisms. IV heparin therapy (E) should be startedif there is a high suspicion of pulmonary embolism, but i in itself does not offer any diagnostic information. Choices (C) and (D) are good diagnostic teste for pulmonary embolism, but only after initial suspicion is established based on the previously mentioned parts of the snilial work-up. ‘Mark this question & => Question Td : 61329 Question 13 of 30 A. 60-year-old female is nated ta have labored breathing soe days after admission for low back pein which required complete hed rest. A pulmonary embolism is suspected. ‘The most appropriate initial intervention would be JY © a) Arterial blood gas. +b) Duplexultrasound of the lower extremities ©) Ventllation- perfusion scan, 4) Pulmonary angiography, ©) IV hepazin tial Question Explanation: ‘An attetial blood gas is the most appropriate intial sudy. A decrease in oxygen saturation and pO would suggest a high possibilty of pulmonary embolism. & history.physical exam, and electrocerdogram ate all part of the initial work-up of a patient with a suspected pulmonary embolism A duplex ultrasound of the lower extremities (B) may establish a diagnosis of DVT, but this does not help establish the diagnosis of pulmonary embolism. While itis thought thet pulmonary embolistns onginate from D'VTs, most patients with DVT do not go on to have pulmonary embolisms. IV heparin therapy (E) should be startedif there is a high suspicion of pulmonary embolism, but i in itself does not offer any diagnostic information. Choices (C) and (D) are good diagnostic teste for pulmonary embolism, but only after initial suspicion is established based on the previously mentioned parts of the snilial work-up. 2/25/2014 10:19:25 AM ‘Mark this question & => Question Id : 63828 Question 14 of 30 A 32 year old woman complains of swollen lymph nedes in her neck and painful nodules on her lower legs. A chest X-ray shows bilateral hilar and right paratracheal lymphadenopathy. The most likely dkagnacis is a) Lymphoma 1b) Metastatic thyroid carcinoma ) Teratoma 4) Sarvoidosis ©) Wegener's gramulomatosis Question Explanation: Sarcoidosis is characterized by nencaseating granulomas in one or more organs and tissues, etiology is unknown. The lungs and lymphatic system are most often affected, but sarcoidosis may affect any orgen. Diagnosis usually is first suspected because of pulmonary involvement and is confirmed by chest X-ray, biopsy, and exclusion of other canses of granulomatous inflammation. Chest Cray, or high-resolution chest CT will show hilar and mediastinal lymphadenopathy. First ine treatment.is with corticosteroids, Prognosis is excellent for limited disease but poor for more advanced disease. 22512014 10:19:25 AM ‘Mark this question & => Question Id : 63828 Question 14 of 30 A 32 year old woman complains of swollen lymph nedes in her neck and painful nodules on her lower legs. A chest X-ray shows bilateral hilar and right paratracheal lymphadenopathy. The most likely dlagnosis is a) Lymphoma 1b) Metastatic thyroid carcinoma ) Teratoma Y © 4) Sarcoidosis ©) Wegener's gramulomatosis Question Explanation: Sarcoidosis is characterized by nencaseating granulomas in one or more organs and tissues, etiology is unknown. The lungs and lymphatic system are most often affected, but sarcoidosis may affect any orgen. Diagnosis usually is first suspected because of pulmonary involvement and is confirmed by chest X-ray, biopsy, and exclusion of other canses of granulomatous inflammation. Chest solution chest CT wll show hilar and mediastinal lymphadenopathy. Fist ine treatmentis with corticosteroids. 3Eray, or high- Prognosis is excellent for limited disease but poor for more advanced disease. 2/25/2014 10:19:39 AM ‘Mark this question & => Question Id : 67115 Question 15 of 30 A.44 year oldman develops fever, cough with sputum production, night sweats, anorexia and weight loss. The expectorated spunum is foul smelling and bad tasting, Imaging shows development of an abscess in his lung. He is HIV negative. Which of the following organistrs cannot cause lung abscess in this patient? a) Preumocystis carini ») Staphylococcus aureus ¢) Klebsiella pneumoria 4) Pseudomonas Question Explanation: ‘Lung abscess can be caused by a bacterial infection that reaches the lungs in several ways. The most common is aspiration of oropharyngeal contents. Patients at the highest risk for developing hung abscess have the following risk factors: poor dentition, seizure disorder and alcohol abuse. In addition the following infectious etiologies of pneumonia may progress to parenchymal necrosis and dung abscess formation: psendomonas aeruginosa, Klebsiella pneumoniae, staphylococcus aureus, streptococcal pneumonia and nocardia species Pneumocystis jroveci formerly Pneumocystis carini) is an opportunistic bug that can cause lung abscess in HIV/AIDS patients by taking advantage of the patients weakened imumune system. 2/25/2014 10:19:39 AM ‘Mark this question & => Question Id : 67115 Question 15 of 30 A.44 year oldman develops fever, cough with sputum production, night sweats, anorexia and weight loss. The expectorated spunum is foul smelling and bad tasting, Imaging shows development of an abscess in his lung. He is HIV negative. Which of the following organistrs cannot cause ling abscess in this patient? ¥ © a) Pneumocystis carina ») Staphylococcus aureus ¢) Klebsiella pneumoria 4) Pseudomonas Question Explanation: ‘Lung abscess can be caused by a bacterial infection that reaches the lungs in several ways. The most common is aspiration of oropharyngeal contents. Patients at the highest risk for developing hung abscess have the following risk factors: poor dentition, seizure disorder and alcohol abuse. In addition the following infectious etiologies of pneumonia may progress to parenchymal necrosis and dung abscess formation: psendomonas aeruginosa, Klebsiella pneumoniae, staphylococcus aureus, streptococcal pneumonia and nocardia species Pneumocystis jroveci formerly Pneumocystis carini) is an opportunistic bug that can cause lung abscess in HIV/AIDS patients by taking advantage of the patients weakened imumune system. 2i25/2014 10:19:51 AM Mark this question & => Question Td : 67346 Question 16 of 30 A.56 year old man, with a history compatible with chronic bronchitis, presents because of progressive shortness of breath on exertion, In the history which of the following would not be anticipated? 2) Increased symptoms on exposure to smog ) An increased incidence of chrenie respiratory disease in farnly members ©) Recurrent episodes of pleurisy 4) A. 20 year history of emoking 6) Increased symptoms with acute respiratory infection Question Explanation: Bronchitis is inflammation of the upper airways, commonly following a URI. The cause is usually a viral infection though iis sometimes a bacterial infection; the pathogen is rarely identified. The most common symotom is cough with or without fever and/or sputum production, In patients with COPD, hemoptysis, burning chest pain, and hypoxemia may also occur. Diagnosis is clinical ‘Treatment is supportive, antbiotics are necessery only for patients with chronic lung disease, Prognosis is excellent in patients without lung disease, but in patients with COPD, acute respiratory falure may resuit, Pleunsy develops when something (usually a virus or bacterium) irritates the pleura, resulting in inflammation. Certain autoimmune diseases (such as systemic lupus erythematosus) can invitate the pleura, Pleurisy may also develop when cancer spreads from the hing or another part of the body to the pleura, initating it. Inhalation of asbestos can also cause pleurisy, as can (rarely) the use of certain drugs, such as nitrofurantoin or procainamide 2i25/2014 10:19:51 AM Mark this question & => Question Td : 67346 Question 16 of 30 A.56 year old man, with a history compatible with chronic bronchitis, presents because of progressive shortness of breath on exertion, In the history which of the following would not be anticipated? 2) Increased symptoms on exposure to smog ) An increased incidence of chrenie respiratory disease in farnly members Y¥ © ©) Recurrent episodes of pleurisy 4) A. 20 year history of emoking 6) Increased symptoms with acute respiratory infection Question Explanation: Bronchitis is inflammation of the upper airways, commonly following a URI. The cause is usually a viral infection though iis sometimes a bacterial infection; the pathogen is rarely identified. The most common symotom is cough with or without fever and/or sputum production, In patients with COPD, hemoptysis, burning chest pain, and hypoxemia may also occur. Diagnosis is clinical ‘Treatment is supportive, antbiotics are necessery only for patients with chronic lung disease, Prognosis is excellent in patients without lung disease, but in patients with COPD, acute respiratory falure may resuit, Pleunsy develops when something (usually a virus or bacterium) irritates the pleura, resulting in inflammation. Certain autoimmune diseases (such as systemic lupus erythematosus) can invitate the pleura, Pleurisy may also develop when cancer spreads from the hing or another part of the body to the pleura, initating it. Inhalation of asbestos can also cause pleurisy, as can (rarely) the use of certain drugs, such as nitrofurantoin or procainamide ‘Marle this question & => Question Ta : 83676 Question 17 of 30 A 36 year old man with a history of rhinitis and asthma presents to his physician with complaints of intermittent severe abdominal pein anda chronic maculopapular rach. Marked eosinophilia is revealed on peripheral blood smear Biopsy of a skin lesion demonstrates necrotizing vasculitis with large numbers of eosinophils. The mostlikely diagnosis is? @) Asthma exacerbation ') Churg-Strauss syndrome ©) Monckeberg arteriosclerosis 9) Urticaria 6) Wegener granulomatosis Question Explanation: ‘The most likely condition is Churg-Strauss syndrome, also known as allergic granulomatosis and angitis. This variant of polyarteitis nodosa is clinically associated with asthma and eosinophilia. The vascular lesions are those descbed in the question stem. Granvlomas ate present in many. but rot all cases. Pulmonery involvement may be prominent, but this is not always the case. Ih this cace the patient's abdorninel pain ic related to Gl vasculitis, and his clin rach is related to dermal vasculitis. Churg-Stranss syndrome should be suspected in any patient with vascultic symptoms eosinophilia, and astira. “Asthma exacesbation (choice A) does not cause abdominal pain and maculopapular rash. It presents with shortness of breath wheezing and decreased exercise tolerance; in severe exacerbation, akered sensorium and respiratory falure can ensue. It is caused bby a triggering agent such as cold air, allergens, pollatants and upper respiratory infections Monckeberg artericaclerosis (choice C) is a benign condition characterized by ring like calcifications of media of the small to ‘medium-sized muscular vessels, it spares the intima and adventitia, Pathogenesis is unknown and iis seen in patients greater than 50 years of age most commonly invelng femoral, tibial, gerital, ulnar and radial arteries. Calciications are nonobstructive and of litle clinical importance. Urticaria (choise D) os hives ie characterized by raised red ckin wheals, a skin condition that reeulte from an allergy and ie caused by erect contact with an allergen, It does nt involve abdominal pain or asthma Lesions are itchy and may have associated angioedema. “Wegener granulomatosis (choice E) affects the respiratory and renal systems. Its characterized by focal necrotizing vascultis and necrotizing granulomas. IE does not cause abdominal pain and asthma. It causes sinusitis, rhinorrhea, epistasis, ottis media, hemoptysis, cough, and renal faitire. Cutaneous manifestations include subcutaneous nodules, papules, vesicles, ulcers, petechiae, and pyogenic gangronosurn, ‘Marle this question & => Question Ta : 83676 Question 17 of 30 A 36 year old man with a history of rhinitis and asthma presents to his physician with complaints of intermittent severe abdominal pein and a chronic maculopapuler rach. Mzrleed eosinophilia is revealed on peripheral blood amear Biopsy of a ckin lesion demonstrates necrotizing vascullis with large numbers of eosinophils. The most likely diagnosis is? @) Asthma exacerbation Y © b) Churg-Strauss syndrome ©) Monckeberg arteriosclerosis 4) Ueticaria ©) Wegener granulematosis Question Explanation: ‘The most likely condition is Churg-Strauss syndrome, also known as allergic granulomatosis and angitis. This variant of polyarteitis nodosa is clinically associated with asthma and eosinophilia. The vascular lesions are those descbed in the question stem. Granvlomas ate present in many. but rot all cases. Pulmonery involvement may be prominent, but this is not always the case. Ih this cace the patient's abdorninel pain ic related to Gl vasculitis, and his clin rach is related to dermal vasculitis. Churg-Stranss syndrome should be suspected in any patient with vascultic symptoms eosinophilia, and astira. “Asthma exacesbation (choice A) does not cause abdominal pain and maculopapular rash. It presents with shortness of breath wheezing and decreased exercise tolerance; in severe exacerbation, akered sensorium and respiratory falure can ensue. It is caused bby a triggering agent such as cold air, allergens, pollatants and upper respiratory infections Monckeberg artericaclerosis (choice C) is a benign condition characterized by ring like calcifications of media of the small to ‘medium-sized muscular vessels, it spares the intima and adventitia, Pathogenesis is unknown and iis seen in patients greater than 50 years of age most commonly invelng femoral, tibial, gerital, ulnar and radial arteries. Calciications are nonobstructive and of litle clinical importance. Urticaria (choise D) os hives ie characterized by raised red ckin wheals, a skin condition that reeulte from an allergy and ie caused by erect contact with an allergen, It does nt involve abdominal pain or asthma Lesions are itchy and may have associated angioedema. “Wegener granulomatosis (choice E) affects the respiratory and renal systems. Its characterized by focal necrotizing vascultis and necrotizing granulomas. IE does not cause abdominal pain and asthma. It causes sinusitis, rhinorrhea, epistasis, ottis media, hemoptysis, cough, and renal faitire. Cutaneous manifestations include subcutaneous nodules, papules, vesicles, ulcers, petechiae, and pyogenic gangronosurn, Mark this question & => Question Td : 8695: Question 18 of 30 A.44 year old female who had a bilateral mastectomy 2 weeks ago for breast cancer complains of shortness of breath. Her doctor notices decreased breath sounds on the right side of the lung. Radiographic examination reveals a large, right sided pleural effusion. The fluid is tapped. Analysis of the fluid reveals a high triglyceride level relative to the serum; effusion cholesterol levels, however, are reduced when compared to serum, No malignant cells are revealed on effusion cytology. Culture is negative for bacieria, The most likely cause of the effusion is a) Congestive heart failure +b) Damage to the thoracic duct ©) Decreased oncotic pressure 4) Fungal infection 6) Upward tracking of abdominal ascites Question Explanation: Datnage to the thoracic duct can occur secondary to neoplastic infiltration; additionally, the thoracic duct could have been damaged in the surgical removal of the breasts. Chylous effisione appear grossly milky white, with high tiglycerices and low to variable cholesterol compared with serum. Ths differential diagnosis inclades lymphoma, damage to the thoracic duct, and other carcinomas Given the history of bilateral mastectomies, mechanical disruption of the thoracic duct curing the surgery is the most likely etiology. Damage ofthe duct leads to leakage of lymphatic Buid in to the pleural cavity. The combination of high trialycerides and low cholesterol is indicative of a chylous effusion. Clongestive heart faitire can lead to bilateral effusions. These are transudates and do not contain Ligh serum levels. The protein content of the transudate and, therefore, specific gravity are low. Certain chemotherapeutic agents can lead to cardiomyopathies. Decreased oncotic pressure not only leads to a pleural effusion but can also lead to systemic generalized edema called anasarca, Fungal infections can lead to a pseudochylous effision. This is caused by the breakdown of cell membranes by the fungus. Cholesterol is high in these effusions: trialycerides are low, clfferentiating fungal infections from chylous ffusicns Abdominal ascites can track through pores in the diaphragm. As with transudates, ascites do act usually contain high ssiglycerides Report An Error Mark this question & => Question Td : 8695: Question 18 of 30 A.44 year old female who had a bilateral mastectomy 2 weeks ago for breast cancer complains of shoress of breath. Her doctor notices decreased breath sounds on the right side of the lung. Radiographic examination reveals allarge, right sided pleural effusion. ‘The fuidis tapped. Analysis of the fuid revels a high triglyceride level relative to the serum, effusion cholesterol levels, however, are reduced when compared te serum. No malignant cells are revealed on effision cytology. Culture is negative for bacteria, The most licely cause of the effision is 8) Congestive heart Pilure Y¥ © b) Damage to the thoracic duct ©) Decreased oncotic pressure 4) Fungal infection 6) Upward tracking of abdominal ascites Question Explanation: Datnage to the thoracic duct can occur secondary to neoplastic infiltration; additionally, the thoracic duct could have been damaged in the surgical removal of the breasts. Chylous effisione appear grossly milky white, with high tiglycerices and low to variable cholesterol compared with serum. Ths differential diagnosis inclades lymphoma, damage to the thoracic duct, and other carcinomas Given the history of bilateral mastectomies, mechanical disruption of the thoracic duct curing the surgery is the most likely etiology. Damage ofthe duct leads to leakage of lymphatic Buid in to the pleural cavity. The combination of high trialycerides and low cholesterol is indicative of a chylous effusion. Clongestive heart faitire can lead to bilateral effusions. These are transudates and do not contain Ligh serum levels. The protein content of the transudate and, therefore, specific gravity are low. Certain chemotherapeutic agents can lead to cardiomyopathies. Decreased oncotic pressure not only leads to a pleural effusion but can also lead to systemic generalized edema called anasarca, Fungal infections can lead to a pseudochylous effision. This is caused by the breakdown of cell membranes by the fungus. Cholesterol is high in these effusions: trialycerides are low, clfferentiating fungal infections from chylous ffusicns Abdominal ascites can track through pores in the diaphragm. As with transudates, ascites do act usually contain high ssiglycerides Report An Error ‘Mark this question & => Question Id : 87749 Question 19 of 30 A.33 year old Agtican American male dancer injures himself during a dance routine and is seen in the emergency room for tender ribs. A chest radiograph is performed and is shown below: ? + No evidence of nb fractures is present Bilateral hilar nodes and middle zone parenchymal infiltrates are noted on radiograph. He developed similar episode approximately 1 year ago in which he experienced fever. ankle swelling, and tender red bumps on his lower extremities. Mo medical complaints are currently present and he is able to perform without difficulty. The patient is suffering from which one of the folowing? a) No disease: the patient is healthy b) Pneumocystis carinii pneumonia c) Systemic sclerosis 4) Wegener granulomatosis 8) Sarcoidosis Question Explanation: This patient has sarcoidosis, a multisystem disease of unkaown etiology characterized by noncaseeting granulomas on histologic examination of various organs. Sarcoidosic ocours mostly in Afkican Americans in the third to th decades. Approximately 90% of cases involve the lungs, with findings such as hilar adenopatby and pulmonary infiltration Other findings include uveitis, exythema nodosum (the lesions on this patient's lower extremities), arthritis, central and peripheral neuropathies, cardiomyopathy, and hypercalcemia Symptoms include fatigue, exertional dyspnea, and nonproductive cough. Diagnosis is made after excliding other cances by examination ofa tistue biopsy. Radiographic staging is performed by serial chect X-rays. Pneumocystic carinii pneumonia (PCP) is a disease almost exclusively of HIV infected patients, usually with a characteristic chest radiograph appearance of bilatera, fluffy infiltrates. Systemic sclerosis, or scleroderma, is a systemic collagen-vascular disease that involves the skin nearly 100% ofthe time, and the esephagus in approximately 75% of patients, Wogener gramiloraatosis is a disorder characterized by focal necrotising vasculitis mainly in the hinge, kidneys, and nasal passages The first sign is commonly rhinitis, followed by hemoptysis and renal faiure. Diagnosis is confirmed by biopsy of a granuloma demonstrating leukocytoclastic vasculitis with necrotic changes and granulomatous inflammation cANCA positive staning of leukocvtes aide in the diagnosis. ‘Mark this question & => Question Id : 87749 Question 19 of 30 A.33 year old Agtican American male dancer injures himself during a dance routine and is seen in the emergency room for tender ribs. A chest radiograph is performed and is shown below: ? + ‘No evidence of rib fractures is present. Bilateral hilar nodes and middle zone parenchymal infiltrates are noted on radiograph. He developed similar episode approximately 1 year ago in which he experienced fever. ankle swelling, and tender red bumps on his lower extremities. No medical complaints are currently present and he is able to perform without difficulty. The patient is suffering from which one of the following? a) No disease: the patient is healthy b) Pneumocystis carinii pneumonia c) Systemic sclerosis d) Wegener granulomatosis Y © 2) Sarcoidosis Question Explanation: This patient has sarcoidosis, a multisystem disease of unkaown etiology characterized by noncaseeting granulomas on histologic examination of various organs. Sarcoidosic ocours mostly in Afkican Americans in the third to th decades. Approximately 90% of cases involve the lungs, with findings such as hilar adenopatby and pulmonary infiltration Other findings include uveitis, exythema nodosum (the lesions on this patient's lower extremities), arthritis, central and peripheral neuropathies, cardiomyopathy, and hypercalcemia Symptoms include fatigue, exertional dyspnea, and nonproductive cough. Diagnosis is made after excliding other cances by examination ofa tistue biopsy. Radiographic staging is performed by serial chect X-rays. Pneumocystic carinii pneumonia (PCP) is a disease almost exclusively of HIV infected patients, usually with a characteristic chest radiograph appearance of bilatera, fluffy infiltrates. Systemic sclerosis, or scleroderma, is a systemic collagen-vascular disease that involves the skin nearly 100% ofthe time, and the esephagus in approximately 75% of patients, Wogener gramiloraatosis is a disorder characterized by focal necrotising vasculitis mainly in the hinge, kidneys, and nasal passages The first sign is commonly rhinitis, followed by hemoptysis and renal faiure. Diagnosis is confirmed by biopsy of a granuloma demonstrating leukocytoclastic vasculitis with necrotic changes and granulomatous inflammation cANCA positive staning of leukocvtes aide in the diagnosis. 9913, “Mark this question = => Question Td Question 20 of 30 ‘The most important strategy in preventing postoperative pneumonia after an elective cholecystectomy performed under general anesthesia is 2) Prophylactic wide spectrum antibiotic b) Tracheal suction ©) Good pulmonary todet with incentive spirometry 4) Steroids ©) Theophylline Answer | Bolanation | Other User's Explanation Report An Error Question Explanation: Preventing atelectasis by encouraging deep breathing and coughing and use of incentive spirometry is the best way to prevent pneumonia in this setting in an otherwise healthy individual Prophylactic use of wide spectrum antibiotics increases the rske of developing a resistant infection ifthe patient becomes infected. Tracheal suction, stercids, and theophylline have no beneficial roles in this semting, 9913, “Mark this question = => Question Td Question 20 of 30 ‘The most important strategy in preventing postoperative pneumonia after an elective cholecystectomy performed under general anesthesia is 2) Prophylactic wide spectrum antibiotic b) Tracheal suction Y © 0) Good pulmonary todet with incentive spirometry 4) Steroids ©) Theophylline Answer | Bolanation | Other User's Explanation Report An Error Question Explanation: Preventing atelectasis by encouraging deep breathing and coughing and use of incentive spirometry is the best way to prevent pneumonia in this setting in an otherwise healthy individual Prophylactic use of wide spectrum antibiotics increases the rske of developing a resistant infection ifthe patient becomes infected. Tracheal suction, stercids, and theophylline have no beneficial roles in this semting, 2/25/2014 10:21:41 AM ‘Mark this question = => Question Ii : 95669 Question 21 of 30 A 24 year old female presents with aoctumel dry cough and dyspnea on exertion, On physical examination, she was found to be wheezing. All of the following tests can make the diagnosis of asthnna EXCEPT? a) Peak dow meier ) Chest X-ray c) Methacholine challenge test ) Empitic tril of beta 2 agonists ¢) Exacerbation of wheezing during allergen exposure Anower (UBQNRIBEMY) otnor Urors Explanation Report An Error Question Explanation ‘A chest X-ray will usually be normal or show hyperinflation, but would not support a definitive diagnosis of asthma, which is a reversible airway disease. A forced expiratory volume of less than one liter in one second is highly supportive of severe asthina exacerbation, A peak flow meter can give us a clue as to the severity of an asthma attack. Inhaled methacholine can cause bronchoconstriction and, thus. wheezing when the diagnosis of asthtra is unclear. Inhaled beta 2 agonists will result in bronchodilation and, thus, alleviation of asthmatic symptoms Environmental asthma triggers, such as dust mites, cockroaches, cat protein dander, mold, and various pollen exposure, can precipitate wheezing and an asthma attack, 2/25/2014 10:21:41 AM ‘Mark this question = => Question Ii : 95669 Question 21 of 30 A 24 year old female presents with aoctumel dry cough and dyspnea on exertion, On physical examination, she was found to be wheezing. All of the following tests can make the diagnosis of asthnna EXCEPT? a) Peak dow meier Y © b) Chest X-ray c) Methacholine challenge test ) Empitic tril of beta 2 agonists ¢) Exacerbation of wheezing during allergen exposure Anower (UBQNRIBEMY) otnor Urors Explanation Report An Error Question Explanation ‘A chest X-ray will usually be normal or show hyperinflation, but would not support a definitive diagnosis of asthma, which is a reversible airway disease. A forced expiratory volume of less than one liter in one second is highly supportive of severe asthina exacerbation, A peak flow meter can give us a clue as to the severity of an asthma attack. Inhaled methacholine can cause bronchoconstriction and, thus. wheezing when the diagnosis of asthtra is unclear. Inhaled beta 2 agonists will result in bronchodilation and, thus, alleviation of asthmatic symptoms Environmental asthma triggers, such as dust mites, cockroaches, cat protein dander, mold, and various pollen exposure, can precipitate wheezing and an asthma attack, 2/25/2014 10:21:51 AM “Mark this question &=> Question Id : 97066 Question 22 of 30 For which one of the following is desensitization with increesirg doses of subcutaneous antigen injection NOT indicated? a) Bee sting sensitivity ) Asthma c) Tree allergy. ) Mold allergy. e) Dust allergy. Question Explanation: Deseasiiization is not indicated for asthma unless a known antigen is responsible for the bronchospasm. Allergies to bee venom, tree pollen, mold, and dust are all amenable, to desensitization. 2/25/2014 10:21:51 AM “Mark this question &=> Question Id : 97066 Question 22 of 30 For which one of the following is desensitization with increesirg doses of subcutaneous antigen injection NOT indicated? a) Bee sting sensitivity Y¥ © b) Asthma c) Tree allergy. ) Mold allergy. e) Dust allergy. Question Explanation: Deseasiiization is not indicated for asthma unless a known antigen is responsible for the bronchospasm. Allergies to bee venom, tree pollen, mold, and dust are all amenable, to desensitization. ‘Mark this question & => Question Td : 107396 Question 23 of 30 ‘A. 66 year old man who is diagnosed with @ new lung nodule notices that he is having difficulty getting out of the bathtub anc out of his favorite easy chair. What is the licelest cause of his symptoms? 4) Polymayalgia cheumatica ) Cachexta ) Polymyostis 4) Trichinosis ) Infection with coxsackievirus Question Explanation: ‘This patient likely has polymyositis in association wth ung cancer. Elderly men are the most common group in which this association occurs. Polymyalaia theumatica would not be associated with a lung lesion and would cause more pain than weakness. Cachexia would not occur this quickly and would nat specifically cause proximal muscle weakness, Trichinosis is a parasitic infection that would be unlikely to present in this fashion. Coxsackicvims infection can cause myalgias, but they arc act usually proximal only, end they are not associated with pulmonary lesions. ‘Mark this question & => Question Td : 107396 Question 23 of 30 ‘A. 66 year old man who is diagnosed with @ new lung nodule notices that he is having difficulty getting out of the bathtub anc out of his favorite easy chair. What is the licelest cause of his symptoms? 4) Polymayalgia cheumatica ) Cachexta Y © ¢) Polymyosits, 4) Trichinosis ) Infection with coxsackievirus Question Explanation: ‘This patient likely has polymyositis in association wth ung cancer. Elderly men are the most common group in which this association occurs. Polymyalaia theumatica would not be associated with a lung lesion and would cause more pain than weakness. Cachexia would not occur this quickly and would nat specifically cause proximal muscle weakness, Trichinosis is a parasitic infection that would be unlikely to present in this fashion. Coxsackicvims infection can cause myalgias, but they arc act usually proximal only, end they are not associated with pulmonary lesions. Mare this question & => Ouestion Id: 109602 Question 24 of 30 A 52 year old mele is brought to the emergency room with a history of methanol ingestion, He is found to have a profound acidosis, with apCO2 of 47. What is mostllkely to be present in this individual? 2) Metabolic acidosis 1b) Mixed metabolic acidesis and respiratory acidosis ©) Respiratory acidosis 6) Mixed metabolic acidosis and respiratory alkalosis ©) Nonmal anion gap Question Explanation: ‘Methanol intoxication produces amixed metabolic acidosis end respiratory acidosis due to methanol induced metabolic acidosis followed by depressed respirations and respiratory acidosis. Because of the direct suopression of breathing due to methancl, a pure ‘metabolic acidosis is not seen, Since methanol itselfis resporsitle for metabolic acidosis, pure respiratory acidosis does not orcur Option D is incorrect because respiratory alkalosis is not a result of methanol intoxicetion. Methanol produses an increased arion gap, not a normal one, Mare this question & => Ouestion Id: 109602 Question 24 of 30 A.52 year old male is brought to the emergency room with a history of methanol ingestion. He is found to have a profound acidosis with apCO2 of 47. What is most likely to be present in this individual? 2) Metabolic acidosis V © b) Mixed metabolic acidosis and respiratory acidosis 6) Respiratory acidasis 6) Mixed metabolic acidosis and respiratory alkalosis ©) Nonmal anion gap Question Explanation: ‘Methanol intoxication produces amixed metabolic acidosis end respiratory acidosis due to methanol induced metabolic acidosis followed by depressed respirations and respiratory acidosis. Because of the direct suopression of breathing due to methancl, a pure ‘metabolic acidosis is not seen, Since methanol itselfis resporsitle for metabolic acidosis, pure respiratory acidosis does not orcur Option D is incorrect because respiratory alkalosis is not a result of methanol intoxicetion. Methanol produses an increased arion gap, not a normal one, 225.2014 10:22:43 AM ‘Mark this question & => Question Id : 111061 Question 25 of 30 A woman aged 58 years with along history of cigarette smoking and chronic cough presents with acute shortness of breath. She is found to have an acute respiratory alkalosis. Her arterial blood gas examination would most likely show a) Hypoxemia ) Hypoxemia. ©) Hypercapnia, <) Nomal pH. ¢) Notmal oxygen level Answer | Exrianation | Other User's Explanation Report An Error Question Explanation: A cigarette smoker with acute shortness of breath and respiratory akcelosis is most licely to have kypoxemia, which is the stimulation for the hyperventilation. The bicarbonate level and pCO2 should be low in this setting, The pH is elevated by defintion in allalosis. Cheice Eis not correct 225.2014 10:22:43 AM ‘Mark this question & => Question Id : 111061 Question 25 of 30 A woman aged 58 years with along history of cigarette smoking and chronic cough presents with acute shortness of breath. She is found to have an acute respiratory alkalosis. Her atterial blood gas examination would most likely show Y © a) Bypoxemia ) Hypoxemia. ©) Hypercapnia, <) Nomal pH. ¢) Notmal oxygen level Answer | Exrianation | Other User's Explanation Report An Error Question Explanation: A cigarette smoker with acute shortness of breath and respiratory akcelosis is most licely to have kypoxemia, which is the stimulation for the hyperventilation. The bicarbonate level and pCO2 should be low in this setting, The pH is elevated by defintion in allalosis. Cheice Eis not correct 2i25/2014 10:22:57 AM ‘Mark this question e& => Question 26 of 30 “Whats the most likely cause of bilateral hilar adenopathy? a) Sarcoidosis. ) Tuberculosis °c) Malignancy 4) Fungal infection ©) Histiocytosis Anower (UEQRSESREN) incr User's Explanation Report An Exot Question Explanation: Question Td : 113182 Sarcoidosis is the number one cause of bilateral hiler adenopathy Malignancy is the number one cause of unilateral kilar adenopathy. ‘All of the causes can present either bilaterally or unilaterally. 2i25/2014 10:22:57 AM ‘Mark this question e& => Question 26 of 30 “Whats the most likely cause of bilateral hilar adenopathy? Y © a Sarcoidosis. ) Tuberculosis ) Malignancy 4) Fungal infection ¢) Histiocytosis Anower (UEQRSESREN) incr User's Explanation Report An Exot Question Explanation: Question Td : 113182 Sarcoidosis is the number one cause of bilateral hiler adenopathy Malignancy is the number one cause of unilateral kilar adenopathy. ‘All of the causes can present either bilaterally or unilaterally. ‘Mark this question —& => Question Td : 136533 Question 27 of 30 A.33 year old man presents with hemoptysis, recurrent sinusits, and large cavitary nodules on chest X-ray. Investigation also demonstrates renal falure. The most lkely diagnosis is a) Goodpasture’s syndrome +b) Hodgkin’s lymphoma ©) Mycoplasma preunonia 4) Wegener's granulomatosis ©) Tuberculosis Anowor [UEQRRNRREN) omer Users Explanation Report An Exo Question Explanation: The main clinical tad is hemeptysis, upper respiratory ilness, and nodules on chest X-ray. Wegener's is a nccectising vesculits and includes granlometous inflammation, as well as glomeruionephriis. Goodpastare’s syndrome is not associated with recusrent sinusitis, and itis characterized by the deposition of complement fixing IgG antibody along the glomerular and alveolar membranes Hodgkin‘s lymphoma usually presents as hilar adenopathy, not cavities, on the chest X-ray. Mycoplasma pneumonia does not usually present as sinusitis, and the chest X- ray patterns interstitial in nature, Pulmonary tuberculosis does not usually have associated renal failure or sinusitis ‘Mark this question —& => Question Td : 136533 Question 27 of 30 A.33 year old man presents with hemoptysis, recurent sinusits, and large cavttary nodules on chest X-ray. Investigation also demonstrates reaal falure, The most lcely diagnosis is a) Goodpasture’s syndrome +b) Hodgkin’s lymphoma c) Mycoplasma pneumonia ¥ © d) Wegener’s granulomatosis ¢) Tuberculosis Anowor [UEQRRNRREN) omer Users Explanation Report An Exo Question Explanation: The main clinical tad is hemeptysis, upper respiratory ilness, and nodules on chest X-ray. Wegener's is a nccectising vesculits and includes granlometous inflammation, as well as glomeruionephriis. Goodpastare’s syndrome is not associated with recusrent sinusitis, and itis characterized by the deposition of complement fixing IgG antibody along the glomerular and alveolar membranes Hodgkin‘s lymphoma usually presents as hilar adenopathy, not cavities, on the chest X-ray. Mycoplasma pneumonia does not usually present as sinusitis, and the chest X- ray patterns interstitial in nature, Pulmonary tuberculosis does not usually have associated renal failure or sinusitis ‘Mark this question & => Question Id : 138654 Question 28 of 30 A woman aged 66 years develops acute thoracic spine pain after a bout of bronchitis. What finding would be present on a chest K- ray? a) Ostsoarthrits, ) Pneumonia. ©) Hilar adenopathy. 4) A thoracic compression fracture ©) A solitary lung mass. Question Explanatior ‘A postmencpausal women with acute pain such as this after relatively mirimal ranma ie., coughing from bronchitis is most Ucely to have a thoracic compression fracture on chest roentgenogram, as the result of osteoporosis. Osteoarthritis, pneumonia, hilar adenopathy, or a solitary lung lesion would not cause a syndrome such as this. ‘Mark this question & => Question Id : 138654 Question 28 of 30 A woman aged 66 years develops acute thoracic spine pain afier a bout of bronchitis. What finding would be present on a chest X- ray? a) Osteoarthritis ') Pheumoria. c) Hilar adenopathy. V © 4) A thoracic compression fracture. e) A solitary lung mass. Question Explanatior ‘A postmencpausal women with acute pain such as this after relatively mirimal ranma ie., coughing from bronchitis is most Ucely to have a thoracic compression fracture on chest roentgenogram, as the result of osteoporosis. Osteoarthritis, pneumonia, hilar adenopathy, or a solitary lung lesion would not cause a syndrome such as this. 2:25/2014 10: 35 AM “Mark this question = Question Id ; 180916 Question 29 of 30 A patient develops an acute febrile illness with shwers, nonproductive cough, and pleunitic chest pain. 5 days later he presents after abruptiy having "coughed up" neatly a cup of blood steined sputum. Most likely to be seen on a CXR is a) Blunting of diaphragmatic costal angles 6) A cavity with a fluid level ©) Complete opacification of one lobe with no additional findings 6) Patchy consolidetion centered on bronchi ©) Frominent bronchi that can be followed far out into the fung fields Avewor [UERRNERER) otror veore Explanation Report An Eror Question Explanatic Thic isa clascio peesertation of a pulmonary abscess. Chronic cources with lece cevere eymptcens (with intermittent improvement following short courses of antibiotics) are also sometimes seen, particularly ifthe diagnosis was not suspected. A chest X-ray fm typically shows paeumonic opaciication i which a cavity, offen with aflud level, is vsible. Puimonic abscesses can be caused by anaerobes (most common, particularly if aspiraticn initiated the abscess). gram negetive aerobic bacill, and staphylococci. Therapy is ‘baced on the organisms isolated and chould be continued for at least 4-6 weeks. In cazes that fal to recolve, the poccibilty of coexisting carcinoma should be considered. Option A is the x-ray appearance of pleural effsion. Option Cis the X-ray appearance of lobar pneumonia. Cption D is the X-ray appearance of broachopneumonia. Option Eis the X-ray appearance of bronchiectasis. 2:25/2014 10: 35 AM “Mark this question = Question Id ; 180916 Question 29 of 30 A patient develops an acute febrile ilness with shwers, nonproductive cough, and pleuritic chest pain. 5 days later he presents after abruptly having "coughed up" neatly a cup of blood stained sputum, Mos‘ likely to be seen ona CXR is a) Blunting of diaphragmatic costal angles ¥ © b) A cavity with a fluid level ©) Complete opacification of one lobe with ne additional findings 6) Patchy consolidation centered on bronchi c) Prominent bronchi that can be followed far out into the hung fields Avewor [UERRNERER) otror veore Explanation Report An Eror Question Explanatic Thic isa clascio peesertation of a pulmonary abscess. Chronic cources with lece cevere eymptcens (with intermittent improvement following short courses of antibiotics) are also sometimes seen, particularly ifthe diagnosis was not suspected. A chest X-ray fm typically shows paeumonic opaciication i which a cavity, offen with aflud level, is vsible. Puimonic abscesses can be caused by anaerobes (most common, particularly if aspiraticn initiated the abscess). gram negetive aerobic bacill, and staphylococci. Therapy is ‘baced on the organisms isolated and chould be continued for at least 4-6 weeks. In cazes that fal to recolve, the poccibilty of coexisting carcinoma should be considered. Option A is the x-ray appearance of pleural effsion. Option Cis the X-ray appearance of lobar pneumonia. Cption D is the X-ray appearance of broachopneumonia. Option Eis the X-ray appearance of bronchiectasis. ‘Mark this question Question Td : 20471 Question 1 of 30 ‘The major problem with the use of continuous positive airway pressure (CPAP) for obstmctive sleep apnea is a) Sneezing b) Dry macous membranes ©) Contined snoring 4) Noncompliance ¢) Rhinorhea Question Explanation Sneezing, rhinorrhea, and dry mucous membranes are common problems - associated with the use of continuous positive airway pressure (CPAP) for obstructive sleep apnea. However, the major problem with CPAP is lack of compliance; patients complain of discomfort, claustrophobia, panic atacks, intrusiveness, difficulty keeping the appliance in place, and noise from the machine. Mark this question => Question Td : 20471 Question 1 of 30 ‘Tae major problem with the use of continous positive airway pressure (CPAP) for obstructive sleep apneais a) Sneezing 'b) Dry macous membranes ©) Contined snoring V © 4) Noncompliance ¢) Rhinorrhea Question Explanation Sneezing, rhinorrhea, and dry mucous membranes are common problems - associated with the use of continuous positive airway pressure (CPAP) for obstructive sleep apnea. However, the major problem with CPAP is lack of compliance; patients complain of discomfort, claustrophobia, panic atacks, intrusiveness, difficulty keeping the appliance in place, and noise from the machine. Mark this question = => Question Td : 28994 Question 2 of 30 ‘Which ofthe following statements is true in regard to diagnose Allergic bronchopulmonary Aspergillosis? a) The immmedite skin test to an extract of Aspergilus fumigatus is positive ) The CO transfer factor is unaffected c) Recurrent hemoptysis is a characteristic feature 4) Circulating IgG precipitins to Aspergilus fimigatue are negative ) Plewal effusion is a complication Question Explanation: Immediate (type 1) reactions occur in virtually all patients wth ABPA following intradermal injections of A. fumigatus extracts, wath oaly 16% developing delayed (type II) reactions. Precipitating IgG antibodies are present in 70% of patients. Transfer factor may be alfected in the later Bbrotic stage of the disease. Haemoptysis is symptom of aspergiloma and bronchiectasis, but is not cheracteristic of ABPA, Mark this question = => Question Td : 28994 Question 2 of 30 ‘Which of the following statements is true in regard to diagnose Allergic bronchopulmonary Aspergillosis? Y © a) The immediate skin test to an extract of Aspergilus fumigatus is positive ) The CO transfer factor is unaffected c) Recurrent hemoptysis is a characteristic fecture 4) Circulating IgG precipitine to Aspergile fnigatas are negative ) Plewal effusion is a complication Question Explanation: Immediate (type 1) reactions occur in virtually all patients wth ABPA following intradermal injections of A. fumigatus extracts, wath oaly 16% developing delayed (type II) reactions. Precipitating IgG antibodies are present in 70% of patients. Transfer factor may be alfected in the later Bbrotic stage of the disease. Haemoptysis is symptom of aspergiloma and bronchiectasis, but is not cheracteristic of ABPA, Mark this question A patient's arterial blood gas analysis gives the following results: & => Question 3 of 30 Question Td : 29104 po 10 kPa/?SearaFlg (11.3-12.6 kPa) co, [7 kPat52 mF (47-60 kPa) St [2.47 (0.36-7.44) Bicarbonate 57 mmol (20-28) ‘Which of the following is the most likely cause? a) Acute exacerbation of chronic obstructive pulmonary disease b) Pyloric obstruction ©) Chronic hyperventilation syndrome 4) Diabetic coma ¢) Pulmonary embolism Anower [REGISRSINY) Other Users Explanation Report An Exot Question Explanation: These resuits demonstrate a metabolic allcalosis and there is respiratory compensation with an elevation of pCOz. Consequently, pO is slightly low. The most probable cause is pyloric stenosis, Mark this question A patient's arterial blood gas analysis gives the following results: & => Question 3 of 30 Question Td : 29104 po 10 kPa/?SearaFlg (11.3-12.6 kPa) co, [7 kPat52 mF (47-60 kPa) St [2.47 (0.36-7.44) Bicarbonate 57 mmol (20-28) ‘Which of the following is the most likely cause? a) Acute exacerbation of chronic obstructive pulmonary disease Y © b) Pyloric obstruction ©) Chronic hyperventilation syndrome 4) Diabetic coma ¢) Pulmonary embolism Anower [REGISRSINY) Other Users Explanation Report An Exot Question Explanation: These resuits demonstrate a metabolic allcalosis and there is respiratory compensation with an elevation of pCOz. Consequently, pO is slightly low. The most probable cause is pyloric stenosis, ‘Mark this question —& => Question Ti : 29683 Question 4 of 30 AAl-year-old male hes bronchiectasis. He is breathless and is a victim of recurrent infections. Inspiratory crackles are present ‘Which of the followings likely to decrease the frequency of his exacerbations? a) Postural drainage ') Cyclical antibiotic therapy c) Inhaled corticosteroids ) Vitamin supplements e) Pancreatic enzyme replacements Question Explanation: As this patient has Bronchiectasis, he wil have regular production of spumm associated with breathlessness, repeated lurg infections and the signs of lnspiratosy bilateral crackles are present. Retained mucus is the most important reason why bronchuectatc patients become infected, Postural drainage is therefore the comerstone to treating bronchiectasis and should be undertaken at least once per day and more equently during exacerbation. There have been trials looking at regular antibiotic therapy versus symptomatic treatment in patients with cystic Bbrosis colonized with pseudomonas but there is currently no evidence that this approach is of benefit in bronchiectass. Similarly inhaled corkcosteroids should not be used routinely in bronchiectasis untl firther evidence of their effect on lung fimction and exacerbation frequency is available, Surgical resection as a curative procedure can be performed for localized direace when uaderlying causec cuch ac primary cliary dyskinesia have been excluded, In this pation: the bileteral crackles suggests widespread disease. ‘Mark this question —& => Question Ti : 29683 Question 4 of 30 AAl-year-old male hes bronchiectasis. He is breathless and is a victim of recurrent infections. Inspiratory crackles are present “Which of the followings likely to decrease the frequency of his exacerbations? WV © a) Posmural drainage ) Cyclical antibiotic therapy ©) Inhaled conticosteroids ) Vitamin supplements ©) Fancreatic enzyme replacements Question Explanation: As this patient has Bronchiectasis, he wil have regular production of spumm associated with breathlessness, repeated lurg infections and the signs of lnspiratosy bilateral crackles are present. Retained mucus is the most important reason why bronchuectatc patients become infected, Postural drainage is therefore the comerstone to treating bronchiectasis and should be undertaken at least once per day and more equently during exacerbation. There have been trials looking at regular antibiotic therapy versus symptomatic treatment in patients with cystic Bbrosis colonized with pseudomonas but there is currently no evidence that this approach is of benefit in bronchiectass. Similarly inhaled corkcosteroids should not be used routinely in bronchiectasis untl firther evidence of their effect on lung fimction and exacerbation frequency is available, Surgical resection as a curative procedure can be performed for localized direace when uaderlying causec cuch ac primary cliary dyskinesia have been excluded, In this pation: the bileteral crackles suggests widespread disease. Mark this question & => Question Td : 29743 Question 5 of 30 A 17-year-old boy with cystic fibrosis (CF) presents with abdontinal pain. Which of the following is most likely to be the cause? a) Invtable bowel syndrome ) Pyelonephritis ) Renal calculi 4) Meconium ileus equivalent syndrome ) Uloerative coltis 8) Whipple's disease Avower [UERINRTEN) osier veers Explanation Report An Eros Question Explanation: ‘Mesonium Jeus equivalent or distal intestinal obstructive syndrome occurs in older children and acutts with CF and presents with colicky abdominal pain, distension, vomiting and failure to pass feces. The plain abominal x-ray confirms small bowel obstruction Titi management incudes rehydration with intravenous Quids and oral N-acetyl cysteine. Other gastrointestinal complications of fassociations with CF include - Liver citthosis - Gall bladder disease - Pancreatitis - Peptic ulceration - Hiatus hemia ~ Celiac disease and - Crohn's disease. Mark this question & => Question Td : 29743 Question 5 of 30 A 17-year-old boy with cystic fibrosis (CE) presents with abdominal pain. Which of the following is most likely to be the cause? 2) Iitable bowel syndrome +) Pyelonephritis ©) Renal calcul Y © 4) Meconium ileus equivalent syndrome «) Ulcerative cottis £) Whipple's disease Avower [UERINRTEN) osier veers Explanation Report An Eros Question Explanation: ‘Mesonium Jeus equivalent or distal intestinal obstructive syndrome occurs in older children and acutts with CF and presents with colicky abdominal pain, distension, vomiting and failure to pass feces. The plain abominal x-ray confirms small bowel obstruction Titi management incudes rehydration with intravenous Quids and oral N-acetyl cysteine. Other gastrointestinal complications of fassociations with CF include - Liver citthosis - Gall bladder disease - Pancreatitis - Peptic ulceration - Hiatus hemia ~ Celiac disease and - Crohn's disease. 2:25/2014 10:26:32 AM Mark this question => Question Id : 29825 Question 6 of 30 A.66 year cld female presented with increasing fatigue, dyspnoea and a dry cough. An area of dense pneumoria-tike consoiidation in the right lower lobe is preseat on CXR. A couse of antibiotics did not improve her symptoms or the findings on chest X-ray. Bronchioalveolar lavage (BAL) retrieved "atypical" cells. What is the most licely diagnosis? 2) Mycoplasma pneumonia ) Bronchioloalveolar cell carcinoma ) Pulncnary alveolar proteinosis 4) Pulmonary em lism with infarction ©) Sarcoidesis, newer [SBIR otror ueorsExptan Question Explanation: Bronchoalveolar carcinoma accounts for between 1-20% of pulmonary neoplasms. The population most affected is middle-aged, with no prediction for either sexx. Interestingly, there is an increased incidence in pationts with ssleroderma or other disease causing localized parenchymal scarring or cifuse interstitial Sbrosis, Diffuse bilateral involvement in bronchoalveolar cell carcinoma occurs late in the disease and is usually spread by the bronchial tree. “Manifestations include both local and diffuse forms. The local form may grow very slowly charging litle for several years. The diffuse form simulates an airspace filling diceare with air bronchograms and air broncholograms. A pleural effvsien develops in 8-10% of n Report An Error 2:25/2014 10:26:32 AM Mark this question => Question Id : 29825 Question 6 of 30 A 66 year old female presented with increasing fatigue, dyspnoea and a dry cough. An area of dense pneumoria-like consolidation in the right lower lobe is present on CXR, A course of antibiotics did not improve her symptoms or the findings on chest X-ray. Bronchioalveolar lavage (BAL) retrieved "atypical" cells. What is the most lcely diagnosis? a) Mycoplasma pneumonia Y © b) Bronchicloalveolar cell carcinoma ©) Pulmonary alveolar proteinosis 4) Pulnonary embelism with infarction ©) Sarcoidesis, newer [SBIR otror ueorsExptan Question Explanation: Bronchoalveolar carcinoma accounts for between 1-20% of pulmonary neoplasms. The population most affected is middle-aged, with no prediction for either sexx. Interestingly, there is an increased incidence in pationts with ssleroderma or other disease causing localized parenchymal scarring or cifuse interstitial Sbrosis, Diffuse bilateral involvement in bronchoalveolar cell carcinoma occurs late in the disease and is usually spread by the bronchial tree. “Manifestations include both local and diffuse forms. The local form may grow very slowly charging litle for several years. The diffuse form simulates an airspace filling diceare with air bronchograms and air broncholograms. A pleural effvsien develops in 8-10% of n Report An Error Marke this question << => Question Td : 29975 Question 7 of 30 All of the following statements are not correct regarding chronic obstructive pulmonary disease except a) Patients show at least a 15 per cent improvement in the FEV after nebulised bronchodilator 'b) Breathlessness is uncommon untl the FEV falls to approximately 50 per cent of predicted ©) Inhaled corticosteroid usage does not improve long-term prognosis 4) Emphysema is associated with increaced transfer factor «) Ih advanced cases there is reduced pulmonary vascular resistance Question Explanation: ‘This level of improvement would mean the presence of asthma High dose inheled steroids have been shown (ISOLDE) to improve quality of if and reduce hospitalization rated by reducing the manber of exacerbations, butit does not slow the rate of dectine of FEV (hence doss not aifect prognosis). Breathlessness is common but subjective, Mild chronic obstructive pulmonary disease (COPD) (60 - 79% predicted FEV1) is often unknown to their GP. Those with moderate COPD (40 -59% predicted) are seen intermittently seen by GP, whilst those with severe disease (less than 40% predicted) have frequent hospital and GP visits. Lis asthma which is associated with normal or increased transfer factor. COPD is ascociated with decreased transfer factor. COPD is associated with secondary pulmonary hypertension. n Report An Error Marke this question << => Question Td : 29975 Question 7 of 30 All of the following statements are not correct regarding chronic obstructive pulmonary disease except 2) Patients show at least a 15 per cent improvement in the FEV after nebulised bronchodilator +b) Breathlessness is uncornmon untl the FEV falls to approximately 50 per cent of predicted Y © ¢) Inhaled corticosteroid usage does not improve long-term prognosis 4) Emphycema is associated with increaced trancfer factor «) In advanced cases there is reduced pulmonary vascular resistance Question Explanation: ‘This level of improvement would mean the presence of asthma High dose inheled steroids have been shown (ISOLDE) to improve quality of if and reduce hospitalization rated by reducing the manber of exacerbations, butit does not slow the rate of dectine of FEV (hence doss not aifect prognosis). Breathlessness is common but subjective, Mild chronic obstructive pulmonary disease (COPD) (60 - 79% predicted FEV1) is often unknown to their GP. Those with moderate COPD (40 -59% predicted) are seen intermittently seen by GP, whilst those with severe disease (less than 40% predicted) have frequent hospital and GP visits. Lis asthma which is associated with normal or increased transfer factor. COPD is ascociated with decreased transfer factor. COPD is associated with secondary pulmonary hypertension. n Report An Error Mark tis question = => Question a 54009 Question § of 30 A.19 year old boy presents with cough, headache, malaise, and fever for 2 weeks Physical examination shows a petient in good general condition with no abnormal findings on anscuation, Gram stain of the sputum is negative and the white blood cell count is normal. Chest X-ray shows bilatoral pneuraonia in the lower lobes without pleural effision. The most lisely cause is 2) Haemophilus infuenzae +) Staphylococcus aureus c) Mycoplasma pneumenia 4) Klebsiella pneumoniae «) Streptococcus pneumoniae Question Explanation: ‘Walking pneumonia (akc.a. mycoplasma pneumonia or atypical pneumonia) is a term used to describe a mild case of pneumonia, Itis a lung infection thatis caused by the mycoplasma pneumoniae organism Walking pneumonia is diferent ftom other types of ppnenmoria in that, even f untreated, walking pneumonia would not cause a person to be confined to a bed or require a hospital stay Signa and syeeptome of walking pasumonia usualy include a severe cough, fever, abdominal pain, headaches, and chills. Mycoplasma pneumoniae, the organism thar causes walking pneamonia, can be contagious if contact with the infected person is prolonged. ‘Walking pneumonia is transmitted through airbome droplets, which are dispersed when a person talks, sneezes, coughs or laughs ‘The onset of walking pneumonia takes approximately 1 to 3 weeks, beginning aradually, with the earliest sign being a dectease in energy. Soon thereafter, cold like symptoms begin. You will be able to distinguish the difference between walking pacumonia and a common cold because walking pneumonia will get worse over 2 weeks Mark tis question = => Question a 54009 Question § of 30 A.19 year old boy presents with cough, headache, malaise, and fever for 2 weeks Physical examination shows a patient in good general condition with no abnormal findings on anscuation, Gram stain of the spntum is negative and the whte blood cell count is normal. Chest X-ray chows bilateral pneumonia in the lower lobes without pleural effusion. The most likely cause is 2) Haemophilus infnenzae b) Staphylococcus aureus Y¥ © 0) Mycoplasma pneumonia 4) Klebsiella pneumoniae ¢) Streptococens pneamoriae Question Explanation: ‘Walking pneumonia (akc.a. mycoplasma pneumonia or atypical pneumonia) is a term used to describe a mild case of pneumonia, Itis a lung infection thatis caused by the mycoplasma pneumoniae organism Walking pneumonia is diferent ftom other types of ppnenmoria in that, even f untreated, walking pneumonia would not cause a person to be confined to a bed or require a hospital stay Signa and syeeptome of walking pasumonia usualy include a severe cough, fever, abdominal pain, headaches, and chills. Mycoplasma pneumoniae, the organism thar causes walking pneamonia, can be contagious if contact with the infected person is prolonged. ‘Walking pneumonia is transmitted through airbome droplets, which are dispersed when a person talks, sneezes, coughs or laughs ‘The onset of walking pneumonia takes approximately 1 to 3 weeks, beginning aradually, with the earliest sign being a dectease in energy. Soon thereafter, cold like symptoms begin. You will be able to distinguish the difference between walking pacumonia and a common cold because walking pneumonia will get worse over 2 weeks “Mark this question & => Question Id ; 64048 Question 9 of 30 Astima has been associated wit all ofthe following agents in susceptible individuals EXCEPT a) Pollen. b) Dus: c) CFTR Mutations 4) Mold. €) Cold Question Explanation CETR Mutations produce cystic fbrosis “Mark this question & => Question Id ; 64048 Question 9 of 30 Astima has been associated wit all ofthe following agents in susceptible individuals EXCEPT a) Pollen. b) Dus: Y © co) CRTR Matations 4) Mold. €) Cold Question Explanation CETR Mutations produce cystic fbrosis 22512014 10:27:34 AM Mark this question & => Question Td : 65881. Question 10 of 30 A 57 year-old foundry worker hes difficuty breathing end dyspnea on exertion with no chest pain, fever, clubbing, or weight loss CHR reveals mubipie cmall nodules in the upper lebes and enlarged hilar lymaph nodes that contain caleitm on the outer rims (eggshell calcifications), Pulmonary function tests show a decrease in Inng volumes. Whats the most likely diagnosis? a) Tuberculosis, +) Histoplasmosis ©) Mycobacterium avinm intracellulere 4) Silicosis ©) Byssinosis Question Explanation: Silicosis is an occupational disorder characterized by inhalation of crystalline silicon dhoxide particles from working in rock mining, sand blasting, or glass blowing The main symptom is shortness of breath, Silica crystals are present in the lang nodales end pulmonary fibrosis can resul from long-term, chronic exposure. ‘The classic radiological sign is "eggshell hilar calcifications." “Antinuclear aniibodies are present in up to 40% of patients with slicosis, Tuberculosis usually presents with fever and weight loss The nedules also may cavitate on the chest X-ray Histoplasmosis con also prevent with hilar adenopathy, but weight loss and fever are common as well as a history of exposure to pigeons prevalent in a geographic location (Ohio's Mississippi Valley). MAT infection is common in end-stage AIDS. High fevers and weight loss are prevalent, Byssinosis is an occupational airway disorder thet occurs in workers exposed to cotton, hemp, or flax. The symptoms of chest tightness and dyspnea are most prominent during the initial ‘working shit following a weekend, and tend to diminish over the course of cach work-week. Long-term exposure to cotton dust can result in interstitial fibrosis Report An Error 22512014 10:27:34 AM Mark this question & => Question Td : 65881. Question 10 of 30 A 57 year-old foundry worker hes difficuty breathing end dyspnea on exertion with no chest pain, fever, clubbing, or weight loss CHR reveals mubipie cmall nodules in the upper lebes and enlarged hilar lymaph nodes that contain caleitm on the outer rims (eggshell calcifications), Pulmonary function tests show a decrease in Inng volumes. Whats the most likely diagnosis? a) Tuberculosis, +) Histoplasmosis ©) Mycobacterium avinm intracellulere Y © *) Silicosis ©) Byssinosis Question Explanation: Silicosis is an occupational disorder characterized by inhalation of crystalline silicon dhoxide particles from working in rock mining, sand blasting, or glass blowing The main symptom is shortness of breath, Silica crystals are present in the lang nodales end pulmonary fibrosis can resul from long-term, chronic exposure. ‘The classic radiological sign is "eggshell hilar calcifications." “Antinuclear aniibodies are present in up to 40% of patients with slicosis, Tuberculosis usually presents with fever and weight loss The nedules also may cavitate on the chest X-ray Histoplasmosis con also prevent with hilar adenopathy, but weight loss and fever are common as well as a history of exposure to pigeons prevalent in a geographic location (Ohio's Mississippi Valley). MAT infection is common in end-stage AIDS. High fevers and weight loss are prevalent, Byssinosis is an occupational airway disorder thet occurs in workers exposed to cotton, hemp, or flax. The symptoms of chest tightness and dyspnea are most prominent during the initial ‘working shit following a weekend, and tend to diminish over the course of cach work-week. Long-term exposure to cotton dust can result in interstitial fibrosis Report An Error ‘Mark this question & => Question Id Question 11 of 30 An old man traveled to North America from Australia After 5 days he develops dyspnea, fever, tachycardia andlis rushed to the hospital. The most likely diagnosis is a) Pulmonary embolism ') Pneumonia c) Myocarcial infarction 4) Aortic dissection Aneweor [REIRIRTRHAN) othe: User's Explanation — Repost An vor Question Explanatior Pulmonary embolicm (PE) is the occlusion of one or more pulmonary arteries by thrombi that originate eleewhere, typioaly in the large veins of the lower extremity or pelvis. Neatly all PEs arise fiom thrombi in the lower extremity or pelvic veins (deep venous thrombosis (DVD). Virchow’s tad’ of stass, typercoagilabilty and endotheloal damage are sisk factors for developing DVT. ‘This man has sat on a very long airplare fight from Australia to North America. Mesting the criteria for stasis. His symptoms are classic for PE which can inctude: acute dyspnea, pleuriic chest pain, tachycardia and tachyrnea. The frst symptom in an older patient may be atered mental stanus ‘Mark this question & => Question Id Question 11 of 30 An old man traveled to North America from Australia After 5 days he develops dyspnea, fever, tachycardia andlis rushed to the hospital. The most licely diagnocis is VW © a) Fulmonary embolism +) Pneumoria c) Myocardial infarction 4) Acttic dissection Aneweor [REIRIRTRHAN) othe: User's Explanation — Repost An vor Question Explanatior Pulmonary embolicm (PE) is the occlusion of one or more pulmonary arteries by thrombi that originate eleewhere, typioaly in the large veins of the lower extremity or pelvis. Neatly all PEs arise fiom thrombi in the lower extremity or pelvic veins (deep venous thrombosis (DVD). Virchow’s tad’ of stass, typercoagilabilty and endotheloal damage are sisk factors for developing DVT. ‘This man has sat on a very long airplare fight from Australia to North America. Mesting the criteria for stasis. His symptoms are classic for PE which can inctude: acute dyspnea, pleuriic chest pain, tachycardia and tachyrnea. The frst symptom in an older patient may be atered mental stanus 2i25/2014 10:28:04 AM Marke this question <=> Question Td : 67175 Question 12 of 36 A. 32 yese eld man has abistory of recurrent pneumoniae and chronic cough dating Som easly childhood. The cough, whichis worse in the morning and on lying down, is productive of foul-smelling purulent sputum whichis occasionally bloody tinged. The patient is chronicaly il end has clubbed fingers. Rales are heard over the posterior lung bases. The most likely dkamnosis is 2) Chronic bronchitis b) Pulnenary aspergillosis ©) Pulmonary neoplasm 4) Chroric obstractive emphysema «) Bronchiectasis Question Explanation: Bronchiectasis is dilation and destruction of larger bronchi caused by chronic infection and inflammation, Common causes are cystic Sbrosis, immune defects, and infections, though some cases appear to be idiopathic. Syreptoms are chronic cough end purulent sputum expectoration, sce patients may also have fever and dyspnea Diagnosis is based on history and imaging, usually involving hich resolution CT, though standard chest x-rays may be diagnostic. ‘Treatment and prevention of acute exacerbation are with antibiotics, drainage of cecretions, and management of ccmplications, such as superinfections and hemoptysis. Treatment of underlying causes is important whenever possible Report An Error 2i25/2014 10:28:04 AM Marke this question <=> Question Td : 67175 Question 12 of 36 A. 32 yese eld man has abistory of recurrent pneumoniae and chronic cough dating Som easly childhood. The cough, whichis worse in the morning and on lying down, is productive of foul-smelling purulent sputum whichis occasionally bloody tinged. The patient is chronicaly il end has clubbed fingers. Rales are heard over the posterior lung bases. The most likely dkamnosis is 2) Chronic bronchitis b) Pulnenary aspergillosis ©) Pulmonary neoplasm 4) Chroric obstractive emphysema Y © @) Bronchiectasis Question Explanation: Bronchiectasis is dilation and destruction of larger bronchi caused by chronic infection and inflammation, Common causes are cystic Sbrosis, immune defects, and infections, though some cases appear to be idiopathic. Syreptoms are chronic cough end purulent sputum expectoration, sce patients may also have fever and dyspnea Diagnosis is based on history and imaging, usually involving hich resolution CT, though standard chest x-rays may be diagnostic. ‘Treatment and prevention of acute exacerbation are with antibiotics, drainage of cecretions, and management of ccmplications, such as superinfections and hemoptysis. Treatment of underlying causes is important whenever possible Report An Error ‘Mark this question —& => Question Id : 83686 Question 13 of 30 A.60 year old man is hospitalized. A chest X-ray film was obtained at the hospital and showed 250 ec of uid with a pleural fuid protein/serum protein ratio of 0.7, and a lactate dehydrogenase level of 240 U/L. The contltion that may lead to development ofthis Binding within plewal cavity is a) Bacterial pleusitis 1b) Cirthasis af the liver c) Congestive near failure 4) ) Protein losing enteropathy Question Explanation: Nephrotic syndrome Exudative effusions are characterized by one or more of the following: pleural uid protein'serum protein 0.5, pleural Quid LDH/serum LDH >0.6, of pleural fuid LDH more than two-thirds the normal upper limit for serum, An exudate results from leakage of protein-rich ud from the plasma into the interstitium. Itis usvally the result of increased vascular permeability caused by inflammation Exudates also contain mmerons acute or chronic inflammatory cells, depencing on the inciting event OF the choices given here, only bacterial pleusitis would produce an exudate. IEpleuntis is caused by pyogenic organisms, the exudate is purulent (heurrophil-tick). If pleural inflammation is due to mycobacterial infection or neoplastic infitration, the resulting exudate will contain chronic inflammatory cells Tn contrast, a transudate contains less protein and few inflammatory cells. There are tao ntain mechanisms of transudate formation: (), decreased oncotic pressure, suck as that which occurs in cirrhosis ofthe liver, nephrotic syndrome, and protein-losing enteropathy (choices B, D, and E); and (2) increased hydrostatic pressure, which may result from congestive heart failare (choice C) ‘Mark this question —& => Question Id : 83686 Question 13 of 30 4.00 year old man is hospitalized, & chest X-ray film was obiained at the hospital and showed 250 ce of fu with a plewral fud protein(serum protein ratio of 0.7, anda lactate dekycrogenase level of 240 U/L. The consition that may lead to development of this finding within pleural cavity is Y © a) Bacterial pleuritis 'b) Cirthosis ofthe fier ©} Congestive near faiure a €) Provein losing enteropathy Question Explanation: Nephrotic syndrome Exudative effusions are characterized by one or more of the following: pleural uid protein'serum protein 0.5, pleural Quid LDH/serum LDH >0.6, of pleural fuid LDH more than two-thirds the normal upper limit for serum, An exudate results from leakage of protein-rich ud from the plasma into the interstitium. Itis usvally the result of increased vascular permeability caused by inflammation Exudates also contain mmerons acute or chronic inflammatory cells, depencing on the inciting event OF the choices given here, only bacterial pleusitis would produce an exudate. IEpleuntis is caused by pyogenic organisms, the exudate is purulent (heurrophil-tick). If pleural inflammation is due to mycobacterial infection or neoplastic infitration, the resulting exudate will contain chronic inflammatory cells Tn contrast, a transudate contains less protein and few inflammatory cells. There are tao ntain mechanisms of transudate formation: (), decreased oncotic pressure, suck as that which occurs in cirrhosis ofthe liver, nephrotic syndrome, and protein-losing enteropathy (choices B, D, and E); and (2) increased hydrostatic pressure, which may result from congestive heart failare (choice C) ‘Marks this question = => Question Ta : 84343 Question 14 of 30 ATS year old man of Korean descent goes to see his primary care physician complaining of chest pain, dlificuky breathing, and general fatigue. When interviewed, he answers the physician's questions respectfily in a soft voice with litle eye contact. Whea questioned about any pain, he indicates that tis always with him, Subsequent examination and testing leads the physician to suspect that the patient might have lung cancer. When the possibility is mentioned to the patient, he becomes very quite, stares at the floor and says sofly, but distinctly, "I do not think I went to know this ifit is true." What would be the physicien’s best reply at this point? a) "Tknow cancer can seem a bit ightening but modem advances in treatment give us some options you may not be aware of" 'b) "Tunderstand the custom of your culture is not to discuss thece things, but I'l need you to work with me co we can beat ie" c) "Tunderstand your hesitation, respected Grandfather. Iwill work with your fanily if thet is what with me co we can beat ee 4) "Tim sorry, tut I will need your cooperation as we arrange treatment, Iwill tellyou and you will have some decisions to make." €) "Tellme abit more about why you do not want te know." 8) "Ifit is your wish, I will aot tell you. Whom in your family should [talk to in your place? ) "a this country, patients have to do everything, T'l let you know when we have more definite information." 1h) "Let's wait until we know something definitive and then we can talk about thing further." 4)"OK. Tl respect your wishes and not tell you." Question Explanation: ‘More information is needed before the physician can decide how to proceed. The requirements of Informed consent require that the patients know his diagnosis so that he can make informed treatment decisions tut this is not the momert to force that Issue. By understanding the patient's thinking end way he does not want to know his cagnosis, the physician is better positioned to address his concems, dispel aay myths, and provide whatever information is needed, By getting to understand the patients conceras in detail, a context is set for the discussion of whatever diagnosis might come down the road. Choice A makes assumptions as to what the patient's concerns ate. The assumptions may be right, but also may be off the mack. Before commenting, Snd out what the patient ic thinking, Choice B also malces an asrumaptioa. This time, the assumption i that the ppatieat does not want to know the diagnosis for reasons linked to his cultural background, This may or may aot be correct. Get information before making assumptions Choice C tries to work within the cutture as the physician understands i. Ibrisks offending the patient by actingin a manner incorsistent with the patients expectations ofthe physician and, again, in making atsumption to the reason for the patient's request Note that the paticnt rmust ultimately be told his diagnosis for reasons of informed consent. Only telling the family and not the paticat is not really an option. Choice Dis a statement of fact, but too harsh and too premature, Find out the patients concerns before laying out what the rules are. ‘The patient must be. told of his diagnosis for reasons of informed consent, Telling the family, (choice F) and not the patient is not an option. Choice Gis a statement of fact, but is also too harsh, and too premature, In addition, this option is insulting, suggesting the patient is an outsider of some kind who does not know "this country.” The issue is up for discussion. Have the discussion now, not later (choice H). Choice Lis incorrect. The patient must be told his eas 2a ei Se TS ee eee ae ‘Marks this question = => Question Ta : 84343 Question 14 of 30 ATS year old man of Korean descent goes to see his primary care physician complaining of chest pain, dlificuky breathing, and general fatigue. When interviewed, he answers the physician's questions respectfily in a soft voice with litle eye contact. Whea questioned about any pain, he indicates that tis always with him, Subsequent examination and testing leads the physician to suspect that the patient might have lung cancer. When the possibility is mentioned to the patient, he becomes very quite, stares at the floor and says sofly, but distinctly, "I do not think I went to know this ifit is true." What would be the physicien’s best reply at this point? a) "Tknow cancer can seem a bit ightening but modem advances in treatment give us some options you may not be aware of" 'b) "Tunderstand the custom of your culture is not to discuss thece things, but I'l need you to work with me co we can beat this." c) "Tunderstand your hesitation, respected Grandfather. Iwill work with your fanily if thet is what with me co we can beat this" 4) "Tim sorry, tut I will need your cooperation as we arrange treatment, Iwill tellyou and you will have some decisions to make." Y © 2)"Tellme abit more about why you do not want to know." £)"Ifit is your wish, I wall aot tell you. Whom in your family should Ital to in your place? 2) "In this country, patients have to do everything, I'l let you know when we have more definitive information." h) "Let's wait unt we know something definitive and then we can talk about thing further.” 4) "OK. I'l respect your wishes and act tell you." Question Explanation: ‘More information is needed before the physician can decide how to proceed. The requirements of Informed consent require that the patients know his diagnosis so that he can make informed treatment decisions tut this is not the momert to force that Issue. By understanding the patient's thinking end way he does not want to know his cagnosis, the physician is better positioned to address his concems, dispel aay myths, and provide whatever information is needed, By getting to understand the patients conceras in detail, a context is set for the discussion of whatever diagnosis might come down the road. Choice A makes assumptions as to what the patient's concerns ate. The assumptions may be right, but also may be off the mack. Before commenting, Snd out what the patient ic thinking, Choice B also malces an asrumaptioa. This time, the assumption i that the ppatieat does not want to know the diagnosis for reasons linked to his cultural background, This may or may aot be correct. Get information before making assumptions Choice C tries to work within the cutture as the physician understands i. Ibrisks offending the patient by actingin a manner incorsistent with the patients expectations ofthe physician and, again, in making atsumption to the reason for the patient's request Note that the paticnt rmust ultimately be told his diagnosis for reasons of informed consent. Only telling the family and not the paticat is not really an option. Choice Dis a statement of fact, but too harsh and too premature, Find out the patients concerns before laying out what the rules are. ‘The patient must be. told of his diagnosis for reasons of informed consent, Telling the family, (choice F) and not the patient is not an option. Choice Gis a statement of fact, but is also too harsh, and too premature, In addition, this option is insulting, suggesting the patient is an outsider of some kind who does not know "this country.” The issue is up for discussion. Have the discussion now, not later (choice H). Choice Lis incorrect. The patient must be told his eas 2a ei Se TS ee eee ae Mark this question & => Question Td : 84900 Question 15 of 30 A 55 year oldman is admitted to the hospital with cough and dyspnea, which over a period of years has progressed to marked dyspnea and cyanosis. Wedge biopsy of the lung demonstrates air spaces filled with macrophages containing lipid, periodic acid- Schiff PAS)-positive granules, and letnellar bodies. Intersitial pneumonitis, hyperplasia of the septal lining epithelial cells, and esquamation of epithelial cells into alveoliis also present. What is the composition of lamellar bodies found in the macrophages? a) Amyloid 6) Calcitonin o) Fibrin & Hemosiderin ) Surfactant Question Explanation: The disease is desquamative interstitial pneumonitis (DIP), which is an idiopathic process related to idiopathic interstitial fbrosis, DIP may respord to steroid therapy but may also progress to end stage lung disease. Desquamative interstitial pneumonitis is characterized by diffuse and uniform involvement of the hung parenchyma. The alveoli are filled with macrophages and hyperplastic type TI pneumocytes. The lamellar (layered) bodies within macrophages contain surfactant derived fiom type TI pneumocytes. Classic findings on chest X-ray are bilateral, lower-lobe, hazy infiltrates resembling; ground glass. “Amyloid (choice A) is deposited extracellularly and is not part of the DIP process Calcitonin (choice B) is foundin medullary carcinoma of the thyroid, Fibrin deposition (choice C) can be part of the DIP process bu occurs as eosinophilic excracelular deposits, rather than as intracellular lamellar boctes ‘Hemosiderin (choice D) can accumulate in pulmonary macrophages, usually in the setting of congestive heart failure with microhemorrhages, and produces golden-brown granules. Mark this question & => Question Td : 84900 Question 15 of 30 A 55 year oldman is admitted to the hospital with cough and dyspnea, which over a period of years has progressed to marked dyspnea and cyanosis. Wedge biopcy of the hing demonstrates air epacee filled with macrophages containing lipid, periocic acid Schill PAS)-positive granules, and lamellar bodies. Interstitial pneumonitis, hyperplasia of the septal lining epithelial cells, and esquamation of epithelial ces into alveoliis also present. Wat is the composition of lamellar bodies found in the macrophages? 2) Amyloid 6) Calcitonin ©) Fibrin & Hemosiderin Y © ¢) Surfactant Question Explanation: The disease is desquamative interstitial pneumonitis (DIP), which is an idiopathic process related to idiopathic interstitial fbrosis, DIP may respord to steroid therapy but may also progress to end stage lung disease. Desquamative interstitial pneumonitis is characterized by diffuse and uniform involvement of the hung parenchyma. The alveoli are filled with macrophages and hyperplastic type TI pneumocytes. The lamellar (layered) bodies within macrophages contain surfactant derived fiom type TI pneumocytes. Classic findings on chest X-ray are bilateral, lower-lobe, hazy infiltrates resembling; ground glass. “Amyloid (choice A) is deposited extracellularly and is not part of the DIP process Calcitonin (choice B) is foundin medullary carcinoma of the thyroid, Fibrin deposition (choice C) can be part of the DIP process bu occurs as eosinophilic excracelular deposits, rather than as intracellular lamellar boctes ‘Hemosiderin (choice D) can accumulate in pulmonary macrophages, usually in the setting of congestive heart failure with microhemorrhages, and produces golden-brown granules. ‘Marle this question & => Question Ti : 86193 Question 16 of 30 A.62 year old male with 15 pack year history of smoking and marked chronic obstructive pulmonary disease (COPD) develops fever and a petsistent cough frequently tinged with blood. X-ray of the chest reveals a complex: cavitary lesion in the right upper lobe Bronchoscopic examination identiies a large Inng abscess from whick pare fisobacterinm nucleatum is cultured, The most licely source of the Fusobacteriam in this pation is which of the following? 2) Blood +) Infected aerosole ©) Oral cavity 4) Stomach 6) Upper respiratory tract Question Explanation: “Anaerobic lung abscesses commenly arise from aspirated oral secretions in patients with compromised neurological status (@.2., anesthesia, coma, intoxication), of in individuals with a depressed cough reflex. The most common anaerobic organisms isolated from lung abscesses include Bacteroides, Fusobacterium, and Feptococcus species, all of which are typical oral flora. Blood bere pathogens producing lung abscesses probably originate as emboli from vegetations on the right heart. The most licely pathogens in endocardtis are Staphylococcus and Streptococcus species; Fusobacterium would be very unusual Fusobacterium usualy is not pathogenic and lives in a commensal relationship with the host. Tnfective aerosols genevally contain organisms that are intrinsically pathogenic (e.g., mycobacteria, influenza virus). Iris the acicity of the gastric aspirate that most seriously injures the lung, not the gastric bacteria Fusobacterium do inhabit the gastrointestinal tract but viable bacteria are much ‘more likely to be transmitted from the oral secretions than from the stomach. The upper respiratory tractis generally sterle, although COPD is frequently complicated by chronic bronchitis. Numerous bacterial and viral pathogens caa be involved in chronic bronchitis, however, Fusobacterium is not an upper respiratory tract pathogen. ‘Marle this question & => Question Ti : 86193 Question 16 of 30 A.62 year old male with 15 pack year history of smoking and marked chronic obstructive pulmonary disease (COPD) develops fever and a petsistent cough frequently tinged with blood. X-ray of the chest reveals a complex: cavitary lesion in the right upper lobe Bronchoscopic examination identiies a large Inng abscess from whick pare fisobacterinm nucleatum is cultured, The most licely source of the Fusobacteriam in this pation is which of the following? 2) Blood +) Infected aerosole VY © 0) Oral cavity 4) Stomach 6) Upper respiratory tract Question Explanation: “Anaerobic lung abscesses commenly arise from aspirated oral secretions in patients with compromised neurological status (@.2., anesthesia, coma, intoxication), of in individuals with a depressed cough reflex. The most common anaerobic organisms isolated from lung abscesses include Bacteroides, Fusobacterium, and Feptococcus species, all of which are typical oral flora. Blood bere pathogens producing lung abscesses probably originate as emboli from vegetations on the right heart. The most licely pathogens in endocardtis are Staphylococcus and Streptococcus species; Fusobacterium would be very unusual Fusobacterium usualy is not pathogenic and lives in a commensal relationship with the host. Tnfective aerosols genevally contain organisms that are intrinsically pathogenic (e.g., mycobacteria, influenza virus). Iris the acicity of the gastric aspirate that most seriously injures the lung, not the gastric bacteria Fusobacterium do inhabit the gastrointestinal tract but viable bacteria are much ‘more likely to be transmitted from the oral secretions than from the stomach. The upper respiratory tractis generally sterle, although COPD is frequently complicated by chronic bronchitis. Numerous bacterial and viral pathogens caa be involved in chronic bronchitis, however, Fusobacterium is not an upper respiratory tract pathogen. 2i25/2014 10:29:40 AM ‘Mark this question e& => ‘Question Id : 88047 Question 17 of 30 A.74 year old man presents with profound weight loss dyspnea, hoarseness, and hemoptysis. He has a 40 pack year smoking history. ‘He complains of anorexia and profound weakness, and appears cachectic on examination Factor that mediate the abnormelities causing cachesia in this pation is a) ACTH-ike peptides ») Cross-reacting antbedies ¢) Parathyroid hormone related peptide d) TNF-f and other cytokines ©) The tumor's tutritional demands Answer | Explanation Other User's Explanation Report An Error Question Explanation Patients with extensive tumor burden develop a wasting syndrome referred to as tumor cachexia, There is a reduction in lean body mass, associated with, profound loss of appetite and weakness. Tumor necrosis factor-L+ (TNF-I4), and probably interferon-y and IL-1, are suspected to mediate tamor cachexsa by increasing catabolism of proteins and fat. Paraneoplastic syndromes affect 10% of cancer patients and are due to different mechanisms, such as production of hormones and, abnormal immune reactions. Paraneoplastic syndromes aesociated with carcinomac of the hing are often due to production of ACTH. like peptides or parathyroid hormone like peptide (choice C) which manifest with Cushing eyndrome and hypercalcemia, respectively. Some paraneoplastic syndromes affecting the brain, peripheral nerves, or skeletal muscle may result from the host's ‘reaction against the tumor. which leads to formation of, cross reacting antibodies to tumor antigens and neuronal or muscle antigens. Ibis well established that the tunor‘s nutritional demands do not play a significant role in cancer cachexia, 2i25/2014 10:29:40 AM ‘Mark this question e& => ‘Question Id : 88047 Question 17 of 30 A.74 year old man presents with profound weight loss dyspnea, hoarseness, and hemoptysis. He has a 40 pack year smoking history. ‘He complains of anorexia and profound weakness, and appears cachectic on examination Factor that mediate the abnormelities causing cachesia in this pation is a) ACTH-ike peptides ») Cross-reacting antbedies ¢) Parathyroid hormone related peptide Y¥ © 4d) TNF-E and other cytokines ©) The tumor's tutritional demands Answer | Explanation Other User's Explanation Report An Error Question Explanation Patients with extensive tumor burden develop a wasting syndrome referred to as tumor cachexia, There is a reduction in lean body mass, associated with, profound loss of appetite and weakness. Tumor necrosis factor-L+ (TNF-I4), and probably interferon-y and IL-1, are suspected to mediate tamor cachexsa by increasing catabolism of proteins and fat. Paraneoplastic syndromes affect 10% of cancer patients and are due to different mechanisms, such as production of hormones and, abnormal immune reactions. Paraneoplastic syndromes aesociated with carcinomac of the hing are often due to production of ACTH. like peptides or parathyroid hormone like peptide (choice C) which manifest with Cushing eyndrome and hypercalcemia, respectively. Some paraneoplastic syndromes affecting the brain, peripheral nerves, or skeletal muscle may result from the host's ‘reaction against the tumor. which leads to formation of, cross reacting antibodies to tumor antigens and neuronal or muscle antigens. Ibis well established that the tunor‘s nutritional demands do not play a significant role in cancer cachexia, Mark this question & => Question Td : 90752 Question 18 of 30 A.24 year old woman who recently had an upper respiratory ilness develops acute severe anterior chest pain and is brought to the emergency rocm by her faraily. She is atherwise healthy and taking no medicaticns The most likely cause of her symptoms is 2) An acute myocardial infarction +) A viral infection ©) A pneumothorax 4) A pulmonary embolus ©) Muscle spasms Question Explanation: ‘The most likely agent in this setting is coxsackiewims, which can produce severe chest pain in an otherwise healthy young person with recent nonspecific upper respiratory syndrome. An otherwise healthy young woman is an unlikely candidate for an acute rayocardial infarction. She has no evidence of any predisposition to a pneumothorax, which is more common in men with a history of previous pneamothoraces. The patient has no apperent predisposition to a pulmonary embolus, Muscle spasms would be unlikely to produce severe anterior chest pain. Mark this question & => Question Td : 90752 Question 18 of 30 A.24 year old woman who recently had an upper respiratory ilness develops acute severe anterior chest pain and is brought to the emergency rocm by her faraily. She is atherwise healthy and taking no medicaticns The most likely cause of her symptoms is 2) An acute myocardial infarction Y © b) A viral infection ©) A pneumothorax 4) A pulmonary embolus ©) Muscle spasms Question Explanation: ‘The most likely agent in this setting is coxsackiewims, which can produce severe chest pain in an otherwise healthy young person with recent nonspecific upper respiratory syndrome. An otherwise healthy young woman is an unlikely candidate for an acute rayocardial infarction. She has no evidence of any predisposition to a pneumothorax, which is more common in men with a history of previous pneamothoraces. The patient has no apperent predisposition to a pulmonary embolus, Muscle spasms would be unlikely to produce severe anterior chest pain. 2725/2014 3 AM “Mark this question & => Question 19 of 30 A.61 year old shipyard worker presents with shoriness ofbreath, malaise, weight loss, end anorexia, CXR shows bilateral pleural plaques, diaphragmatic plaques, and pleural thickening, The patent presents two weeks later with complains of pleurine chest pan. Pleural biopsy reveals a mesothelioma The most likely cause of this dow growing tumor is a) Benzedrine 6) Asbestos ©) Vinyl chloride 6) Isopropyl alcohol ©) Tobacso Anower [UERPIENSHEN) otnor User's explana Question Explanation: Asbestos exposure results in pleural plaques, peritonzal plaques, and, eventually, malignant mesothelioma of the hings if the exposure has been great. Asbestosis a group of fbrous silicates that are inhaled by workers exposed to this substance. Crocidolite fibers are most strongly associated with mesotheliomas Bengidine is a carcinogen that can resut in bladder cancer. Vinyl chloride has been associated with liver cancer. Long term exposure to isopropyl alcohol and wood dusts a tisk factor for cancer of the nasal cavity Cigarette smoking is the dominant cause of ng cancer in the United States and other Westem nations, Cigarette smolcing also increases the risk of laryngeal and oral cavity cancer. Report An Error 2725/2014 3 AM “Mark this question & => Question 19 of 30 4.61 year old shipyard worker presents with shoress of breath, malaise, weight loss, and anorexia. CXR shows bilateral pleural plaques, diaphragmatic plaques, and pleural thickening, The patert presents two weeks later with complains of pleurite chest pam. Pleural biopsy reveals a mesothelioma, The most Ukely cause of this dow growing tumor is a) Benzedrine ¥ © b) Asbestos c) Vinyl chloride ©) Isopropyl alcohol £) Tobacco Anower [UERPIENSHEN) otnor User's explana Question Explanation: Asbestos exposure results in pleural plaques, peritonzal plaques, and, eventually, malignant mesothelioma of the hings if the exposure has been great. Asbestosis a group of fbrous silicates that are inhaled by workers exposed to this substance. Crocidolite fibers are most strongly associated with mesotheliomas Bengidine is a carcinogen that can resut in bladder cancer. Vinyl chloride has been associated with liver cancer. Long term exposure to isopropyl alcohol and wood dusts a tisk factor for cancer of the nasal cavity Cigarette smoking is the dominant cause of ng cancer in the United States and other Westem nations, Cigarette smolcing also increases the risk of laryngeal and oral cavity cancer. Report An Error ‘Mark this question e& => Question Td : 92024 Question 20 of 30 Flu shots should NOT be given to which of the following groups anmuclly? 2) Physicians. b) Nurses. c) Healthy young adults. 4) Healthy elderly adults ¢) People with purmonery diseace. Question Explanation: Unless healthy young adults are medical caregivers, annual fu shots are not considered necessary All ofthe other groups are at risk: for contracting infiuenza or for very severe illness and sequelae (c.g, the elderly and people with hing disease), and flu shots are stroagly recommended for these groups ‘Mark this question e& => Question Td : 92024 Question 20 of 30 Flu shots should NOT be given to which of the following groups anmuclly? 2) Physicians. b) Nurses. Y © c} Healthy young adults, 4) Healthy elderly adults ¢) People with purmonery diseace. Question Explanation: Unless healthy young adults are medical caregivers, annual fu shots are not considered necessary All ofthe other groups are at risk: for contracting infiuenza or for very severe illness and sequelae (c.g, the elderly and people with hing disease), and flu shots are stroagly recommended for these groups 2i25/2014 10:30:39 AM Marke this question & => Question Td : 95875 Question 21 of 30 A 65 year old man complains of fever, chills, and coughing. The patient is a diabetic who admits to abusing alcohol. Physical examination shows a consolidation, which is reinforced by chest X-ray. Gram stain of the sputum demonstrates an encapsulated rod bacillus. The most lkely first line drug therapy to treat this pneumonia is a) Penicillin . ) Erythromycin, ) Ceftriaxone 4) Aminogiycoside. €) None of the above. Question Explanation: “Although both Haemophilus influenzae and Klebsiella pneumoniae are tivo important Grain negative bacteria, which cause lobar consolidating pneumonias, only Klebsiella has been shown to be associated with alcohol abusers and diabetics. Haemophilus influenzae pneumoria occurs often in patients with a chronic cardiopulmonary disease. However, the first line crug therapy for the treatment of pneumonia secondary to either Klebsiella or Haemophilus influenzae infection is ceftriaxone, Penicillin is usefil for Gram positive bacterial infections such as Streptecocons pneumoniae and Staphylococens aureus. Erythromycin is usefil for the treatment Report An Error of Mebsiella pneumonia, 2i25/2014 10:30:39 AM Marke this question & => Question Td : 95875 Question 21 of 30 A.65 year old man complains of fever, chills, and coughing. The patientis a diabetic who admits to abusing alcohol. Physical exanination shows a consolidation, which is reinforced by chest X-ray. Gram stain of the sputurn demonstrates an encapsulated 10d ‘bacillus. The most likely first line drug therepy to treat this pneumonia is a) Penicilin ) Erythromycin, Y © © Ceftriaxone. 4) Aminoglycocide ©) None of the above. Question Explanation: “Although both Haemophilus influenzae and Klebsiella pneumoniae are tivo important Grain negative bacteria, which cause lobar consolidating pneumonias, only Klebsiella has been shown to be associated with alcohol abusers and diabetics. Haemophilus influenzae pneumoria occurs often in patients with a chronic cardiopulmonary disease. However, the first line crug therapy for the treatment of pneumonia secondary to either Klebsiella or Haemophilus influenzae infection is ceftriaxone, Penicillin is usefil for Gram positive bacterial infections such as Streptecocons pneumoniae and Staphylococens aureus. Erythromycin is usefil for the treatment Report An Error of Mebsiella pneumonia, 2i25/2014 10:30:50 AM Marke this question & => Question Td : 97281 Question 22 of 30 A.27 year old woman had a dry cough that had been worsening for five months, She presented because of an acute bout of coughing associated with hemoptysis. CXR and follow-up CT scan of the lungs revealed a large anterior mediastinuin mass, Which of the following is NOT associated with anterior mediastinum mass? a) Thymoma. ) Retrosternal thyroid tumor. ¢) Germ cell mmor. 4) Neurogenic tumor. ) Teratoma. Question Explanation: Neurogenic tumors ate the most common primary mediastinal neoplasm. However, they present inost commonly as posterior mediastinal masses and majority are berign. Ten percent of primary mediastinal neoplasms are thymomas. They are usually present anteriorly and 25% are usually found to be malignant, Retrostemal thyroid tumors can present as an anterior, superior, and middle mediastinal mass. Germn cell tumors, bronchogenic cysts, aortic aneurysms, and pericardial cysts can al present as anterior mediastinal masses. Teratomas arise from the pericardium and often attach to the base ofthe great vessels. They are rare and occur most often in infants. Report An Error 2i25/2014 10:30:50 AM Marke this question & => Question Td : 97281 Question 22 of 30 A.27 year old woman had a dry cough that had been worsening fer five months. She presented because of an acute bout of coughing associated with hemoptysis, CR and follow-up CT scan of the lungs revealed a large anterior mediasiinurn mass, Which of the following is NOT associated with enterior mediastinum mass? a) Thymoma. ) Retrosternal thyroid tumor. ©) Germ cell mmor. Y © 4) Neurogenic tamor. «) Teratoma. Question Explanation: Neurogenic tumors ate the most common primary mediastinal neoplasm. However, they present inost commonly as posterior mediastinal masses and majority are berign. Ten percent of primary mediastinal neoplasms are thymomas. They are usually present anteriorly and 25% are usually found to be malignant, Retrostemal thyroid tumors can present as an anterior, superior, and middle mediastinal mass. Germn cell tumors, bronchogenic cysts, aortic aneurysms, and pericardial cysts can al present as anterior mediastinal masses. Teratomas arise from the pericardium and often attach to the base ofthe great vessels. They are rare and occur most often in infants. Report An Error ‘Mark this question & => Question Td : 100754 Question 23 of 30 A 33 year old woman found unconscious in an apartment fire is taken to the hospital and placed on an artificial respirator. She died after 3 weeks in the hospital due to an infection secondary to her burns. Lung tissue examination at autopsy shows hyaline deposits within the alveolar walls, a smal neutrophilic infltrate, but no hemosicerin deposits. What is the most likely mechanisma of injury? a) Asthma ) Bacterial pneumonia ©) Desquatrative interstitial ©) Diffuse alveolar damage ©) Hemodynamic pulmonary edema Question Explanation: Diffise alveolar damage, clinically referred to as adult respiratory distress syndrome (ARDS), is characterized by diffise damage to the alveolarfcapillary rier, which may result from diverse acut= conditions. The four most frequent canses are trauma, sepsis shack, and gastric aspiration. The pathogenesis is not entirely clear butinfux of neutrophils and release of cytokines, cicosancids, and fice radivals seem to be crucial in promoting alveolar damage. The most characteristic histopathologic hallmark of diffuse alveolar damage is formation of hyaline membranes within the alveclar cavities These consist of proteinaceous material of plasma origin and necrotic debris from desquamated epithelium. Tae condition kas a 60% mertaity and manifests with acute respiratory failure, The pathologic features of esthma are relatively nonspecific and are similar, for example, to those of chronic bronchitis, including chronic inflammatory infiltration, hyperplasia of mucous glands, and hypertrophy of smooth muscle. The presence of numerous eosinophils, however, is more characteristic of asthma, Bacterial pneumonia is characterized by intra-alveolar exudation of neutrophils, brn, and erythrocytes, Bacteria are also present within the alveolar cavities. Desquatnative interstitial pneumoritis (DIP) is a fortn of interstitial disease referred to as idiopathic pulmonary fibrosis, In contrast to usual interstitial pneumonitis (another form of idiopathic pulmonary fitrosis), DIP is more responsive to steroid treatment. Histopathologicaly. DIP leads to hyperplasia of pneumocytes and accumulation of histiocytes that till the alveclar cavities. Hemodynamic pulmonary edema is caused by increased hydrostatic pressure, occurs in acute left ventricular failure. Itis dus to escape of fluid from the intravascular compastinent into the alveoli, Report An Error ‘Mark this question & => Question Td : 100754 Question 23 of 30 A 33 year old woman found unconscious in an apartment fire is taken to the hospital and placed on an artificial respirator. She died after 3 weeks in the hospital due to an infection secondary to her burns. Lung tissue examination at autopsy shows hyaline deposits within the alveolar walls, a smal neutrophilic infltrate, but no hemosicerin deposits. What is the most likely mechanisma of injury? a) Asthma +) Bactesial preumenia ©) Desquatrative interstitial Y © @ Diffuse alveolar damage ©) Hemodynamic pulmonary edema Question Explanation: Diffise alveolar damage, clinically referred to as adult respiratory distress syndrome (ARDS), is characterized by diffise damage to the alveolarfcapillary rier, which may result from diverse acut= conditions. The four most frequent canses are trauma, sepsis shack, and gastric aspiration. The pathogenesis is not entirely clear butinfux of neutrophils and release of cytokines, cicosancids, and fice radivals seem to be crucial in promoting alveolar damage. The most characteristic histopathologic hallmark of diffuse alveolar damage is formation of hyaline membranes within the alveclar cavities These consist of proteinaceous material of plasma origin and necrotic debris from desquamated epithelium. Tae condition kas a 60% mertaity and manifests with acute respiratory failure, The pathologic features of esthma are relatively nonspecific and are similar, for example, to those of chronic bronchitis, including chronic inflammatory infiltration, hyperplasia of mucous glands, and hypertrophy of smooth muscle. The presence of numerous eosinophils, however, is more characteristic of asthma, Bacterial pneumonia is characterized by intra-alveolar exudation of neutrophils, brn, and erythrocytes, Bacteria are also present within the alveolar cavities. Desquatnative interstitial pneumoritis (DIP) is a fortn of interstitial disease referred to as idiopathic pulmonary fibrosis, In contrast to usual interstitial pneumonitis (another form of idiopathic pulmonary fitrosis), DIP is more responsive to steroid treatment. Histopathologicaly. DIP leads to hyperplasia of pneumocytes and accumulation of histiocytes that till the alveclar cavities. Hemodynamic pulmonary edema is caused by increased hydrostatic pressure, occurs in acute left ventricular failure. Itis dus to escape of fluid from the intravascular compastinent into the alveoli, Report An Error “Mark this question & => Question 24 of 30 Laboratory test that should be obiained specifically for a paticnt with a suspected chylous pleural effusion is a) Serum triglycerides ) Pleural uid triglycerides ©) Serum amylase 4) Pleural fic amylase ©) Adenosine deaminase levels Question Explanation: ‘A pleural fluid triglyceride level of > 110 mg/ill is consistent with a chylous pleural effusion, which occurs with eupture of the thoracic uct from malignancy (usually lymphoma), surgery, or trauma. The other choices are not helpful “Mark this question & => Question 24 of 30 Laboratory test that should be obtained specifically for a paticnt with a suspected chylous pleural effusion is 2) Serum triglycerides V © ») Pleural uid triglycerides ©) Serum armylase 4) Pleural fluid araylace ©) Adenosine deaminase levels Question Explanation: ‘A pleural fluid triglyceride level of > 110 mg/ill is consistent with a chylous pleural effusion, which occurs with eupture of the thoracic uct from malignancy (usually lymphoma), surgery, or trauma. The other choices are not helpful 2/25/2014 10:31:32 AM Mark this question & => Question Td : 112508 Question 25 of 30 “What is the most likely etiology of unilateral hilar adenopathy? 2) Sarcoidosis, +) Tuberculosis ©) Malignancy. 6) Fungal infection. ©) Histiocytosis. Anower [UEQPISNBER) otter User Explanation Report An Error Question Explanation: "Sarcoidosis is the number one cause of bilateral hilar adenopathy. Malignancy is the number ons cause of unilateral hilar adenopathy. “All of the causes can presen either bilaterally or unilaterally. 2/25/2014 10:31:32 AM Mark this question & => Question Td : 112508 Question 25 of 30 “What is the most likely etiology of unilateral hilar adenopathy? 8) Sarcoidosis +) Tuberculosis Y © ©) Malignancy. 6) Fungal infection. 6) Histiocytosis Anower [UEQPISNBER) otter User Explanation Report An Error Question Explanation: "Sarcoidosis is the number one cause of bilateral hilar adenopathy. Malignancy is the number ons cause of unilateral hilar adenopathy. “All of the causes can presen either bilaterally or unilaterally. Marke this question e& => Question Td : 118350 Question 26 of 30 “Medications that CANNOT be administered via endotracheal tube inchide which one of the fallowing? 2) Naloxone +) Calcium chloride ) Epinephrine 4) Atvopine €) Lidocaine Answer | Explanation Other User's Explanation Report An Error Question Explanation: Emergency medications that can be administered via the endotracheal tube include "NAVEL'-naloxone, atropine, valium (diazepam), epinephrine, and lidocaine. This can be extremely important if intravenous access is not available. Calcium chloride, used in the treatment of li threatening hyperkalemia, is ouly effective intravencusly. Marke this question e& => Question Td : 118350 Question 26 of 30 “Medications that CANNOT be administered via endotracheal tube inchide which one of the fallowing? 2) Naloxone Y © ») Calcium chlonde ) Epinephrine 4) Atvopine €) Lidocaine Answer | Explanation Other User's Explanation Report An Error Question Explanation: Emergency medications that can be administered via the endotracheal tube include "NAVEL'-naloxone, atropine, valium (diazepam), epinephrine, and lidocaine. This can be extremely important if intravenous access is not available. Calcium chloride, used in the treatment of li threatening hyperkalemia, is ouly effective intravencusly. ‘Mark this question >> Question Td : 136735 Question 27 of 30 ‘The most common bacterial pneumonia in HIV/AIDS is which one of the following? a) Pneumococcal b) Staphylococcal ©) Mycobacteriam tuberculosis, d) Mycobacterium avium intracellulare €) Pneumocystis carinii Answor [BBIFIRNRIAY) other Users Explanation Repost An Enos Question Explanation: Pneumococcal pneumonia is the most common form of bacterial pneumonia in patients with HIVIAIDS, as tis in the general population in this country. Staphylococcal pneumonia is not particularly common. While tuberculosis and pneumonia dae to infection with Mycobacterium avum intracellulare are more commen in patients with HIV/AIDS than in healthy individuals, they are not more common than pneumococcal pneumenia in these patients. Pneumocystis cariniiis a plasmodium, nota bacterium, and is not more common than pneumococcal pneumonia ‘Mark this question >> Question Td : 136735 Question 27 of 30 ‘The most common bacterial pneumonia in HIV/AIDS is which one of the following? Y © a)Pheumococcal b) Staphylococcal ©) Mycobacteriam tuberculosis, d) Mycobacterium avium intracellulare €) Pneumocystis carinii Answor [BBIFIRNRIAY) other Users Explanation Repost An Enos Question Explanation: Pneumococcal pneumonia is the most common form of bacterial pneumonia in patients with HIVIAIDS, as tis in the general population in this country. Staphylococcal pneumonia is not particularly common. While tuberculosis and pneumonia dae to infection with Mycobacterium avum intracellulare are more commen in patients with HIV/AIDS than in healthy individuals, they are not more common than pneumococcal pneumenia in these patients. Pneumocystis cariniiis a plasmodium, nota bacterium, and is not more common than pneumococcal pneumonia Mark this question e& => Question Td : 138704 Question 28 of 30 A. 26 year cld HIV positive non-smoker person presents with massive hemoptysis, fever, and punilent sputum. CXR reveals a large ‘ight upper lobe cavitary lesion. The CT of the chest reveals a mass within the cavity. Bronchoscopy was performed and the culture grew branching hyphae. The moct licely diagnocie is a) Squamous cell carcinoma +) Tuberculosis ©) Bronchiectasis 2) Aspergillus fimigatas «) Lung abscess Question Explanation: ‘These “fungal bails" are found most often in patients wah underlying lung disease and iamunosuppression. Hemoptysis results from mechanical trauma sesulting from the presence ofthe ballin the cavity. Squamous cell carcinoma ofthe luagis usually centrally located This cancer is associated with smoking, Active tuberculosis pneumonitis can cause bronchiolar wiceration and necrosis of adjacent vessels and distant alveoli leading to hemoptysis. Eroachiectasis is umaly caused by a mycobacterium, not a fungus; therefore, hyphae would not grow in cultured tissue spurum samples. However, patients with prolonged neutropenia ot with bronchiectasis induced by toxic exposures are predisposed to infection with Aspergillus organisms. Samples taken from these patients would grow hyphae in culture These patients would also exhibit hemeptysis. Chest X-ray findings are nonspecific: in many of these patients, they appear normal. The xecrotizing effects cf the primary infection on the hung parenchyma and vasculehuxe can lead to hemoptysis. Predisposing factors for the development of lung abscesses include alcchol use, seizures, and poor oral hygiene. Mark this question e& => Question Td : 138704 Question 28 of 30 A. 26 year cld HIV positive non-smoker person presents with massive hemoptysis, fever, and punilent sputum. CXR reveals a large ‘ight upper lobe cavitary lesion. The CT of the chest reveals a mass within the cavity. Bronchoscopy was performed and the culture grew branching hyphae. The moct licely diagnocie is a) Squamous cell carcinoma +) Tuberculosis ©) Bronchiectasis Y © 4) Acpergilne famaigatas «) Lung abscess Question Explanation: ‘These “fungal bails" are found most often in patients wah underlying lung disease and iamunosuppression. Hemoptysis results from mechanical trauma sesulting from the presence ofthe ballin the cavity. Squamous cell carcinoma ofthe luagis usually centrally located This cancer is associated with smoking, Active tuberculosis pneumonitis can cause bronchiolar wiceration and necrosis of adjacent vessels and distant alveoli leading to hemoptysis. Eroachiectasis is umaly caused by a mycobacterium, not a fungus; therefore, hyphae would not grow in cultured tissue spurum samples. However, patients with prolonged neutropenia ot with bronchiectasis induced by toxic exposures are predisposed to infection with Aspergillus organisms. Samples taken from these patients would grow hyphae in culture These patients would also exhibit hemeptysis. Chest X-ray findings are nonspecific: in many of these patients, they appear normal. The xecrotizing effects cf the primary infection on the hung parenchyma and vasculehuxe can lead to hemoptysis. Predisposing factors for the development of lung abscesses include alcchol use, seizures, and poor oral hygiene. 2252014 :29 AM ‘Mark this question & => Question Id ; 149716 Question 29 of 30 An 18 year old stadent presents complainng of weakness, malaise and a chronic cough. He has a fever of 1000F (380C) and a dry! cough. Sputum cannot be obtained for analysis. so a bronchial lavage is peeformed for analysis. According to the lab report the organicm is 'slow- growing," Sero diagnocis reveals Strep MG agglutirin in the patient's serum. What is the most lely cause of his iiiness? a) Klebsiella pneumoniae 'b) Mycoplasma pneumoniae ©) Parainduenza virus 6) Respiratory syncytial virus 6) Streptococens pneumoniae Question Explanation: ‘The patient has primary atyrical pneumonia caused ty Mycoplasma pneumonia, This organism is fastidious and difficult to culbare in the laboratory however serodiagnosis can be most helpful. Itis an exiracelular prokaryote with no cell wall, but with sterols in the membrane. Patients typically produce one or two heterophile antibodies during the course of the infection; one agelutinates human (O+RBCs in the cold (he cold hemagalutnin) while the other causes the agglutination of a strain of Streptococcus salivarius tesmned strain MG (the Strep MG agglutinins). The treatment is macrolides (.e.. erythromycin azithromycin, or clarithromycin). Klebsiella pneumoniae is readily cultured on routine laboratory media and characteristically produces pneumonia with blood clots in the sputum (red currant jelly sputum), which may be indicative of pulmonary abscess development, Parainfluenza viruses cause croup, which is characterized by a cry, "barking" cough. Ibis more of a tracheitis, bronchitis, and bronchitis its than a pneumonitis. No heterophile antibodies are produced in these patients. Respiratory syncytial virus causes an atypical pneumonitis in infants, Itis usually diagnosed bby the observation of syncytial masses in respiratory secretions. Cold hemagelutinins and Strep MG agglutinins are absert. Streptococcus pneumoniae is the number one cause of pneumonia in adults. It also causes septicemia and meningitis in the elderly. ‘The patient has a classical acute pneumonia with a productive cough, high fever with chills, leukocytosis, tachycardia, rapid respiretions, and other signs of serious respiratory disease. A vaccine. composed of the capsular carbohydrate of 23 serotypes of this organism, is routinely given to individuals over the age of 60, as well as to individuals with splenic abnormalities (e.g, sickle-cell disease) who are at increased tisk for pneumococcal sepsis, Report An Error 2252014 :29 AM ‘Mark this question & => Question Id ; 149716 Question 29 of 30 An 18 year old stadent presents complainng of weakness, malaise and a chronic cough. He has a fever of 1000F (380C) and a dry! cough. Sputum cannot be obtained for analysis, so a bronchial lavage is performed for analysis. According to the lab report the organicm is 'slow- growing," Sero diagnocis reveals Strep MG agglutirin in the patient's serum. What is the most lely cause of his iiiness? a) Klebsiella pneumoniae ¥ © &) Mycoplasma pneumoniae ©) Parainfuenza virus 6) Respiratory syncytial virus ©) Streptococcus pneumoniae Question Explanation: ‘The patient has primary atyrical pneumonia caused ty Mycoplasma pneumonia, This organism is fastidious and difficult to culbare in the laboratory however serodiagnosis can be most helpful. Itis an exiracelular prokaryote with no cell wall, but with sterols in the membrane. Patients typically produce one or two heterophile antibodies during the course of the infection; one agelutinates human (O+RBCs in the cold (he cold hemagalutnin) while the other causes the agglutination of a strain of Streptococcus salivarius tesmned strain MG (the Strep MG agglutinins). The treatment is macrolides (.e.. erythromycin azithromycin, or clarithromycin). Klebsiella pneumoniae is readily cultured on routine laboratory media and characteristically produces pneumonia with blood clots in the sputum (red currant jelly sputum), which may be indicative of pulmonary abscess development, Parainfluenza viruses cause croup, which is characterized by a cry, "barking" cough. Ibis more of a tracheitis, bronchitis, and bronchitis its than a pneumonitis. No heterophile antibodies are produced in these patients. Respiratory syncytial virus causes an atypical pneumonitis in infants, Itis usually diagnosed bby the observation of syncytial masses in respiratory secretions. Cold hemagelutinins and Strep MG agglutinins are absert. Streptococcus pneumoniae is the number one cause of pneumonia in adults. It also causes septicemia and meningitis in the elderly. ‘The patient has a classical acute pneumonia with a productive cough, high fever with chills, leukocytosis, tachycardia, rapid respiretions, and other signs of serious respiratory disease. A vaccine. composed of the capsular carbohydrate of 23 serotypes of this organism, is routinely given to individuals over the age of 60, as well as to individuals with splenic abnormalities (e.g, sickle-cell disease) who are at increased tisk for pneumococcal sepsis, Report An Error ‘Mark this question = Question Td : 213625 Question 30 of 30 A.69-year-old women hac productive cough with yellow-green sputum, rzld bilateral chest pain, and fever for lact 3 days. Now che has shortness of breath, Temperature =39.8 °C (103.6 °F), BP= 128/66 mmHg, pulse= 120/nin, respirations =32/mnin, and sPO2= 94% on oxygen 2 Linin by way of nasal cannula, Auscultaticn reveals decreased breath sounds and crackles bilaterally. CBC shows leukocytosis CXR is as follows © tnere Mest Enon ‘Which of the folowing is the most aporopriate management for this patient? ) Azithromycin ) Azithromycin and ceftriaxone ©) Cefepime and ciprofloxacin 4) Doxycycline ©) Iinipenern Question Explanation: Ths patient presents with signs and symptoms of bilateral community-acquired pneumonia, Streptococcus pneumoniae is responsible for community-acquired pneumonia in 20 to 60% of cases Because this patient is older than 65 years and has an underlying cardiac disease, inpatient management is required. The criteria for admitting a patient who has commanity-acquired rneumoria are (CURB- 65 cntesia) + Confusion © Urea >7 mmoVL o 20 mg/dL, © Respiratory rate > 30min Blood pressure <90/60 mmHg + 65 years ofage or older Other criteria that can be considered are unstable vital signs, high fever, presence of comorbid disease (alcoholism, chronic obstructive pulmonary disease, malnutrition), and immunosuppression. The recoramended therapeutic regimen in this group of hospitalized patients who have com wnity-acquired pneumoniae by the American Thoracic Socisty inchudes intravenous beta-lactam antibiotics, second- or third- generation cephalosporn (e.g. cefbtastime, ceftriaxone), plus anIV or oral (PO) macrolide antibiotic or doxyeyeline (¢.g., azithromycin) or an IV fucroqunnolone alone (gatifloxacin) or a beta-lactam/beta- lactamase combination plus a macrolide or doxyeyalire. Adithromycin , clavittromycin, or doxycycline can be used for treatment of commnunity-acquited pneumonia on an ouppatient basis only. This patient, however, requires admission for intravenous antibiotic therapy. Aathroraycin, clarithromycin, or doxycycline is not sufficient for the management of this patient Cefepime and ciprofloxacin are is a recommended regimen for the treatment ofhospital-acquired pneumonia and sommusity- acquired pneumonia in immunocompromised in patients who have confirmed severe pneumonia and who require admission to intensive care and have risk of Pseudomonas aeruginosa infection. Re ce a eae Gp ce Fae as clad ea Ge RIS GE ‘Mark this question = Question Td : 213625 Question 30 of 30 A.69-year-old women hac productive cough with yellow-green sputum, rzld bilateral chest pain, and fever for lact 3 days. Now che has shortness of breath, Temperature =39.8 °C (103.6 °F), BP= 128/66 mmHg, pulse= 120/nin, respirations =32/mnin, and sPO2= 94% on oxygen 2 Linin by way of nasal cannula, Auscultaticn reveals decreased breath sounds and crackles bilaterally. CBC shows leukocytosis CXR is as follows © tnere Mest Enon ‘Which of the folowing is the most aporopriate management for this patient? ) Azithromycin Y © b) Asithromycin and ceftriazone ©) Cefepime and ciprofloxacin 4) Doxycycline ©) Iinipenern Question Explanation: Ths patient presents with signs and symptoms of bilateral community-acquired pneumonia, Streptococcus pneumoniae is responsible for community-acquired pneumonia in 20 to 60% of cases Because this patient is older than 65 years and has an underlying cardiac disease, inpatient management is required. The criteria for admitting a patient who has commanity-acquired rneumoria are (CURB- 65 cntesia) + Confusion © Urea >7 mmoVL o 20 mg/dL, © Respiratory rate > 30min Blood pressure <90/60 mmHg + 65 years ofage or older Other criteria that can be considered are unstable vital signs, high fever, presence of comorbid disease (alcoholism, chronic obstructive pulmonary disease, malnutrition), and immunosuppression. The recoramended therapeutic regimen in this group of hospitalized patients who have com wnity-acquired pneumoniae by the American Thoracic Socisty inchudes intravenous beta-lactam antibiotics, second- or third- generation cephalosporn (e.g. cefbtastime, ceftriaxone), plus anIV or oral (PO) macrolide antibiotic or doxyeyeline (¢.g., azithromycin) or an IV fucroqunnolone alone (gatifloxacin) or a beta-lactam/beta- lactamase combination plus a macrolide or doxyeyalire. Adithromycin , clavittromycin, or doxycycline can be used for treatment of commnunity-acquited pneumonia on an ouppatient basis only. This patient, however, requires admission for intravenous antibiotic therapy. Aathroraycin, clarithromycin, or doxycycline is not sufficient for the management of this patient Cefepime and ciprofloxacin are is a recommended regimen for the treatment ofhospital-acquired pneumonia and sommusity- acquired pneumonia in immunocompromised in patients who have confirmed severe pneumonia and who require admission to intensive care and have risk of Pseudomonas aeruginosa infection. Re ce a eae Gp ce Fae as clad ea Ge RIS GE Mark this question => Question Td : 19423 Question 1 of 30 A. 70-year-old Male with 2 year old history of MI has dyspnoea. MI was complicated by ventricular arrhythmias. His oxygen saturations were 85% on air and a chest X-ray revealed bilateral patchy infitration of both ling fields with a cardiothoracic ratio of 20/30 cm. Which of the following drugs is the culprit? a) Amiodarone ) Sotalol ©) Aspirin 4) Proceinamide €) Lidocaine Question Explanation: This patient has desaturation with patchy infiltration on CXR, suggesting a diagnosis of amiodarone-induced tung disease, Usually the presentation is insidious and the disorder associated with the cumulative dose. Treatment depends on withdrawing amiodarone and initiation of steroid therapy. Differential diagnosis is any lymphangitis/paeumontitis but high resolution CT can help by demonstration ofradio-dense plaques etc. Procamnamide causes increased arrhythmias, hypotension and hnpus-like syndrome. Lidocaine causes increased archythmias and CNS excitation andis either IV or IM so no long term use Sotalol will cause dose-related torsade de pointes and cardkac depression. ‘The scenario is unlikely to pick Aspirin, Mark this question => Question Td : 19423 Question 1 of 30 A. 70-year-old Male with 2 year old history of MI has dyspnoea. MI was complicated by ventricular arrhythmias. His oxygen saturations were 85% on air and a chest X-ray revealed bilateral patchy infitration of both ling fields with a cardiothoracic ratio of 20/30 cm. ‘Which of the following drugs is the culprit? Y © a) Amiodarone ) Sotalol ©) Aspirin 4) Proceinamide €) Lidocaine Question Explanation: This patient has desaturation with patchy infiltration on CXR, suggesting a diagnosis of amiodarone-induced tung disease, Usually the presentation is insidious and the disorder associated with the cumulative dose. Treatment depends on withdrawing amiodarone and initiation of steroid therapy. Differential diagnosis is any lymphangitis/paeumontitis but high resolution CT can help by demonstration ofradio-dense plaques etc. Procamnamide causes increased arrhythmias, hypotension and hnpus-like syndrome. Lidocaine causes increased archythmias and CNS excitation andis either IV or IM so no long term use Sotalol will cause dose-related torsade de pointes and cardkac depression. ‘The scenario is unlikely to pick Aspirin, 2252014 :30 AM Mark this question & => Question Id : 20327 Question 2 of 30 A.44 yer old man sustained a fracture of his left tibia and fibula 4 months ago. He hes been bed ridden in sehabsitation for past 3 months. Three hours aga he suddenly developed chest pain and skorness of breath. On examination he describes an acking ciscomfort over the right superior anterior chest. Ta questioning the patient further, an important point in the history would be the relationship of the pain to which of the following? 2) Change of postion b) Deep breathing ©) Eating 4) Swallowing ©) Walling newer [UEIVIRIRAN) omer users Explanation Report An Error Question Explanation: ‘Three months of immobilization is like demanding a pulmonary embolus. Itis surprising that it took so long, and that he is not yet moderately ambulatory. J this case, the patient has already flipped an embolns to his ling, and that area of the lung and the overlying pleura is infarcted, Hence, the pain and discomfort will be related to deep breathing, and not to change in postion. Pain with eating and swallowing. ‘odynophagia is more commonly encountered with lesions of the esophagus. Pain with walking, depending on the site, can be claudication in the lower extreraities, or ischemia of the myocardiun and chest pain, 2252014 :30 AM Mark this question & => Question Id : 20327 Question 2 of 30 A.44 yer old man sustained a fracture of his left tibia and fibula 4 months ago. He hes been bed ridden in sehabsitation for past 3 months. Three hours aga he suddenly developed chest pain and skorness of breath. On examination he describes an acking ciscomfort over the right superior anterior chest. Ta questioning the patient further, an important point in the history would be the relationship of the pain to which of the following? 2) Change of postion Y © b) Deep breathing ©) Eating 4) Swallowing ©) Walling newer [UEIVIRIRAN) omer users Explanation Report An Error Question Explanation: ‘Three months of immobilization is like demanding a pulmonary embolus. Itis surprising that it took so long, and that he is not yet moderately ambulatory. J this case, the patient has already flipped an embolns to his ling, and that area of the lung and the overlying pleura is infarcted, Hence, the pain and discomfort will be related to deep breathing, and not to change in postion. Pain with eating and swallowing. ‘odynophagia is more commonly encountered with lesions of the esophagus. Pain with walking, depending on the site, can be claudication in the lower extreraities, or ischemia of the myocardiun and chest pain, Mark this question e-= Question 3 of 30 Question Td : 29503 A lifelong non-smoker is diagnosed with emphysema, Which of the following would be the most licey etiological agent? 2) Isosyanates b) Steel ) Zins Exposure 4) Cadmium ¢) Asbestos Exposure newer [UEIRERIBNY Other Users Explanation Report An Exes Question Explanation: Cadmium fume inhalation is a recognized cause of emphysema, Other industrial associations with COPD include coal, cotton grain and cement, Mark this question e-= Question 3 of 30 Question Td : 29503 A lifelong non-smoker is diagnosed with emphysema, Which of the following would be the most licey etiological agent? 2) Isosyanates b) Steel ) Zins Exposure Y¥ © @ Cadmium ¢) Asbestos Exposure newer [UEIRERIBNY Other Users Explanation Report An Exes Question Explanation: Cadmium fume inhalation is a recognized cause of emphysema, Other industrial associations with COPD include coal, cotton grain and cement, 2/25/2014 10:34:55 AM “Mark this question => Question Id : 29653 Question 4 of 30 ‘Which ofthe followingis comrect regarding long-acting beta-2 agonists? 2) Are beneficial in acute viral croup b) Tolerance does not develop ©) Protect against allergen challenge for up to 48 hours 6) Prevent activity-induced symptoms along anti-inflammetory therapy 6) Should not be used in association with erythromycin Question Explanation: Long-acting beta -2 agonists, for example salmeterol, can be used twice daily to assist in prophylaxis in chronic asthma es step 3 of the Bekich Thoracic Society Acthna Guidelizes There is no evidence that the bronchodilator effects wanes with time, though there is debate that itmay become less effective in protecting against exercise or methacholine induced bronchospasm. Its duration of action is around 12 hours, and goes completely by 36 hours. Aminophylline interacts with erythromycin, giving an increased risle of toxicity There is no evidence that sakneterol works in vial croup, though oral steraids are highly effective. Report An Error 2/25/2014 10:34:55 AM “Mark this question => Question Id : 29653 Question 4 of 30 ‘Vihich ofthe following is correct regarding long-acting beta-2 agonists? 2) Are beneficial in acute vieal croup VW © b) Tolerance does not develop 6) Protect against allergen challenge for up to 43 hours 6) Prevent activity-induced symptoms along anti-inflammatory therapy 6) Should not be used in assaciation with erythromycin Question Explanation: Long-acting beta -2 agonists, for example salmeterol, can be used twice daily to assist in prophylaxis in chronic asthma es step 3 of the Bekich Thoracic Society Acthna Guidelizes There is no evidence that the bronchodilator effects wanes with time, though there is debate that itmay become less effective in protecting against exercise or methacholine induced bronchospasm. Its duration of action is around 12 hours, and goes completely by 36 hours. Aminophylline interacts with erythromycin, giving an increased risle of toxicity There is no evidence that sakneterol works in vial croup, though oral steraids are highly effective. Report An Error ‘Mark this question & => Question Ti : 29854 Question 5 of 30 Ahhotel worker of 44 years of age, who has been human immunodeficiency winus (HIV) positive for eight years, presents with complaint of progressive shortmess of breath on exercise. Chest X-ray reveals normal lung Selds with prominent pulmonary arteries Pulse oximeiry demonstrates that ie desaturates on exercise. What is the licely diagnosis? a) Anemia ) Intracardiac shunt across an abil septal defect c) Primary pulmoaary hypertension 6 Pulmonary emboic disease ¢) Pneumocystis carinii pneumonia (PCP) Question Explanation: ‘The history of dyspnea and exercise induced dyspnea in an HIV postive male with no signs on the chest X-ray andlittle other data would suggest PCP until proved otherwise. Pulmonary hypertension is another possiblity and occurs in HIV patiants, but the most likely choice would be PCP. ‘Mark this question & => Question Ti : 29854 Question 5 of 30 Ahhotel worker of 44 years of age, who has been human immunodeficiency winus (HIV) positive for eight years, presents with complaint of progressive shortmess of breath on exercise. Chest X-ray reveals normal lung Selds with prominent pulmonary arteries Pulse oximeiry demonstrates that ie desaturates on exercise. What is the licely diagnosis? a) Anemia ) Intracardiac shunt across an abil septal defect c) Primary pulmoaary hypertension 6 Pulmonary emboic disease Y © #) Pneumocystis carinii pneumonia PCP) Question Explanation: ‘The history of dyspnea and exercise induced dyspnea in an HIV postive male with no signs on the chest X-ray andlittle other data would suggest PCP until proved otherwise. Pulmonary hypertension is another possiblity and occurs in HIV patiants, but the most likely choice would be PCP. ‘Mark this question = >> Question Ta : 29986 Question 6 of 30 An asthmatic female of age 25 years is acmitted with acute severe asthma. Which one of the following is correct? a) Agitation should be managed with a benzodiazepine ) Inhaled salmeterol is indicated as first ine therapy ©) A high inspired cxygen concentration should be used routinely 4) Normal arterial pCO is reassuring €) Pulsus paradouus is a reliable sign of severity Question Explanation: ‘A nomnal or raised arterial pCO is an indication of severe asthma, Fulsus peradomsis not reliable and is not part of the criteria in assessing svetity of asthma attack. Salmeterol is used in management of chronic asthma (Step 3). High dose oxygen (40-60% should be ucedin severe asthma attack together with steroids and nsbuliced bronchodilators. Sedation must be avoided ac it can cauce respiratory failure and arrest. ‘Mark this question = >> Question Ta : 29986 Question 6 of 30 An asthmatic female of age 25 years is acmitted with acute severe asthma. Which one of the following is correct? a) Agitation should be managed with a benzodiazepine ) Inhaled salmeterol is indicated as first ine therapy Y © ©) A high inspired oxygen concentration should be used routinely &) Normal arterial pCO. is reassuring €) Pulsus paradouus is a reliable sign of severity Question Explanation: ‘A nomnal or raised arterial pCO is an indication of severe asthma, Fulsus peradomsis not reliable and is not part of the criteria in assessing svetity of asthma attack. Salmeterol is used in management of chronic asthma (Step 3). High dose oxygen (40-60% should be ucedin severe asthma attack together with steroids and nsbuliced bronchodilators. Sedation must be avoided ac it can cauce respiratory failure and arrest. 2/25/2014 10:35:31 AM “Mark this question => Question 7 of 30 ‘The correct statement regarding Bullous emphysema is? 2) The carbon monoxide transfer factor is reduced b) Helum dilution is more accurate than body plethysmography in measuring residual volume. ©) Hypoxaemia at rest will improve with exercise 6) Pulmonary compliance is reduced ©) Reduced elastic recoil opposes airway collapse in expiration Question Explanation: ‘Whole body piethysmography also measures trapped gas, that is intrathoracic gas Gnchding within bullae and other poorly ventilated areas) which barely commurictes with the airway. Standard gas-diution measures gas that communicates with the sirway. Mixing in helium dilution is more dificult in airways obstruction requiring mubi-breath methods lasting ive minutes rather than single breath test. Exertion will exacerbate breathlessness and hypoxia. The characteristic changes of severe emphysema are increase in static compliance and reduction in img recoil pressure, Loss of hing recoil causes a reduction of alveolar pressure (elastic recoil pressure of Jung + pleural pressure) leading to cellapas of peripheral airways on expiration, Emphysematcus patients purse their lips in expiration to increase airway pressure to prevent this collapse. CO transfer factor is reduced, Report An Error 2/25/2014 10:35:31 AM “Mark this question => Question 7 of 30 “The correct statement regarding Bullous emphysema is? Y © a) The carbon monexide transfer factor is reduced +b) Helum dilution is more accurate than body plethysmography in measuring residual volume ©) Hypoxaemia at rect will improve with exercise 6) Pulmonary compliance is reduced ©) Reduced elastic recoil opposes airway collapse in expiration Question Explanation: ‘Whole body piethysmography also measures trapped gas, that is intrathoracic gas Gnchding within bullae and other poorly ventilated areas) which barely commurictes with the airway. Standard gas-diution measures gas that communicates with the sirway. Mixing in helium dilution is more dificult in airways obstruction requiring mubi-breath methods lasting ive minutes rather than single breath test. Exertion will exacerbate breathlessness and hypoxia. The characteristic changes of severe emphysema are increase in static compliance and reduction in img recoil pressure, Loss of hing recoil causes a reduction of alveolar pressure (elastic recoil pressure of Jung + pleural pressure) leading to cellapas of peripheral airways on expiration, Emphysematcus patients purse their lips in expiration to increase airway pressure to prevent this collapse. CO transfer factor is reduced, Report An Error ‘Mark this question & => ‘Question Td : 42747 Question 8 of 30 Al of the following are typical features of asbestosis, except a) Increased risk of cancer +) Obstructive pattem on pulmonary function tests c) Plewral thickening and calcification 4) Interstitial fibrosis Answer | Berienaton | Other User's Explanation Report An Error Question Explanation: Asbestosis is a chronic and restrictive lung disease. Itis caused by inhalation of asbestos particles over prolonged periods of time. “Asbestosis takes approximately 15 to 20 years to showiits effects. The inhaled asbestos fibers iritclly damage the cell membranes in the Ings, causing the hung tissues to harden and forma scars, Thic is known as Gbrosis During the initial stages of disease only the outer lung tissues get scarred, but as the disease progresses, it affects the entire lungs. The hardening oflung tissues makes breathing difficult, thus redacing the oxygen intake. Pressure on the lungs increases because ofits inability to remave carbon dioxide and replace # with fresh oxygen This is the reasonit is known as a restrictive hing disease Persons suffering from asbestosis are more prone to develop bronchitis and pneumonia, Symptoms of Asbestosis may include one or ‘more of the following: shortness of breath, difficulty in breathing, constant cry cough, constant pain in the chest, pulmonary hypertension, and excess phlegm. Mesothelioma ie a cancer ofthe pleura (hing lining). Past documented records show that mesothelioma is cauced mainly by the inhalation of the crocidolite and amosite variety of asbestos. ‘Mark this question & => ‘Question Td : 42747 Question 8 of 30 All ofthe following are typical features of asbestosis, except a) Increased risle of cancer Y © b) Obstructive pattem on pulmonary function tests c) Pleural thickening and calcification 4) Interstitial ttrosis Answer | Berienaton | Other User's Explanation Report An Error Question Explanation: Asbestosis is a chronic and restrictive lung disease. Itis caused by inhalation of asbestos particles over prolonged periods of time. “Asbestosis takes approximately 15 to 20 years to showiits effects. The inhaled asbestos fibers iritclly damage the cell membranes in the Ings, causing the hung tissues to harden and forma scars, Thic is known as Gbrosis During the initial stages of disease only the outer lung tissues get scarred, but as the disease progresses, it affects the entire lungs. The hardening oflung tissues makes breathing difficult, thus redacing the oxygen intake. Pressure on the lungs increases because ofits inability to remave carbon dioxide and replace # with fresh oxygen This is the reasonit is known as a restrictive hing disease Persons suffering from asbestosis are more prone to develop bronchitis and pneumonia, Symptoms of Asbestosis may include one or ‘more of the following: shortness of breath, difficulty in breathing, constant cry cough, constant pain in the chest, pulmonary hypertension, and excess phlegm. Mesothelioma ie a cancer ofthe pleura (hing lining). Past documented records show that mesothelioma is cauced mainly by the inhalation of the crocidolite and amosite variety of asbestos. “Mark this question => Question Id : 46637 Question 9 of 30 A 39 year old epileptic manis brought unconscious in the middle of the night. He has a shallow respiration at arate >40/minute andis cyanotic. His pulse is 130/minute and bis blood pressure is £0/50 mmHg What should you do first? a) Calla close relative and inquire about any new drug intake ') Install a nasogastric tube c) Administer 35-40 % FiO2 by nasal canula 4) Order EKG and chest x-ray and reassess e) Perform endotracheal intubation Anewor [UEWIRIRARY) omer teers Explanation Report An Error Question Explanatio: ABC is a mnemonic for memorizing estentiel steps in dealing with an unconscious or uaresponsive patient. TE stands for Airway, Breathing and Circulation Tithe patient's airway is blocked, oxygen cannot reach the lungs and so caniot be transported around the body in the blood. Ensuring a clear eirway is the fist step in treating any patient. Common problems with the airway involve blockage by the tongue or orn Initial opening of the airway is often achieved by a "head tit-chin lit" or jaw thrust technique, although further maneuvers such as intubation may be necessary. The patients next assessed for breathing, Common finding: during an assessment of breathing may include normal breathing, noisy breathing, gasping or coughing. The rescuer proceeds to act on these based on hishher training Generally at this poiat t will become clear whether or not the casualy aeeds supportive care (such as the recovery position) or rescue breathing. Once oxygen can be delivered to the lungs by a clear airway and efficient breathing, there needs to be a circulation to doliverit to the rest of the body. This can be ascesced in a murcber of ways, inchuding a pulse check, ECG analysis, or capillary refill time, “Mark this question => Question Id : 46637 Question 9 of 30 A.39 year old epileptic man is brought unconscious in the midele of the night. He has a shallow respiration at arate >40/mimute and is cyanotic. His pulse is 130/minute and bis blood pressure is £0/50 mmHg What should you do first? a} Call a clese relative and ingpire about any new drug intake ') Install a nasogastric tube ©) Administer 35-40 % FiO2 by nasal camula 4) Order EKG and chest x-ray and reassess W © 6) Perform endotracheal intubation Anewor [UEHRIEARY) omer teers Explanation Report An Er Question Explanatio: ABC is a mnemonic for memorizing estentiel steps in dealing with an unconscious or uaresponsive patient. TE stands for Airway, Breathing and Circulation Tithe patient's airway is blocked, oxygen cannot reach the lungs and so caniot be transported around the body in the blood. Ensuring a clear eirway is the fist step in treating any patient. Common problems with the airway involve blockage by the tongue or orn Initial opening of the airway is often achieved by a "head tit-chin lit" or jaw thrust technique, although further maneuvers such as intubation may be necessary. The patients next assessed for breathing, Common finding: during an assessment of breathing may include normal breathing, noisy breathing, gasping or coughing. The rescuer proceeds to act on these based on hishher training Generally at this poiat t will become clear whether or not the casualy aeeds supportive care (such as the recovery position) or rescue breathing. Once oxygen can be delivered to the lungs by a clear airway and efficient breathing, there needs to be a circulation to doliverit to the rest of the body. This can be ascesced in a murcber of ways, inchuding a pulse check, ECG analysis, or capillary refill time, Marke this question = => Question Td : 49794 Question 10 of 30 AAD year old man presents to the clinic complaining of a chronic cough over the past 4 months, which has now been accompanied bby hemoptysis. His hangs have diffise bilateral rales. Laboratory findings reveal a sodium of 142 mEdyL, a potassinm of 4 3 mEqidL, aBUN of 39 mg/dL, anda creatinine of 2.9 mg/dL. Urinalysis reveals microscopic hemamuna and 4+ proteinuria. The serologic blood tests thar would most help confitm the suspected diamosis is a) Anti-glomenilar hasement membrane anthodies b) Anti-mitochondrial antibodies ©) Anticnentrophilic antibodies 4) Anti-parietal cell antibodies ©) Anti-smooth rmscle antibodies Question Explanation: ‘The combination of hematuria and hemoptysis should always raise the possibilty of Geodpasture syndrome. Anti-glomenslar bacement membrane antibodies are pathognomonic for this diagnosis Anti-titochondrial antibodies are found in patients with primary biliary citthosis. The anti-neutrophilic cytoplasmic antibodies are found i patents with Wegener granulomatosis. Wegener granulomatosis may also present with pulmonary and renel involvement but will have associated upper respiratory tract findings e.g.. sinastis and sinus abscesses. “Anti-parietal cell antibodies are found in patients with the autoimmune disease known as pemicious anemia, Anti-smocth muscle antibodies are found in patients with autoimmune hepatitis. Marke this question = => Question Td : 49794 Question 10 of 30 A.40 year old man presents to the clinic complaining of a chronic cough over the past 4 months, which has now been accompanied ‘by hemoptysis. His hnngs have diffuse bilateral rales. Laboratory fincings reveal a sodium of 142 mEa/L., a potassium of 4 3 mEqidL, aBUN of 39 mg/dL, and a creatinine of 2.9 mg/dL. Usinalysis reveals microscopic hematuria and 4+ proteinuria The serologic blood tests that would most help confitm the suspected diagnosis is J © 2) Anti-glomenilar basement membrane antbodies +b) Anti-mitochondrial antibodies ©) Antisneutrophilc antibodies 4) Anti-parietal cell antibodies ¢) Anti-smooth muscle antibodies Question Explanation: ‘The combination of hematuria and hemoptysis should always raise the possibilty of Geodpasture syndrome. Anti-glomenslar bacement membrane antibodies are pathognomonic for this diagnosis Anti-titochondrial antibodies are found in patients with primary biliary citthosis. The anti-neutrophilic cytoplasmic antibodies are found i patents with Wegener granulomatosis. Wegener granulomatosis may also present with pulmonary and renel involvement but will have associated upper respiratory tract findings e.g.. sinastis and sinus abscesses. “Anti-parietal cell antibodies are found in patients with the autoimmune disease known as pemicious anemia, Anti-smocth muscle antibodies are found in patients with autoimmune hepatitis. ‘Mark this question —& => Question Td : 55252 Question 11 of 30 ‘A patient presents with puimonery edema and has a blood pressure of 180/95. The most appropriate intial treatments a) Furosemide 6) Metoprolol c) Morphine 4) Enalapril Question Explanation: ‘Sometimes increased pressure in the blood vessels in the lungs forces fluid into the air sacs, preventing them from absorbing oxygen, a.condtion called pulmonary edema, The treatment of pulmonary edema is as follows: Preload secucers: Diuretics, such as furosemide (Lasix) axe used to treat pulmonary edema. These medications dilate the veins in the lungs and elsewhere in the body, which decreases fuid pressure going into the heart and hnngs. Morphine, a narcotic, is a mainstay in treating cardiac pulmonary edema, may be used to relieve shortness of breath and associated anxiety “Afterload reducers: These drags dilate the peripheral vessels and take a pressure load offthe left ventricle. Scme examples of afterload reducers medications include nitroprusside and enalapri. n Report An Error ‘Mark this question —& => Question Td : 55252 Question 11 of 30 A patient presents with pulmonary edema and has a blood pressure of 180/95, The most appropriate intial treatment is Y © a) Furosemide b) Metoprolol ©) Morphine 4) Enalapeil Question Explanation: ‘Sometimes increased pressure in the blood vessels in the lungs forces fluid into the air sacs, preventing them from absorbing oxygen, a.condtion called pulmonary edema, The treatment of pulmonary edema is as follows: Preload secucers: Diuretics, such as furosemide (Lasix) axe used to treat pulmonary edema. These medications dilate the veins in the lungs and elsewhere in the body, which decreases fuid pressure going into the heart and hnngs. Morphine, a narcotic, is a mainstay in treating cardiac pulmonary edema, may be used to relieve shortness of breath and associated anxiety “Afterload reducers: These drags dilate the peripheral vessels and take a pressure load offthe left ventricle. Scme examples of afterload reducers medications include nitroprusside and enalapri. n Report An Error “Mask this question €& => Question Id : 59731 Question 12 of 30 A.59 year old man presents with polycythemia, He is found to have low oxygen saturation, increased red blood cells mass, notmnal plasina volume and increased erythropoletin levels. He is most licely suffering from? a) Polycythemia rubra vera +b) Chronic cbstructive pulmonary disease (COPD) Stress polycythemia Renal adenocarcinoma 9 ad ©) Congenital spherocytosis Avowor [UBINSHER) otnerUsors Explanation Report An Error Question Explanation: Chronic obsiructive pulmonery disease (COPD) is persistent obstruction of the airways occurring with emphysema, chronic bronchitis, or both disorders Tn the eater stages of COPD, oxygen levels in the blood may be decreased, but carbon dioxide levels remain normal. In the later stages, carbon dioxide levels increase and oxygen levels fall The decrease in oxygen levels in the blood stimulates the bone marrow to send more red blood cells into the bloodstream, a condition known az secondary polycythemia. The decrease in oxygen levels in the blood also increases the pressure in the artery through which blood flows from the heart to the lungs (pulmonary artery) “Mask this question €& => Question Id : 59731 Question 12 of 30 A.59 year old man presents with polycythemia, He is found to have low oxygen saturation, increased red blood cells mass, notmnal plasina volume and increased erythropoletin levels. He is most licely suffering from? a) Polycythemia rubra vera Y © b) Chronic bstructive pulmonary disease (COPD) Stress polycythemia Renal adenocarcinoma 9 ad ©) Congenital spherocytosis Avowor [UBINSHER) otnerUsors Explanation Report An Error Question Explanation: Chronic obsiructive pulmonery disease (COPD) is persistent obstruction of the airways occurring with emphysema, chronic bronchitis, or both disorders Tn the eater stages of COPD, oxygen levels in the blood may be decreased, but carbon dioxide levels remain normal. In the later stages, carbon dioxide levels increase and oxygen levels fall The decrease in oxygen levels in the blood stimulates the bone marrow to send more red blood cells into the bloodstream, a condition known az secondary polycythemia. The decrease in oxygen levels in the blood also increases the pressure in the artery through which blood flows from the heart to the lungs (pulmonary artery) 2252014 1 AM “Mark this question => Question 13 of 30 A70 year old smoker presents with trancal, atm, and facial swelling for 1 week. Physical examination is remarkable for facial erythema and facial, trincal, and arm edema with prominence of thoracic and neck veins. CXR reveals amass in the ght ‘mediastinam with adenopathy. The most ikely diagnosis is 8) Adenocarcinoma 6) Hodgkin lymphoma ©) Large cell carcinoma ) Non-small carcinoma ©) Small cell carcinoma Question Explanation: Superiorvena cava (SVC) syndrome is characterized by obstruction of venous retum from the head neck, and upper extremities Over 85% of cases of SYC syndrome are related to malignancy. Bronchogenic carcinomas (most commonly small cel cancer and squamous cell cancer) account for over 80% of these cases. Among bronchogenic carcinomas, the most common causes of SVC symdrome (in order of frequency) are small cel carcinoma, epidermoid carcinoma, adenocarcinoma (choice A) and large cell carcinoma (choice C). Lymphomas such as Hodgkin disease (choice B) and non-Hodgkin lymphoma ate uncommon causes of SVC syndrome. Rare tumors associeted with SVC syndrome include primary leiomyosarcomas and plasmacytomas. Infectious etiologies include tuberculosis, syphilis, and Histoplasmosis SVC syndrome can also occur as a result of an enlarged goiter, and from thrombus formation caused by in-duvelling intravenous lixes or pacemaker wires, A Non- small cell carcinoma (cheice D) is not commonly associated with SVC syndrome, 2252014 1 AM “Mark this question => Question 13 of 30 A70 year old smoker presents with trancal, atm, and facial swelling for 1 week. Physical examination is remarkable for facial erythema and facial, trincal, and arm edema with prominence of thoracic and neck veins. CXR reveals amass in the ght ‘mediastinam with adenopathy. The most ikely diagnosis is 8) Adenocarcinoma 6) Hodgkin lymphoma ©) Large cell carcinoma ) Non-small carcinoma © & Small cell carcinoma Question Explanation: Superiorvena cava (SVC) syndrome is characterized by obstruction of venous retum from the head neck, and upper extremities Over 85% of cases of SYC syndrome are related to malignancy. Bronchogenic carcinomas (most commonly small cel cancer and squamous cell cancer) account for over 80% of these cases. Among bronchogenic carcinomas, the most common causes of SVC symdrome (in order of frequency) are small cel carcinoma, epidermoid carcinoma, adenocarcinoma (choice A) and large cell carcinoma (choice C). Lymphomas such as Hodgkin disease (choice B) and non-Hodgkin lymphoma ate uncommon causes of SVC syndrome. Rare tumors associeted with SVC syndrome include primary leiomyosarcomas and plasmacytomas. Infectious etiologies include tuberculosis, syphilis, and Histoplasmosis SVC syndrome can also occur as a result of an enlarged goiter, and from thrombus formation caused by in-duvelling intravenous lixes or pacemaker wires, A Non- small cell carcinoma (cheice D) is not commonly associated with SVC syndrome, Mark this question = => Question Id : 87709 Question 14 of 30 ATA year old female with along history of cigarette smoking is found to have a smell tumor at the periphery of her sight upper lobe Thitally the tarmor was believed to be a stage [carcinoma (T1 NO MO), but after surgery it is found to be stage TE (T1N'1 M0) The finding that changed the staging of the tumor is 2) Involvement of the chest well b) Positive bronchial lymph nodes ©) small cell histology Tumor to the carina ) Tumor size greater than 3 em Question Explanation: Although itis nearly impossible to memorize the staging rules of the different kinds of tumors, there are several basic principles common to all The TNM stage of a tumor is determined by the tumor size and extent (T for tumor) lymph node involvemert, (NT for nodes), and metastasis (M for metastasis), NO always means no lymph nodes are involved. NI lesion has postive nodes, and only choioe B, positive bronchial lymph nodes, changes the petient’s nodal statue. Involvement of the chest well is afeature of tumor size and extent-thus itis a component of the Tin TNM staging T2 lesiors involve the chest wall. Histological fearures of the tumor such as small cell morphology are not considered in tumor stage but rather in tumor grade. Small cell tumors are considered high-grade carcinomas ‘Tumor extending to the carina reflects the tumor size and extert-thus, iis a component of the T in TNM staging, T3 lesions involve the carina, Tumor size is a consideration in the Tin TM staging, Tumors greater than 3 cm are at least T2 lesions. Mark this question = => Question Id : 87709 Question 14 of 30 ATA year old female with along history of cigarette smoking is found to have a smell tumor at the periphery of her sight upper lobe Thitally the tarmor was believed to be a stage [carcinoma (T1 NO MO), but after surgery it is found to be stage TE (T1N'1 M0) The finding that chengod the staging of the tumor is 2) Involvement of the chest well Y © &) Positive bronchial lymph nodes ©) small cell histology ) Tumor te the eatin ) Tumor size greater than 3 em Question Explanation: Although itis nearly impossible to memorize the staging rules of the different kinds of tumors, there are several basic principles common to all The TNM stage of a tumor is determined by the tumor size and extent (T for tumor) lymph node involvemert, (NT for nodes), and metastasis (M for metastasis), NO always means no lymph nodes are involved. NI lesion has postive nodes, and only choioe B, positive bronchial lymph nodes, changes the petient’s nodal statue. Involvement of the chest well is afeature of tumor size and extent-thus itis a component of the Tin TNM staging T2 lesiors involve the chest wall. Histological fearures of the tumor such as small cell morphology are not considered in tumor stage but rather in tumor grade. Small cell tumors are considered high-grade carcinomas ‘Tumor extending to the carina reflects the tumor size and extert-thus, iis a component of the T in TNM staging, T3 lesions involve the carina, Tumor size is a consideration in the Tin TM staging, Tumors greater than 3 cm are at least T2 lesions. ‘Mark this question & => Question Td : 89324 Question 15 of 30 A 55 year old morbidy obese man is brought in to the physician's office by his wife because of excess and loud snoring at night. The wife complains that she cannot sleep at night and sometimes she feels as if her husband is "gasping for air" The man only complains of excess daylime sleepiness. Whatis the most likely diagnosis in this patient? a) Narcolepsy ») Obstructive sleep apnea ©) Central sleep apnea & Sinusitis ®) Voral cord paralysis Question Explanation: Obstmctive sleep apneais common in obese people who have loud, excess snoring at night and daytime sleepiness Tis thought thet there is excess fatty tissue around the pharyngeal area, causing blockage of air at night. The patient becomes intermittently hypoxic turing the night. A polysomnography test is needed to make the official diagnosis. Narcclepsy is a sleep disorder where the patient suddenly drops to sleep in the middle ofthe day, night, or anytime, with no prior warning Central sleep apneais rate and it does not hhave to occurin obese patients. The mechanism of apnea is in the CNIS. The patient would complan of more than daytime sleepiness in sinusitis, He would also have symptoms of nasal congestion, fever, and sinus tenderness. In vocal cord paralysis, the patient would be unable to talk in his usual voice. He would not have difficulty sleeping ‘Mark this question & => Question Td : 89324 Question 15 of 30 A 55 year old morbidy obese man is brought in to the physician's office by his wife because of excess and loud snoring at night. The wife complains that che cannot sleep at night and cometimes che feele ac ifher husband ic "gasping for air" The man only complains of excess dayiime sleepiness. What is the most likely diagnosis in this patient? a) Narcolepsy Y © d) Obstmetive sleep apnea ©) Central sleep apnea a) Sinusitis 2) Vocal card paralysis Question Explanation: Obstmctive sleep apneais common in obese people who have loud, excess snoring at night and daytime sleepiness Tis thought thet there is excess fatty tissue around the pharyngeal area, causing blockage of air at night. The patient becomes intermittently hypoxic turing the night. A polysomnography test is needed to make the official diagnosis. Narcclepsy is a sleep disorder where the patient suddenly drops to sleep in the middle ofthe day, night, or anytime, with no prior warning Central sleep apneais rate and it does not hhave to occurin obese patients. The mechanism of apnea is in the CNIS. The patient would complan of more than daytime sleepiness in sinusitis, He would also have symptoms of nasal congestion, fever, and sinus tenderness. In vocal cord paralysis, the patient would be unable to talk in his usual voice. He would not have difficulty sleeping ‘Mark this question & => Question Td : 90874 Question 16 of 30 All of the following are sources of pulmonary embolus, EXCEPT. a) Lower extremity superficial vein thrombi b) Renal veins c) Neoplasms d) Deep veins of pelvis e) Right atrium Answer | Geplanation Other User's Explanation Report An Error Question Explanation: Puimonary emboli are exogenous materials that have impacted into branches of the pulmonary bed. Most pulmonary emboli are thromboemboli, which result from dislodgement of thrombi in the deep veins of the pelvis or thighs or from the deep veins of the calf, ‘Thrombi in the superficial veins of the lower extremities do not embolize. Renal vein thrombi may embolize, particularly in persons with nephrotic smdrome. Mural thrombi in the sight cardiac chambers may also embolize, especially ifindwelling catheters or wires are present, Other causes of pulmonary emboli include neoplasms, fat, air, and exogenous materials, such as tale contaminants of sntravenously injecied drugs. ‘Mark this question & => Question Td : 90874 Question 16 of 30 All ofthe following are sources of pulmonary embolus, EXCEPT Y © a) Lower extremity superficial vein thrombi b) Renal vaine c) Neoplasms ) Deep veins of pelvis e) Right atrium Answer | Geplanation Other User's Explanation Report An Error Question Explanation: Puimonary emboli are exogenous materials that have impacted into branches of the pulmonary bed. Most pulmonary emboli are thromboemboli, which result from dislodgement of thrombi in the deep veins of the pelvis or thighs or from the deep veins of the calf, ‘Thrombi in the superficial veins of the lower extremities do not embolize. Renal vein thrombi may embolize, particularly in persons with nephrotic smdrome. Mural thrombi in the sight cardiac chambers may also embolize, especially ifindwelling catheters or wires are present, Other causes of pulmonary emboli include neoplasms, fat, air, and exogenous materials, such as tale contaminants of sntravenously injecied drugs. ‘Mark this question e& => Question Td : 94484 Question 17 of 30 “What sign is consistent with an overdose of morphine? a) Tremulousness b) Nystagmus: ©) Mydriasis @) Hypertension, ©) Respiratory depression Question Explanation: Respiratory depression is the chicf hazard of all morphine preparations. Respiratory depression occurs more fiequently in elderly or debilitated patients and in those suffering from conditions accompanied by byposta, hypercapnia, or upper airway obstruction in whom even moderate therapeutic doses mey significantly decrease pulmonary ventilation. Acute overdosage with morphine is manifested by respiratory depression, somnolence progressing to stapor or coma, skeletal muscle flaccidity, cold and clammy skin and constricted pupils. ‘Mark this question e& => Question Td : 94484 Question 17 of 30 “What sign is consistent with an overdose of morphine? a) Tremulousness b) Nystagmus: ©) Mydriasis @) Hypertension, Y © €) Respiratory depression Question Explanation: Respiratory depression is the chicf hazard of all morphine preparations. Respiratory depression occurs more fiequently in elderly or debilitated patients and in those suffering from conditions accompanied by byposta, hypercapnia, or upper airway obstruction in whom even moderate therapeutic doses mey significantly decrease pulmonary ventilation. Acute overdosage with morphine is manifested by respiratory depression, somnolence progressing to stapor or coma, skeletal muscle flaccidity, cold and clammy skin and constricted pupils. 2/25/2014 10:38:09 AM ‘Mark this question & => Question Td : 94637 Question 18 of 30 A. 36 year old man wants to go to the mountains to ski, He has experienced moderate symptems of acute mountain sickness (AMS) in the past, incliding headache, nausea, shortness of breath, and sleep disturbance. Which one of the following could be used to ‘minimize the effects of AMS? a) Furosemide (Lasix) b) Erythromycin 6) Acetazolamide Diamox) D Beta-Blockers ©) Fluid restriction (Question Explanation: ‘Acute mountain sickness (AMS) is a clinical syndrome which may affect as many as 12%-57% of persons ascending to altindes of S000 feet or greater While a standard definition of AMS does not exist, persons having three or mare cf the following symptoms may be considered to have AMS: headache, nausea, vomiting, sleep disturbance, anorexia, fatigue, or dyspnea. Gradual ascent is recommended to allow acclimatization, Measures which may help minimize symptoms include avoidance of alcohol, increased flud intake, and a high carbohydrate diet, Fluid restriction and diuretics should be avoided because of the diuresis associated with, acchmatization, which may by iself cause dehydration, Antibiotics are ofno benefit. B-Blockers would be harmfil, by interfering with the physiologic responses of tachycardia and increased cardiac ouput zt higher altindes. Acetazolamide speeds the process of acclimatization The drugis a carbonic anhydrase mbibitor which resuits in a renal bicarbonate diuresis and metabole acidosis, thereby increasing ventilation and arterial oxygenation, The respiratory stimulation is particularly important during sleep, when reduces the severe hypoxemia caused by periodic breathing. The drug alse lowers cerebrospinal fluid volume and pressure, which may play an additional role in its therapeutic and prophylactic utility 2/25/2014 10:38:09 AM ‘Mark this question & => Question Td : 94637 Question 18 of 30 A. 36 year old man wants to go to the mountains to ski, He has experienced moderate symptems of acute mountain sickness (AMS) in the past, incliding headache, nausea, shortness of breath, and sleep disturbance. Which one of the following could be used to ‘minimize the effects of AMS? a) Furosemide (Lasix) b) Erythromycin Y © ©) Acetazolamide Diamex) D Beta-Blockers ©) Fluid restriction (Question Explanation: ‘Acute mountain sickness (AMS) is a clinical syndrome which may affect as many as 12%-57% of persons ascending to altindes of S000 feet or greater While a standard definition of AMS does not exist, persons having three or mare cf the following symptoms may be considered to have AMS: headache, nausea, vomiting, sleep disturbance, anorexia, fatigue, or dyspnea. Gradual ascent is recommended to allow acclimatization, Measures which may help minimize symptoms include avoidance of alcohol, increased flud intake, and a high carbohydrate diet, Fluid restriction and diuretics should be avoided because of the diuresis associated with, acchmatization, which may by iself cause dehydration, Antibiotics are ofno benefit. B-Blockers would be harmfil, by interfering with the physiologic responses of tachycardia and increased cardiac ouput zt higher altindes. Acetazolamide speeds the process of acclimatization The drugis a carbonic anhydrase mbibitor which resuits in a renal bicarbonate diuresis and metabole acidosis, thereby increasing ventilation and arterial oxygenation, The respiratory stimulation is particularly important during sleep, when reduces the severe hypoxemia caused by periodic breathing. The drug alse lowers cerebrospinal fluid volume and pressure, which may play an additional role in its therapeutic and prophylactic utility “Marke this question & => Question 19 of 30 A.44 year old man preseats with sinustis, fever, and malaise. Exemination demonstrates rales at the right lung base and pallor. Serum crestinine level of 5.0 is presert CXR shows nodular caxities bilaterally. The most licely dicenosis is 8) Goodpasmure’s syndrome +) Sarcoidosis ©) Wegener's granulomatosis 6) Tuberculosis ©) Folyarteritis nodosa Question Explanatic ‘Wegener's granviomatosis presents more commonly in men and almost always presents with en upper respiratory tract infection, Biopsy of the nodular hung tissue would reveal the necrotizing vasculitis. The ANCA test would be a good soreering test, but the gold standard is biopsy. Renal fature with rapidly progressive glomeruionephritis is common Goodpasture’s syndrome is also more common in men, Both renal and pulmonary problems are also sncountsred, but it would be unasual to have an upper respiratory tract infection as well. Anti glomerular basement antibody would be a useful test. Sarcoidosis can present this way, but renal dysfunction is rare and the ches: X-ray would usually reveal bilateral hilar adenopathy. Tuberculosis can also present this way, but renal dysfinction, is rare, and the chest X-ray would more commonly have upper lobe infltrates. Polyarteriis nodosa is a vasculiis which affects the medium to large arteries and commonly involves the kidneys. This disease would not intially involve the upper respiratory tract. “Marke this question & => Question 19 of 30 A.44 year old man preseats with sinustis, fever, and malaise. Exemination demonstrates rales at the right lung base and pallor. Serum crestinine level of 5.0 is presert CXR shows nodular caxities bilaterally. The most licely dicenosis is 8) Goodpasmure’s syndrome b) Sarcoidosis W © oc) Wegener's granulomatosis 6) Tuberculosis ©) Folyarteritis nodosa Question Explanatic ‘Wegener's granviomatosis presents more commonly in men and almost always presents with en upper respiratory tract infection, Biopsy of the nodular hung tissue would reveal the necrotizing vasculitis. The ANCA test would be a good soreering test, but the gold standard is biopsy. Renal fature with rapidly progressive glomeruionephritis is common Goodpasture’s syndrome is also more common in men, Both renal and pulmonary problems are also sncountsred, but it would be unasual to have an upper respiratory tract infection as well. Anti glomerular basement antibody would be a useful test. Sarcoidosis can present this way, but renal dysfunction is rare and the ches: X-ray would usually reveal bilateral hilar adenopathy. Tuberculosis can also present this way, but renal dysfinction, is rare, and the chest X-ray would more commonly have upper lobe infltrates. Polyarteriis nodosa is a vasculiis which affects the medium to large arteries and commonly involves the kidneys. This disease would not intially involve the upper respiratory tract. 2:25/2014 10: Mark this question & => Question Td : 100733 Question 20 of 30 40 AM ATI year old man has smoked for years despite a chronic productive cough He notices blood in his spnium one day. CXR. reveals a ‘mass in one lung, which is biopsisd. When assumed that this patient's canceris etiologically related to his smoking, the diagnosis that is likely to be made by leboratory would be a) Adenocarcinoma or Bronchicloalveolar carcinoma 6) Adenocarcinoma or stnall cell carcinoma c) Bronchioloalveolar carcinoma or smell cell carcinoma Bronchioloalveolar carcinoma ot Squamous cell carcinoma ©) Smell cell carcinoma or Squamous cell carcinoma Question Explanation: Smoking does not increase the risk of all types of lang cancers to the same degree, Oat (small) cell carcinoma has a very strong association with smoking, with only 1% of cases occurring in nonsmokers. Squamous cell carcinoma is also stronaly associated with smoking, because smoking predisposes for squamous metaplasia, a precancerous condition. The association of smoking with broachogenic adenocarcinoma and with bronchioloalveolar carcinoma is auch weaker. 2:25/2014 10: Mark this question & => Question Td : 100733 Question 20 of 30 40 AM ATH yeer eld man has smeked for year: despite a chronic productive cough He natices blood in his sputum one day. CXR reveals a siass in one lung, which is biopsied When assumed that this patient's cancer is etiologically related to his smoking, the diagnosis that is likely to be made by laboratory would be 2) Adenocarcinema or Bronchicloalveolar ceroinoma ) Adenocarcinoma or small cell carcinoma ¢) Bronchioloalveclar carcinoma or smell cell carcinoma Bronchioloalveolar carcinoma ot Squamous cell carcinoma Y © ¢) Smell cell carcinoma or Squamous cell carcinoma Question Explanation: Smoking does not increase the risk of all types of lang cancers to the same degree, Oat (small) cell carcinoma has a very strong association with smoking, with only 1% of cases occurring in nonsmokers. Squamous cell carcinoma is also stronaly associated with smoking, because smoking predisposes for squamous metaplasia, a precancerous condition. The association of smoking with broachogenic adenocarcinoma and with bronchioloalveolar carcinoma is auch weaker. 2:25/2014 10:38:53 AM Mark this question & => Question 21 of 30 Hypertrophic ostcoarthropathy is most commonly associated with disorders of which of the folowing? a) The central nervous system ') Calcium metabolism. ©) Osteoblasts 4) The kings ) The stomach, Question Explanation: Diseases of the lungs are the most common cause of hypertrophic osteoarthropathy Disorders of the ceniral nervous system, calcium metabolism, osteoblasts, and the stomach are not specifically associated with this condition. 2:25/2014 10:38:53 AM Mark this question & => Question 21 of 30 Hypertrophic ostzoarthropathy is most commonly associated with disorders of which of the Following? 2) The central nervous system. +) Calcium metab ols ©) Osteoblasts SY © 4d) The hangs. «) The stomach. Question Explanation: Diseases of the lungs are the most common cause of hypertrophic osteoarthropathy Disorders of the ceniral nervous system, calcium metabolism, osteoblasts, and the stomach are not specifically associated with this condition. 225/204 10: 206 AM ‘Mark this question & => Question Td : 104643 Question 22 of 30 ‘Which of the following patient is NOT expected to be at greater risk for aspiration pneumonia? a) A heallhy neonate with an Apgar of 9. b) A comatose 50 year old man. ©) 46 year old alcoholic san 4) A 23 year old woman with severe seizures. ©) A.63 year old woman with raubiple cerebral infarcts Question Explanation A healthy newbom is at little risk for the development of aspiration pneumonia, while the risk is increased in all of the other groups. 225/204 10: 206 AM ‘Mark this question & => Question Td : 104643 Question 22 of 30 ‘Which of the following patient is NOT expected to be at greater risk for aspiration pneumonia? Y © a) Ahealthy neonate with an Apgar of 9. b) A comatose 50 year old man. ©) 46 year old alcoholic san 4) A 23 year old woman with severe seizures ©) A.63 year old woman with raubiple cerebral infarcts Question Explanation A healthy newbom is at little risk for the development of aspiration pneumonia, while the risk is increased in all of the other groups. 2252014 1 ‘Merk this question & => Question 23 of 30 aM “What is the most appropriate treatment for small cell carcinoma ofthe lung? a) Hydroxyurea plus pilebotomy. £) Cisplatin plus etoposide. ©) Cisplatin plus morouracil ©) Hydroxyarea plus alpha interferon ©) Chlorembucil plus prednisone. Question Explanation: Small cell carcinoma is one of the most aggressive lung neoplasins and requires aggressive combination chemotherapy. The first ine treatment for small cell lung carcinoma is cisplatin plus etoposide. In cancers that are insensitive to this treatment, cyclophosamide, doxorubicin, and vincristine can be employed, None oftthe other choices are indicated for the treatment of small cell carcinoma of the hung, 2252014 1 ‘Merk this question & => Question 23 of 30 aM “What is the most appropriate treatment for small cell carcinoma of the lung? a) Hydroxyurea plus phlebotomy. ¥ © b) Cisplatin plus etoposide, c) Cisplatin plus nucrouracil. ©) Hydroxyurea pins aipha interferon e) Chlorambucil plus prednisone. Question Explanation: Small cell carcinoma is one of the most aggressive lung neoplasins and requires aggressive combination chemotherapy. The first ine treatment for small cell lung carcinoma is cisplatin plus etoposide. In cancers that are insensitive to this treatment, cyclophosamide, doxorubicin, and vincristine can be employed, None oftthe other choices are indicated for the treatment of small cell carcinoma of the hung, Mark this question & => Question Td : 136819 Question 24 of 30 ‘Which one of the following is NOT a non-cardiac cause of dyspnea? 2) Pneumothorax ) Pulmonary embolism. ©) Annicty. 6) Lumbar lordosis ©) Exertion, Question Explanation: ‘The breathing pattem is controlled by both central and peripheral mechanisms. Dyspnea occurs whenever the work of breathing is excessive, Metabolic demands increase daring fear and araety resulting in a sensation of dyspnea, During and following mild to moderate exertion, some perscns become aware of the act of breathing AAs the activity level ncreases to exhausting exercise, most individuals become unpleasontly aware of breathing. Conditions that cause hypoxia, such as pneumothorax and pulmonary emboli, stimulate the respiratory drive centers, producing dyspnea Severe chest wall diseases, such as kyphoscolosis, may produce dyspnea. n Report An Error Mark this question & => Question Td : 136819 Question 24 of 30 “Which one of the following is NOT a non-cardiac cause of dyspnea? 2) Fneumothorax ) Pulmonary embolism. ©) Amicty. Y © 6) Lumbar lordosis ©) Exertion. Question Explanation: ‘The breathing pattem is controlled by both central and peripheral mechanisms. Dyspnea occurs whenever the work of breathing is excessive, Metabolic demands increase daring fear and araety resulting in a sensation of dyspnea, During and following mild to moderate exertion, some perscns become aware of the act of breathing AAs the activity level ncreases to exhausting exercise, most individuals become unpleasontly aware of breathing. Conditions that cause hypoxia, such as pneumothorax and pulmonary emboli, stimulate the respiratory drive centers, producing dyspnea Severe chest wall diseases, such as kyphoscolosis, may produce dyspnea. n Report An Error Mark this question & => Question Td : 139175 Question 25 of 30 ‘A patient with long standing, progressive congestive heart failure develops worsening shortness of breath over 2 weeks’ petiod and dies in respiratory distress. At autopsy lungs are 3 times their normal weight. Histologically, the alveoli show a proteinaceous granular precipitate, engorged alveolar capillaries, and Hemosiderin laden macrophages. The following is the most likely diagnosis is ) Candida pneumonia +b) Pneumococcal pneumonia ©) Paeumocystis pneumonia 4) Pulmonary edema ¢) Pulmonary infarction Anower [FEIGIRIRHON) Other User's Explanat Question Explanation: ‘These are the characteristic findings of pulmonary edema The edema finid is apparent as a protemaceans gramular precipitate after histologic processing of the tissue. Hemosiderin laden macrophages in the tissue, called "heart failure calls" are the cad result of ingestion of red blood cells by alveolar macrophages. Pulmonary edema develops in her failure when the pulmonary venous pressure tises and the capacity for the tissue to reabsorb the fluid into the venous system is exceeded Th Candida pneumonia fingal hyphae and spores would be described. Tn pneumocystis pneamonia aram positive cocci (Gtreptococcus pneumoriae) would be described. In Pneumocystis pneumonia hat shaped, silver-siained cysts would be described. In pulmonary infarciioa, ischemic necrosis of alveoli would be described, Report An Error Mark this question & => Question Td : 139175 Question 25 of 30 ‘A patient with long standing, progressive congestive heart failure develops worsening shortness of breath over 2 weeks’ petiod and dies in respiratory distress. At autopsy lungs are 3 times their normal weight. Histologically, the alveoli show a proteinaceous granular precipitate, engorged alveolar capillaries, and Hemosiderin laden macrophages. The following is the most likely diagnosis is ) Candida pneumonia +b) Pneumococcal pneumonia ©) Paeumocystis pneumonia Y © @) Pubnonary edema ¢) Pulmonary infarction Anower [FEIGIRIRHON) Other User's Explanat Question Explanation: ‘These are the characteristic findings of pulmonary edema The edema finid is apparent as a protemaceans gramular precipitate after histologic processing of the tissue. Hemosiderin laden macrophages in the tissue, called "heart failure calls" are the cad result of ingestion of red blood cells by alveolar macrophages. Pulmonary edema develops in her failure when the pulmonary venous pressure tises and the capacity for the tissue to reabsorb the fluid into the venous system is exceeded Th Candida pneumonia fingal hyphae and spores would be described. Tn pneumocystis pneamonia aram positive cocci (Gtreptococcus pneumoriae) would be described. In Pneumocystis pneumonia hat shaped, silver-siained cysts would be described. In pulmonary infarciioa, ischemic necrosis of alveoli would be described, Report An Error Mark this question & => Question Td : 150315 Question 26 of 30 A.48 year old man presents with oliguria, elevated blood area nitrogen and creatinine and hematuria. Nasal congestion and epistaxis ace also present. Systemic review is notable for occasional cough and hemopiysis. On exam mucosal ulceration and nasal septal perforation is present, butne polyps are seen, What serum markers would likely be present in this case? 2) Antceatromere antibody b) Ant-Ro ©) Ant-SS-B Oc ANCA ©) Decreased erythrocyte sedimentation rate Question Explanation: ‘This patient has Wegener granulomatosis, which is characterized by renal involvement, severe upper respiratory wract symptoms, and pulmonary involvement. Other organ systems may also be involved. The renal syndrome is a crescentic, rapidly progressive omenulonephritis leacing to renal feitare. The upper respiratory tract findings include sinus pain and crainage, and punulent or bloody aasal discharge with or without nssal ulserations. Nasal septal perforation may follow. Pulmonary invelvement may be clinically slent ‘with only inflates present on X-ray, or itmay present as cough and hemoptysis. ANCA stands for antineutroptil cytoplasmic antibody, and c-AINCA refers to antibody that localizes staining to the cytoplasin of neutrophils. The most common target entigen is proteinase 3 PR 3) C-ANCA is a marker for Wegener gramloatosis, present in a high percentage of patents Anticentromare antibody is associated with eppronimately 90% of cases with CREST syndrome (calcinosis, Raynaud phenomenon, esophageal ‘motlity syndrome, clerodactyly, and telangiectasia), which is also called limited systemic sclerosis. Anti-Ro is also called ani-SS-A ands associated with Sjogren syndrome (70 to 95%). Anti-SS-B is associated with Sjogren syadrome (60 to 90%). Decreased ecythrocste sedimentation rate (ESR) is not a marker of Wegener. Instead, amarkedyy elevated ESR is seen, Additionally, mild anomia thrombocytosis, leukocytosis, mild hypergammeglobulinenia (gA), aad mildly elevated sheumatoid factor are seen in this disorder. Mark this question & => Question Td : 150315 Question 26 of 30 A.48 year old man presents with oliguria, elevated blood area nitrogen and creatinine and hematuria. Nasal congestion and epistaxis ate also presert. Systemic review is aotable for occasional cough and hemoptysis. On exam mucosal ulceration and nasal septal perforatica is precert, butme polyps are ceen, What serum markere would likely be present in thic cace? 2) Antceatromere antibody ) Anti-Ro ©) Ant-SS-B ¥ @ dc ANCA ©) Decreased erythrocyte sedimentation rate Question Explanation: ‘This patient has Wegener granulomatosis, which is characterized by renal involvement, severe upper respiratory wract symptoms, and pulmonary involvement. Other organ systems may also be involved. The renal syndrome is a crescentic, rapidly progressive omenulonephritis leacing to renal feitare. The upper respiratory tract findings include sinus pain and crainage, and punulent or bloody aasal discharge with or without nssal ulserations. Nasal septal perforation may follow. Pulmonary invelvement may be clinically slent ‘with only inflates present on X-ray, or itmay present as cough and hemoptysis. ANCA stands for antineutroptil cytoplasmic antibody, and c-AINCA refers to antibody that localizes staining to the cytoplasin of neutrophils. The most common target entigen is proteinase 3 PR 3) C-ANCA is a marker for Wegener gramloatosis, present in a high percentage of patents Anticentromare antibody is associated with eppronimately 90% of cases with CREST syndrome (calcinosis, Raynaud phenomenon, esophageal ‘motlity syndrome, clerodactyly, and telangiectasia), which is also called limited systemic sclerosis. Anti-Ro is also called ani-SS-A ands associated with Sjogren syndrome (70 to 95%). Anti-SS-B is associated with Sjogren syadrome (60 to 90%). Decreased ecythrocste sedimentation rate (ESR) is not a marker of Wegener. Instead, amarkedyy elevated ESR is seen, Additionally, mild anomia thrombocytosis, leukocytosis, mild hypergammeglobulinenia (gA), aad mildly elevated sheumatoid factor are seen in this disorder. 2i25/2014 10:40:10 AM ‘Mark this question & => Question Id : 161333 Question 27 of 30 A.55 year old man with emphysema has a blood pressure of 157/101 mim Hg. Over the next several months the doctor prescribes several different medications, but the patient hes to discontinue each agent because of undesirable side effects. He then decides to prescribe a beta antagonist. Which beta antagonists would be most appropnate for him? a) Metoprolol b) Nadolol ¢) Froprandlol od) Sotalol ) Timolel Answer (Bxpianaton) Other User's Explanation Report An Error Question Explanation: Patients with nonallergic bronchospastic conditions such as emphysema and chronic bronchitis, are generally not prescribed beta receptor blocking agents since these agents can cause bronchocenstriction by blocking beta-2 receptors. However, relatively lowr doses of selective beta 1 receptor antagonists, such as metoprolol and atsnolel, are relatively well tolerated in patients with emphyseina As a precautionary measure, emphysema patients receiving beta-L-selective blocking agents should use a bronchodilator with beta 2-stimulating activity. All the other answer choices are nonselective beta receptor blocking agents and should not be used in the treatment of hypertension in patients with nonallergic bronchospasm since they are likely to exacerbate the signs and symptoms of the condiion 2i25/2014 10:40:10 AM ‘Mark this question & => Question Id : 161333 Question 27 of 30 A.55 year old man with emphysema has a blood pressure of 157/101 mim Hg. Over the next several months the doctor prescribes several different medications, but the patient hes to discontinue each agent because of undesirable side effects. He then decides to prescribe a beta antagonist. Which beta antagonists would be most appropnate for him? Y © a) Metoprolol b) Nadolol ¢) Froprandlol od) Sotalol ) Timolel Answer (Bxpianaton) Other User's Explanation Report An Error Question Explanation: Patients with nonallergic bronchospastic conditions such as emphysema and chronic bronchitis, are generally not prescribed beta receptor blocking agents since these agents can cause bronchocenstriction by blocking beta-2 receptors. However, relatively lowr doses of selective beta 1 receptor antagonists, such as metoprolol and atsnolel, are relatively well tolerated in patients with emphyseina As a precautionary measure, emphysema patients receiving beta-L-selective blocking agents should use a bronchodilator with beta 2-stimulating activity. All the other answer choices are nonselective beta receptor blocking agents and should not be used in the treatment of hypertension in patients with nonallergic bronchospasm since they are likely to exacerbate the signs and symptoms of the condiion Mark this question & => Question Td : 17422: Question 28 of 30 ‘A 22 year old female attempts suicide by taking an overdose of barbiturates, Th the emergency department, her blood pressure is 95/65 mm Hg, and her pulse is 105tmin. The physician in the intensive care orders arterial blood gases, What value would you expect in her? a) PO, = 45, PCO, = 45, pH=7.45 6) PO, = 55, PCo, = 70, pH=7.50 6) POy=65,PCO7=35,pH=7.45 d) PO, =75, PCO, = 60, pH=7.30 6) PO, = 98, PCO, = 60, pH=7.20 Anower (UBIRIEN) omer veore Explanation Report AnEror Question Explanation: Barbiturate overdose causes respiratory depression, resulting in carbon dioxide retention (producing increased PCO, and decreased pH) and hypouemia (decreased 202), In other words, the patient has respiratory acidosis. You should look for low POs, high PCO, and acidic pH. Options A, B, C, and E do not full these requirements. Mote that choice C might be expected in a patient who is hyperventilating to the point of respiratory alkalosis: diminished O, (the usual chive for hyperventilation in nonpsychiatric hyperventilation), diminished CO, end mildly allcalotic pH. Mark this question & => Question Td : 17422: Question 28 of 30 ‘A 22 year old female attempts suicide by taking an overdose of barbiturates, Th the emergency department, her blood pressure is 95/65 mm Hg, and her pulse is 105tmin. The physician in the intensive care orders arterial blood gases, What value would you expect in her? a) PO, = 45, PCO, = 45, pH=7.45 6) PO, = 55, PCo, = 70, pH=7.50 6) POy=65,PCO7=35,pH=7.45 Y © dPO,=75, PCO, = 60, pH=7.30 6) PO, = 98, PCO, = 60, pH=7.20 Anower (UBIRIEN) omer veore Explanation Report AnEror Question Explanation: Barbiturate overdose causes respiratory depression, resulting in carbon dioxide retention (producing increased PCO, and decreased pH) and hypouemia (decreased 202), In other words, the patient has respiratory acidosis. You should look for low POs, high PCO, and acidic pH. Options A, B, C, and E do not full these requirements. Mote that choice C might be expected in a patient who is hyperventilating to the point of respiratory alkalosis: diminished O, (the usual chive for hyperventilation in nonpsychiatric hyperventilation), diminished CO, end mildly allcalotic pH. ‘Mark this question & => Question Id : 213603, Question 29 of 30 A 19-year-old man of normal built and height has hemoptysis for | day. He also had sputum streaked with blood along prolonged coughing He has had a slight fever, runny nose, and a productive cough for the last 3 days. Temperature = 37.8 °C (100 °F), BP is 118/76 meFlg, pulse is 90min, and respiratory rate is 16/min. Laboratory studies are normal. What is the next appropriate step? a) Check antinuclear cytoplasmic antibody 1) Check urinalysis ) Obtain a chest CT scan ® Obtain a chest X-ray 8) Schedule a bronchoscopy Question Explanation: This patient most licely has benign streaking-with blood in the setting of prolonged coughing, in as much as he has an upper respiratory infection. He is young and does not report risk factors for lung cancer. Findings on chest x-ray in conjunction with the patient history will give good clues to the diagnosis and further workup that may be indicated, his chest x-ray is normal, he can be observed until his infection resolves to see ifhis symptoms also resolve at that point ifthe streaking does not resolve, firther workup is indicated Antinuclear cytoplasmic antibodies can be checked to determine if a patient has Wegener granulomatosis. This patient does not provide syinptoms consistent with a vascultis, ‘There is litle reason to suspect a renal-pulmonary syndrome such as Goodpasture syndrome. F the patient's chest x-ray is normal, he is unlely to have an abnormal urinalysis. Bronchoscopy before a chest x-ray would not be indicated. Similarly, a chest CT scan before a chest x-ray is not appropriate. ‘Mark this question & => Question Id : 213603, Question 29 of 30 A 19-year-old man of normal built and height has hemoptysis for ] day. He also had sputum streaked with blood along prolonged coughing. He has hada slisht Fever, runny nose, and a productive cough fer the last 3 days. Temperature = 37.8 °C (100 °F), BP is 118/76 mmHg, pulse is 90/min, and respiratory rate is 16/min. Laboratory studies are normal. What is the next appropriate step? a) Check antinuclear cytoplasmic antibody +b) Check urinalysis +c) Obtain a chest CT scan Y © & Obtain a chest X-ray 2) Schedule a bronchoscopy Question Explanation: This patient most licely has benign streaking-with blood in the setting of prolonged coughing, in as much as he has an upper respiratory infection. He is young and does not report risk factors for lung cancer. Findings on chest x-ray in conjunction with the patient history will give good clues to the diagnosis and further workup that may be indicated, his chest x-ray is normal, he can be observed until his infection resolves to see ifhis symptoms also resolve at that point ifthe streaking does not resolve, firther workup is indicated Antinuclear cytoplasmic antibodies can be checked to determine if a patient has Wegener granulomatosis. This patient does not provide syinptoms consistent with a vascultis, ‘There is litle reason to suspect a renal-pulmonary syndrome such as Goodpasture syndrome. F the patient's chest x-ray is normal, he is unlely to have an abnormal urinalysis. Bronchoscopy before a chest x-ray would not be indicated. Similarly, a chest CT scan before a chest x-ray is not appropriate. Mark this question <= ‘Question Td : 213866 Question 30 of 30 A 67-year-old has productive cough with yellow-green sputum, mid bilateral chest pain, and fever along shortness of breath, ‘Temperature =39.3 °C (102.7°F), BP = 120/65 mmllg, pulse =12B8/nin, respirations= 30/min and SPO2 = 96% on oxygen 20 min by way of nasal canmula, Bilateral decreased breath sounds and crackles exist CXR shows © intertace Medical Educate “Which of the followings the most appropsiate action? a) Admit her to the floor and start agaressive therepy against congestive heart falure secondary to acute pericardtis ') Admit her to the floor and start intravenous cefiriazone and azithromycin, ©) Admit her to the intenswve care unt and start intravenous ceftriaxone and azithromycin 4) Outpatient treatment with oral moxifloxacin and re-evahiation in 24h ) Start intravenous ceftriaxone and azithromycin and observe her in the emergency departinent to assess the clinical response Answer | Explanation Other User's Explanation Report An Error Question Explanation: ‘The patient presents with severe the classic symptoms of pneumonia: fever, productive cough, and shortness of breath, Findings on physical examination, such as decreased breath sounds and bilateral crackles, are consistert with pneumonia. Also, a complete blood count shews leukocytosis, which indicates the presence of inflammation. A chest x-ray is usually the next step to confirm the suspected diagnosis. Tae chest x-ray shown is consistent with extensive, bilateral, parenchymal ling disease most consistent with pneumonia She meets the criteria for adnission to an intensive care unit These criteria include the presentation of septic shock requiring vasopressor support or acute respiratory distress syndrome requiring mechanical ventilation as major eriteria. Addtionally the need for ICU care is suagested by the presence of at least three minor criteria: respirator rate >30hmin, PaO2/FiO2 ratio 250, smuliloba:iefitrates, confusion, BUN >20 mg/dL, leulcopenia, thrombocytopenia, hypothermia, or hypotension requiring duid support Pencarditis can cause congestive heart failure with fever. ‘This answer choice is not the most probable, however, because the patient ‘s presentation has evidence of bilateral parenchymal hing disease, which is most consistent with oneumonia, not congestive heart failure, Her heart sounds are nernzal with no murnaurs or rubs, aad she does nothave peripheral edema. Chest radiograph in a patient ‘who has congestive heartfalure shows enlarged heart and evidence of pulmonary venous engorgement (the vessels are seen to extend further than normal into the lung field) Also, pleural effusion may be seen ‘This patienthas clear criteria for admission to an intensive care unit This severe pneumonia needs close monitoring to detect any ee ae Mark this question <= ‘Question Td : 213866 Question 30 of 30 A 67-year-old has productive cough with yellow-green sputum, mid bilateral chest pain, and fever along shortness of breath, ‘Temperature =39.3 °C (102.7°F), BP = 120/65 mmllg, pulse =12B8/nin, respirations= 30/min and SPO2 = 96% on oxygen 20 min by way of nasal canmula, Bilateral decreased breath sounds and crackles exist CXR shows © intertace Medical Educate “Vihich ofthe following is the most appropriate action? a) Admit her to the floor and start aggressive therepy against congestive heart falute secondary to acute pericardtis +b) Aduait her to the door and start intravenous cefiriazone and azithromycin Y © c) Admither to the intensive care unt and start intravenous ceftriaxone and azithromycin 6) Outpatient treatment with oral moxifloxacin and re-evaluation in 24 h ) Start intravenous ceftriaxone and azithromycin and observe her in the emergency departinent to assess the clinical response Answer | Explanation Other User's Explanation Report An Error Question Explanation: ‘The patient presents with severe the classic symptoms of pneumonia: fever, productive cough, and shortness of breath, Findings on physical examination, such as decreased breath sounds and bilateral crackles, are consistert with pneumonia. Also, a complete blood count shews leukocytosis, which indicates the presence of inflammation. A chest x-ray is usually the next step to confirm the suspected diagnosis. Tae chest x-ray shown is consistent with extensive, bilateral, parenchymal ling disease most consistent with pneumonia She meets the criteria for adnission to an intensive care unit These criteria include the presentation of septic shock requiring vasopressor support or acute respiratory distress syndrome requiring mechanical ventilation as major eriteria. Addtionally the need for ICU care is suagested by the presence of at least three minor criteria: respirator rate >30hmin, PaO2/FiO2 ratio 250, smuliloba:iefitrates, confusion, BUN >20 mg/dL, leulcopenia, thrombocytopenia, hypothermia, or hypotension requiring duid support Pencarditis can cause congestive heart failure with fever. ‘This answer choice is not the most probable, however, because the patient ‘s presentation has evidence of bilateral parenchymal hing disease, which is most consistent with oneumonia, not congestive heart failure, Her heart sounds are nernzal with no murnaurs or rubs, aad she does nothave peripheral edema. Chest radiograph in a patient ‘who has congestive heartfalure shows enlarged heart and evidence of pulmonary venous engorgement (the vessels are seen to extend further than normal into the lung field) Also, pleural effusion may be seen ‘This patienthas clear criteria for admission to an intensive care unit This severe pneumonia needs close monitoring to detect any ee ae 2/25/2014 1:42:02 PM ‘Mark this question => Question Td : 29044 Question 1 of 30 A 61-year-old man has cough, weight loss and tiredness. CAR shows chronis upper lobe Abrosis. Three spun test stain positive for Acid fast bacilli but are negative for Mycobacterium nuberculosis on culture, Which is the licely cause? @) Micropolyspora facai +) Allergic Bronchopumonery Asperaillosis ©) Mycobacterium avium intracellalare complex: ) Cryptococcus Neoformans ©) Coccidiodes Tinmitis Question Explanation: ‘The presence of AFB yet absence of TB suggest an atypical AFB such as M. aviun, 2/25/2014 1:42:02 PM ‘Mark this question => Question Td : 29044 Question 1 of 30 A 61-year-old man has cough, weight loss and tiredness. CXR shows chronic upper lobe dbrosis. Three spunam test stain positive for Acid fast bacili but are negative for Mycobacterium tuberculosis on culture, Which is the lixely cause? 2) Micropolyspora facai +) Allergic Bronchopumonery Asperaillosis ¥ © © Mycobacterium avium intracellalare complex ) Cryptococcus Neoformans ©) Coccidiodes Tnnitis Question Explanation: ‘The presence of AFB yet absence of TB suggest an atypical AFB such as M. aviun, ‘Mark this question —& => Question Td : 29473 Question 2 of 30 A 62-year-dld, heavy smoker, with a BMI of 37, has complains of impotence, nocturia and depression. He is hypoxic at rest on ait aad har aakle edema, Whichis the most appropriate investigation te determine the etiology? a) Liver Function Tests +) Sleep study c} Chest x-ray 4) Spizometry ¢} Thyroid fiction test Question Explanation: “The clinical scenasio describes cor pulmonale secondary to diurnal respiratory failare. This occurs in patients with severe obstructive sleep apnea (OSA). Most patients who develop this complication have lower airways obstruction (from smoking, gross obesity or respiratory muscle wealeness. Hypercapnia out of proportion to the degree of lng disease should suggest OSA as a possible agnosis, ‘Mark this question —& => Question Td : 29473 Question 2 of 30 A 62-year-dld, heavy smoker, with a BMI of 37, has complains of impotence, nocturia and depression. He is hypoxic at rest on ait aad har aakle edema, Whichis the most appropriate investigation te determine the etiology? a) Liver Function Tests SY © b) Sleep study c} Chest x-ray 4) Spizometry ¢} Thyroid fiction test Question Explanation: “The clinical scenasio describes cor pulmonale secondary to diurnal respiratory failare. This occurs in patients with severe obstructive sleep apnea (OSA). Most patients who develop this complication have lower airways obstruction (from smoking, gross obesity or respiratory muscle wealeness. Hypercapnia out of proportion to the degree of lng disease should suggest OSA as a possible agnosis, 272512014 1:12:40 PM “Mark this question e=> Question 3 of 30 Question Id : 29583 A.49-year-old has shortness of breath, cough with heavy sputum production, and a low grade fever; she has stoked 20 cigarettes per day for 30 years Her arterial blood gases revealed: ee [a4 [C36-7.44) [pCO [45 moma (35-45) [pO [75 mamttg (0-110) Diagnose: a) Emphysema 1b) Cryptogenic fbrosirg alveottis, Paraneoplastic syndrome o) }) Chronic bronchitis 9) ©) Pulmonary embotsm Avewor (UBIPIRNSHER) otnorUsors Explanation Report An Error Question Explanation: The most likely explanation based on the symptoms and the relative byposia with high pC'Qa is an acute exacerbation of COPD- ‘towards the chronic bronchitic end of the spectrum. Excessive sputum is unsupporiive of emphysema. 272512014 1:12:40 PM “Mark this question e=> Question 3 of 30 Question Id : 29583 A.49-year-old has shortness of breath, cough with heavy sputum production, and a low grade fever; she has stoked 20 cigarettes per day for 30 years Her arterial blood gases revealed: ee [a4 [C36-7.44) [pCO [45 moma (35-45) [pO [75 mamttg (0-110) Diagnose: v a) Emphysema 1b) Cryptogenic fbrosirg alveottis, Paraneoplastic syndrome o) }) Chronic bronchitis 9) ©) Pulmonary embotsm Avewor (UBIPIRNSHER) otnorUsors Explanation Report An Error Question Explanation: The most likely explanation based on the symptoms and the relative byposia with high pC'Qa is an acute exacerbation of COPD- ‘towards the chronic bronchitic end of the spectrum. Excessive sputum is unsupporiive of emphysema. “Mark this question & => Question 4 of 30 Folowing an accident a 25-year-old male was managed according to ATLS protocol before being brought to emergency department, On exam he is acutely short of breath and has temperature of 37.5°C. He seems to be confised on speaking and you also note petechial hemorrhages. The likely diagnosis is a) Fat embolism ») Asthma attack c) Chest infection 4) Pulmonary embolism ¢) Teasion pneumothorax Question Explanation: The two diagnoses which should be considered rst in this scenario are pulmonary erabolism and fat embolism Fat embolism is thought to occur as aresult of release of lipid globules from damage bone marrow fat cells Another mecharism that hes been suggest is the increased mobilization of fatty acids peripherally The effects that are seen clinically depend on what part of the misrovasculamre is affected by the linid alobules Pulmonary symptoms are caused by ventiletion perfusion saismatch. Confusion (cerabral effects) may be sean as well as a peteckial rash caused by capillary damage in the skin, “Mark this question & => Question 4 of 30 Folowing an accident a 25-year-old male was managed according to ATLS protocol before being brought to emergency department, On exam he is acutely short of breath and has temperature of 37.5°C. He seems to be confised on speaking and you also note petechial hemorrhages. The likely diagnosis is ¥ © a)Fat embolism ) Asthina attack: c) Chest infection ) Pulmonary embolism ¢) Teasion pneumothorax Question Explanation: The two diagnoses which should be considered rst in this scenario are pulmonary erabolism and fat embolism Fat embolism is thought to occur as aresult of release of lipid globules from damage bone marrow fat cells Another mecharism that hes been suggest is the increased mobilization of fatty acids peripherally The effects that are seen clinically depend on what part of the misrovasculamre is affected by the linid alobules Pulmonary symptoms are caused by ventiletion perfusion saismatch. Confusion (cerabral effects) may be sean as well as a peteckial rash caused by capillary damage in the skin, 225/204 1:43:17 PM ‘Mark this question & => Question Id : 29723 Question 5 of 30 A 55-year-old plumber presented with a dry noctumal cough and increasing exertional breathlessness. Oa examination he had early finger chibhing, cyanosis and bilateral baral crackles. A chest X-ray showed bilateral lower zone shadowing Investigations revealed: |fpa0z Coreathing air) 2.2 bPa [a13-12.6) |FEVYEVC ratio 25% “Which of the following investigations is most lkely to establish the diagnosis? 2) Echocardiography ) Measurement of difusion capacity ¢) High resolttion CT scan of chest 4) Serum angiotensin-converting enzyme (ACE) level «) Repeat Spiromeiry Question Explanation: This paticnt has a restrictive lung defect and hypoxia, with clinical features of hing fibrosis With the occupational history, there might have been previous asbestos exposure, although the CX is not reported to show pleural thickening or plaques. The next test should be one to confirma pulmonary fibrosis. High resolution CT chest is often diagnostic with good correlation to histological abnormalities. A ground-glass appearance is associated with predominantly cellular appearance on biopsy and more active disease, which responds to treatment and has a better prognosis. A reticular paltern is suggestive or destroyed Abrotie lings. 225/204 1:43:17 PM ‘Mark this question & => Question Id : 29723 Question 5 of 30 A 55-year-old plumber presented with a dry noctumal cough and increasing exertional breathlessness. Oa examination he had early finger chibhing, cyanosis and bilateral baral crackles. A chest X-ray showed bilateral lower zone shadowing Investigations revealed: |fpa0z Coreathing air) 2.2 bPa [a13-12.6) |FEVYEVC ratio 25% “Which of the following investigations is most lkely to establish the diagnosis? 2) Echocardiography ) Measurement of difusion capacity VW © 6) High resolution CT scan of chest & Serum angiotensin-converting enzyme (ACE) level «) Repeat Spiromeiry Question Explanation: This paticnt has a restrictive lung defect and hypoxia, with clinical features of hing fibrosis With the occupational history, there might have been previous asbestos exposure, although the CX is not reported to show pleural thickening or plaques. The next test should be one to confirma pulmonary fibrosis. High resolution CT chest is often diagnostic with good correlation to histological abnormalities. A ground-glass appearance is associated with predominantly cellular appearance on biopsy and more active disease, which responds to treatment and has a better prognosis. A reticular paltern is suggestive or destroyed Abrotie lings. 2252014 1:13:32 PM “Mark this question & => Question 6 of 30 A 65-year-old obese man presents with night time sweats, nocturia, poor concentration and day time somnolence. To which of the following conditions does this diagnosis predispose? 2) Hypoglycaemia b) Hypotension 6) Insulin sensitivity 4) Osteoporosis 6) Sudden death Question Explanation: This history is typical of sleep apnoea. Sleep apnoca is an independent risk factor for stroke (and death from ell causes) and is associated with hypertension, IGT, and insulin resistance. 2252014 1:13:32 PM “Mark this question & => Question 6 of 30 A 65-year-old obese man presents with night time sweats, nocturia, poor concentration and day time somnolence. To which of the following conditions does this diagnosis predispose? 2) Hypoglycaemia b) Hypotension ¢) sulin sensitivity 4) Osteoporosis Y © 28) Sudden death Question Explanation: This history is typical of sleep apnoea. Sleep apnoca is an independent risk factor for stroke (and death from ell causes) and is associated with hypertension, IGT, and insulin resistance. 2252014 1:13:45 PM “Mark this question €& => Question Id : 29964 Question 7 of 30 A.man aged 57 years who has a 25 year pack history smoking presents with productive cough with mucoid sputum of two year duration, Scattered rhonchi and wheezing are found on examination The likeliest diagnosis is: a) Chronic bronchitis ) Bronchial asthma ¢) Bronchiectasis 4) Fibrosing alveolitis ¢) Pneumonitis Anowor (UBERSEREN) oiner veers Explanation Report An Evor Question Explanation: Chronic bronchitis is one of the most common respiratory disease due to cigarette smoking, The smoking history and productive cough for atleast two years is indicative of chronic bronchitis, 2252014 1:13:45 PM “Mark this question €& => Question Id : 29964 Question 7 of 30 A.man aged 57 years who has a 25 year pack history smoking presents with productive cough with mucoid sputum of two year duration, Scattered rhonchi and wheezing are found on examination The likeliest diagnosis is: © a) Chronic bronchitis ) Bronchial asthma ¢) Bronchiectasis 4) Fibrosing alveolitis ¢) Pneumonitis Anowor (UBERSEREN) oiner veers Explanation Report An Evor Question Explanation: Chronic bronchitis is one of the most common respiratory disease due to cigarette smoking, The smoking history and productive cough for atleast two years is indicative of chronic bronchitis, Mark this question & => Question Id : 30070 Question 8 of 30 Out of the following, which one is correct regarding cryptogenic fibrosing alveolitis? a) Peak flow rate is a good guide to severity +) 80 per cent of patients initially respond well to immanosuppression ©) Peak incidence seen in the fourth decade © Active inflammation may be suggested by CT scan, €) Lung volumes show a raised residual volume / total lung capacity ratio Quostion Explanation: 2 ~ also the presence of a predominantly ground glass appearance is an independent predictor of survival. ' - Peak flow measure arway obstruction. CFA is characterized by a restricitive defect on lung function testing ¢ - About 40% of patients have an improvement in their symptoms with steroids and 25% of patients have an tmprowement in their symptoms with steroids and 25% have improved huag function 4 - Peak incidence is in tae 6th decade. ¢ - Residual volume (RV) increases with airways obstruction, total lang capacity (TLC) reduces with restrictive disorders like CEA, raised RV/TLC ratio suggests a combination of airways obstruction ard restrictive defect NOT just CFA as mentioned in the question. Mark this question & => Question Id : 30070 Question 8 of 30 Out of the following, which one is correct regarding cryptogenic fibrosing alveolitis? a) Peak flow rate is a good guide to severity +) 80 per cent of patients initially respond well to immanosuppression ©) Peak incidence seen in the fourth decade Y © & Active inflammation may be suggested by CT scan €) Lung volumes show a raised residual volume / total lung capacity ratio Quostion Explanation: 2 ~ also the presence of a predominantly ground glass appearance is an independent predictor of survival. ' - Peak flow measure arway obstruction. CFA is characterized by a restricitive defect on lung function testing ¢ - About 40% of patients have an improvement in their symptoms with steroids and 25% of patients have an tmprowement in their symptoms with steroids and 25% have improved huag function 4 - Peak incidence is in tae 6th decade. ¢ - Residual volume (RV) increases with airways obstruction, total lang capacity (TLC) reduces with restrictive disorders like CEA, raised RV/TLC ratio suggests a combination of airways obstruction ard restrictive defect NOT just CFA as mentioned in the question. 225/204 1:44:16 PM Mark this question & => Question 9 of 30 A 35 year old black female is diagnosed with pulmonary sarcoidosis. Which one of the following should be performed on the patient? a) Slit lamp examination of the eyes b) HLA serotype testing ¢) Helper-to suppressor T-cell ratio 4) Eveim ckin test Anower [UEHRNSIEN oer Ueors Expan Question Explanation: Patients with sarcoidosis should have a slit lamp examination to rule out uveitis, pulmonary finction testing, @ serum calcium level, and electrocardiography. In addition, serum angictensin converting enzyme and anergy skin testing might be considered, Certain HLA types have been associated with sarcoidosis in Japan, but testing is not specific. T-cell ratios may explain the reason for anergy, but are not specific or recommended. The Kveim skin test is not used clinically now because ofthe danger of injecting carcoid antigen into another patient. Report An Error 225/204 1:44:16 PM Mark this question & => Question 9 of 30 A 35 year old black female is diagnosed with pulmonary sarcoidosis. Which one of the following should be performed on the patient? Y © a Slitlamp examination of the eyes b) HLA serotype testing ¢) Helper-to suppressor T-cell rato 4) Kiveiea cin test Anower [UEHRNSIEN oer Ueors Expan Question Explanation: Patients with sarcoidosis should have a slit lamp examination to rule out uveitis, pulmonary finction testing, @ serum calcium level, and electrocardiography. In addition, serum angictensin converting enzyme and anergy skin testing might be considered, Certain HLA types have been associated with sarcoidosis in Japan, but testing is not specific. T-cell ratios may explain the reason for anergy, but are not specific or recommended. The Kveim skin test is not used clinically now because ofthe danger of injecting carcoid antigen into another patient. Report An Error ‘Marke this question ==> ‘Question Ta : 54899 Question 10 of 30 A 30-year-old smoker has to week old cough, fever and hemoptysis. CAR demonstrates diffuse alveolar infiltrates. A urine dipstick: demonstrates red cell casts, Hb=10.8g/dl, WCC= 5.1 x 103, Platelets=376 x10? end ANCA positive at titer is 1 in 3600. Diegnose? a) Aiport's syndrome b) Goodpasure’s syndrome ©} Polymyositis ) ad ©) Churg-Strauss Syndrome Anower [JEIPIRNSHER) otnerUsors Explanation Repert An Eixor Question Explanation: Goodpasiure’s syndcome presents in young men in their twenties and men and women in their siztis. It fequently has an eruptive presentation in the young, with: - Cough - Fever Relapsing polychondritis - Hemoplysis -Hematwia ~ Proteinuria and -Red cell caste “The pulmonary heemorhage resul in a drop in haemoglobin and the ank-neutrophi cytoplasmic antbodies (AINCA) is postive Alport's syndrome presents with: ~ hematuria - proteinuria and - progressive renal failure and ~ sensorineural deafness Both conditions are disorders of type IV’ collagen assemble. Churg-Strauss Synckome maybe characterized by Allergic rhinitis, asthma, p-ANCA antibodies (70% of cases) and eosnophilia ‘Marke this question ==> ‘Question Ta : 54899 Question 10 of 30 A 30-year-old smoker has to week old cough, fever and hemoptysis. CAR demonstrates diffuse alveolar infiltrates. A urine dipstick: demonstrates red cell casts, Hb=10.8g/dl, WCC= 5.1 x 103, Platelets=376 x10? end ANCA positive at titer is 1 in 3600. Diegnose? a) Aiport's syndrome ¥ © b) Goodpasmire’s syndrome ©} Polymyositis ) ad ©) Churg-Strauss Syndrome Anower [JEIPIRNSHER) otnerUsors Explanation Repert An Eixor Question Explanation: Goodpasiure’s syndcome presents in young men in their twenties and men and women in their siztis. It fequently has an eruptive presentation in the young, with: - Cough - Fever Relapsing polychondritis - Hemoplysis -Hematwia ~ Proteinuria and -Red cell caste “The pulmonary heemorhage resul in a drop in haemoglobin and the ank-neutrophi cytoplasmic antbodies (AINCA) is postive Alport's syndrome presents with: ~ hematuria - proteinuria and - progressive renal failure and ~ sensorineural deafness Both conditions are disorders of type IV’ collagen assemble. Churg-Strauss Synckome maybe characterized by Allergic rhinitis, asthma, p-ANCA antibodies (70% of cases) and eosnophilia Marke this question = => Question Td : 61074 Question 11 of 30 A.21 year old tall thin boy presents to the ER with difficulty breathing and chest pain when he tries to breath. A STAT Chest X-ray shows the Following — bs Wet ‘5 the most appropriate imme diate intervention? 2) Antibiotics ) Needle thoracentesis ©) Chest tube 4) CT of chest Answer | Exvianaton) Other User's Explanation Report An Error Question Explanation: Pneumothoraxis air in the pleural space causing partial or complete lung collapse. Pneumothora can occur spontaneously or from underlving pulmonary disease, trauma, or medical procedures Symptoms inckide dyepnea, pleuritic chest pain, and arstety. Dyspnea may be cuclen or gradual in onset depending on the rate of development and size of the pneumothorax. Physical findings classically consist of absert tactle fremitus, hyperresonance to percussion, and decreased breath sounds on the side with the pneumothorax Diagnosis is made with chest X-ray. Radiolucent air and the absence of hing merkings juxtaposed between a shmnien lobe or lang end the parietal pleura are diagnostic of pneumothorax. Tracheal deviation and medicstinal shift ocour with large pacumotheraces. Tension pneumothorax is a medical emergency It should de treated immediately by inserting a 14 or 16 gauge needle with catheter through the chest wall in the 2nd intercostal space at the mid-clavicular ine. The sound of high-pressure air escaping confirmas Lisoracie teneraenbe clenentieccice enive be C-liscoed smenechatels bar chest nase tinraecolaner alter woh w te patketer ie vemcoed Marke this question = => Question Td : 61074 Question 11 of 30 A.21 year old tall thin boy presents to the ER with difficulty breathing and chest pain when he tries to breath. A STAT Chest X-ray shows the Following — bs Wet ‘5 the most appropriate imme diate intervention? 2) Antibiotics Y © b) Needle thoracentesis ©) Chest tube 4) CT of chest Answer | Exvianaton) Other User's Explanation Report An Error Question Explanation: Pneumothoraxis air in the pleural space causing partial or complete lung collapse. Pneumothora can occur spontaneously or from underlving pulmonary disease, trauma, or medical procedures Symptoms inckide dyepnea, pleuritic chest pain, and arstety. Dyspnea may be cuclen or gradual in onset depending on the rate of development and size of the pneumothorax. Physical findings classically consist of absert tactle fremitus, hyperresonance to percussion, and decreased breath sounds on the side with the pneumothorax Diagnosis is made with chest X-ray. Radiolucent air and the absence of hing merkings juxtaposed between a shmnien lobe or lang end the parietal pleura are diagnostic of pneumothorax. Tracheal deviation and medicstinal shift ocour with large pacumotheraces. Tension pneumothorax is a medical emergency It should de treated immediately by inserting a 14 or 16 gauge needle with catheter through the chest wall in the 2nd intercostal space at the mid-clavicular ine. The sound of high-pressure air escaping confirmas Lisoracie teneraenbe clenentieccice enive be C-liscoed smenechatels bar chest nase tinraecolaner alter woh w te patketer ie vemcoed 2/25/2014 1:45:11 PM ‘Mark this question & => Question Td : 81512 Question 12 of 30 A 32 year old naval officer went diving in the ocean in search of a lost scuba diver. After 30 minutes of submersion, he sudderly experienced joint pain, headache, confusion, dizziness, and dyspnea, The most licely diagnosis is a) Non-cardiogenic pulmonary edema ) Rupture of a berry aneurysm ©) Decompression sickness ) Hypoxic encephalopathy e) Eypercapnea Question Explanation: Decompression sickness is secondary to increased pressure from submersion, Gas bubble formation in the joints (bends), ear and sinus barotrauma, and arterial gas embolism occurs about 30 minutes laier. Treatment is oxygen supplementation and intravenous hydration Non-cardiogenic pulmonary edema (A) would be associated with shortness of breath, not joint pains. Rupture of a berry aneurysm (B) is not associated with joint pain and submersion, Hypoxic encephalopathy (D) would resuit in confusion and mental status changes. This can occur in acute mountain sickness secondary to high altitudes. Hypercapnea (E) occurs when the arterial PCO; is greater than 50 mmHg, This can occur secondary te pulmonary hypertension. 2/25/2014 1:45:11 PM ‘Mark this question & => Question Td : 81512 Question 12 of 30 A 32 year old naval officer went diving in the ocean in search of a lost scuba diver. After 30 minutes of submersion, he sudderly experienced joint pain, headache, confusion, dizziness, and dyspnea, The most licely diagnosis is a) Non-cardiogenic pulmonary edema ) Rupture of a berry aneurysm Y © 0) Decompression sickness ) Hypoxic encephalopathy e) Eypercapnea Question Explanation: Decompression sickness is secondary to increased pressure from submersion, Gas bubble formation in the joints (bends), ear and sinus barotrauma, and arterial gas embolism occurs about 30 minutes laier. Treatment is oxygen supplementation and intravenous hydration Non-cardiogenic pulmonary edema (A) would be associated with shortness of breath, not joint pains. Rupture of a berry aneurysm (B) is not associated with joint pain and submersion, Hypoxic encephalopathy (D) would resuit in confusion and mental status changes. This can occur in acute mountain sickness secondary to high altitudes. Hypercapnea (E) occurs when the arterial PCO; is greater than 50 mmHg, This can occur secondary te pulmonary hypertension. Mark this question & => Question Td : 82441 Question 13 of 30 A.5S yeer old man with mild chronic obstructive pulmonary disease retuned from a business convention and developed a fever and cough. He was treated for pneumonia with amoxicillin’ clavulanic acid (Augmentin®) but failed to improve. What antimicrobial is most likely to be helpful in tis situation? 2) timethaprim/salfamethoxazole 6) Erythromycin, ) Tobramycin, 4) Amantadine ©) Aztreonam. newer (UEIIRIRAN) omer users Explanation Report An Error Question Explanation: “Most community acquired pneamonia in this age group is due to S. pneumoniae and, hecanse of the patient's history of mild chronic obstructive pulmonary discase, TH influenzae should also be considered. However, amoxicillin clavulenate should be active against ‘most strains of these pathogens, so other organisms should be considered. The patient's recent retum from a convention suggests that he spenttime in buildings (convention centers, hotels) with large air-handling systems which can occasionally be contaminated with Legionella pneumophila This underlying lg disease would also predispose him to disease after an exposure that might not lead to iness in others. Erythromycin would also cover Mycoplasma pneumoniae, another common cause of community-acquired pheumotia, although usually in a younger population. The coverage of timethoprim/sulfamethoxazcle (A) significantly overlaps that of amoxicilin/clavulanete and is unlikely to include other important organisms Tobramycin (C) and aztreonam (E) are beth excellent agents against a variety of moderately resistant Gram-negative bacilli, but these organisms are urlikely to be involved in this cave Asnantacine (D) is an anti-irfluenza drug but the patientis not reported as having other influenza-like syraptoms Mark this question & => Question Td : 82441 Question 13 of 30 A.5S yeer old man with mild chronic obstructive pulmonary disease retuned from a business convention and developed a fever and cough. He was treated for pneumonia with amoxicillin’ clavulanic acid (Augmentin®) but failed to improve. What antimicrobial is most likely to be helpful in tis situation? 2) timethaprim/salfamethoxazole ¥ © ») Erythromycin, ¢) Tobramycin, 4) Amantadine «) Aztreoncm, newer (UEIIRIRAN) omer users Explanation Report An Error Question Explanation: “Most community acquired pneamonia in this age group is due to S. pneumoniae and, hecanse of the patient's history of mild chronic obstructive pulmonary discase, TH influenzae should also be considered. However, amoxicillin clavulenate should be active against ‘most strains of these pathogens, so other organisms should be considered. The patient's recent retum from a convention suggests that he spenttime in buildings (convention centers, hotels) with large air-handling systems which can occasionally be contaminated with Legionella pneumophila This underlying lg disease would also predispose him to disease after an exposure that might not lead to iness in others. Erythromycin would also cover Mycoplasma pneumoniae, another common cause of community-acquired pheumotia, although usually in a younger population. The coverage of timethoprim/sulfamethoxazcle (A) significantly overlaps that of amoxicilin/clavulanete and is unlikely to include other important organisms Tobramycin (C) and aztreonam (E) are beth excellent agents against a variety of moderately resistant Gram-negative bacilli, but these organisms are urlikely to be involved in this cave Asnantacine (D) is an anti-irfluenza drug but the patientis not reported as having other influenza-like syraptoms Mark this question & => Question Tid : 84707 Question 14 of 30 A patient with previously diagnosed lung cancer develops a large pleural effsion, ‘The effusion is tapped and demonstrates an unusval silky white duid that separates into a thin white layer over a thicker clear layer upon standing. Pleural duid to serum protein ratio is 20.5, pleural uid to serum lactate dehydrogenase ratio is >0.6. Thic is most licely an example of which of the following? 2) Chylous effision +) Empyema ) Fibsinous pleuritis 4) Fibrous pleusits 6) Transudate Question Explanation: ‘This is an exudative pleural effusion, and the uid is lymphatic fid, Exudative effusions have a pleural uid 10 serum protein ratio of0.5 and a pleural fid to serum LDH ratio of >0.6, or pleural fuid LDH > two-thirds the upper limit of normal serum value, A Jarge tumor in the chest can damage the thoracic duct, allowing the chylomicron-rich iymphatic fic from the abdomen to drain into the pleural cavity (chylothorax) rether than the vena cava. Chylous effusions can also develop secondary to trauma, Empyema (choice B) is an exudative effusion that is a resuit of infection, and the pleural dud will be frankly purulent. ‘Transudative pleural effusions (choice E) are filtrates of sesurn. Fibrinous plentitis (choice C) is characterized by deposition of fibrin on the pleural surface Fibrous pleuritis (choice D) is characterized by doposition of collagen and formation of cormective tiseue along the pleural membranes. Mark this question & => Question Tid : 84707 Question 14 of 30 ‘A patient wih previously diagnosed lung cancer develops a large rleural effusion. The effusion is tapped and demonstrates an unusual nilley white duid that separates into a thin white layer over a thicker clear layer upon standing. Pleural uid to serum protein ratio is 0.5, pleural fluid to serum lactate dehydrogenase ratio is >0.6. This is most lcely an example of which of the following? Y © a) Chylous effusion b) Empyema ©) Hibrinous pleusitis © Fibrous pleurits e) Transudate Question Explanation: ‘This is an exudative pleural effusion, and the uid is lymphatic fid, Exudative effusions have a pleural uid 10 serum protein ratio of0.5 and a pleural fid to serum LDH ratio of >0.6, or pleural fuid LDH > two-thirds the upper limit of normal serum value, A Jarge tumor in the chest can damage the thoracic duct, allowing the chylomicron-rich iymphatic fic from the abdomen to drain into the pleural cavity (chylothorax) rether than the vena cava. Chylous effusions can also develop secondary to trauma, Empyema (choice B) is an exudative effusion that is a resuit of infection, and the pleural dud will be frankly purulent. ‘Transudative pleural effusions (choice E) are filtrates of sesurn. Fibrinous plentitis (choice C) is characterized by deposition of fibrin on the pleural surface Fibrous pleuritis (choice D) is characterized by doposition of collagen and formation of cormective tiseue along the pleural membranes. 2i25/2014 1:46:01 PM ‘Mark this question & => Question Td : 85003 Question 15 of 30 An 18 year old boy brought to the emergency room is noted to be hyperventilating. Blood gas studies show both a respiratory alkelosis and metabolic acidosis. This presentation is most suggestive of poisoning with which of the following agents? a) Acetaminophen ) Carbon monoxide ) Lead ) Mereary ¢) Salicylates Question Explanation: Tis worth leaming the presentations of the common poisons isted in the answer choices, because a timely diagnosis can sometimes save a patient's life. This boy's presentation is typical for salicylate poisoning, including poisoning with aspirin, Fatalities are typically ue to dehydration and hypokalemia, Salicylate toxicity has cifferent effects on the body. First it causes respiratory alkalosis by rectly stimulating the respiratory center in the brain stem and causing centrally mediated hyperventilation and hyper apnea. It also interferes with the Krebs cycle and leads to metabolic acidosis through the reversion to anaerabic glycolysis as a method of energy production in the body. This causes lactic acidosis, and eventually metabolic acidosis with an elevated anion gap. ‘Tn contrast to the acute toxcity of salicylates, acetaminophen (choice A) poisoning produces nausea, vomiting, abdominal pain, and shock in some cases it can cause irreversible hepatic failure Carbon monoxide (choice B) causes hypozia, and a cherry-red coloration of the lps and mucous membranes maybe evident. Lead and mercury poisoning are both usually seen in their chroric forms, although mercury ingestion can produce an acute toxicity with renal tubular necrosis and necrosis of gastrointestinal epithelium. Chronic lead poisoning (choice C) can cause anemia (with basophilic stippling of RBCs), neuropathy, and abdominal pain. Chronic mercury poisoning (choice D) canses CNS atrophy, gngiviis, gastits, and renal tubvlar changes. 2i25/2014 1:46:01 PM ‘Mark this question & => Question Td : 85003 Question 15 of 30 An 18 year old boy brought to the emergency room is noted to be hyperventilating. Blood gas studies show both a respiratory alkelosis and metabolic acidosis. This presentation is most suggestive of poisoning with which of the following agents? a) Acetaminophen ) Carbon monoxide ) Lead ) Mereary Y © 2) Salicylates Question Explanation: Tis worth leaming the presentations of the common poisons isted in the answer choices, because a timely diagnosis can sometimes save a patient's life. This boy's presentation is typical for salicylate poisoning, including poisoning with aspirin, Fatalities are typically ue to dehydration and hypokalemia, Salicylate toxicity has cifferent effects on the body. First it causes respiratory alkalosis by rectly stimulating the respiratory center in the brain stem and causing centrally mediated hyperventilation and hyper apnea. It also interferes with the Krebs cycle and leads to metabolic acidosis through the reversion to anaerabic glycolysis as a method of energy production in the body. This causes lactic acidosis, and eventually metabolic acidosis with an elevated anion gap. ‘Tn contrast to the acute toxcity of salicylates, acetaminophen (choice A) poisoning produces nausea, vomiting, abdominal pain, and shock in some cases it can cause irreversible hepatic failure Carbon monoxide (choice B) causes hypozia, and a cherry-red coloration of the lps and mucous membranes maybe evident. Lead and mercury poisoning are both usually seen in their chroric forms, although mercury ingestion can produce an acute toxicity with renal tubular necrosis and necrosis of gastrointestinal epithelium. Chronic lead poisoning (choice C) can cause anemia (with basophilic stippling of RBCs), neuropathy, and abdominal pain. Chronic mercury poisoning (choice D) canses CNS atrophy, gngiviis, gastits, and renal tubvlar changes. ) Mark this question = => Question Td : 85126 Question 16 of 30 A32 year old man complains of bloody sputum, fever and night sweats. Lung biopsy shows multiple noduler lesions consisting of large epithelicid cells and surrounded by lymphocytes and fibroblasts, as shown in the photomicrograph below. There is an area of necrosis in the center ofsome nodules. Numerous acid-fast bacili are demonstrated by Ziehl-Neelsen staining within the cytoplasm of epithelioid cells. Silver stains for fingi are negative. The condition that predisposed the patient to this pulmonary disease is a) Acqtired immunodeficiency syndrome (AIDS) b) Common variable immunodeficiency ©) Cystic flbrosis 4d) Depressed level of consciousness ©) Sarcoidosis Question Explanation: The lesion shown is a necratising granuloma developing as a result of infection by acid-fast bacilli mast lleely mycobacteria). Grannlomatons inflarsmaticn is a specialized form of chronis inflemmation which begins with the uptalce of forcign antigens by macrophages. These cells process and presert the antigen to helper T lymphocytes, which in turn activate macrophages by interferon- secretion. Granulomas may be due to infectious and norinfectious causes. Usually, infectious granulomas are necrotizing and non-infections granuloma are non-necrotizing, but there are many exceptions to this rule, Among the infectious causes, mycobacteria infections are of the utmost importance, especially in patients with AIDS. Fungi may also cause granulomas, which may be ofthe necrotizing or non- necrotizing type. By definition, however, caseating necrosis is pathognomonic of mycobacterial infection. Common variable immunodeficiency (choice B) is one of the most common forms of congenital immune deficiency syndrome. It is due to inability of lmmphocytes to mabire into plasma cells. Consequently, levels of zmmunoglobulins are depressed, and patients sulfer from depressed humoral immuity. Cell-nediated immunity (on which granuloma formation is largely dependent) is intact Cystic fibrosis (choice C) predisposes to recurrent bronchopneumonia and development of bronchiectass. Depressed level of consciousness (choice D) predisposes to aspiration of gastric contents and development of aspiration preumcnia Sheets of macrophages are often observed, but aot necrotizing gramulomas (unless there are additional predisposing conditions, such as AIDS). Sarcoidosis (choice E) is a diagnosis of exclusion. The condition is characterized by widespread granuloma formation without User Sse ee a eta Res sas ele cag ak ce GI ) Mark this question = => Question Td : 85126 Question 16 of 30 A32 year old man complains of bloody sputum, fever and night sweats. Lung biopsy shows multiple noduler lesions consisting of large epithelicid cells and surrounded by lymphocytes and fibroblasts, as shown in the photomicrograph below. There is an area of necrosis in the center ofsome nodules. Numerous acid-fast bacili are demonstrated by Ziehl-Neelsen staining within the cytoplasm of epithelioid cells. Silver stains for fingi are negative. The condition that predisposed the patient to this pulmonary disease is Y © a) Acquired inmnodeficiency syndrome (AIDS), 'b) Common variable immunodeiiciency c) Cystic fbrosis d) Depressed level of consciousness 2) Sarcoidosis Question Explanation: The lesion shown is a necratising granuloma developing as a result of infection by acid-fast bacilli mast lleely mycobacteria). Grannlomatons inflarsmaticn is a specialized form of chronis inflemmation which begins with the uptalce of forcign antigens by macrophages. These cells process and presert the antigen to helper T lymphocytes, which in turn activate macrophages by interferon- secretion. Granulomas may be due to infectious and norinfectious causes. Usually, infectious granulomas are necrotizing and non-infections granuloma are non-necrotizing, but there are many exceptions to this rule, Among the infectious causes, mycobacteria infections are of the utmost importance, especially in patients with AIDS. Fungi may also cause granulomas, which may be ofthe necrotizing or non- necrotizing type. By definition, however, caseating necrosis is pathognomonic of mycobacterial infection. Common variable immunodeficiency (choice B) is one of the most common forms of congenital immune deficiency syndrome. It is due to inability of lmmphocytes to mabire into plasma cells. Consequently, levels of zmmunoglobulins are depressed, and patients sulfer from depressed humoral immuity. Cell-nediated immunity (on which granuloma formation is largely dependent) is intact Cystic fibrosis (choice C) predisposes to recurrent bronchopneumonia and development of bronchiectass. Depressed level of consciousness (choice D) predisposes to aspiration of gastric contents and development of aspiration preumcnia Sheets of macrophages are often observed, but aot necrotizing gramulomas (unless there are additional predisposing conditions, such as AIDS). Sarcoidosis (choice E) is a diagnosis of exclusion. The condition is characterized by widespread granuloma formation without User Sse ee a eta Res sas ele cag ak ce GI 2252014 1:46:41 PM Mark this question €c=> Question 17 of 30 Question Td : 92064 A 30-year-old dyspneic AIDS patient has fever and a non-productive cough A CXR shows bilateral lfuse interstal infiltrates without hilar adenopathy. Oxygen partial pressure is reduced The most likely causative agentis @) Candida Albicans $) Histoplasma Capsulatum, ©) Coceidiodes Tinmitis 6) Aspergillas flavus, ©) Pneumocystis carinis Anewer [UEIRINN) omer sors Explanation Report An Error Question Explanation: Pneumocystis carini is a common cause of infection in HIV-positive individuals, It can cause pneumocystic carini pneumonia, which presents as bilateral diffuse interstitial infiltrates without hilar adenopathy. 2252014 1:46:41 PM Mark this question €c=> Question 17 of 30 Question Td : 92064 A 30-year-old dyspneic AIDS patient has fever and a non-productive cough A CXR shows bilateral lfuse interstal infiltrates ‘without hilar adenopathy. Oxygen partial pressure is reduced The mostlikely causative agentis @) Candida Albicans ) Histoplasma Capsulatumn. ©) Coceidiodes Tinmitis 4) Aspergillus Barus Y © ®) Pneumocystis carini, Anewer [UEIRNN) omer user's Explanation Report An Error Question Explanation: Pneumocystis carini is a common cause of infection in HIV-positive individuals, It can cause pneumocystic carini pneumonia, which presents as bilateral diffuse interstitial infiltrates without hilar adenopathy. ‘Mark this question & => Question 18 of 30 Question Td : 94617 A 67 year old man with metastatic ling cancer is hospitalized because of decreased appetite, lethargy, and confusion of 2 weeks duration. Laboratory evalvation reveals the fellownng Serum calcium —‘|[15.8 mg/dL QW 8.4-10.0) [Serum phosphorus |B.9 mfdL QT 2.6-4.2) [Serum creatinine [1.1 mg/dL AY 0.7-1.3) [Total serum protein||7.3 aldL (NT 6.0-8.0) [Albumin |B gal 09 3.7-48) ‘Tho most appropriate iritidl management is which of the fellowing? a) Calcitonin (calcimar) subcutaneously +b) Pamideonate dicodiura (Aredia) by intravencus infusion c) Normal saline intravenously 4) Plicamycin (Mithrareycin) intravenously ©) Furosemide (Lasix) intravenously Question Explanation Thitial management of hypercaloenia of malignancy calls for Quid replacement with normal saline to correct the volume depletion that 1s invariably present and to enhance renal calcium excretion, The use of loop diuretics such as furosemide should be restricted to patients in danger of fuid overload, since these drugs can aggravate volume depletion anc are not very effective clone in prompting renal calcium excretion, Aithough intravenous pamidronate has become the manstay for the hypercalcemia of malignancy, itis considered only after the hypercalomic patient has been rendered cuvolemic by saline repletion. The same is tric for the other calcium lowering agents listed, ‘Mark this question & => Question 18 of 30 Question Td : 94617 A 67 year old man with metastatic ling cancer is hospitalized because of decreased appetite, lethargy, and confusion of 2 weeks duration. Laboratory evalvation reveals the fellownng Serum calcium —‘|[15.8 mg/dL QW 8.4-10.0) [Serum phosphorus |B.9 mfdL QT 2.6-4.2) [Serum creatinine [1.1 mg/dL AY 0.7-1.3) [Total serum protein||7.3 aldL (NT 6.0-8.0) [Albumin |B gal 09 3.7-48) ‘Tho most appropriate iritidl management is which of the fellowing? a) Calcitonin (calcimar) subcutaneously +b) Pamideonate dicodiura (Aredia) by intravencus infusion Y © c) Normal saline intravenously 4) Plicamycin (Mithrareycin) intravenously ©) Furosemide (Lasix) intravenously Question Explanation Thitial management of hypercaloenia of malignancy calls for Quid replacement with normal saline to correct the volume depletion that 1s invariably present and to enhance renal calcium excretion, The use of loop diuretics such as furosemide should be restricted to patients in danger of fuid overload, since these drugs can aggravate volume depletion anc are not very effective clone in prompting renal calcium excretion, Aithough intravenous pamidronate has become the manstay for the hypercalcemia of malignancy, itis considered only after the hypercalomic patient has been rendered cuvolemic by saline repletion. The same is tric for the other calcium lowering agents listed, 2i25:2014 1:47:11 PM “Mark this question €&c=> Question 19 of 30 A 74 year old man with a histosy of emphysema presents to his physician after he develops the acute onset of fevers, rigors, and a cough productive of green spubam, His temperature is 38.3 C (100.9 F), blood pressure is 155/90 minHg, pulse is 90‘min, and respirations are 34/min He weights 80 eg An arterial blood gas reveals a pH of 7 20, apCO2 of 60 mmHg, and a pO2 of 52mm Hg. He becomes increasingly tachypaeic, and his pCO2 rises to 74 mm¥g over the next 2 hours. The decision is made to intubate hitn at thet point, Most appropriate setting for his tidal volume on the respirator? 8) 400 mL breath ) 800 mL/breath ©) 1200 mL/breath 6) 1600 mL/breath Question Explanation: ‘The tidal volume for a patient is generally estimated as 10 mike of weight, which for this patient would be 850 mL/breath. Giving a ower tidal volurne vill yield hypoventilation and he insufficient to eliminate pCO2 Providing a tidal wchume greater then 10 ral increases the risk of pneumcthoracr, particularly in a patient with longstanding emphysema who may have thin-walled alveoli Report An Error 2i25:2014 1:47:11 PM “Mark this question €&c=> Question 19 of 30 4.74 year old man with a history of emphysema presents to his physician after he develops the acute onset of fevers, rigors, anda cough productive of green sputim. His temperature is 38.3 C (100.9 F). blood pressure is 155/90 mmHg, puise is 90/min, and respirations are 34/min He weights 20 kg An arterial blood gas reveals a pH of 7.20, apCO2 of 60 mmHg, and a pO2 of 52mm. ‘Hg, He becomes increasingly tachypneic, and his pCO2 rises to 74 mmHg over the next 2 hours. The decision is made to intubate hain at thet point. Most appropriate setting for his tidal volume on the respirator? a) 400 mL/breath V © 6) 800 mLibreath c) 1200 mL/breath 4) 1600 mLfbreath Question Explanation: ‘The tidal volume for a patient is generally estimated as 10 mike of weight, which for this patient would be 850 mL/breath. Giving a ower tidal volurne vill yield hypoventilation and he insufficient to eliminate pCO2 Providing a tidal wchume greater then 10 ral increases the risk of pneumcthoracr, particularly in a patient with longstanding emphysema who may have thin-walled alveoli Report An Error 2/25/2014 1:47:23 PM Mark this question €=> Question Id : 115471 Question 20 of 30 Factor that does NOT determine the extent of pulmonary injury in gastric acid aspiration include a) pH of the aspirate ) Presence of food particles. ©) Volume of the aspirate 4) Distribution of the aspirate. €) Overgrowth of normal oropharyngeal fora Answer ( Expionation) Other User's Explanation Report An Error Question Explanation: “The extent of ilness caused by aspiration of gastric contents is determined by several factors. The acidity of the aspirated material is the most important factor, with a oH <2.5 being proposed as a cut off value, Aspiration of food particles causes severe pneumonitis “Aspirating more than 0.4 mileg body weight of gastne acid is sufficient to cause preumonits Although conghing is protective, in some cases it enhances dispersion of acid over a greater pulmonary surface area, The majority of normal fora of the oropharyngeal cavity has little invasiveness in the normal host. Overgrowth of this area with pathogenic bacteria predisposes to pneumonia 2/25/2014 1:47:23 PM Mark this question €=> Question Id : 115471 Question 20 of 30 Factor that does NOT determine the extent of pulmonary injury in gastric acid aspiration include 2) pH ofthe aspirate +b) Presence of food particles. ) Volume of the aspirate. d) Distribution of the aspirate. Y © ©) Overgrowth of normal oropharyngeal flora Answer ( Expionation) Other User's Explanation Report An Error Question Explanation: “The extent of ilness caused by aspiration of gastric contents is determined by several factors. The acidity of the aspirated material is the most important factor, with a oH <2.5 being proposed as a cut off value, Aspiration of food particles causes severe pneumonitis “Aspirating more than 0.4 mileg body weight of gastne acid is sufficient to cause preumonits Although conghing is protective, in some cases it enhances dispersion of acid over a greater pulmonary surface area, The majority of normal fora of the oropharyngeal cavity has little invasiveness in the normal host. Overgrowth of this area with pathogenic bacteria predisposes to pneumonia 2252014 1:47:45 PM ‘Mark this question & => Question Id : 119358 Question 21 of 30 A 24 year old man was brought to the emergency room in an unconscious state. His lips and nail beds are cherry red. His carhozyhemoglobin level is sixty peccent. Ling auscultation revealed ronchi, and this patient was diagnosed with respiratory Palure ‘The most immediate intervention would be a) Hyperbaric oxygen therapy +) One hundred percent oxygenation via a non rebreabher venlimasie 6) Indirect larmgoscopy 4) Intubation and ventilation with one hundred percent oxygen ) Diuretic therapy Question Explanation: ‘The most immediate concem for patients suffering fom stoke inhalation is to overcome loss of ventilation induced by CO. which damages lung tistue. Patients should be given 100% Oy immediately by a tight fitting mask or intubation. This patient may also require hyperbaric O, therapy, as indicated by a very high carboxyhemoglobin concentration, This method is considered adjunctive to respiratory support and is not the first step in therapy, but reduces mortality rates when initiated within sis hours of poisoning. In a patient who is unconscious and who is in respiratory failure, 100% oxygen given by non retreather ventimasks delivering only about 50% of the oxygen supplied, would not be effective for an unconscious person with an elevated carboxyhemoglobin level, which suggests a high concentration of CO in the blood Indtect larmgoscopy would be done to visualize thermal or toxic injury to the airway if stridor is present. Diuretic therapy would be given if non cardiogenic pulmonary edemais present after smoke inhalation This medication would be given only efter the patient has been intubated and given oxygen, 2252014 1:47:45 PM ‘Mark this question & => Question Id : 119358 Question 21 of 30 A 24 year old man was brought to the emergency room in an unconscious state. His lips and nail beds are cherry red. His carhozyhemoglobin level is sixty peccent. Ling auscultation revealed ronchi, and this patient was diagnosed with respiratory Palure ‘The most immediate intervention would be a) Hyperbaric oxygen therapy +) One hundred percent oxygenation via a non rebreabher venlimasie 6) Indirect larmgoscopy Y © 4) Inmubation and ventilation with one hundred percent oxygen €) Diuretic therapy Question Explanation: ‘The most immediate concem for patients suffering fom stoke inhalation is to overcome loss of ventilation induced by CO. which damages lung tistue. Patients should be given 100% Oy immediately by a tight fitting mask or intubation. This patient may also require hyperbaric O, therapy, as indicated by a very high carboxyhemoglobin concentration, This method is considered adjunctive to respiratory support and is not the first step in therapy, but reduces mortality rates when initiated within sis hours of poisoning. In a patient who is unconscious and who is in respiratory failure, 100% oxygen given by non retreather ventimasks delivering only about 50% of the oxygen supplied, would not be effective for an unconscious person with an elevated carboxyhemoglobin level, which suggests a high concentration of CO in the blood Indtect larmgoscopy would be done to visualize thermal or toxic injury to the airway if stridor is present. Diuretic therapy would be given if non cardiogenic pulmonary edemais present after smoke inhalation This medication would be given only efter the patient has been intubated and given oxygen, 2/25/2014 1:47:59 PM ‘Merk this question & => Question 22 of 30 A 24 year old medical student was found to have symmetrical bilateral hilar adenopathy on a routine chest X-ray. He was asymptomatic. On skin testing, he was found to he anergic. Chest CT revealed symmatrical non narcetiring hilar adenopathy. The treatment of choice in this patient would be a) Isoniazid, rifampin, pyrazinamide b) Steroids ©) Chemotherapy 6) Do nothing ©) Amphotericin B. Question Explanation: This patient has stage T sarcoid He is not symptomatic and thus routine follow up is the only thing to do at this point. Aneray is common because of in decrease in circulating CDA cells in the blood. Tuberculosis would most llcely show up as azymmetrical adenopathy. The chest CT would showy necrotizing hilar adenopathy. This patient does not have tuberculosis, and thus isoniazid, tfamnpin, and PZA are not indicated, Steroids are indicated in symptomatic active pulmonary sarcoid as well as neurosarcoid and eye involvement. Chemotherapy would be the treatment of choice ifthis patient had lymphoma, Amphotericin B would be indicated if this paticnt hed a fingal infection such as histoplasmosis or coccidioomycosis, 2/25/2014 1:47:59 PM ‘Merk this question & => Question 22 of 30 A 24 year old medical student was found to have symmetrical bilateral hilar adenopathy on a routine chest X-ray. He was asymptomatic. On skin testing, he was found to he anergic. Chest CT revealed symmatrical non narcetiring hilar adenopathy. The treatment of choice in this patient would be a) Isoniazid, rifampin, pyrazinamide b) Steroids ©} Chemotherapy oY © &Do nothing ®) Amphotericin B Question Explanation: This patient has stage T sarcoid He is not symptomatic and thus routine follow up is the only thing to do at this point. Aneray is common because of in decrease in circulating CDA cells in the blood. Tuberculosis would most llcely show up as azymmetrical adenopathy. The chest CT would showy necrotizing hilar adenopathy. This patient does not have tuberculosis, and thus isoniazid, tfamnpin, and PZA are not indicated, Steroids are indicated in symptomatic active pulmonary sarcoid as well as neurosarcoid and eye involvement. Chemotherapy would be the treatment of choice ifthis patient had lymphoma, Amphotericin B would be indicated if this paticnt hed a fingal infection such as histoplasmosis or coccidioomycosis, 2125/2014 1:18:18 PM Mark this question => ‘Question Id : 140383, Question 23 of 30 A.45 year oldman preseats for an employment physical examninaiion, On examination ihe patient's nail beds are thickened, widened, and convex. He also finds tenderness over the distal ends of the ractus, uina, and fibula, The most appropriate diagnostic step at this time is 2) Chest radiography 6) Endoscopy ©) MEI of the upper sight quadrent 4) Penal sonography €) Sorotal sonography Question Explanation: ‘This patient hes the finger clubbing and hypertrophic pulmonary osteoarthropathy that can be associated with bronchogenic carcinoma (other than sqiamous cell carcinoma berign mesothelioma, and diaphragmatic neurlemmoma, X-ray of the bones gensrelly shows formation of new periosteal bone; arthritis may be present: The cticlogy of these changes remains a mystory. An alert, clinician may identify a cancer at an earlier, potentially curable stage by investigating a possible paraneoplastic syndrome Report An Error 2125/2014 1:18:18 PM Mark this question => ‘Question Id : 140383, Question 23 of 30 A.45 year oldman preseats for an employment physical examninaiion, On examination ihe patient's nail beds are thickened, widened, and convex. He also finds tenderness over the distal ends of the radus, ulna, and fibula, ‘The most appropricte diagnostic step at this time is oY © 2) Chestradiography 6) Endoscopy ©) MEI of the upper sight quadrent 4) Penal sonography €) Sorotal sonography Question Explanation: ‘This patient hes the finger clubbing and hypertrophic pulmonary osteoarthropathy that can be associated with bronchogenic carcinoma (other than sqiamous cell carcinoma berign mesothelioma, and diaphragmatic neurlemmoma, X-ray of the bones gensrelly shows formation of new periosteal bone; arthritis may be present: The cticlogy of these changes remains a mystory. An alert, clinician may identify a cancer at an earlier, potentially curable stage by investigating a possible paraneoplastic syndrome Report An Error 2:25/2014 1:18:33 PM ‘Mark this question & => Question Id : 141844 Question 24 of 30 A T7 year old high school studert presents with low grade fever pleuritc pain and anongroductive cough, Her serum agglutinates Streptococcus saliverius strain MG, Which one of the following is the appropriate therapy? a) Ampicillin ) Erythromycin c) Oxygen and external cooling ) Penicillin G ) Ribavirin Question Explanation: ‘The patient hes primary atypical pneumonia caused by Mycoplasma pneumoniae. These organisms are fastidicus and difficult to culture in the Inboratory however, serodiagnosis can be mozt helpfid. Patierts produce one or two heterophile antibodies during the course of the infection one agalutinates human C+ RECs in the cold (the cold hemagelutinins), while the other causes the agelutnetion of a strain of Streptococcus selivarius termed strain MG (the strep MG agglutinins). Mycoplasma are susceptible to the macrolide farily of antibiotics (erythromycin, clarithromycin, etc.). These organisms lack a cell wall; hence, they are indifferent to antibiotics that interfere with peptidoglycan syuthesis, such as penicilin and arcpicillin, Oxygen and external cooling are therapeutic ineasures that are used in the treatment of severe respiratory diseases, such as pneumococcal pneumonia Ribavinin is used in the treatment of respiratory syncytial vis infection in infants. This is the most common cause of hospitalization for respiratory disease. in the very young, probably because aerosol administration of the antiviral compound is best accomplished in a hospital setting 2:25/2014 1:18:33 PM ‘Mark this question & => Question Id : 141844 Question 24 of 30 A.17 year old high school student presents with low grade fever pleurtic pein and a nonproductive cough, Her serum agglutinetes Streptococcus caliverine strain MG. Which one of the following ic the appropriate therapy? a) Ampicilln ¥ © b) Erythromycin c) Oxygen and external cooling 4) Penicillin G ) Ribavirin Question Explanation: ‘The patient hes primary atypical pneumonia caused by Mycoplasma pneumoniae. These organisms are fastidicus and difficult to culture in the Inboratory however, serodiagnosis can be mozt helpfid. Patierts produce one or two heterophile antibodies during the course of the infection one agalutinates human C+ RECs in the cold (the cold hemagelutinins), while the other causes the agelutnetion of a strain of Streptococcus selivarius termed strain MG (the strep MG agglutinins). Mycoplasma are susceptible to the macrolide farily of antibiotics (erythromycin, clarithromycin, etc.). These organisms lack a cell wall; hence, they are indifferent to antibiotics that interfere with peptidoglycan syuthesis, such as penicilin and arcpicillin, Oxygen and external cooling are therapeutic ineasures that are used in the treatment of severe respiratory diseases, such as pneumococcal pneumonia Ribavinin is used in the treatment of respiratory syncytial vis infection in infants. This is the most common cause of hospitalization for respiratory disease. in the very young, probably because aerosol administration of the antiviral compound is best accomplished in a hospital setting 2/25/2014 1:48:45 PM ‘Mark this question & => Question Td : 177811 Question 25 of 30 A 24 year old has left sided sudden onset chest pain at night and shortness of breath, Exantination shows shallow, rapid respirations, and hypesresonance to percussion on the left side, Breath sounds are absent on the left side. Arterial blood gases measurement would likely reveal a) Hlevated pE b) Elevated PCO2 ©) Hevated PO2 6) Reduced HCO3 ©) Reduced PCO2 Anewor [UERFIRISHAN) otner usar Exptan Question Explanation: This patient is suffering from spontaneous pneumothorax, in which the left ling collapsed. The collapse of the lung explains the absence of sound and the hyperresonance to percussion on the left side. This dysfimctional hing would decrease the amount of CO2 that can be exhaled, causing a buildup of PCO2, Therefore, reduced PCO2 is incorrect. When PCO2 nises, the body becomes more acidic thereby reducing pH rather than increasing i, PO2 would not be increased in a patient with pneumothorax, but would be normal to redused, The body's response to a decrease in pH due to elevaied PCO2 is to reabsorb more HCO3- in the kidney. Asa result, HCO3- would be elevated rather than reduced as a compensatory mechanism. Report An Error 2/25/2014 1:48:45 PM ‘Mark this question & => Question Td : 177811 Question 25 of 30 A 24 year old has left sided sudden onset chest pain at night and shortness of breath, Exantination shows shallow, rapid respirations, aad hyperreronance to percussion on the left cide. Breath sounde are absent on the lei side. Arterial blood gases measurement would likely reveal a) Hevated pE v © b) Hevated PCO2 6) Hlevated PO2 6) Reduced HCO3 6) Reduced PCO2 Anewor [UERFIRISHAN) otner usar Exptan Question Explanation: This patient is suffering from spontaneous pneumothorax, in which the left ling collapsed. The collapse of the lung explains the absence of sound and the hyperresonance to percussion on the left side. This dysfimctional hing would decrease the amount of CO2 that can be exhaled, causing a buildup of PCO2, Therefore, reduced PCO2 is incorrect. When PCO2 nises, the body becomes more acidic thereby reducing pH rather than increasing i, PO2 would not be increased in a patient with pneumothorax, but would be normal to redused, The body's response to a decrease in pH due to elevaied PCO2 is to reabsorb more HCO3- in the kidney. Asa result, HCO3- would be elevated rather than reduced as a compensatory mechanism. Report An Error 2:25/2014 1:49:00 PM Mark this question & => Question Id : 178983 Question 26 of 30 A 12 year old boys brought after collapsing while playing at his school Whie olaving he had difficulty breathing, beceme tired and fall to the ground without losing consciousness. At hospital, he appears lethargic and in mederate respiratory distress, with a respiratory rate of 30 per minute. Chest exam reveals decreased breath sound in all hing fields with coarse thoucki and wheezes throughout. ABG on 50% oxygen shows: pH=7.34; PCO2=38, PO2=55, bicerbonate=20. Which set of pulmonary function test result would likely be obtained in him? a) High FVC, high FEV, high TLC ) High FVC, high FFV1, low TLC ¢) Low FVC, high FEV1, low TLC 4 Low FVC, low FEVI, high TLC ¢) Low FVC, low FEVI, low TLC Answer | Boplanation Other User's Explanation Report An Error Question Explanation: This patient is having an acute asthma attack. Asthma is an obstructive lung disease primarily affecting air movement out of the lungs (@shalation). The airways (especialy the large bronchioles) are hypersensitive to inritating stimu, such as allergens and smoke. Local rast cell histamine release causes mucous secretion and smooth muscle contraction, leading to bronchoconstriction, Certain allergic triggers (cats, pollen, hay, etc) can precipitate attacks. Symptoms include coughing, dificulty breathing, and wheezing. Physical examination of the iungs often reveals wheezes and coarse lung sounds, although in severe cases one tray hear orly decreased breath sounds and ne wheezes dus to very poce sow. On pulmonary finction tests, asthmatice show an obstructive pattern. Both the FEV, (masimum volume exhaled in one second) and the FVC (maximum volume of air that can be exthaled in one breath) are decreased. The halimark of obstructive hing disease, however, is a decreased FEVEVC ratio, The total hing capacity (TLC) is often increaced ia acute astra attacke as patients tend to hyperinilate to obtain more oxygen whle being unable to extaale effcienty ‘Thus, this patient has a low FEV1, a low FVC, and a high TLC. 2:25/2014 1:49:00 PM Mark this question & => Question Id : 178983 Question 26 of 30 A 12 year old boys brought after collapsing while playing at his school Whie olaving he had difficulty breathing, beceme tired and fall to the ground without losing consciousness. At hospital, he appears lethargic and in mederate respiratory distress, with a respiratory rate of 30 per minute. Chest exam reveals decreased breath sound in all hing fields with coarse thoucki and wheezes throughout. ABG on 50% oxygen shows: pH=7.34; PCO2=38, PO2=55, bicerbonate=20. Which set of pulmonary function test result would likely be obtained in him? a) High FVC, high FEV, high TLC ) High FVC, high FFV1, low TLC ¢) Low FVC, high FEV1, low TLC Y © 4 Low FVC, low FEVI, high TLC ¢) Low FVC, low FEVI, low TLC Answer | Boplanation Other User's Explanation Report An Error Question Explanation: This patient is having an acute asthma attack. Asthma is an obstructive lung disease primarily affecting air movement out of the lungs (@shalation). The airways (especialy the large bronchioles) are hypersensitive to inritating stimu, such as allergens and smoke. Local rast cell histamine release causes mucous secretion and smooth muscle contraction, leading to bronchoconstriction, Certain allergic triggers (cats, pollen, hay, etc) can precipitate attacks. Symptoms include coughing, dificulty breathing, and wheezing. Physical examination of the iungs often reveals wheezes and coarse lung sounds, although in severe cases one tray hear orly decreased breath sounds and ne wheezes dus to very poce sow. On pulmonary finction tests, asthmatice show an obstructive pattern. Both the FEV, (masimum volume exhaled in one second) and the FVC (maximum volume of air that can be exthaled in one breath) are decreased. The halimark of obstructive hing disease, however, is a decreased FEVEVC ratio, The total hing capacity (TLC) is often increaced ia acute astra attacke as patients tend to hyperinilate to obtain more oxygen whle being unable to extaale effcienty ‘Thus, this patient has a low FEV1, a low FVC, and a high TLC. 2:25/2014 1:49:16 PM “Mark this question >= Question 27 of 30 A 63 year old woman presents with stridor associated with a retrostermal goitre. The most appropriate investigation of her airways, obstruction would be which one of the following? a) EVV ratio £) low-volume loop ©) Beak Expiratory Flow Rate 9 Spiromeiry @) Transfer factor Question Explanation: The flow volume loop is the best method of for detecting an obstruction associated with a retrosternal mass 2:25/2014 1:49:16 PM “Mark this question >= Question 27 of 30 A 63 year old woman presents with stridor associated with a retrostermal goitre. The most appropriate investigation of her airways, obstruction would be which one of the following? a) FEVUFYVC ratio ¥ © b) Flow-volume loop c) Peak Expiratory Flow Rate 9 Spiromeiry e) Transfer factor Question Explanation: The flow volume loop is the best method of for detecting an obstruction associated with a retrosternal mass 2:25/2014 1:19:37 PM ‘Mark this question << => Question 28 of 30 4.37 year old woman was successfully treated for Pneumocystis jirovecii pncumonia, Shs was re-admitted with acute breathlessness with left sided chest pain 10 days after discharge. Exam revealed hypoxia and she was found to have dittinished breath sounds on the left side of chest. The likely cause of her recent admission is which one of the following? a) Acate myocardial infarction b) Acute pericarditis ©) Acute pulmonary embolism 4) Left lobar pneumonia €) Pheumothorax Question Explanation: Pneumothorax is a well-known complication of PCP. An acute history of chest pain with breathlessness and dininished breath sounds is typical of pneumothorax, Diminished breath sounds are not a feature of acute myocardial infarction or acut= pericarditis, Acute puknonary embolism should be considered due to her recent edmission but diminished breath sounds are not ¢ feature There are no signs of consolidation to consider lobar pneumonia Report An Error 2:25/2014 1:19:37 PM ‘Mark this question << => Question 28 of 30 4.37 year old woman was successfully treated for Pneumocystis jirovecii pncumonia, Shs was re-admitted with acute breathlessness with left sided chest pain 10 days after discharge. Exam revealed hypoxia and she was found to have dittinished breath sounds on the left side of chest. The likely cause of her recent admission is which one of the following? a) Acate myocardial infarction b) Acute pericarditis ©) Acute pulmonary embolism 4) Left lobar pneumonia Y © ¢) Pheumothorax Question Explanation: Pneumothorax is a well-known complication of PCP. An acute history of chest pain with breathlessness and dininished breath sounds is typical of pneumothorax, Diminished breath sounds are not a feature of acute myocardial infarction or acut= pericarditis, Acute puknonary embolism should be considered due to her recent edmission but diminished breath sounds are not ¢ feature There are no signs of consolidation to consider lobar pneumonia Report An Error GO Mark this question = => ‘Question Td : 213800 Question 29 of 30 A 56-year-old alcoholic with severe shortness of breath is diagnosed with upper lobe pneumonia His respiratory fimction worsens and he is intubated and broad-spectrum antibiotics initated, Next morning, his saturation drops to 70% and he deteriorates. There are decreased breath sounds on the right side with dullness to percussion CXR is as follows “What is the most appropriate next step in management? a) Aggressive diuresis ») Bronchoscopy ©) Chest tube placement 4) Corticosteroid administration ¢) Thoracocentesis Anower [IEIRIRIRHBH) Other Users Explanation, Report An Evor Question Explanation: Because airway obstruction is the most lcely eticlogy ofthe acute hypoxia in this patient, as evidenced by the complete opacification of the right lung and dullness to percussion, bronchoscopy is most appropriate, Bronchoscopy allows for the removal of a mucus plug. If suction through the BT tube had been an answer choice for this question, that would have been the best next step, followed bby bronchoscopy if ET tube suction failed to resolve the hypoxia “Aggressive diuresis is helpful in patients with pulmonary edema Chest tube placement and thoracocentesis should be considered with pleural effusion or pneumotheras, but net in airway obstruction. ‘The present patient does not have evidence of a pneumothorax, which would have inchided hyperresonaace to percussion and blackout on chest x-ray. A pleural effision would also have been seen on the chest x-ray. a GO Mark this question = => ‘Question Td : 213800 Question 29 of 30 A 56-year-old alcoholic with severe shortness of breath is diagnosed with upper lobe pneumonia His respiratory fimction worsens and he is intubated and broad-spectrum antibiotics initated, Next morning, his saturation drops to 70% and he deteriorates. There are decreased breath sounds on the right side with dullness to percussion CXR is as follows “What is the most appropriate nest step in management? a) Aggressive diuresis Y © b) Bronchoscopy ©) Chest tube placement 4) Corticosteroid administration ¢) Thoracocentesis Anower [IEIRIRIRHBH) Other Users Explanation, Report An Evor Question Explanation: Because airway obstruction is the most lcely eticlogy ofthe acute hypoxia in this patient, as evidenced by the complete opacification of the right lung and dullness to percussion, bronchoscopy is most appropriate, Bronchoscopy allows for the removal of a mucus plug. If suction through the BT tube had been an answer choice for this question, that would have been the best next step, followed bby bronchoscopy if ET tube suction failed to resolve the hypoxia “Aggressive diuresis is helpful in patients with pulmonary edema Chest tube placement and thoracocentesis should be considered with pleural effusion or pneumotheras, but net in airway obstruction. ‘The present patient does not have evidence of a pneumothorax, which would have inchided hyperresonaace to percussion and blackout on chest x-ray. A pleural effision would also have been seen on the chest x-ray. a 2:25/2014 1:50:08 PM “Mark tis question Question 30 of 30 A 65-year-old man is currently recovering from an aortic valve replacement procedure. He gets a pulmonary embolus. He also has a tistory of hemorthagic stroke earlier. Whatis the accurate management? a) Caval ter +) Low molecular-weight heparin ©) Thrombolytic therapy ) Unlkactionated heparin ©) Uniractionated heparin end warfarin Question Explanation: ‘This patient has two absolute contraindications to anticoaguaton or thrombolytic therapy: recent surgery and a recent hemorrhagic stroke. He is in a dificult simation regarding his pulmonary embolus, as he is at high risk for a securrent event. Usually, recusrent clots can kill a patient, co come form of prophylactic therapy is necessary. In patients with cortraindications to anticoagulation, a vena caval filter (Greenfield fiter) is appropriate, Recent surgery and hemorhagic stroke are also a contraindication to thrombolytic, thesapy. Low molecular-weight heparin, unffactionated heparin and unfractionated heparin and warfarin would all be reasonable treatments to start in a patient who had no coniraindications to anticoagulation. Given the present patient's recent surgery, stroke, and septic emboli, however, anticoagulation is not appropriate. 2:25/2014 1:50:08 PM “Mark tis question Question 30 of 30 A 65-year-old man is currently recovering from an aortic valve replacement procedure. He gets a pulmonary embolus. He also has a tistory of hemorthagic stroke earlier. Whatis the accurate management? Y © a Caval fiter +) Low molecular-weight heparin ©) Thrombolytic therapy ) Unlkactionated heparin ©) Uniractionated heparin end warfarin Question Explanation: ‘This patient has two absolute contraindications to anticoaguaton or thrombolytic therapy: recent surgery and a recent hemorrhagic stroke. He is in a dificult simation regarding his pulmonary embolus, as he is at high risk for a securrent event. Usually, recusrent clots can kill a patient, co come form of prophylactic therapy is necessary. In patients with cortraindications to anticoagulation, a vena caval filter (Greenfield fiter) is appropriate, Recent surgery and hemorhagic stroke are also a contraindication to thrombolytic, thesapy. Low molecular-weight heparin, unffactionated heparin and unfractionated heparin and warfarin would all be reasonable treatments to start in a patient who had no coniraindications to anticoagulation. Given the present patient's recent surgery, stroke, and septic emboli, however, anticoagulation is not appropriate. 225/204 1:51:23 PM ‘Mark this question => Question Td : 28984 Question 1 of 30 ‘A 25-year-old woman complains of a 4-month old cough productive of mucoid sputum streaked with bright red blood, wheezing and diarthea, Her chest and abdominal examination is normal. You will proceed by? a) Chest X-ray +) Computed tomegraphy (CT) of chest ©) Echovardiogratn ® Bronchoscopy 8) Ceftriaxone+Ampicilin Question Explanation The scenario is of bronchial carcinoid which is a highly vascular ‘cherry-like’ mmour causing recurrent hemoptysis and bronchial obstruction It may rarely produce the classical syrmptoms of carcinoid syncrome such as cyanotic fushing, intestinal cramps and, diacrhea following liver metastases in 5% cases. Bronchoscopy identifies up to 80% of carcinoid tumours in the main bronchi, Biopey is usually followed with brisk blesding and should be done via rigid bronchoscopy. 225/204 1:51:23 PM ‘Mark this question => Question Td : 28984 Question 1 of 30 ‘A 25-year-old woman complains of a 4-month old cough productive of mucoid sputum streaked with bright red blood, wheezing and diarthea, Her chest and abdominal examination is normal. You will proceed by? a) Chest X-ray +) Computed tomegraphy (CT) of chest ©) Echovardiogratn Y © @ Bronchoscopy 8) Ceftriaxone+Ampicilin Question Explanation The scenario is of bronchial carcinoid which is a highly vascular ‘cherry-like’ mmour causing recurrent hemoptysis and bronchial obstruction It may rarely produce the classical syrmptoms of carcinoid syncrome such as cyanotic fushing, intestinal cramps and, diacrhea following liver metastases in 5% cases. Bronchoscopy identifies up to 80% of carcinoid tumours in the main bronchi, Biopey is usually followed with brisk blesding and should be done via rigid bronchoscopy. “Marke this question & => Question 2 of 30 A patient with Rheumatoid arthritic complains of progressive breathlessness. Which of the following is the most icely cause? a) Pulmonary Eosinephilia 1b) Hibrosing Aiveotitis c) Progressive Massive Fibrosis ) Centri-Acinar Emphysema ¢) Pulmonary Embolas newer [RESIEREA) other Users Explanation Report AnEvor Question Explanation: Hbrosing alveitis association with theumatoid arthritis is incistinguishable ftom cryptogeric Sbrosing alveolitis. It presents with progressive breathlessness and cough, Signs inchde finger clubbing, cyanosic and bilateral end inspiratory crackles. Other pulmonary complications of Rheumatoid arthits inckusle pleural effusions, empyema, cryptogenic organizing pneumonia, bronchiectasis and pulmonary nodules. The latter are usually asymptomatic but may cavitate resulting in hemoptysis and when occurring with coal hrocicers oneumorotiosis (Cuplans Syndrome) is associated with breathlessness: “Marke this question & => Question 2 of 30 ‘A patient wih Rheumatoid arthritis complains of progressive breathlessness. Which of the following is the most llcely cause? a) Pulmonary Eosinophilia J © b) Fibrosing Alveolitis ) Progressive Massive Fibrosis d) Centri-Acinar Emphysema ¢) Fulnonary Embolus newer [RESIEREA) other Users Explanation Report AnEvor Question Explanation: Hbrosing alveitis association with theumatoid arthritis is incistinguishable ftom cryptogeric Sbrosing alveolitis. It presents with progressive breathlessness and cough, Signs inchde finger clubbing, cyanosic and bilateral end inspiratory crackles. Other pulmonary complications of Rheumatoid arthits inckusle pleural effusions, empyema, cryptogenic organizing pneumonia, bronchiectasis and pulmonary nodules. The latter are usually asymptomatic but may cavitate resulting in hemoptysis and when occurring with coal hrocicers oneumorotiosis (Cuplans Syndrome) is associated with breathlessness: 2252014 1:51:53 PM ‘Mark this question —& => Question Td : 29805 Question 3 of 30 ‘You ate asked to see a patient who attends A +E with shortness of breath. The chest X-ray shows right lobe consolidation Which of the following features should prompt admission to hospital? a) A respiratory rate 32.mia ) Audible Bronchial breathing ©) A PaO2 of 98 kPa (11-13) 4) A Sa02 of 95% €) A white cell count of 16.82109/1 (4-10) Anower [REIRRRHBRY) Other Users Explanation Report An E01 Question Explanation: ‘The British Thoracic Society guideline for community acquired paeumonia in aduits recommends use of the CURB-65, A 6-point score, one point for each of Confusion, Urea =7 mmol/l, Respiratory rate 30imnin or more. systolic Blood pressure below 90rnmEg (or diastolic below 60mmHg). Age 65 years or older. Ifthe CURB score is 1-2 then risk of death is increased and hospital admission should be considered ACTIRE score of 3 or more puts the patient at high risk of death and hospital admission is warranted, 2252014 1:51:53 PM ‘Mark this question —& => Question Td : 29805 Question 3 of 30 ‘You ate asked to see a patient who attends A +E with shortness of breath. The chest X-ray shows right lobe consolidation Which of the following features should prompt admission to hospital? Y © a) A respiratory rate 32min ) Audible Bronchial breathing ©) A PaO2 of 98 kPa (11-13) 4) A Sa02 of 95% €) A white cell count of 16.82109/1 (4-10) Anower [REIRRRHBRY) Other Users Explanation Report An E01 Question Explanation: ‘The British Thoracic Society guideline for community acquired paeumonia in aduits recommends use of the CURB-65, A 6-point score, one point for each of Confusion, Urea =7 mmol/l, Respiratory rate 30imnin or more. systolic Blood pressure below 90rnmEg (or diastolic below 60mmHg). Age 65 years or older. Ifthe CURB score is 1-2 then risk of death is increased and hospital admission should be considered ACTIRE score of 3 or more puts the patient at high risk of death and hospital admission is warranted, 2i25/2014 1:52:06 PM Mark this question & => Question Id : 30028 Question 4 of 30 Tihas been shown by Randomized Controlled Trials that long-term oxygen therapy (LTOTD) reduces mortality in 2) Cryptogenic fibrosing alveolitis ) Asthma ©) Cystic fibrosis 4) Cor pulmonale due to chronic airlow obstruction ©) Pulmonary sarcoidosis Anowor (UEQVIRNBHEN) otter Usovs Explanation Report An Error Question Explanation: Adequate data for LTOT prolonging survival exist only for chronic obstructive pulmonary disease (COPD) although in practice itis assumed to apply in other chronic hypoxaemic lung conditions 2i25/2014 1:52:06 PM Mark this question & => Question Id : 30028 Question 4 of 30 Tihas been shown by Randomized Controlled Trials that long-term oxygen therapy (LTOTD) reduces mortality in 2) Cryptogenic fibrosing alveolitis ) Asthma ©) Cystic fibrosis Y © 6) Cor pulmonale dae to chronic airlow obstruction ©) Pulmonary sarcoidosis Anowor (UEQVIRNBHEN) otter Usovs Explanation Report An Error Question Explanation: Adequate data for LTOT prolonging survival exist only for chronic obstructive pulmonary disease (COPD) although in practice itis assumed to apply in other chronic hypoxaemic lung conditions 2:25/2014 1:52:21 PM “Mark this question & => Question 5 of 30 ‘Which beta blocker used to treat hypertension is contraindicated an asthina patient? a) Acebutolol ) Atenolol ©) Labetalol 4) Esmolol 8) Metoprolol Question Explanation: Beta blockers are either beta | selective where they only block the receptors on the heart. Or they can be non-selective and block: toth beta 1 and beta 2 receptors. Blockade of beta Ireceptors leads to a decrease in heart rate. Blockade offthe beta 2 receptor Ieads to bronchoconstrction. This should be avoided in asthmatic patients. Therefore itis best to orly give beta 1 selective beta lockers to asthma patients when treating their hypertension. This group includes the following four: acebutelol, atenolol, esmolol and metoprolol. n Report An Error 2:25/2014 1:52:21 PM “Mark this question & => Question 5 of 30 ‘Which beta blocker used to treat hypertension is contraindicated an asthma patient? 2) Acebutotol b) Atenolol Y © © Labetalol ¢) Esmolol €) Metoprolol Question Explanation: Beta blockers are either beta | selective where they only block the receptors on the heart. Or they can be non-selective and block: toth beta 1 and beta 2 receptors. Blockade of beta Ireceptors leads to a decrease in heart rate. Blockade offthe beta 2 receptor Ieads to bronchoconstrction. This should be avoided in asthmatic patients. Therefore itis best to orly give beta 1 selective beta lockers to asthma patients when treating their hypertension. This group includes the following four: acebutelol, atenolol, esmolol and metoprolol. n Report An Error Mat tis question & => Question Td : 49045, Question 6 of 30 A.75 year old woman is brought to the emergency room She is unresponsive and restless, Her breathing is laboured, Her fanily indicates that she has a history of confusion and dementia. They also indicate that she ingested “anti-freeze” from the garage. Blood gas resuits are: pH.7.09, PCO, 7 mmElg, PO, 133 mig, HCO, 2 mmol, O, saturation 97%. The interpretation of the blood gas status is a) Primary metabolic acidosis with respiratory compensation ) Primary metabolic acidesis with no compensation c) Primary respiratory acidosis with no metabolic compensation, ) Primary respiratory acidosis with metabolic compensation ©) Primary metabolic alkalosis with respiratory compensation Answer | Boplanation | Other User's Explanation Report An Error Question Explanation: The pHis less than 7.4. Therefore this patients in acidosis. Tae PCO, would be expected to be high (greater than 40) ifthe acidosis was due to a recpizatory cause (eg hypoventilation), Since these patiente PCO, i quite low, che ie in metabolic acidosis ‘The following table describes the various acidfbase states and the corresponding pH, CO and HCO;-(bicarb) values: PH |[PCO2|HCO; |ffxterpretion [7.40 [+24 [Respiratory acidosis with compensaied [<7.4]>40_|NormallRespiratory acidosis pure IS7.4)=40 |<24 [Metabolic acidosis with compensated [>7.4[<40 |<24 [Respiratory alkalosis with compensated 7.4]<40 [Normal Respiratory alkalosis pure P?4|40 [> 24 |[Mctabolic alkalosis with compensated In this patient. pH = 7.09, PCO =7 mraHlg, Bicarb = 2 (Overall change is acid. Respiratory change is alkaline, therefore NOT contsibuting to the acidosis. Metabolic change is acid, therefore responsible for acidosis. The components are puling in opposte directors, So itis primarily a metabolic problem with. somne respiratory compensation, Interpretation: metabolic acidosis with respiratory compensation. Mat tis question & => Question Td : 49045, Question 6 of 30 A.75 year old woman is brought to the emergency room She is unresponsive and restless, Her breathing is laboured, Her fanily indicates that she has a history of confusion and dementia. They also indicate that she ingested “anti-freeze” from the garage. Blood gas resuits are: pH.7.09, PCO, 7 mmElg, PO, 133 mig, HCO, 2 mmol, O, saturation 97%. The interpretation of the blood gas status is Y © a) Primary metabolic acidosis with respiratory compensation ) Primary metabolic acidesis with no compensation c) Primary respiratory acidosis with no metabolic compensation, ) Primary respiratory acidosis with metabolic compensation ©) Primary metabolic alkalosis with respiratory compensation Answer | Boplanation | Other User's Explanation Report An Error Question Explanation: The pHis less than 7.4. Therefore this patients in acidosis. Tae PCO, would be expected to be high (greater than 40) ifthe acidosis was due to a recpizatory cause (eg hypoventilation), Since these patiente PCO, i quite low, che ie in metabolic acidosis ‘The following table describes the various acidfbase states and the corresponding pH, CO and HCO;-(bicarb) values: PH |[PCO2|HCO; |ffxterpretion [7.40 [+24 [Respiratory acidosis with compensaied [<7.4]>40_|NormallRespiratory acidosis pure IS7.4)=40 |<24 [Metabolic acidosis with compensated [>7.4[<40 |<24 [Respiratory alkalosis with compensated 7.4]<40 [Normal Respiratory alkalosis pure P?4|40 [> 24 |[Mctabolic alkalosis with compensated In this patient. pH = 7.09, PCO =7 mraHlg, Bicarb = 2 (Overall change is acid. Respiratory change is alkaline, therefore NOT contsibuting to the acidosis. Metabolic change is acid, therefore responsible for acidosis. The components are puling in opposte directors, So itis primarily a metabolic problem with. somne respiratory compensation, Interpretation: metabolic acidosis with respiratory compensation. 2/25/2014 1:52:50 PM “Mark this question e& => Question Id : 54209 Question 7 of 30 A patient worked in silica mine for 20 years. Now he presents with 6 month history of cough and weightloss. CAR shows fibrosis of upper lobes. The most likely diagnosis is a) Pneumonia b) Tuberculosis c) Mesothelioma 4) Rapidly progressive silicosis Question Explanation Pulmonary tuberculosis is associated with silica dust exposure as an occupational disease. It is caused by Mycobacterium ‘tuberculosis in employees who have been exposed to crystalline silica dustin the workplace ‘Symptoms include cough, hemoptysis, fever, chills and weight loss. Diagnosis is made by sputum cultures and CXR, Treatment is usually with a combination of medicines such as Tsoniarid, Rifampin, Ethambutol and Pyrannamice Pacumonia would not be this long ofa duration and would not present with weight loss. Mesothelioma usually affects the lower lobes ‘with pleural thickening. Rapidly progressive silicosis develops within 6 months of acute exposure. 2/25/2014 1:52:50 PM “Mark this question e& => Question Id : 54209 Question 7 of 30 A patient worked in silica mine for 20 years. Now he presents with 6 month history of cough and weightloss. CKR shows abrosis of upper lobes. The most lely diagnosis is a) Pneumonia Y © b) Tuberculosis c) Mesothelioma 4) Rapidly progressive silicosis Question Explanation Pulmonary tuberculosis is associated with silica dust exposure as an occupational disease. It is caused by Mycobacterium ‘tuberculosis in employees who have been exposed to crystalline silica dustin the workplace ‘Symptoms include cough, hemoptysis, fever, chills and weight loss. Diagnosis is made by sputum cultures and CXR, Treatment is usually with a combination of medicines such as Tsoniarid, Rifampin, Ethambutol and Pyrannamice Pacumonia would not be this long ofa duration and would not present with weight loss. Mesothelioma usually affects the lower lobes ‘with pleural thickening. Rapidly progressive silicosis develops within 6 months of acute exposure. Mark this question & => Question Id : 5475: Question 8 of 30 False statement about COPD is 8) The type of emphysema accociated with amcking ie usvally centriacinar +) Chibbing is not a clinical feature 6) Long-term oral steroide chould be avoided 4) Smoking cessation does not lead to improvement of pulmonary function, 6) The aim of supplemental O; therapy is to provide relief of chortess of breath Anower [UBRPINEHER) otnor Usov' Exptan Question Explanation: Chronic obstructive pulmonary disease is partially reversible airflow obstruction caused by an inflammatory response to inhaled toxins, offen cigarette smoke, el -antitrypsin deficiency and several of occupational exposures are less cornmon causes in nonsmokers. Symptoms are productive cough and dyspnea that develop over years; common signs include decreased breath sounds, prolonged expiratory phase of respiration, and wheezing Long-ierm 0, therapy prolongs life in COPD patients whose PaQ3 is chronically < 55 mm Hg, Continual 24-h use is more effective than a 12-hnoctumal regimen, Q, therapy brings Hct toward normal levels; moderately improves neuropsycholegic factors, possibly try facilitating sleep; and amelicrates pulmonary hemodynamic abnormalities Oz therapy alzo increases exercise tolerance in many n _Roport An Error patients. Mark this question & => Question Id : 5475: Question 8 of 30 False statement about COPD is a) The type of emphysema associated with smoking is usually centriacinar +) Clubbing is not a clinical feature ©) Long-term oral steroids should be avoided Y © 4) Smokng cessation does not lead to improvement of pulmonary function €) The aim of supplemental O, therapy is to provide relief of chormese of breath Anower [UBRPINEHER) otnor Usov' Exptan Question Explanation: Chronic obstructive pulmonary disease is partially reversible airflow obstruction caused by an inflammatory response to inhaled toxins, offen cigarette smoke, el -antitrypsin deficiency and several of occupational exposures are less cornmon causes in nonsmokers. Symptoms are productive cough and dyspnea that develop over years; common signs include decreased breath sounds, prolonged expiratory phase of respiration, and wheezing Long-ierm 0, therapy prolongs life in COPD patients whose PaQ3 is chronically < 55 mm Hg, Continual 24-h use is more effective than a 12-hnoctumal regimen, Q, therapy brings Hct toward normal levels; moderately improves neuropsycholegic factors, possibly try facilitating sleep; and amelicrates pulmonary hemodynamic abnormalities Oz therapy alzo increases exercise tolerance in many n _Roport An Error patients. 2/25/2014 1:53:19 PM ‘Mark this question & => Question Td : 55733 Question 9 of 30 A 22 year old man presents with respiratory distress, He has a severe acute asthmatic attack. In addition to oxygen, which one of the following is the mast important medication he requires? a) Morphine +) Corticosteroids c) Montehucast sodium 4) Albuterol £) Sodium cromoglycate Question Explanation: ‘An asthma attack can be frightening, both to the person experiencing it and to others around, Even when relatively mild, the symptoms provoke antety and alarm. A severe asthma attack is a life threatening emergency that recuites immediete, skilled, professional care, Fnot ireated adequately and quickly, a severe asthma attack can cause death. Supplemental oxygen may be given during aitacks. However, in severe attacks, a doctor also needs to monitor carbon dioxide levels, and this test requires a sample of blood from an artery. Short acting beta-adrenergic agonists (e.g., albuterol) are usually the best drugs for relieving asthma attacks Most beta-receptor agonists, especially the inhaled ones, act within minutes, but the effects last only 2 to 6 hours. They are given by mouth in higher doses for people experiencing severe attacks. Corticosteroids given by mouth are generally continued for at least several days after a severe attack, 2/25/2014 1:53:19 PM ‘Mark this question & => Question Td : 55733 Question 9 of 30 A 22 year old man presents with respiratory distress, He has a severe acute asthmatic attack. In addition to oxygen, which one of the following is the mast important medication he requires? a) Morphine +) Corticosteroids c) Montehucast sodium Y © @) Albuterol £) Sodium cromoglycate Question Explanation: ‘An asthma attack can be frightening, both to the person experiencing it and to others around, Even when relatively mild, the symptoms provoke antety and alarm. A severe asthma attack is a life threatening emergency that recuites immediete, skilled, professional care, Fnot ireated adequately and quickly, a severe asthma attack can cause death. Supplemental oxygen may be given during aitacks. However, in severe attacks, a doctor also needs to monitor carbon dioxide levels, and this test requires a sample of blood from an artery. Short acting beta-adrenergic agonists (e.g., albuterol) are usually the best drugs for relieving asthma attacks Most beta-receptor agonists, especially the inhaled ones, act within minutes, but the effects last only 2 to 6 hours. They are given by mouth in higher doses for people experiencing severe attacks. Corticosteroids given by mouth are generally continued for at least several days after a severe attack, 2/25/2014 1:53:32 PM ‘Mark this question & => Question Ti : 59581 Question 10 of 30 Sleep apneais known to be associated with which of the following? a) Respiratory muscle dysfimction ) Hypertension ¢) Carpal tunnel syndrome ) Hypercalcemia ©) Previous tonsillectomy Avewor (UERVIRNSEN) otnorUrore Explanation Report An Error Question Explanation: Sleep apnea is associated with hypertension, and may in fact be a cause ofhypertension Improvement of sleep apnea cen be associated with improved blood pressure. 2/25/2014 1:53:32 PM ‘Mark this question & => Question Ti : 59581 Question 10 of 30 Sleep apneais knovn to be associated with which of the following? a) Respiratory muscle dysfimction © b) Hypertension ¢) Carpal tunnel eyndrome 6 Hypercalcemia ©) Previous tonsillectomy Avewor (UERVIRNSEN) otnorUrore Explanation Report An Error Question Explanation: Sleep apnea is associated with hypertension, and may in fact be a cause ofhypertension Improvement of sleep apnea cen be associated with improved blood pressure. ‘Mark this question e& => Question Td : 64376 Question 11 of 30 A.36 year old man with a recent history of severe rmultiple traumas to chest have dyspnea andis diagnosed as having adult respiratory distress syndrome (ARDS) Which of the following statement is false about the management of ARDS? a) Management may inclide artificial ventilation with endotracheal intubation and oxygen therapy ') A sudden deterioration in 2 ventlated patient should raise the suspicion of pneumothorast ¢) Early management may inclde oxygen mask by careful monitoring of respiratory rate and arterial blood gases 4) Furosemide is routinely used to keep the Ings as dry as possible ¢) Frequent asterial blood gas determinations wil be required to assess the adequacy of therapy in any ventlated petient Question Explanation: People with ARDS are treated in an intensive care uni. Successful treatment depends on correcting the underlying cause; oxygen thesapy, which is vial to correcting low oxygen levels, is combined with treatment of the underlying cause. Ff oxygen delivered by a face mack or nasal prongs does act correct the low blood oxygen levels, or if excessively hich doses of inhaled oxygen are recuired, a ventilator must be used, this treatment is called mechanical ventilation, A ventilator delivers oxygen-rich air under pressure using a tube inserted through the mouth into the trachea, ‘The pressure supplied by the ventilator during and after a breath opens colapsed (atelectatic) regions of the lung aad allows oxygen to move through the walls of the injured lungs inte the blood. The pressure and volume of air that the ventilator delivers to the lungs with each breath must be adjusted to help keep the small airways and alveoli open while avoiding rupturing the fragile air sacs, which can lead to air accumulating around the lung and collapsing t (called pneumothorax’). Monitoring and adjusting the pressure also ensures that the lungs do not receive an excessive concentration of oxygen, which can damage the lungs and worsen ARDS. In some cases. diuretic drags may be needed to help remove fluid from the fangs. Antibiotics are usvally needed for people who develop bacterial pneumonia, Some people may benefit fom the use of intravenous corticosteroids in the later stages of ARDS, ‘Mark this question e& => Question Td : 64376 Question 11 of 30 A.36 year old man with a recenthistory of severe multiple traumas to chest have dyspnea andis diagnosed as having adult respiratory distress syndrome (ARDS) Which of the fellowing statement is false about the rranagement of ARDS? a) Management may inchude artificial ventilation with endotracheal intubation and oxygen therapy ') A sudden deterioration in a ventlated patient should raise the suspicion of pneumothorasz ¢) Barly management may include oxygen mack by carefll monitoring of respiratory rate and arterial blood gases Y © 4) Furosemide is routinely used to keep the lungs as dry as possible «) Frequent arterial blood gas determinations wil be required to assess the adequacy of therapy in any ventlated patient Question Explanation: People with ARDS are treated in an intensive care uni. Successful treatment depends on correcting the underlying cause; oxygen thesapy, which is vial to correcting low oxygen levels, is combined with treatment of the underlying cause. Ff oxygen delivered by a face mack or nasal prongs does act correct the low blood oxygen levels, or if excessively hich doses of inhaled oxygen are recuired, a ventilator must be used, this treatment is called mechanical ventilation, A ventilator delivers oxygen-rich air under pressure using a tube inserted through the mouth into the trachea, ‘The pressure supplied by the ventilator during and after a breath opens colapsed (atelectatic) regions of the lung aad allows oxygen to move through the walls of the injured lungs inte the blood. The pressure and volume of air that the ventilator delivers to the lungs with each breath must be adjusted to help keep the small airways and alveoli open while avoiding rupturing the fragile air sacs, which can lead to air accumulating around the lung and collapsing t (called pneumothorax’). Monitoring and adjusting the pressure also ensures that the lungs do not receive an excessive concentration of oxygen, which can damage the lungs and worsen ARDS. In some cases. diuretic drags may be needed to help remove fluid from the fangs. Antibiotics are usvally needed for people who develop bacterial pneumonia, Some people may benefit fom the use of intravenous corticosteroids in the later stages of ARDS, ‘Mark this question e& => Question Id : 64552 Question 12 of 30 A 36 year old man consults you about a vague chest pain he developed while sitting at his desk earlier in the day. ‘The pain is sight- sided and was sharp for abrieftime when t began, but it rapidly subsided. There was no hemoptysis and the pain does not seem plewitis. His physical examination, EEG, and oxygen saturation are unremarkable. A chect fm shows a 10% right preumcthoras. ‘True statement regarding this sitzation is a) He is licely to be an overweight smoker with a chrenic cough ') Rupture of subpleural bullae would be an unlikely cause of his problem A chest tube should be placed expedtiously 4) Outpatient observation with a repeat chest radiograph in 24 hours is indicated ) After treatment his probability of recurrence is less than 15% Question Explanation: ‘The majonty of patients presenting with spontaneous pneumothorax are tall, thin individuals under 40 years of age. Most do not have clinically apparent lung disease, and the chest pain is sometimes minimal at onset and may resolve within 24 hours even if untreated, Patients with small pneumothorax involving less thaa 15% of the hemithoraxx may have a normal physical examination, although tachycardia is occasionally noted. The diagnosis is confirmed by chestradiographs. Studies of recurrence have found that an average of 30% of patients will have a recurrence within 6 months to 2 years. The treatment of an initial pneumothorax of less than 20% may be monitored if'a patient has few symptoms. Follow up should inciude a chest radiograph to assess stability at 24-48 hours Indications for treatment include progression, delayed expansion, or the development of symptoms. The majceity of pationts with spontaneous pneumothoraces, and perhaps aimost all ofthem, will have subcutaneous bulae on a CT scat. o ) Report An Error ‘Mark this question e& => Question Id : 64552 Question 12 of 30 A 36 year old man consults you about a vague chest pain he developed while sitting at his desk earlier in the day. ‘The pain is sight- sided and was sharp for abrieftime when t began, but it rapidly subsided. There was no hemoptysis and the pain does not seem plewitis. His physical examination, EEG, and oxygen saturation are unremarkable. A chect fm shows a 10% right preumcthoras. ‘True statement regarding this sitzation is a) He is licely to be an overweight smoker with a chrenic cough ') Rupture of subpleural bullae would be an unlikely cause of his problem ©) A chest tube should be placed expeditiously V © 4) Outpatient cbservation with a repeat chest radiograph in 24 hours is indicated ©) After treatment his probability of recurrence is less then 15% Question Explanation: ‘The majonty of patients presenting with spontaneous pneumothorax are tall, thin individuals under 40 years of age. Most do not have clinically apparent lung disease, and the chest pain is sometimes minimal at onset and may resolve within 24 hours even if untreated, Patients with small pneumothorax involving less thaa 15% of the hemithoraxx may have a normal physical examination, although tachycardia is occasionally noted. The diagnosis is confirmed by chestradiographs. Studies of recurrence have found that an average of 30% of patients will have a recurrence within 6 months to 2 years. The treatment of an initial pneumothorax of less than 20% may be monitored if'a patient has few symptoms. Follow up should inciude a chest radiograph to assess stability at 24-48 hours Indications for treatment include progression, delayed expansion, or the development of symptoms. The majceity of pationts with spontaneous pneumothoraces, and perhaps aimost all ofthem, will have subcutaneous bulae on a CT scat. Report An Error 225/204 1:54:17 PM Mark this question = => Question 13 of 30 Formation of an anaerobic lang abscess is characterized by all of the following EXCEPT 2) Nonnal oropharyngeal flora are the typical pathogens. +) Aspiration of upper away contents is common, ©) Periodontal infection is present. &) In healthy persons, coughing assists in clearing aspirated upper airway contents, ©) The upright position favors involvement of the superior segments of the lower lobes Question Explanation: Nearly all anaerobic hing abscesses involve normal fora of the oropharynx. Predisposing factors include periodontal disease and aspration of upper airway contents. In heathy persons, coughing, as well as other pulmonary defense mechanisms assistin clearing the small inocula of normally aspirated upper airway contents. Dependent pulmonary segments are the typical sites of involvement. “Aspiration occurring in the upright position favors involvement of the basilar segmerts of the lower lobes; recumbency favors either the superior segments of the lower lobes or the posterior segments of the upper lobes. Report An Error 225/204 1:54:17 PM Mark this question = => Question 13 of 30 Formation of an anaerobic lang abscess is characterized by all ofthe following EXCEPT 2) Nomnal oropharyngeal flora are the typical pathogens. +) Aspiration of upper away contents is common, ©) Periodontal infection is present 6) In healthy persons. coughing assists in clearing aspirated upper airway contents Y © © The upright position favors involvement of the superior segments of the lower lobes. Question Explanation: Nearly all anaerobic hing abscesses involve normal fora of the oropharynx. Predisposing factors include periodontal disease and aspration of upper airway contents. In heathy persons, coughing, as well as other pulmonary defense mechanisms assistin clearing the small inocula of normally aspirated upper airway contents. Dependent pulmonary segments are the typical sites of involvement. “Aspiration occurring in the upright position favors involvement of the basilar segmerts of the lower lobes; recumbency favors either the superior segments of the lower lobes or the posterior segments of the upper lobes. Report An Error 2252014 1:54.28 PM ‘Mark this question & => Question Td : 82076 Question 14 of 30 All ofthe following are factors detesmining the extent of pulmonary injury in gastric acid aspiration, EXCEPT a) PH of the aspirate. ') Presence of food particles ¢) Volume of the aspirate 4) Distribution of the aspire ©) Overgrowtt of normal oropharyngeal flora Answer | Baplanation Other User's Explanation Report An Error Question Explanation: ‘The majority of notnal flora of the oropharyngeal cavity has little invasiveness in the normal host. Overgrowth of this area with pathogenic bacteria precio oses to paeumonia (B). The extent ofillaece caused by aspiration of gastric content ic determined by several factors. The acidity of the aspirated materia is the most important factor, with a pH <2.5 being proposed as a cut-off value (A). Aspiration of food particles causes severe pneumonitis (B). Aspirating more than 0.4 mllkg body-weight of gastric acid is sufficient to cause pneumoritis (C), Although coughing is protective, in same cases it enhances dispersion of acid over a greater pulnonary surface area (D). 2252014 1:54.28 PM ‘Mark this question e& => Question Td : 82076 Question 14 of 30 All ofthe following are factors detesmining the extent of pulmonary injury in gastric acid aspiration, EXCEPT a) PH of the aspirate. b) Presence of food particles. ) Volume of the aspirate 4) Distribution of the aspirate. Y © €) Overgrowth of normal oropharyngeal flora, Answer | Baplanation Other User's Explanation Report An Error Question Explanation: ‘The majority of notnal flora of the oropharyngeal cavity has little invasiveness in the normal host. Overgrowth of this area with pathogenic bacteria precio oses to paeumonia (B). The extent ofillaece caused by aspiration of gastric content ic determined by several factors. The acidity of the aspirated materia is the most important factor, with a pH <2.5 being proposed as a cut-off value (A). Aspiration of food particles causes severe pneumonitis (B). Aspirating more than 0.4 mllkg body-weight of gastric acid is sufficient to cause pneumoritis (C), Although coughing is protective, in same cases it enhances dispersion of acid over a greater pulnonary surface area (D). Mark this question & => Question Td : 90376 Question 15 of 30 Tn the case of an incompetent patient who receives life sustaining treatment and a surrogate decision maker is identified, which of the following sitation may nct require intervention in the decision making process? 2) A céspute occurs among the family members, and no decision maker is designated in an advance directive 4b) A health care provider ie confident that the decision made by the surrogate decision maker is not in the patients hest interests ) No family members are willing to be the patient's surrogate decision maker. 4) There is no adequate evidence of the incompetent patient's preferences and values. €) A health care provider is confident that the cecision made by the surrogate decision maker is not whet the patient would have decided, had he or she been competent: Question Explanation: Choices A.B. C, and E. are all simations in which institutional or judicial review and/or intervention may be required in the decision- making process. Choice D alone is not enough to require the need for intervention unless the situation is accompanied by a dispute which develops among the fanily members and the primary caregiver. Mark this question & => Question Td : 90376 Question 15 of 30 Tn the case of an incompetent patient who receives life sustaining treatment and a surrogate decision maker is identified, which of the following sitation may nct require intervention in the decision making process? 2) A céspute occurs among the family members, and no decision maker is designated in an advance directive 4b) A health care provider ie confident that the decision made by the surrogate decision maker is not in the patients hest interests ) No family members are willing to be the patient's surrogate decision maker. ¥ © 4) There is no adequate evidence of the incompetent patient's preferences and values. €) A health care provider is confident that the cecision made by the surrogate decision maker is not whet the patient would have decided, had he or she been competent: Question Explanation: Choices A.B. C, and E. are all simations in which institutional or judicial review and/or intervention may be required in the decision- making process. Choice D alone is not enough to require the need for intervention unless the situation is accompanied by a dispute which develops among the fanily members and the primary caregiver. 2:25/2014 1:54:56 PM Mark this question @&-_> Question Id : 91789 Question 16 of 30 A156 year old male undergoes a thoracentesis for a left pleural effusion. The pH of the fluidis 6.90 and the fluid is found to be an exaidate, What is the appropriate next step in management? a) Observation, ) Immediete drainage via chest tube ¢) Repeat thoracentesis. 4) Diuresis ¢) Left pneumonectomy. Anower [UBIRIBEN) ter veors Explanai Question Explanation: “An exadative pleural effisicn with a pH < 7.1 is consistent with an empyema The treatment of choice is immediate drainage “Antibiotics should also be started concurrently (although antibiotics alone are insufficient). Observation (A), repeat thoracentesis (C), diuresis (D), and pneumonectomy (E) are definitely NOT indicated. Report An Error 2:25/2014 1:54:56 PM Mark this question @&-_> Question Id : 91789 Question 16 of 30 A156 year old male undergoes a thoracentesis for a left pleural effusion. The pH of the fluidis 6.90 and the fluid is found to be an exaidate, What is the appropriate next step in management? a) Observation, Y © b) Immediete drainage via chest tube ¢) Repeat thoracentesis. 4) Diuresis ¢) Left pneumonectomy. Anower [UBIRIBEN) ter veors Explanai Question Explanation: “An exadative pleural effisicn with a pH < 7.1 is consistent with an empyema The treatment of choice is immediate drainage “Antibiotics should also be started concurrently (although antibiotics alone are insufficient). Observation (A), repeat thoracentesis (C), diuresis (D), and pneumonectomy (E) are definitely NOT indicated. Report An Error ‘Mark this question & => Question Ti : 93874 Question 17 of 30 A patient with serve COPD is being evaliated by the physician prior to oxygen supplementation. The physician decides to draw arterial blood for blood gas stadies. He introduces room air into the syringe while pulling on the plunger as he is drawing the syringe out of the patient, The patterns of changes that would be most likely to be produced by this exposure of arterial blood to room ai is a) PO2 Decreased), PCO2 Decreased, pH Decreased) ») PO2 Decreased), PCO2 (Elevated), pH (Decreased) ©) PO2 Elevated), PCO2 Decreased), pH Decreared) a) PO2 Clevated, PCO2 (Decreased), pH Elevated) Answer (Explanation | Other User's Explanation Report An Error Question Explanation: The technical part of the collection of arterial blood samples is dificult. Some hospitals allow only physicians to collect the samples, while other hospitals allow nurses or technicians with additional special training to collect the samples. No matter who performs the arterial draw, care must be taken to avoid exposing the blood to room air, as such exposure tends to cause the blood to partially eqnilibrate with the room ait. Room air would have a higher PO2 and a lower PCO2 than this patient's blood, so the sample would have a higher PO2 and a lower PCO2. In the atmosphere, PO2=150 mim Hg and PCO2 is near 0 mmiHig higher in the arterial blood ofa healthy patient, PO2 = 100mm Hg, PCO2 =40 mma Hg PO2 could be lower and PCO? higher in a ciceased individual) Because CO2 is decreased, there will be less carbonic acid present in the blood, thus raising the pH. ‘Mark this question & => Question Ti : 93874 Question 17 of 30 A patient with serve COPD is being evaliated by the physician prior to oxygen supplementation. The physician decides to draw arterial blood for blood gas stadies. He introduces room air into the syringe while pulling on the plunger as he is drawing the syringe out of the patient, The patterns of changes that would be most likely to be produced by this exposure of arterial blood to room ai is a) PO2 Decreased), PCO2 Decreased, pH Decreased) ») PO2 Decreased), PCO2 (Elevated), pH (Decreased) ©) PO2 Elevated), PCO2 Decreased), pH Decreared) Y © 4) P02 Clevated, PCO2 (Decreased), pH Elevated) Answer (Explanation | Other User's Explanation Report An Error Question Explanation: The technical part of the collection of arterial blood samples is dificult. Some hospitals allow only physicians to collect the samples, while other hospitals allow nurses or technicians with additional special training to collect the samples. No matter who performs the arterial draw, care must be taken to avoid exposing the blood to room air, as such exposure tends to cause the blood to partially eqnilibrate with the room ait. Room air would have a higher PO2 and a lower PCO2 than this patient's blood, so the sample would have a higher PO2 and a lower PCO2. In the atmosphere, PO2=150 mim Hg and PCO2 is near 0 mmiHig higher in the arterial blood ofa healthy patient, PO2 = 100mm Hg, PCO2 =40 mma Hg PO2 could be lower and PCO? higher in a ciceased individual) Because CO2 is decreased, there will be less carbonic acid present in the blood, thus raising the pH. ‘Marke this question e& => Question Td = 96266 Question 18 of 30 A patient presents with chest wall pain, Which of the following is the most likely diagnosis? a) Teetze’s syndrome. ) Reiter's syndrome Q 9 ‘Ward-Romano syndrome Cryoglobulinemia, Sarcoidosis Answer [Botanaton | Other User's Explanation Report An Error Question Explanation: Reiter’s syndrome is an inflammatory condition that can present with urethritis, uveitis, and arthritis. Tietze's syndrome is an inflammatory costochondriti, demonstrated by chest wall pain. Ward Romano presents as long QT syndrome. Cryoglobulinemia can present with a vasculitis and glomervlouephiitis Sarcoidosis presents with hilar adenopathy and eye findings e ‘Marke this question e& => Question Td = 96266 Question 18 of 30 A patient presents with chest wall pain. Which of the followings the mast likely diagnosis? Y © a) Tietze's syndrome. b Reiter's syndrome Q 9 ‘Ward-Romano syndrome Cryoglobulinemia, Sarcoidosis Answer [Botanaton | Other User's Explanation Report An Error Question Explanation: Reiter’s syndrome is an inflammatory condition that can present with urethritis, uveitis, and arthritis. Tietze's syndrome is an inflammatory costochondriti, demonstrated by chest wall pain. Ward Romano presents as long QT syndrome. Cryoglobulinemia can present with a vasculitis and glomervlouephiitis Sarcoidosis presents with hilar adenopathy and eye findings e 2/25/2014 1:55:11 PM “Mark this question €& => Question 19 of 30 ‘Which of the following is NOT a feature of a transudative pleural effusion? a) Pleural hid protein serum protein < 0.5 ) Pleural fitid LDH: serum LDH < 0.5, ©) Pleural fid LDH < 2/3 the upper normal limit for serum, «) All ofthe above are features of a transudative pleural effusion 6) Allofthe above are features of an exndative pleural effision Question Explanation: Features of a transudetive pleural effusion include a pleural faid protein to sesum protein ratio of less than 0.5, pleural fluid LDH to serum LDE ratio of less than 0.6, and pleural fuid LDH < 2/3 the upper normal limit for serum, An emidate has the opposite features 2/25/2014 1:55:11 PM “Mark this question €& => Question 19 of 30 ‘Which of the following is NOT a feature of a transudative pleural effusion? 4) Pleural fi protein: serum protein < 0.5. ») Pleural fuid LDEE serum LDH < 0.56 6) Pleural fnid LDH < 2/3 the upper normal limit for serum. ¥ © 4) Allofthe above are features of a transudative pleural effusion, «) All ofthe above are features of an exudative pleural effision Question Explanation: Features of a transudetive pleural effusion include a pleural faid protein to sesum protein ratio of less than 0.5, pleural fluid LDH to serum LDE ratio of less than 0.6, and pleural fuid LDH < 2/3 the upper normal limit for serum, An emidate has the opposite features Mark this question & => Question Td : 114750 Question 20 of 30 ‘All of the folowing conditions are associated with sleep apnea, EXCEPT 2) Chronic obstructive pulmonary disease 'b) Muscular dystrophy ©) Pharyngeal tumors 4) Microglossia ©) Alcoholism Question Explanation: Sleep apnea is an intermittent cessation of airflow at the mouth and nose during sleep. Ttmay result from either obstructive ot central abnormalities. Any lesion that obstructs upper airway flow, such as pharyngeal tumors, tonsillar hypertrophy, or obesity, mey produce sleep apnea. The airway may also be narrowed by structural abnormalities, such as retrognathia and macroglossia, Alcchol finctions as a co-factor because i depresses the fnnction of the muscles of the upper airway. In central-sleep apnea, there is 2 transient abolition of the central drive to the ventilatory muscles. Ultimately. there is a bhinting ofthe brain stem sensitivity to hypercarbia hypoxia. Peripheral neuromuscular disorders may alse produce weakness in the respiratory muscles, which are often combined with desensitization of the brainstem respiratory centers Report An Error Mark this question & => Question Td : 114750 Question 20 of 30 All of the folowing conditions are associaied with sleep apnea, EXCEPT a) Chronic obstructive pulmonary disease +b) Muscular dystrophy ©) Pharyngeal tumors Y © 4) Microglossia ©) Aleoholiem Question Explanation: Sleep apnea is an intermittent cessation of airflow at the mouth and nose during sleep. Ttmay result from either obstructive ot central abnormalities. Any lesion that obstructs upper airway flow, such as pharyngeal tumors, tonsillar hypertrophy, or obesity, mey produce sleep apnea. The airway may also be narrowed by structural abnormalities, such as retrognathia and macroglossia, Alcchol finctions as a co-factor because i depresses the fnnction of the muscles of the upper airway. In central-sleep apnea, there is 2 transient abolition of the central drive to the ventilatory muscles. Ultimately. there is a bhinting ofthe brain stem sensitivity to hypercarbia hypoxia. Peripheral neuromuscular disorders may alse produce weakness in the respiratory muscles, which are often combined with desensitization of the brainstem respiratory centers Report An Error 2:25/2014 1:56:06 PM ‘Mark this question ys Question Id : 114939 Question 21 of 30 A.25 year old woman presents with nght sweats, hemoptysis, fever, weight loss, and anorexia. Her PPD is greater than 12 min indurated. She lives in an urban shetter. The next appropriate step in management is a) Isoniazid and vitamin B6 ») Place the patient in an isolation room, but there is no need for masks «) Radiation therapy @) Obtain sputum for acid fast bacilli smears and cultures. ¢) Rifampin Anowor (UBQARESHEH) ner veers Explanation Report An Exot Question Explanation: This patient most likely has Mycobacteria tuberculosis, which can spread via the respiratory route if she is not isolated. She does not need chemoprophylaxis, but tiple drug treatment. Obtaining sputum for cubures and sensitivities can guide treatment of this disease, especially in this day of raubidrug resistance. Adding isoniazid alone with vitarrin B6 wall only permit furure resistance, Placing the patient in an isolation soom until the diagnosis is established is prudent, along with personal respiratory protective devices Radiation therapy is contraindicated in pulmonary tuberculosis. IFthe patient had lung cancer, this would be a good choice. Rifampin alone is not a good choice, triple anti-ruberculosis therapy is indicated in suspected pulmonary tuberculosis. 2:25/2014 1:56:06 PM ‘Mark this question ys Question Id : 114939 Question 21 of 30 A 25 year old woman presents with nght sweats, hemoptysis, fever, weight loss, and anoresia Her PPD is greater than 12 mm indurated. She lives in an urban shetter. The next appropriate step in management is a) Isoniazid and vitamin B6 ) Place the patient in an isoletion room, but there is no need for masks. ©) Radiation therapy Y © 4) Obtan sputum for acid fast bacilli smears and cultures. ¢) Rifampin Anowor (UBQARESHEH) ner veers Explanation Report An Exot Question Explanation: This patient most likely has Mycobacteria tuberculosis, which can spread via the respiratory route if she is not isolated. She does not need chemoprophylaxis, but tiple drug treatment. Obtaining sputum for cubures and sensitivities can guide treatment of this disease, especially in this day of raubidrug resistance. Adding isoniazid alone with vitarrin B6 wall only permit furure resistance, Placing the patient in an isolation soom until the diagnosis is established is prudent, along with personal respiratory protective devices Radiation therapy is contraindicated in pulmonary tuberculosis. IFthe patient had lung cancer, this would be a good choice. Rifampin alone is not a good choice, triple anti-ruberculosis therapy is indicated in suspected pulmonary tuberculosis. Mark this question = => Question Td : 118461 Question 22 of 30 A.21 year cld nursing studeat was PPD negative before entering nursing schocl. She is found to be PPD positive after her frst ward rotation, What should be the first action to be taken by the Student Health physician who sees her? 2) Repest the PED. ‘b) Check her liver functions. ©) Obtain a sputum sample for culture 4) Order a chest radiograph. 6) Obtain a urine culture Question Explanation: Tn a young indwidual who is known to have converted from a negative to a positive PED, a chest radiograph should be obtained to ‘identify the presence of active pulmonary disease, which would affect the choice of therapy Repeating the PPD is not advised, since a false positive would be unusual and repeating a positive test would likely cause the patient a great deal of discomfort atthe sto of antigen application. There is no reason to check her liver functions at this point in time. Such testing would be beneficial on a regular tbasis once treatment is inticted to identify early adverse hepatic reactions to drugs. A sputumn sample for cuiture would not be helpful at this time, since it ic unlikely to be positive and would take several weeks to grow. Such an individual would not be expected to have infection in her kidneys or bladder, so a urine culture would not be indicated, Report An Error Mark this question = => Question Td : 118461 Question 22 of 30 A.21 year old mursing student was PPD negative before entering nursing school, She is found to be PPD positive after her first ward rotation. What should be the fist action to be taken by the Student Health physician who cees hee? 2) Repest the PED. +) Check her liver functions. ©) Obtain a sputum sample for culture VM © & Order a chest radiograph. 6) Obtain a urine culture Question Explanation: Tn a young indwidual who is known to have converted from a negative to a positive PED, a chest radiograph should be obtained to ‘identify the presence of active pulmonary disease, which would affect the choice of therapy Repeating the PPD is not advised, since a false positive would be unusual and repeating a positive test would likely cause the patient a great deal of discomfort atthe sto of antigen application. There is no reason to check her liver functions at this point in time. Such testing would be beneficial on a regular tbasis once treatment is inticted to identify early adverse hepatic reactions to drugs. A sputumn sample for cuiture would not be helpful at this time, since it ic unlikely to be positive and would take several weeks to grow. Such an individual would not be expected to have infection in her kidneys or bladder, so a urine culture would not be indicated, Report An Error ‘Mark this question & => Question Td : 136756 Question 23 of 30 “Which of the following does NOT predispose an otherwise normal host to infection by Pseudomonas aenuginosa? a) Breach of cutaneous or mucosal barriers ) Compromized immunologic defense mechanisms ¢) Disruption of protective function of normal bacterial 4) Exposure to a hospital environment e) Exposure to a person with a drug resistant abscess Answer | Bolanation Other User's Explanation Report An Error Question Explanation: ‘An otherwise healthy person is rarely infected with Pseudomonas aeruginosa unless there has been a breach of cutaneous or nmcosal bamiers, such as that occurring with burns or postoperatively. Compromise of immunclogic defense mechanisms, such as that occurring with diabetes meliins, cancer, or AIDS, also predisposes to infection, Disruption of the normal protective flora, such as occurs following treatment with broad spectrum antibiotics, is another factor predisposing to infection, Exposure to reservoirs of the organism, as occurring in the hospital environment, may also predispose to infection, although briaf exposure to an infected person is nota tisk factor for infection. ‘Mark this question & => Question Td : 136756 Question 23 of 30 “Which of the following does NOT predispose an otherwise normal host to infection by Pseudomonas aenuginosa? a) Breach of cutaneous or mucosal barriers ) Compromized immunologic defense mechanisms ¢) Disruption of protective function of normal bacterial 4) Exposure to a hospital environment ¥ © ) Exposure to a person with a drug resistant abscess Answer | Bolanation Other User's Explanation Report An Error Question Explanation: ‘An otherwise healthy person is rarely infected with Pseudomonas aeruginosa unless there has been a breach of cutaneous or nmcosal bamiers, such as that occurring with burns or postoperatively. Compromise of immunclogic defense mechanisms, such as that occurring with diabetes meliins, cancer, or AIDS, also predisposes to infection, Disruption of the normal protective flora, such as occurs following treatment with broad spectrum antibiotics, is another factor predisposing to infection, Exposure to reservoirs of the organism, as occurring in the hospital environment, may also predispose to infection, although briaf exposure to an infected person is nota tisk factor for infection. 2i25/2014 1:56:55 PM ‘Mark this question & => Question Id : 139020 Question 24 of 30 Aman is kiled in RTA. He had a history of fever and chronic cough with production of copious amounts of foul smelling sputum. Autopsy showed massively dlated airway and amploidasis is diagnosed made after kistopathological analysis. The Amyloid proteins thai are involved include aAA b) AB c) AB2 M DAL e) ATTR Answer | Bevianaton | Other User's Explanation Report An Error Question Explanation: This man’s history of producing foul smelling sputum and the massively dilated airways that are grossly visible strongly suggest bronchiectasis. One complicetion associated with chrotic infections such as tuberculosis end broxchiectasis (and vations noa infectious inflammatory disorders) is secondary amyloidosis. The AA type of amyloid protein is most often involved in secondary amyloidosis. This type of amyloid is derived from serum amyloid associated protein or SAA which is an acute-phase reactant. Amyloid|) ccociated Disease [Clinical Manifestations ‘Type I laa Secondary amyloidosis (autcimmune [Deposition in kidney, spleen, liver, most commonly presents as renal ldisease, chromic infecticn) linsufficiency/nephrotc syndrome AB |Acheimer (Dementia [Most commonly musculoskeletal, with deposits in muscles, igaments, synovium, ApM 2 I PM) [Long term hemodialysis Sees ne eee earn came acd ‘AL and VAL right chan deposition, systenuc, includes heart, kidney ATR; mostly cardiac [Prinary amyloidosis, ATTR deposition 2i25/2014 1:56:55 PM ‘Mark this question & => Question Id : 139020 Question 24 of 30 Aman is kiled in RTA. He had a history of fever and chronic cough with production of copious amounts of foul smelling sputum. Autopsy showed massively dlated airway and amploidasis is diagnosed made after kistopathological analysis. The Amyloid proteins that are nvolved include ¥OadAs b) Ap ©) AB2 M A AL 2) ATTR Answer | Bevianaton | Other User's Explanation Report An Error Question Explanation: This man’s history of producing foul smelling sputum and the massively dilated airways that are grossly visible strongly suggest bronchiectasis. One complicetion associated with chrotic infections such as tuberculosis end broxchiectasis (and vations noa infectious inflammatory disorders) is secondary amyloidosis. The AA type of amyloid protein is most often involved in secondary amyloidosis. This type of amyloid is derived from serum amyloid associated protein or SAA which is an acute-phase reactant. Amyloid|) ccociated Disease [Clinical Manifestations ‘Type I laa Secondary amyloidosis (autcimmune [Deposition in kidney, spleen, liver, most commonly presents as renal ldisease, chromic infecticn) linsufficiency/nephrotc syndrome AB |Acheimer (Dementia [Most commonly musculoskeletal, with deposits in muscles, igaments, synovium, ApM 2 I PM) [Long term hemodialysis Sees ne eee earn came acd ‘AL and VAL right chan deposition, systenuc, includes heart, kidney ATR; mostly cardiac [Prinary amyloidosis, ATTR deposition ‘Mark this question = => Question Td : 139888 Question 25 of 30 A.42 year old Caucasian male presents with persistent cough and dyspnea. 4 months ago he remembers developing scme red bumps on his legs and some joint swelling His blood testis notable for elevated ACE levels CXR shows bilateral hilar masses with lung parenchyma prominence. Biopsy of the masces chowe noncaseating grenulomas. Ziehl Nelsen and silver staine ace aegative. The most ikely diagnosis is a) Asbestosis b) Beryliun exposure c) Lyrnphoma od) Primary tuberculosis ©) Sarcoidosis Answer | explanation Other User's Explanation Report An Error Question Explanation: Sarcoidosis is a multisystem disease characterized by noncaseating gramulomata in a variety of organs. The disease may be symptomatic (respiratory and constitutional symptoms: erythema nodosum, uveitis, arthritis) or may be discovered incidentally when chest X-ray or autopry reveels bilateral hilar adenopathy. Angiotensin converting enzyme (ACE) is elevated in 60% of sarcoidosis patients, Defistive diagnosis is based on biopsy, which reveals noncascating granulometa that are negetive for fungi or acid fast bacill! Sarcoidosis is more common in individuals of Aftican descent. Asbestosis exposure is characterized by calcified pleural plaques and preferential inrolvement ofthe lung bases. Although beryllium exposure may have similar findings, this patient has 20 apparent occupational risk of exposure, Lymphoma could lead to hilar adenopathy and ing parenchyma prominence. It would not be aszociated with granilomes. Primary tuberculosis, in an immanocompromised patient, may chow noncaseating granulomas. The acid-fast stain, however, would be positive. ‘Mark this question = => Question Td : 139888 Question 25 of 30 A.42 year old Caucasian male presents with persistent cough and dyspnea. 4 months ago he remembers developing scme red bumps on his legs and some joint swelling His blood testis notable for elevated ACE levels CXR shows bilateral hilar masses with lung parenchyma prominence. Biopsy of the masces chowe noncaseating grenulomas. Ziehl Nelsen and silver staine ace aegative. The most ikely diagnosis is a) Asbestosis b) Beryliun exposure c) Lyrnphoma od) Primary tuberculosis Y © 6) Sarcoidosis Answer | explanation Other User's Explanation Report An Error Question Explanation: Sarcoidosis is a multisystem disease characterized by noncaseating gramulomata in a variety of organs. The disease may be symptomatic (respiratory and constitutional symptoms: erythema nodosum, uveitis, arthritis) or may be discovered incidentally when chest X-ray or autopry reveels bilateral hilar adenopathy. Angiotensin converting enzyme (ACE) is elevated in 60% of sarcoidosis patients, Defistive diagnosis is based on biopsy, which reveals noncascating granulometa that are negetive for fungi or acid fast bacill! Sarcoidosis is more common in individuals of Aftican descent. Asbestosis exposure is characterized by calcified pleural plaques and preferential inrolvement ofthe lung bases. Although beryllium exposure may have similar findings, this patient has 20 apparent occupational risk of exposure, Lymphoma could lead to hilar adenopathy and ing parenchyma prominence. It would not be aszociated with granilomes. Primary tuberculosis, in an immanocompromised patient, may chow noncaseating granulomas. The acid-fast stain, however, would be positive. 2125/2014 1 Merk this question & => Question Td : 141886 Question 26 of 30 26 PM Autopsy ofa 24 year old male reveals a small cluster of caseating granulomas in the right lang just above the interlobar fissure, and similar granulomas in the hilar lymph nodes. Acid fast staining demonstrates acid-fast bacilli within these lesions. Remaining organs and systems were normal, The most likely interpretation of these findings is 8) Disseminated infection &) Primary infection ©) Recctivational disease 6) Re-exposure disease ©) Remote healed infection Anower [JERVIRNBHER otner User's Exptan Question Explanation: ‘The Ghon complex is the most frequent pathologe form of primary pulmonary tuberculosis. Mycobacterium tuberculosis first localizes nthe lung parenchyma, then in the hilar lymph nodes. In both of these locations, a granulomatous reaction takes place ‘These lesions usually heal by fibrosis, leaving only small scars at the sites of remote tuberculous infection, Th some cases, owing to immunosuppression (c.g, AIDS, immunesuppressant treatment, and lymphomas), reactivation of dormant bacilliin old lesions or additional re-exposure leads to secondary tuberculosis, with progression oflesions. Sometimes, active lesions of the Ghon complexe are discovered by chance at autopsy. Disseminated infection is a sign of secondary miberculosis. The military form is dae to Iymphohematogenous dissemination and subsequent seeding of tubercle bacill throughout the body, with a myriad of small granulomas forming in the lungs, spleen, iver, bone marrow, retina and adrenals. Reactivational isease occurs when immanosuppression or immunecompromise results in the activation of bail in old lesions with progression to more severe disease Cavitary disease and miliary tuberculosis may result Re-exposure diseas® may resuit in secondary miberculosis and progression of Iesions. Scars due to remote healed tuberculosis are fiequently found postmoriem and listed in autopsy reports as incidental findings. Report An Error 2125/2014 1 Merk this question & => Question Td : 141886 Question 26 of 30 26 PM Autopsy of a 24 year old male reveals a small cluster of caseating granulomas in the right lang just above the interlobar fissure, and similar granulomas in the hilar lymph nodes. Acid fast staining demonstrates acid-fast bacili within these lesions. Remaining organs and systems were normal. The most likely interpretation of these findings is a) Disseminated infection Y¥ © ») Primary nfection c) Recctivational disease 6) Re-emposure disease ¢) Remote healed infection Anower [JERVIRNBHER otner User's Exptan Question Explanation: ‘The Ghon complex is the most frequent pathologe form of primary pulmonary tuberculosis. Mycobacterium tuberculosis first localizes nthe lung parenchyma, then in the hilar lymph nodes. In both of these locations, a granulomatous reaction takes place ‘These lesions usually heal by fibrosis, leaving only small scars at the sites of remote tuberculous infection, Th some cases, owing to immunosuppression (c.g, AIDS, immunesuppressant treatment, and lymphomas), reactivation of dormant bacilliin old lesions or additional re-exposure leads to secondary tuberculosis, with progression oflesions. Sometimes, active lesions of the Ghon complexe are discovered by chance at autopsy. Disseminated infection is a sign of secondary miberculosis. The military form is dae to Iymphohematogenous dissemination and subsequent seeding of tubercle bacill throughout the body, with a myriad of small granulomas forming in the lungs, spleen, iver, bone marrow, retina and adrenals. Reactivational isease occurs when immanosuppression or immunecompromise results in the activation of bail in old lesions with progression to more severe disease Cavitary disease and miliary tuberculosis may result Re-exposure diseas® may resuit in secondary miberculosis and progression of Iesions. Scars due to remote healed tuberculosis are fiequently found postmoriem and listed in autopsy reports as incidental findings. Report An Error ‘Mark this question & => Question Td : 158186 Question 27 of 30 A. 12 year old boy has chronic cough. He has just recovered from his fourth bout of pneumonia in the past S months, On examination, digtal clibbing, hyperresonance to percussion, and baslar crackles are noted. His sweat chloride concentration is 87mEq/L. What agent would most Ikely serve ta alleviate kis chronic signs and symptoms? a) Dextromethorphan @O) b) Ipratropium (aerosolized) c) N-acetylcysteine (aerosolized) d) Pentamidine (aerosolized) ©) Vancomycin 7) Question Explanation: The patient is presenting with signs and oymptoms of cystic brosis (CF). CF ic an autosomal recessive disorder of the exocrine glands. The pulmonary manifestations include acutz end chronic bronchitis bronchiectasis, chronic bouts of pneumoria hemoptysis, and cor pulmonale, which can occur late in the disease. Other common findings include chronic cough, exercise intolerance, recurrent sespiratory infections, digital clubbing, increased anteroposterior ctameter, and basilar crackles It the pilocarpine sweat testreveals, sodium and chloride levels greater than 30mEqy/L, a diagnosis of CF can be made. The primary goals of treatment inclnde thinning the mucus sccretions, keeping the airways open, and treating recurrent infections. Thinning of mucus can bs achieved with mucelytics such as N- acetyloysteine. N-Acetyleysteine (Mucomyst) splits the disulfide linkages between these mucoproteins, resulting in a decrease in mucous viscosity. Ibis indicated as adjuvant therapy in the treatment of cbnormal viscid or inspissated rucus secretions in CE, chronic tung disease, post traumatic chest complications and atelectasis secondary to macus obstriction irhaled bronchodilators are used to open the airways. Furthermore prednisone haz been shown to increase pulmonary function and increase bodyweight. The definiive treatment is lung transplantetion Dexomethorphan, a cough suppressant, is contraindicated in patients with CF since it wil prevent the removal of mucus from the fangs. Ipratropium is an anticholinergic that will cause a drying and thickening of the mucus in this patient, therefore, itis contraindicated, Pentamidine (aerosclized) is an antiprotozoal agent primarily used in the treatment of Pneumocystis carinii pneumonia in HIV-infected patients. Vancomycin is an anti-infective agent used in the treatment of life threatening, gram-positive infections. ‘Mark this question & => Question Td : 158186 Question 27 of 30 A. 12 year old boy has chronic cough. He has just recovered from his fourth bout of pneumonia in the past S months, On examination, digtal clibbing, hyperresonance to percussion, and baslar crackles are noted. His sweat chloride concentration is 87mEq/L. What agent would most Ikely serve ta alleviate kis chronic signs and symptoms? a) Dextromethorphan @O) b) Ipratropium (aerosolized) Y © c) N-acetylcysteine (aerosolized) d) Pentamidine (aerosolized) ©) Vancomycin 7) Question Explanation: The patient is presenting with signs and oymptoms of cystic brosis (CF). CF ic an autosomal recessive disorder of the exocrine glands. The pulmonary manifestations include acutz end chronic bronchitis bronchiectasis, chronic bouts of pneumoria hemoptysis, and cor pulmonale, which can occur late in the disease. Other common findings include chronic cough, exercise intolerance, recurrent sespiratory infections, digital clubbing, increased anteroposterior ctameter, and basilar crackles It the pilocarpine sweat testreveals, sodium and chloride levels greater than 30mEqy/L, a diagnosis of CF can be made. The primary goals of treatment inclnde thinning the mucus sccretions, keeping the airways open, and treating recurrent infections. Thinning of mucus can bs achieved with mucelytics such as N- acetyloysteine. N-Acetyleysteine (Mucomyst) splits the disulfide linkages between these mucoproteins, resulting in a decrease in mucous viscosity. Ibis indicated as adjuvant therapy in the treatment of cbnormal viscid or inspissated rucus secretions in CE, chronic tung disease, post traumatic chest complications and atelectasis secondary to macus obstriction irhaled bronchodilators are used to open the airways. Furthermore prednisone haz been shown to increase pulmonary function and increase bodyweight. The definiive treatment is lung transplantetion Dexomethorphan, a cough suppressant, is contraindicated in patients with CF since it wil prevent the removal of mucus from the fangs. Ipratropium is an anticholinergic that will cause a drying and thickening of the mucus in this patient, therefore, itis contraindicated, Pentamidine (aerosclized) is an antiprotozoal agent primarily used in the treatment of Pneumocystis carinii pneumonia in HIV-infected patients. Vancomycin is an anti-infective agent used in the treatment of life threatening, gram-positive infections. ‘Mark this question rm Question 28 of 30 A 52 year old male hes 2 years of worsening dyspnea upon exercise. He hes a persistent cough and has smoked cigarettes for the past 30 years. Pulmonary function tests reveal decreased FEV1 and FVC. Which set of laboratory result is most likely frorn him? a) PO2 (mm Hg)|O2 Concentration ml./100m1.)||02 Concentration||pH 60 18 60 [7.35 b) PO2 (mm Hg)|O2 Concentration ml./100m1.)||02 Concentration||pH 0 6 60 [7.29 2 PO2 (mmm Hg)|O2 Concentration (ml/100ml,)|(O2 Concentration||pH 0 10 ho [7.42 d PO2 (mm Hg)|O2 Concentration ml./100mL,)|(02 Concentration||pH 105 te bo [7.42 ) PO2 (mm Hg)|O2 Concentration ml./100ml,)|(02 Concentration||pH 105 20 bo [7.29 Question Explanation: ‘This patient has symptoms ineicating chronic obstructive puimonary disease (COPD). The mnismatching of ventilation and perfision causes hypoxemia (PO2 less than 100) with or without some CO2 retention (CO2 greater than 40). FCO2 retention occurs the pH. will decrease, but becomes close to normal overtime because of renal compensation (pH slightly less than 7.4). When the O2 dissociation curve is in the normal position (pH 7.4 and PO2= 40 mum Hg), a PO2 of 60 min Hg rosults ia an arterial O2 saturation of 90%. This would give an ©2 concentration of 18mL/100mL. However, a ftequent accompaniment of chronic hypoxiais a degree of polycythemia, which would resut in a higher value than 18ml. /100mL. ‘This is offSet by the Bchretrect in which increased PO2 and acidosis reduce affinity of hemoglobin for oxygen. Option B would reflect severe, incaparitating hypoxemia, not merely exertional dyspnea, Option C hes an oxygen content that is inconsistent with the PO2 also the CO2 should be elevated with this severe WQ mismatching. Option D is inconsistent with the clinical description, because it reflects increased ventiation, but decreased oxygen content. In option E the pHis not consistent with hyperventilation (which would lead to a respiratory alkalosis), which would not be herein es auc ‘Mark this question rm Question 28 of 30 A 52 year old male hes 2 years of worsening dyspnea upon exercise. He hes a persistent cough and has smoked cigarettes for the past 30 years. Pulmonary function tests reveal decreased FEV1 and FVC. Which set of laboratory result is most likely frorn him? You PO2 (mm Hg)|O2 Concentration ml./100m1.)||02 Concentration||pH 60 18 60 [7.35 b) PO2 (mm Hg)|O2 Concentration ml./100m1.)||02 Concentration||pH 0 6 60 [7.29 2 PO2 (mmm Hg)|O2 Concentration (ml/100ml,)|(O2 Concentration||pH 0 10 ho [7.42 d PO2 (mm Hg)|O2 Concentration ml./100mL,)|(02 Concentration||pH 105 te bo [7.42 ) PO2 (mm Hg)|O2 Concentration ml./100ml,)|(02 Concentration||pH 105 20 bo [7.29 Question Explanation: ‘This patient has symptoms ineicating chronic obstructive puimonary disease (COPD). The mnismatching of ventilation and perfision causes hypoxemia (PO2 less than 100) with or without some CO2 retention (CO2 greater than 40). FCO2 retention occurs the pH. will decrease, but becomes close to normal overtime because of renal compensation (pH slightly less than 7.4). When the O2 dissociation curve is in the normal position (pH 7.4 and PO2= 40 mum Hg), a PO2 of 60 min Hg rosults ia an arterial O2 saturation of 90%. This would give an ©2 concentration of 18mL/100mL. However, a ftequent accompaniment of chronic hypoxiais a degree of polycythemia, which would resut in a higher value than 18ml. /100mL. ‘This is offSet by the Bchretrect in which increased PO2 and acidosis reduce affinity of hemoglobin for oxygen. Option B would reflect severe, incaparitating hypoxemia, not merely exertional dyspnea, Option C hes an oxygen content that is inconsistent with the PO2 also the CO2 should be elevated with this severe WQ mismatching. Option D is inconsistent with the clinical description, because it reflects increased ventiation, but decreased oxygen content. In option E the pHis not consistent with hyperventilation (which would lead to a respiratory alkalosis), which would not be herein es auc Matte this question = => Question Td : 213734 Question 29 of 30 A S2-year-old man with oxygen-dependent COPD has worsening shoriness of breath and right-sided, constant chest pain worse with deep inspiration. Taese awoke kim from sleep at 5.AM. Temperature = 37.0 °C (98.6 °F), BP = 170/70 mmlfg, pulse = 133/mnin, and respirations = 28/min. SPO2 =83% on 6L of oxygen by way of nasal cannula, CXRis as follows ‘What is the most appropriate next step? a) Administration of albuterol, ipratropium bromide, oxygen, antibiotics, and steroids +b) Administration of aspirin, heparin, oxygen, metoprolol, and nitrate ©) Administration of intravenous ceftriaxone and azithromycin 4) Adninistration ofintravenous ceftriaxone and azithromycin €) Insertion of an intercostal chest tube Answer | Bolanation | Other User's Explanation Report An Error Question Explanation: This patient has a pneumothorax secondary to emphysema Although a pneumothorax, # small enough, can resolve spontaneously, many require chest tube insertion. This patient has a fairy large pneumothorax that can be recognized on chest x-ray by the lack of peripheral iung markings on the right. The treatment of clinically significant pneumothorax is insertion of a chest tube with negative pressure to allow hing reexpansion. ‘Administration of albuterol, ipratropium bromide oxygen, antibiotics, and steroids is the treatment for a chronic cbstructive pulmonary disease (COPD) execerbatien. Although this petient has ahistory of CODD, it is more lcely her pneumothorax is causing her acute ‘worsening symptoms. ‘The combination of aspirin, heparin, oxygen, Metoprolol, and nitrates is the treatment for acute coronary syndrome. There is no evidence presented to suggest that this patient has cardiac ischemia, Intravenous ceftriaxone and azithromycin is the treatment for pneumonia, Although pacumonia is in the differential diagnosis of a patient with shortzess of breath and chest pain, the x-ray should suggest the correct diagnosis. Inravenous heparin adiniristration is the treatment for palmonary embolism (PE). PE could explain shortness of breath and chest ae Matte this question = => Question Td : 213734 Question 29 of 30 A S2-year-old man with oxygen-dependent COPD has worsening shoriness of breath and right-sided, constant chest pain worse with deep inspiration. Taese awoke kim from sleep at 5.AM. Temperature = 37.0 °C (98.6 °F), BP = 170/70 mmlfg, pulse = 133/mnin, and respirations = 28/min. SPO2 =83% on 6L of oxygen by way of nasal cannula, CXRis as follows “What is the most appropriate next step? 8) Administration of albutercl, ipratrepium bromide, oxygen, antibiotics, and steroids +b) Administration of aspirin, nepatin, oxygen, metoprolol, and nitrate ©) Administration ofintravenous cefhiaxone and asithramycin 4) Administration of intravenous ceftriaxone and azithromycin JY © @) Insertion of an intercostal chest tube Answer | Bolanation | Other User's Explanation Report An Error Question Explanation: This patient has a pneumothorax secondary to emphysema Although a pneumothorax, # small enough, can resolve spontaneously, many require chest tube insertion. This patient has a fairy large pneumothorax that can be recognized on chest x-ray by the lack of peripheral iung markings on the right. The treatment of clinically significant pneumothorax is insertion of a chest tube with negative pressure to allow hing reexpansion. ‘Administration of albuterol, ipratropium bromide oxygen, antibiotics, and steroids is the treatment for a chronic cbstructive pulmonary disease (COPD) execerbatien. Although this petient has ahistory of CODD, it is more lcely her pneumothorax is causing her acute ‘worsening symptoms. ‘The combination of aspirin, heparin, oxygen, Metoprolol, and nitrates is the treatment for acute coronary syndrome. There is no evidence presented to suggest that this patient has cardiac ischemia, Intravenous ceftriaxone and azithromycin is the treatment for pneumonia, Although pacumonia is in the differential diagnosis of a patient with shortzess of breath and chest pain, the x-ray should suggest the correct diagnosis. Inravenous heparin adiniristration is the treatment for palmonary embolism (PE). PE could explain shortness of breath and chest ae 2:25/2014 1:58:40 PM ‘Mark this question ea Question Id : 217108 Question 30 of 30 A 59-year-old man chronic smoker now has shortness of breath, chronic cough and recent hemoptysis. CXR shows nght sided pleural effusion with no masses. Pleural fic has a hematocrit = 10%, protein = 60% of the serum protein value, ghucose = 50 mg/dl, anda differential cell count of 60% lymphocytes. Which of the following is the most appropriate next step in management? 2) Bronchoscopy with bronchoalveolar lavage +) Determine arnylace level in the pleural Bid ¢) High-resolution spiral CT scan of the chest 4) MBI of the chest «) Send cytology of the pleural uid Question Explanation: all the parameters suggest malignant effusion in this case, therefore cytology would be the most appropriate next step. Pleural fuid amylase levels are used to nde in acute pancreatitis or esophageal perforation, which is not the clinical picture in this case. Bronchoscopy with bronchoalveolar lavage is not the best step in this case since we akteady have the fiuid for analysis. Bronchoscopy and lavage is indicated for the diagnosis of pulmonary infections, interstitial disease, and centrally located malignancies. ‘High-resolution spiral CT scan ofthe chest and MRI are detailed imaging modalities used to determine size and staging of ling cancers. The chest x-ray failing to reveal the presence of a tumor does not mean that no tumoris there. A high-resolution spiral CT scan with Imm Cuts will give a much more accurate diagnosis. After Buid is sent for cytology, the next step would be to order a spiral CT scan to determine the location of the tuner. Report An Error 2:25/2014 1:58:40 PM ‘Mark this question ea Question Id : 217108 Question 30 of 30 A 59-year-old man chronic smoker now has shortness of breath, chronic cough and recent hemoptysis. CXR shows nght sided pleural effusion with no masses. Pleural fic has a hematocrit = 10%, protein = 60% of the serum protein value, ghucose = 50 mg/dl, anda differential cell count of 60% lymphocytes. Which of the following is the most appropriate next step in management? 2) Bronchoscopy with bronchoalveolar lavage +) Determine arnylace level in the pleural Bid ¢) High-resolution spiral CT scan of the chest 4) MBI of the chest Y © 2) Send cytology of the pleural fd Question Explanation: all the parameters suggest malignant effusion in this case, therefore cytology would be the most appropriate next step. Pleural fuid amylase levels are used to nde in acute pancreatitis or esophageal perforation, which is not the clinical picture in this case. Bronchoscopy with bronchoalveolar lavage is not the best step in this case since we akteady have the fiuid for analysis. Bronchoscopy and lavage is indicated for the diagnosis of pulmonary infections, interstitial disease, and centrally located malignancies. ‘High-resolution spiral CT scan ofthe chest and MRI are detailed imaging modalities used to determine size and staging of ling cancers. The chest x-ray failing to reveal the presence of a tumor does not mean that no tumoris there. A high-resolution spiral CT scan with Imm Cuts will give a much more accurate diagnosis. After Buid is sent for cytology, the next step would be to order a spiral CT scan to determine the location of the tuner. Report An Error 2252014 2:14:25 PM ‘Mark this question Question 1 of 30 ‘The following statement is tue of Sarcoidosis a) Prognosis is poor when Sarcoidosis presents acutely with bilateral hiler Imohadenopathy and erythema nodosum +) Hypercalcemia due to increased renal synthesis of I-tydroxylase ¢) Serum angiotensin converting enzyme (ACE) is useful for diagnosis of Sercoidosis 4) Iris synonymous to Miklicz’s syndrome ¢) Dyspnea is the most common symptom Question Explanation: Dyspnea is the most common symptom in Sarcoidosis. Lofgren’s syndrome is the combination of Erythema nodosum and bilateral ilar lymphadenopathy (Stage 1 radiograph). The prognoses are good with 80% resolving spontaneously, and have anortral CAR after Lyear. Hypercalcemia 2-10%) and hypercalciuria (up to 50%) are well recognized in saroidosis. The pattern resembles hypervitaminosis D, wwith elevated serum calciam, normal serum phosphate and normal/cightly raised alkaline phosphotase. There is elevated 1, 25- dlibydrozycholecalciferol due to increased production by alveclar pulmonary macrophages and macrophages in granulomata, Treat with rehdration and corticosteroids Serum ACE is produced by sarcoid granulomata from activation and differentiation of monocyte-macrophage system. Itis a membrane bound glycoprotein, found mainly in the lung capillary endothelium. ACE has poor diagnostic sensitivity (ability to detect disease) and specificity (ability to exclude disease), but is raised in active sarcoidosis, It is useful in monitoring of disease activi. ‘Mikulcz’s syndrome is the enlargement of lacrimal glands and parotid glands, caused by Sarcoidosis and not synonymous to it. Other causes include lymphoma and leukemia. Sarcoidosis is chronic multisystem non-caseating granulomatous disease, Central fibrinoid necrosis may occur, but tends to be focal and limited unlike the purulent necrosis/caseation seen in tuberculosis. 2252014 2:14:25 PM ‘Mark this question Question 1 of 30 ‘The following statement is true of Sarcoidosis a) Prognosis is poor when Sarccidosis presents acutely with bilateral hiler lmmphadenopathy and erythema nodosum +b) Hypercalcemia due to increased renal synthesis of 1-hydroxylase ©) Serum angiotensin converting enzyme (ACE) is useful for diagnosis of Sarcoidosis 4) Ibis synonymous to Mikulicz’s syndrome Y © 6) Dyspnea is the most common symptom Question Explanation: Dyspnea is the most common symptom in Sarcoidosis. Lofgren’s syndrome is the combination of Erythema nodosum and bilateral ilar lymphadenopathy (Stage 1 radiograph). The prognoses are good with 80% resolving spontaneously, and have anortral CAR after Lyear. Hypercalcemia 2-10%) and hypercalciuria (up to 50%) are well recognized in saroidosis. The pattern resembles hypervitaminosis D, wwith elevated serum calciam, normal serum phosphate and normal/cightly raised alkaline phosphotase. There is elevated 1, 25- dlibydrozycholecalciferol due to increased production by alveclar pulmonary macrophages and macrophages in granulomata, Treat with rehdration and corticosteroids Serum ACE is produced by sarcoid granulomata from activation and differentiation of monocyte-macrophage system. Itis a membrane bound glycoprotein, found mainly in the lung capillary endothelium. ACE has poor diagnostic sensitivity (ability to detect disease) and specificity (ability to exclude disease), but is raised in active sarcoidosis, It is useful in monitoring of disease activi. ‘Mikulcz’s syndrome is the enlargement of lacrimal glands and parotid glands, caused by Sarcoidosis and not synonymous to it. Other causes include lymphoma and leukemia. Sarcoidosis is chronic multisystem non-caseating granulomatous disease, Central fibrinoid necrosis may occur, but tends to be focal and limited unlike the purulent necrosis/caseation seen in tuberculosis. 2:25/2014 2:14:39 PM ‘Mark this question <—& => Question Td : 19572 Question 2 of 30 A.61-year-old male who has been prescribed lisinopril for hypertension presents with an irtating cough. What is the mechanism responsible for ACE-induced cougt? 8) Angiotensin T accumulation 6) ACE inhibition 6) Laryngeal inrtation 6) Bradykinin accumulation ©) Rerin accumulation Answer [MEBRRNN) other Users Expan Question Explanation: The erzyme ACK is also responsible for the metabolism of bradykinin in mast cells. The accurmulation ofthis substance is responsible for the cough foundin up to 30% of subjects taking ACE-inhibitors. This phenomenon is net seen in subjects taking Angiotensin receptor blockers such as Losartan. jon Report An Error 2:25/2014 2:14:39 PM ‘Mark this question <—& => Question Td : 19572 Question 2 of 30 A 61-year-old male who has been prescribed lisinopril for hypertension presents with an imtating cough. What is the mechanism responsible for ACE-induced cough? a) Angiotensin I accumulation ) ACE inhibition ¢) Laryngeal isritation v © & Bradykinin accumulation ©) Rerin accumulation Answer [MEBRRNN) other Users Expan Question Explanation: The erzyme ACK is also responsible for the metabolism of bradykinin in mast cells. The accurmulation ofthis substance is responsible for the cough foundin up to 30% of subjects taking ACE-inhibitors. This phenomenon is net seen in subjects taking Angiotensin receptor blockers such as Losartan. jon Report An Error 225.2014 2:14:54 PM ‘Mark this question —& => Question Td : 28242 Question 3 of 30 A 30-year-old professional singer presents with a prolonged history of epistaxis and rapidly progressive shorness of breath. The ECO and eosinophil count are raised, Which of the following is the most likely diagnosis? a) Wegener's gramulomatosis +b) Goodpasnure’s syndrome ¢} Microscopic polyangiitis ) Churg- Strauss syndrome ¢) Alveolar proteinosis Answer (Bxrianation | Other User's Explanation Report An Error Question Explanation: ‘The patient with breathlessness and a raised KCO has alveolar hemorshage until proven otherwise. A prolonged history of epistaxis or sinusitis ic commonly found in Wegerer‘s granulomatesic, which i some patients is also associated with an eosinophil. A history of asttena must usually be present to diagnose the Churg-Strauss syadrome. 225.2014 2:14:54 PM ‘Mark this question —& => Question Td : 28242 Question 3 of 30 A 30-year-old professional singer presents with a prolonged history of epistaxis and rapidly progressive shorness of breath. The ECO and eosinophil count are raised, Which of the following is the most likely diagnosis? Y © a) Wegener's gramilomatosis +b) Goodpasnure’s syndrome ¢} Microscopic polyangiitis ) Churg- Strauss syndrome ¢) Alveolar proteinosis Answer (Bxrianation | Other User's Explanation Report An Error Question Explanation: ‘The patient with breathlessness and a raised KCO has alveolar hemorshage until proven otherwise. A prolonged history of epistaxis or sinusitis ic commonly found in Wegerer‘s granulomatesic, which i some patients is also associated with an eosinophil. A history of asttena must usually be present to diagnose the Churg-Strauss syadrome. 2:25/2014 2:15:12 PM “Mark this question => Question Id : 28934 Question 4 of 30 A 42-year-old female is admitted with a suspected pulnonery embolism, A ventilation perfusion scan is requested. Which of the followings true of lung ventilation perfusion scanning in suspected pulmonary embolism (PE)? 2) Radiation to fetus is minimal ifit is done in & pregnant woman ) Ibis contraindicated in those which iodine hypersensitivity ©) A normal perfusion scan does not necessarly exclude pulmonary embolism 6) There is reduced perfusion in the upper lobes in mitral stenosis ©) The appearances of the scan would resemble those in pulmonary embolism ifthe patienthad emphysema Anower [JENSEN] ter vsors Exptan Question Explanation: Radiation to the fetus is smal Increased pulmonary venous pressure, especially secondary to mitral valve disease, causes increased flow to the upper lobes Chronic obstructive pulmonary disease (COPD) usualy gives rise to matched defects, Ventilation quotient (VQ) scan is not contraindicated in pregnant women, although the perfusion only scan is adequate Xenon is used for imaging ventilation, whilst technetium labeled macroggregated human serum albumin (MAA) is used to image perfusion. A normal perfusion scan has a sensitivity of 98% bur a specificity of only 40% Report An Error 2:25/2014 2:15:12 PM “Mark this question => Question Id : 28934 Question 4 of 30 A.42-year-old female is admitted with a suspected pulmonary embolism. A ventilation perfusion scan is requested. Which of the followings true of lung ventilation perfision scanning in suspected pulmonary embolism (PE)? ¥ © 2 Radiation to fetus is minimal ifit is done in a pregnant woman, +) Itis contraindicated in those which iodine hypersensitivity ©) Anommal perfusion scan does net necessarily exclude pulmonary embolism 4) There is reduced perfusion in the upper lobes in mitral stenosis «) The appearances of the scan would resemble those in pulmonary embolism ifthe patient had emphysema Anower [JENSEN] ter vsors Exptan Question Explanation: Radiation to the fetus is smal Increased pulmonary venous pressure, especially secondary to mitral valve disease, causes increased flow to the upper lobes Chronic obstructive pulmonary disease (COPD) usualy gives rise to matched defects, Ventilation quotient (VQ) scan is not contraindicated in pregnant women, although the perfusion only scan is adequate Xenon is used for imaging ventilation, whilst technetium labeled macroggregated human serum albumin (MAA) is used to image perfusion. A normal perfusion scan has a sensitivity of 98% bur a specificity of only 40% Report An Error 2:25/2014 2:15:27 PM ‘Mark this question & => Question Td : 29084 Question 5 of 30 A 68-year-old presents with a pyresia, productive cough and shortness of breath for 5 days. She has RLL consclidation and a stall unilateral pleural effusion on CXR. Which is a marker of poor prognosis? a) Temp >38°C b) Cough ) WOC>15 A Her age e) Her CKE. signs Answer | Bevionation | Other User's Explanation Report An Error Question Explanatiot Indicators of poor prognosis in pneumonia inclide: age >65, co-existing morbidity inclide diabetes melitas, chronic renal failure, stroke, coronary artery disease, respiratory rate>30 and mental impairment. Biochemicalhematological markers include white count <4 or =30, hypoxia needing CPAP or FiO2>60%, positive blood culture aid blood urea>7, 2:25/2014 2:15:27 PM ‘Mark this question & => Question Td : 29084 Question 5 of 30 A 68-year-old presents with a pyrexia, productive cough and shortness of breath for 5 days. She has RLL consolidation and a stnall unileteral pleural effision on CIR, Which is a marker of poor peagnasis? a) Temp >38°C +) Cough 0) WCC > 15 Y © d Her age e) Her CXR. signs Answer | Bevionation | Other User's Explanation Report An Error Question Explanatiot Indicators of poor prognosis in pneumonia inclide: age >65, co-existing morbidity inclide diabetes melitas, chronic renal failure, stroke, coronary artery disease, respiratory rate>30 and mental impairment. Biochemicalhematological markers include white count <4 or =30, hypoxia needing CPAP or FiO2>60%, positive blood culture aid blood urea>7, 2125/2014 2:15:45 PM “Mark this question => Question Id : 29493 Question 6 of 30 A 21-year-old non-smoker has shortness of breath whilst running. He has no other symptoms. Examination, fil blood count, and chest X-ray are normal. Which of the following is helpful? a) Arterial blood gas studies before and after exercise 6) Spirometry before and after exercise ©) Determination oflung volume and difising capacity 6) Measurement of venous blood lactate before and afer exercise ©) Spirometry before and after administration of bronchodilators Anower [UERENERER) otner User's Exp Question Explanation: ‘The most likely diagnosis is exorcise induced asthma aad this would be best diagnosed with Spiromotry before and after sxercise where a typical obstructive pattem may be displayed following exercise. No abnormalties may be displayed following bronchodilator therapy iftrue exercise induced asthma persists. Similarly lung volumes and diffusion capacity are likely to be unaffected, blood gas analysis would be relatively unhelpful in this scenario as litle change in partial pressures would be expected This patient does not have a glycogen storage disease where weakness rather than shortness of breath is more typical. Hence lactate measurements aro unnecessary, jon Report An Error 2125/2014 2:15:45 PM “Mark this question => Question Id : 29493 Question 6 of 30 A 21-year-old non-smoker has shortness of breath whilst running He has no other symptoms. Examination, full blood count, and chest X-ray are normal, Which of the following is helpful? a) Arterial blood gas studies before and after exercise V © b) Spirometry before and after exercise c) Determination oflung volume and diffusing capacity 6) Measurement of venous blood lactate before and after exercise ©) Spirometry before and after administration of bronchodilators Anower [UERENERER) otner User's Exp Question Explanation: ‘The most likely diagnosis is exorcise induced asthma aad this would be best diagnosed with Spiromotry before and after sxercise where a typical obstructive pattem may be displayed following exercise. No abnormalties may be displayed following bronchodilator therapy iftrue exercise induced asthma persists. Similarly lung volumes and diffusion capacity are likely to be unaffected, blood gas analysis would be relatively unhelpful in this scenario as litle change in partial pressures would be expected This patient does not have a glycogen storage disease where weakness rather than shortness of breath is more typical. Hence lactate measurements aro unnecessary, jon Report An Error 2/25/2014 2:15:59 PM ‘Mark this question & => Question Td : 29513 Question 7 of 30 Shortness of breath and insomria are complains of a 17-year-old gitl, Clinical examination is normal. CXR and PEFR are normal ‘Which of the following investigations is most suggestive of asthma? a) Postive skin prick test to common allergens 'b) Past medical history of hay fever and eczema Increased total [gE c} }) Diumal variation in PEFR> 20% d) ¢} Resolution of symptoms the day after the exam Question Explanation: “The history of atopy in this patient is consistent with a diagnosis of asthma. Ig being elevated really tells you litle other than suggesting atopy and the skin prick would suggest allergy. The resoluion of symptoms after the exam suggests anxisty. However diurnal variations of PEFR. > 20% is one of the diagnosis criteria for asthma, and we feel this is the most appropriate answer in this case. 2/25/2014 2:15:59 PM ‘Mark this question & => Question Td : 29513 Question 7 of 30 Shortness of breath and insomria are complains of a 17-year-old gitl, Clinical examination is normal. CXR and PEFR are normal ‘Which of the following investigations is most suggestive of asthma? a) Postive skin prick test to common allergens 'b) Past medical history of hay fever and eczema Increased total [gE ¢) ¥ © 4) Diumal variation in PEFR> 20% ¢} Resolution of symptoms the day after the exam Question Explanation: “The history of atopy in this patient is consistent with a diagnosis of asthma. Ig being elevated really tells you litle other than suggesting atopy and the skin prick would suggest allergy. The resoluion of symptoms after the exam suggests anxisty. However diurnal variations of PEFR. > 20% is one of the diagnosis criteria for asthma, and we feel this is the most appropriate answer in this case. 2:25/2014 2:16:17 PM Mark this question & => Question Id : 29613 Question § of 30 A 68-year-old tan who hes @ long history of chronic bronchitis is admitted from home with an acute exacerbation, pCO =11 kPa and pO7=6 7 kPa, Which of the following would be expected in this patient? 2) A metabolic acidosis with a low bicarbonate would be expected +) Gentamicin would be a reasonable initial treatment until cultures are available c) Extensor plantar responses may be expected 4) Oxygen therapy should aim to increase the pO3 to above 8 kPa (60mmEa) €) Peripheral edema indicates coexisting heart failure Question Explanation: Tn chronic bronchitis, a low pO with high pCO, and compenseted respiratory acidosis with high bicarbonate is expected. Consequently with the help of pCO} an encephalopathy may be precipitated resubing in extensor plantar seponses. The Sgures given, suggest a respiratory acidosis. Extensor plantar responses are a feature often reflecting high pCO2, There may be dependent (postural) edema and this does not necesvarly indicate heart failure. 2:25/2014 2:16:17 PM Mark this question & => Question Id : 29613 Question § of 30 A 68-year-old man who hes 2 long history of chronic bronchitis is admitted from home with an acute exacerbation, pCO;=11 kPa and p07-67 kPa Which of the following would be expected in this patient? 2) A metabolis acidocis with a low bicarbonate would be expected +) Gentamicin would be a reasonable intel treatment until cultures are available W © ©) Extensor plantar responses may be expected 4) Oxygen therapy should aim to increase the pO; to above 8 kPa (60mmiHa) €) Petipheral edema indicates coexisting heart failure Question Explanation: Tn chronic bronchitis, a low pO with high pCO, and compenseted respiratory acidosis with high bicarbonate is expected. Consequently with the help of pCO} an encephalopathy may be precipitated resubing in extensor plantar seponses. The Sgures given, suggest a respiratory acidosis. Extensor plantar responses are a feature often reflecting high pCO2, There may be dependent (postural) edema and this does not necesvarly indicate heart failure. “Mark this question e& => Question 9 of 30 A 30-year-old man has pulmonary tuberculosis He is a fast acetylator. He was subsequently started on anti-tuberculous therapy that included ssoniazid Which of the following statements is correct? 2) There is an increased riske of convulsions ‘) There is an increased tisk of drug resisiance ©) There is an increased risk of megaloblastic anemia 6) There is an increased tisk of hepatitis ©) There is an increased risk of peripheral neuropathy Question Explanation: Tsoniatid is metabolized primarily by acetylation and dehydrazination, The rate of acetylation is genetically determined. Approximately 50 percent of Blacks and Caucasians are "slow acetylators", and the rest are "rapid acetylators'. Fast acetylation leads to higher ‘blood levels of the toxis metabolite acetyl-isoniarid and thus, to an increase in toxic reactions- hepatitis which ie 250 more cemmon than in slow acetylators. Fact acetylators on the other hand require higher doses but there increased tubercle resistance is not a problem. n Report An Error “Mark this question e& => Question 9 of 30 A 30-year-old man has pulmonary tuberculosis. He is a fast acetylator. He was subsequently started on anti-tuberculous therapy that included isoniazid. Which of the following statements is comrect? 2) There is an increased risk of comvulsions ') There is an increased tisk of dmg resistance ©) There is an increased risk of megaloblastic anemia © 4) There is an increased tisk of hepatitis ©) There is an increased risk of peripheral neuropathy Question Explanation: Tsoniatid is metabolized primarily by acetylation and dehydrazination, The rate of acetylation is genetically determined. Approximately 50 percent of Blacks and Caucasians are "slow acetylators", and the rest are "rapid acetylators'. Fast acetylation leads to higher ‘blood levels of the toxis metabolite acetyl-isoniarid and thus, to an increase in toxic reactions- hepatitis which ie 250 more cemmon than in slow acetylators. Fact acetylators on the other hand require higher doses but there increased tubercle resistance is not a problem. n Report An Error 2125/2014 2:16:44 PM ‘Mark this queston => Question 10 of 30 A man aged 35 years complains ofa persistent cough associated with chronic spumum production, Fibrosis of both upper lobes is, seen on chest X-ray. Whatis the most likely diagnosis? 2) Systemic Sclerosis ) Allergic bronchopulmonary aspergillosis ©) Primary Pulmonary Hypertension 4) Cystic Fibrosis, €) Ankylosing spondylitis Answer ( Expionation | Other User's Explanation Report An Error Question Explanation: The persistent cough i lcely to be a symptom of asthma, On the other hand, only avout 1% of patients with advanced arleylosing spondylitis develop apical fbrosis ie. rare Even then early lesions are asymptomatic. Tis orly when cavitation develops that symptoms lice cough, infected sputum and haemoptysis start, With ABPA, Sbrosis and loss of volume in the upper lobes are common, Within these upper lobes, there may be bronchiectasis. 2125/2014 2:16:44 PM ‘Mark this queston => Question 10 of 30 A man aged 35 years complains ofa persistent cough associated with chronic spumum production, Fibrosis of both upper lobes is, seen on chest X-ray. Whatis the most likely diagnosis? 2) Systemic Sclerosis ¥ © b) Allergic bronchopulmonary aspergillosis ©) Primary Pulmonary Hypertension 4) Cystic Fibrosis €) Ankylosing spondylitis Answer ( Expionation | Other User's Explanation Report An Error Question Explanation: The persistent cough i lcely to be a symptom of asthma, On the other hand, only avout 1% of patients with advanced arleylosing spondylitis develop apical fbrosis ie. rare Even then early lesions are asymptomatic. Tis orly when cavitation develops that symptoms lice cough, infected sputum and haemoptysis start, With ABPA, Sbrosis and loss of volume in the upper lobes are common, Within these upper lobes, there may be bronchiectasis. 2:25/2014 2:16:58 PM “Mark this question & => Question Td : 29920 Question 11 of 30 4.45 year old Carisbean female complains of slowly increasing breathlessness. No history of smoking is present. Investigations reveal she has bilateral enlarged hilar lymph nodes, elevated serum calciuen, interstitial lung disease, and enlarged liver and spleen. “What is the most likely diagnosis? a) Coccidiodomycosis ») Sarcoidosis ¢) Hyperparathyroidism ) Hypervitaminosis D 8) Tuberculosis Answer | Explanation Other User's Explanation Report An Error Question Explanation: Sarccid is common in subjects of Caribbean origin, Subacute increasing breathlessness also suggests the diagnosis of sarcoid, and an elevated calciim narrows the differential diagnosis futher. Hyperparathyroidicm would act cause eymptome of breathlessness Coctidiodomycosis is ancn-infective fingal infection caused by inhalation of the spores of Coccidiodomycesis. I: usually presents with fever, cough, myalgia and rash. Ibis endemic mainly to the south western USA. and North Mexico. TB is the major differential in this case Hypercalcaemia and bilateral hilar lymphadenomegaly (BEL) however are in favour ofa diagnosis of sarcoid. 2:25/2014 2:16:58 PM “Mark this question & => Question Td : 29920 Question 11 of 30 4.45 year old Carisbean female complains of slowly increasing breathlessness. No history of smoking is present. Investigations reveal she has bilateral enlarged hilar lymph nodes, elevated serum calciuen, interstitial lung disease, and enlarged liver and spleen. “What is the most likely diagnosis? a) Coccidiodomycosis ¥ © d) Sarcoidosis ¢) Hyperparathyroidism ) Hypervitaminosis D 8) Tuberculosis Answer | Explanation Other User's Explanation Report An Error Question Explanation: Sarccid is common in subjects of Caribbean origin, Subacute increasing breathlessness also suggests the diagnosis of sarcoid, and an elevated calciim narrows the differential diagnosis futher. Hyperparathyroidicm would act cause eymptome of breathlessness Coctidiodomycosis is ancn-infective fingal infection caused by inhalation of the spores of Coccidiodomycesis. I: usually presents with fever, cough, myalgia and rash. Ibis endemic mainly to the south western USA. and North Mexico. TB is the major differential in this case Hypercalcaemia and bilateral hilar lymphadenomegaly (BEL) however are in favour ofa diagnosis of sarcoid. 2252014 2:17:14 PM Mark this question & => Question Td : 29943 Question 12 of 30 A finding of upper lobe Abrosis is seen on a CKR. What is the most likely canse? a) Cryptogenic fibrosing alveoltis ) Rheumatoid artorits ) Ankylosing spondylitis 2) Scleroderma ¢) Systemic lupus erythematosus Question Explanation: Ceyptogenic fibrosing alveoliis (which may be associated with connective tissue disorders) affects lower lobes and is associated with clubbing Extrinsic allergic alveclitis is not associated with clubbing and affects middle/upper zones 2252014 2:17:14 PM Mark this question & => Question Td : 29943 Question 12 of 30 A finding of upper lobe Abrosis is seen on a CKR. What is the most likely canse? a) Cryptogenic fibrosing alveoltis ) Rheumatoid artorits Y © 6) Akylosing spondylitis 2) Scleroderma ¢) Systemic lupus erythematosus Question Explanation: Ceyptogenic fibrosing alveoliis (which may be associated with connective tissue disorders) affects lower lobes and is associated with clubbing Extrinsic allergic alveclitis is not associated with clubbing and affects middle/upper zones Mark this question & => Question Ti : 46699 Question 13 of 30 A.44 year cld female smoker, on steroids for seropositwe rheumatoid arthstis, presents with cough and hemoptysis. A periiler lang nodule is revealed on chest X-ray. What is the best way to establish the diagnosis? 2) Tomography +) Computed tomography scan of chest ©) Pulmonary finction test 4) Bronchoscopy ©) Tuberculin skin test Question Explanation: Tuberculosis (TE) infection usvally occurs intially in the upper part (lobe) of the lungs. The body's immune system, however, can stop the bacteria from contimiing to reproduce. Thus, the immune system can make the ing infection inactive (dormant). Oa the other hand, if the body's immune systema cannot contain the TB bacteria, the bacteria will reproduce (become active or reactivate) in the fangs and spread elsewhere in the body. Tr may talce many ronths from the time the infection initially gets into the hangs until symptoms develop. The usual symptoms thet oocur with an active TE irfectien are a generalized tiedaess or wealeness, weight, loss, fever, and night sweats, Ifthe infection in the lung worsens, then further symptoms can include coughing, chest pain, coughing up of sputum (material from the lungs) ancYor blood, and shortness of breath. Ifthe infection spreads beyond the lungs, the symptoms will depend pon the organs involved TB can be diagnosed in several diferent ways, incinding chest x-rays, analysis of sputum, and skin tests. Sometimes, the chest x-rays can reveal evidence of active tuberculosis pneumonia. Other times, the x rays may show scarring (Gbrosis) or hardening (calcification) in the lungs, suggesting that the TE is contained and inactive. Several types of skin tests are used to screen for TB infection. These so-called mberculin skin tests include the Tine test and the ‘Mantoux test, also known as the PPD (purified protein derivative) test In each of these tests, a stall ammount of purified extract from dead tuberculosis bacteria i injected uader the skin, Ifa person is infected with tuberculosis, a raised and reddened area will occur around the site of the test injection. Report An Error Mark this question & => Question Ti : 46699 Question 13 of 30 A.44 year cld female smoker, on steroids for seropositwe rheumatoid arthstis, presents with cough and hemoptysis. A periiler lang nodule is revealed on chest X-ray. What is the best way to establish the diagnosis? 2) Tomography £) Computed tomography scan of chest ©) Pulmonary fiction test & Bronchoscopy ¥ © ©) Tuberculn skin test Question Explanation: Tuberculosis (TE) infection usvally occurs intially in the upper part (lobe) of the lungs. The body's immune system, however, can stop the bacteria from contimiing to reproduce. Thus, the immune system can make the ing infection inactive (dormant). Oa the other hand, if the body's immune systema cannot contain the TB bacteria, the bacteria will reproduce (become active or reactivate) in the fangs and spread elsewhere in the body. Tr may talce many ronths from the time the infection initially gets into the hangs until symptoms develop. The usual symptoms thet oocur with an active TE irfectien are a generalized tiedaess or wealeness, weight, loss, fever, and night sweats, Ifthe infection in the lung worsens, then further symptoms can include coughing, chest pain, coughing up of sputum (material from the lungs) ancYor blood, and shortness of breath. Ifthe infection spreads beyond the lungs, the symptoms will depend pon the organs involved TB can be diagnosed in several diferent ways, incinding chest x-rays, analysis of sputum, and skin tests. Sometimes, the chest x-rays can reveal evidence of active tuberculosis pneumonia. Other times, the x rays may show scarring (Gbrosis) or hardening (calcification) in the lungs, suggesting that the TE is contained and inactive. Several types of skin tests are used to screen for TB infection. These so-called mberculin skin tests include the Tine test and the ‘Mantoux test, also known as the PPD (purified protein derivative) test In each of these tests, a stall ammount of purified extract from dead tuberculosis bacteria i injected uader the skin, Ifa person is infected with tuberculosis, a raised and reddened area will occur around the site of the test injection. Report An Error Mark this question & => Question Td : 50549 Question 14 of 30 A.20 year old boy presents with fever and non productive cough. He is otherwise healthy. Chest X-ray shows bilateral basal Infitration. The most appropriate treatment for suspected Mycoplasma pneumonia is 2) Erythromycin b) Doxyeyeline c) Levaquin 4) Penicillin Answer [FEIRIRIRHAN) other User's Explanation Report An For Question Explanation: Community acquired pneumonia develops in people with limted or no contact with medical institutions or settings. The most commonly identified pathogens are Streptococcus pneumoniae, Haemophilus induenzae, and atypical organisms (ie, Chlamydia pneumoniae, Mycoplasma pneumoniae, Legorella). Symptoms and signs are fever, cough, dyspnea, tachypnea, and tachycardia, Diagnosis is based on clinical presentation and chest X-ray. Treatwnent is with empirically chosen antibiotics. Prognosis is excellent for relatively young andlor healthy patients, but many pneumonias especially when caused by S. pneumoniae and influenza viruc, are fatal in older, sicker patients. Treatment for mycoplasma pneumonia is mostly with a macrolide such as erythromycin. Mark this question & => Question Td : 50549 Question 14 of 30 A.20 year old boy presents with fever and non productive cough. He is otherwise healthy. Chest X-ray shows bilateral basal Tnfilration, The most appropricte treatment for suspected Mycoplasma pneumonia is Y © a) Erythromycin £) Doxyeyeline ©) Levaquin 4) Pericilin Answer [FEIRIRIRHAN) other User's Explanation Report An For Question Explanation: Community acquired pneumonia develops in people with limted or no contact with medical institutions or settings. The most commonly identified pathogens are Streptococcus pneumoniae, Haemophilus induenzae, and atypical organisms (ie, Chlamydia pneumoniae, Mycoplasma pneumoniae, Legorella). Symptoms and signs are fever, cough, dyspnea, tachypnea, and tachycardia, Diagnosis is based on clinical presentation and chest X-ray. Treatwnent is with empirically chosen antibiotics. Prognosis is excellent for relatively young andlor healthy patients, but many pneumonias especially when caused by S. pneumoniae and influenza viruc, are fatal in older, sicker patients. Treatment for mycoplasma pneumonia is mostly with a macrolide such as erythromycin. Mark this question —& => Question Td : 50639 Question 15 of 30 A.65 year old woman has a lung tumor. Routine blood test reveals thatthe patient has hyporatemia You suspect the cause of hyponatremia is antidiuretic hormone (ADH) secretion by the tumor. This patient has which one of the following lung tumors? 2) Squamous cell carcinoma ') Adenocarcinoma ¢) Large cell carcinoma 4) Small cell carcinoma ¢) Bronchioalveolar carcinoma Answer (UEIRIRRRHNY) Other Users Explanason Repost An Exes Question Explanation: Small cell hing cancer can present with a paraneoplastic syndrome called STADH (syndrome of inappropriate anti-diuretic hormone) This caused by ectopic production of ADH. Abnomnalities in water and electrolyte balance, inching hyponatremia and calcium disturbances, may result from production of ADH and parathyroid hormone-like hormones from small cell lng cancer. Mark this question = > Question 15 of 30 Question Td : 50639 A.65 year old woman has a lung tumor. Routine blood test reveals thatthe patient has hyporatemia You suspect the cause of hyponatremia is antidiuretic hormone (ADH) secretion by the tumor. This patient has which one of the following lung tumors? 2) Squamous cell carcinoma ') Adenocarcinoma ¢) Large cell carcinoma Y © & Small cell carcinoma ¢) Bronchioalveolar carcinoma Answer (UEIRIRRRHNY) Other Users Explanason Repost An Exes Question Explanation: Small cell hing cancer can present with a paraneoplastic syndrome called STADH (syndrome of inappropriate anti-diuretic hormone) This caused by ectopic production of ADH. Abnomnalities in water and electrolyte balance, inching hyponatremia and calcium disturbances, may result from production of ADH and parathyroid hormone-like hormones from small cell lng cancer. “Mark this question & => Question 16 of 30 ‘When compared with younger adults, healthy older adults a) Fall acleep more quickly at bedtime ) Awakenless fequently dunng the night ©) Spendlese time awake in bed 4) Spend fewer hours in stages of deep sleep Question Explanation: ‘Nontnal older adults require less total sleep time. This change begins by the age of 50, and gradually increases with time. By age 75, total sleep time is reduce to 6 or 7 hours per night, and by age 85, 5 te 6 hours of total sleep time is biologically and physiologcally normal. Older adults take longer to fall asleep, avraken more frequently at night, spend more time awake in bed, and spend far less time in stages of deep sleep. “Mark this question & => Question 16 of 30 ‘When compared with younger adults, healthy older adults a) Fall acleep more quickly at bedtime ) Awakenless fequently dunng the night ©) Spendlese time awake in bed Y © 4) Spend fewer hours in stages of deep sleep Question Explanation: ‘Nontnal older adults require less total sleep time. This change begins by the age of 50, and gradually increases with time. By age 75, total sleep time is reduce to 6 or 7 hours per night, and by age 85, 5 te 6 hours of total sleep time is biologically and physiologcally normal. Older adults take longer to fall asleep, avraken more frequently at night, spend more time awake in bed, and spend far less time in stages of deep sleep. 2:25/2014 2:18:23 PM Mark this question & => Question Td : 55632 Question 17 of 30, In HIV infected patient, the threshold for inating treatment for tuberculosis after PPD screenings induration greater than or equal to a) 2.5mm. 6) 5.0mm 7.5mm. &) 10 min 6) 15mm newer [UERIRIBEN) otrerveors Explan Question Explanation: ‘Treatment should be initiated when a PPD causes induration > 5mm in a patient with HIV infection. Report An Error 2:25/2014 2:18:23 PM Mark this question & => Question Td : 55632 Question 17 of 30, In HIV infected patient, the threshold for inating treatment for tuberculosis after PPD screenings induration greater than or equal to 225mm oY © b)5.0mn 7.5mm. 4) 10 mia 6) 15mm newer [UERIRIBEN) otrerveors Explan Question Explanation: ‘Treatment should be initiated when a PPD causes induration > 5mm in a patient with HIV infection. Report An Error 2/25/2014 2:18:33 PM ‘Mark this question & => Question Id : 58062 Question 18 of 30 A boy aged 11 year fell down a steep hill while riding his scooter. He presented in the ED with a fractured wrist and a lacerated spleen, which required surgical removal. Two years following splenectomy he is diagnosed with bacterial pneumenia, Which bacteria is the most likely cause of this patient’s pneumonia? a) Escherichia coli ») Klebsiella pneumoniae c) Neisseria meningitidis © Staphylococcus aureus ) Streptococcus pneumoniae Question Explanation: This pation! underwent a splenectomy afier his injury, and thus. is susceptible to encapsulated organisms. S. pneumoniae is the most important pathogen in asplenic children. Less common causes ofinfection include Hemophis infuenzae type B, E. coli, Staph aureus, and gram-negative bacill such as Salmonella, Klebsiella and Pseudomonas. Asplenic children are also at increased risk for fatal malaria and babesiosis. This child should receive a pneumococcal vaccine 2/25/2014 2:18:33 PM ‘Mark this question & => Question Id : 58062 Question 18 of 30 A boy aged 11 year fell down a steep hill while riding his scooter. He presented in the ED with a fractured wrist and a lacerated spleen, which required surgicel removal. Twro years following splenectomy he is diagnosed with bacterial pnoumonia, Which bactoria is the most likely cause of this patient's pneumonia? a) Becherichia coli ) Klebsiella pneumonias «) Neisseria meningitidis @) Staphylococcus aureus WV © 2) Streptococcus pneumoniae Question Explanation: This pation! underwent a splenectomy afier his injury, and thus. is susceptible to encapsulated organisms. S. pneumoniae is the most important pathogen in asplenic children. Less common causes ofinfection include Hemophis infuenzae type B, E. coli, Staph aureus, and gram-negative bacill such as Salmonella, Klebsiella and Pseudomonas. Asplenic children are also at increased risk for fatal malaria and babesiosis. This child should receive a pneumococcal vaccine 2:25/2014 2:18:46 PM ‘Mark this question & => Question Td : 61339 Question 19 of 30 A 60-year-old female is noted to have labored breathing six days after admission for low back pain which required complete bed rest, A pulmonary embolism is suspected. The most diagnostic test for determination of pulmonary embolism in this patient would be a) Arterial blood gas. b) Duplex ultrasound of the lower extremities ©) Ventilation-perfusion soan, ) 4) Pulmonary angiography. @) TY heparin trial Anewor [UEIRIEARY) omer teers Explanation Report An Error Question Explanation: Pulmonary angography remains the gold standard for diagnosis of pulmonary embolism. While ventlation-perfision scans (C) are obtained more often than pulmonary angiograms in the work-up of pubnonary embolism, this is because of the increased risks associated with pulmonary angiography. Studies have shown pulmonary angiography to be more sensitive and specific than ventlation-perfision scanning (C)) A duplex ultrasound (B) and TW herparin trial (E) are incorrect for the reasons mentioned above An arterial blood gas (A) is an important part of the iniial work-up, but is not diagnosis. 2:25/2014 2:18:46 PM ‘Mark this question & => Question Td : 61339 Question 19 of 30 A 60-year-old female is noted to have labored breathing six days after admission for low back pain which required complete bed rest, A pulmonary embolism is suspected. The most diagnostic test for determination of pulmonary embolism in this patient would be a) Arterial blood gas. b) Duplex ultrasound of the lower extremities ©) Ventilation-perfision scan, ) Y¥ © 4) Pulmonary angiography. €) TV heparin tril. Anewor [UEIRIEARY) omer teers Explanation Report An Error Question Explanation: Pulmonary angography remains the gold standard for diagnosis of pulmonary embolism. While ventlation-perfision scans (C) are obtained more often than pulmonary angiograms in the work-up of pubnonary embolism, this is because of the increased risks associated with pulmonary angiography. Studies have shown pulmonary angiography to be more sensitive and specific than ventlation-perfision scanning (C)) A duplex ultrasound (B) and TW herparin trial (E) are incorrect for the reasons mentioned above An arterial blood gas (A) is an important part of the iniial work-up, but is not diagnosis. 2i25:2014 2:19:03 PM ‘Mark this question & => Question Ti : 64913 Question 20 of 30 A. 16 year old boy has been taking medication over the past 10 years that has successfly controlled his severe asthma. He has had tno exacerbations in the past two years. His parents come to see the boy's playsician seeking her advice. The boy recently says that he doer not wantto take hic asthma medication any longer Instead, he believes that changing hic dist to one that ic free of al “toxinc" ic all that is requited to limit his exacerbations, The parents want to know what they should do, What should be the physician's next action? 8) Arrange to speake with the boy and convince hita that he aust stay on his medicetion +) Arrange to speak with the boy and ask him the reasons for his decision ) Have the bey evaluated by a pulmonologist and fellow the resommendations ofthe specialist 6) Take the bey offliis medication and bey to a newly available medication 6) Tell peronts that you will switch the boy to a newly avaiable medication 4) Tell the parents that itis essential that their soa stay his medications and that must convince him to do so Answer [Extension Other User's Explanation Report An Error Question Explanation: Before reaching any treatment secommendation or encouraging either the parents or the boy to take any particular course of action, the physician needs more information. All he knows is the boy's views as represented by the parents. The boy is old enough to exprece himself and articulate bis reasons, co the physician should go directly to the source. Getting enough information before you act is a good rile for the exam, and a good mule for medical practice. Speakang with the boy is the right idea. But walking into the conversation with an agenda of selling a particular course of action (choice A) is likely to lead to confrontation, not communication. Be open-minded anc’ hear what the boy's experience and reasons hhave bea. This is your patient, and you chould malce the required desisione. The rule: “Mever pase off" appliec here. Giving your patient to someone else (choice C) will yew sarely be aright answer on the Step 1 exam. ‘Taking the boy off his medication (choice D) is premature, Talk to the boy and get his views before deciding on any course of action. Choices B and F not only force a solution before all the relevant information is known, they force the parents to do the work of the physician, namely, talking directly with the patient. Yee, the parents, at thie age, have the final cay regarding treatment choices, but cooperation of the patient is essertial for adequate treatment of asthma. 2i25:2014 2:19:03 PM ‘Mark this question & => Question Ti : 64913 Question 20 of 30 A. 16 year old boy has been taking medication over the past 10 years that has successfly controlled his severe asthma. He has had tno exacerbations in the past two years. His parents come to see the boy's playsician seeking her advice. The boy recently says that he doer not wantto take hic asthma medication any longer Instead, he believes that changing hic dist to one that ic free of al “toxinc" ic all that is requited to limit his exacerbations, The parents want to know what they should do, What should be the physician's next action? 8) Arrange to speake with the boy and convince hita that he aust stay on his medicetion Y © ¥) Atrange to speak with the boy and ask him the reasons for his decision ) Have the bey evaluated by a pulmonologist and fellow the resommendations ofthe specialist 6) Take the bey offliis medication and bey to a newly available medication 6) Tell peronts that you will switch the boy to a newly avaiable medication 4) Tell the parents that itis essential that their soa stay his medications and that must convince him to do so Answer [Extension Other User's Explanation Report An Error Question Explanation: Before reaching any treatment secommendation or encouraging either the parents or the boy to take any particular course of action, the physician needs more information. All he knows is the boy's views as represented by the parents. The boy is old enough to exprece himself and articulate bis reasons, co the physician should go directly to the source. Getting enough information before you act is a good rile for the exam, and a good mule for medical practice. Speakang with the boy is the right idea. But walking into the conversation with an agenda of selling a particular course of action (choice A) is likely to lead to confrontation, not communication. Be open-minded anc’ hear what the boy's experience and reasons hhave bea. This is your patient, and you chould malce the required desisione. The rule: “Mever pase off" appliec here. Giving your patient to someone else (choice C) will yew sarely be aright answer on the Step 1 exam. ‘Taking the boy off his medication (choice D) is premature, Talk to the boy and get his views before deciding on any course of action. Choices B and F not only force a solution before all the relevant information is known, they force the parents to do the work of the physician, namely, talking directly with the patient. Yee, the parents, at thie age, have the final cay regarding treatment choices, but cooperation of the patient is essertial for adequate treatment of asthma. ‘Mark this question = => Question Ta : 64995 Question 21 of 30 A.44 year old man has probleme sleeping for the past year. He typical goze to bed after watching the news at 11:30PM and no trouble falling asleep, however, he keeps awakening and abays feels tired all day while at work. His primary physician refers him to the university’ sleep center where an overnight polysomnogram is perforined, during the study, he is found to have decreased stage 1 sleep. and decreased delta and REM sleep. He has frequent awakenings associated with gasping breaths, oxygen desaturations (monitored by pulee oximetry), and bradycardia. What would be the best iritial medical therapy for thie problem? a) Behavioral therapy ) Imipranine ) Tonsillectomy and adenoidectomy 4) Triazoleen @) Weight loss and use of continuous positive always pressure (CPAP) Answer | Brotanation Other User's Explanation Report An Error Question Explanation: ‘This isa classic descnpton of a person with obstructive sleep apnea (OSA) syndrome. The patientis typically a middle-age, cbese sale who snores loudly during sleep. The problem stems from obstruction from tissues of the aasopharymx and hypopharyns. The obstruction recuits in significant periods of apnea accociated with arterial hyp oxemia and bradycardia. The patient may develop both system and pulmonary hypertension andi at a high risk of sudden death during sleep due to severe hypoxemia and arrhythmias Ofien the patient will have a long apneic period followed by gasping respirations and awakening, This cycle repeats over the course of the night and therefore. the petient actually gets litle sleep resulting in dastime somnolence. On a pelysomnogram (mutichannel recording of sleep), one sees a decrease in stage 1 sleop. This isthe fist stage of non REM sleep. Most adults spend most ofthe sleep cycle in stage 2 REM sleep. Stage 3 and 4 NREM sleep is deep sleep and togetter constitutes celta sleep. The time in REM sleep gets longer as the night progresses and therefore occurs mostly during the last half of the night Tn OS.A, we also see a decrease in detta sleep and REM sleep is also decreased. The inital medical treatment for OSA is weiaht loss with a trial of nasal prong or face maske continuous positive airway pressure (CPAP). Mary are noncompliant with this therapy because the presence of the CPAP is felt to be uncomfortable and prevents the patient from falling asleep. Ultimately, one of several surgical procedures involving the semoval of tissue fiom the naso- and hypopherynx may be performed. Behavioral modification (choice A) has no role in the management of OSA. Tis the most effective therapy fr insorania, Imipramine (choice B) may be used along with tehavior modification in children with noctumal enuresis, The most common cause of obstructive sleep apnea in children is adeno tonsilar hypertrophy, and the treatment of choice is adenctonsilectomy (choice C) Triazolam (alcian) (choice D) may be used in combination with behavioral modification for insomnia ‘Mark this question = => Question Ta : 64995 Question 21 of 30 A.44 year old man has probleme sleeping for the past year. He typical goze to bed after watching the news at 11:30PM and no trouble falling asleep, however, he keeps awakening and abays feels tired all day while at work. His primary physician refers him to the university’ sleep center where an overnight polysomnogram is perforined, during the study, he is found to have decreased stage 1 sleep. and decreased delta and REM sleep. He has frequent awakenings associated with gasping breaths, oxygen desaturations (monitored by pulee oximetry), and bradycardia. What would be the best iritial medical therapy for thie problem? a) Behavioral therapy ) Imipranine ) Tonsillectomy and adenoidectomy 4) Triazoleen Y © 6) Weight loss and use of continuous positive always pressure (CPAP) Answer | Brotanation Other User's Explanation Report An Error Question Explanation: ‘This isa classic descnpton of a person with obstructive sleep apnea (OSA) syndrome. The patientis typically a middle-age, cbese sale who snores loudly during sleep. The problem stems from obstruction from tissues of the aasopharymx and hypopharyns. The obstruction recuits in significant periods of apnea accociated with arterial hyp oxemia and bradycardia. The patient may develop both system and pulmonary hypertension andi at a high risk of sudden death during sleep due to severe hypoxemia and arrhythmias Ofien the patient will have a long apneic period followed by gasping respirations and awakening, This cycle repeats over the course of the night and therefore. the petient actually gets litle sleep resulting in dastime somnolence. On a pelysomnogram (mutichannel recording of sleep), one sees a decrease in stage 1 sleop. This isthe fist stage of non REM sleep. Most adults spend most ofthe sleep cycle in stage 2 REM sleep. Stage 3 and 4 NREM sleep is deep sleep and togetter constitutes celta sleep. The time in REM sleep gets longer as the night progresses and therefore occurs mostly during the last half of the night Tn OS.A, we also see a decrease in detta sleep and REM sleep is also decreased. The inital medical treatment for OSA is weiaht loss with a trial of nasal prong or face maske continuous positive airway pressure (CPAP). Mary are noncompliant with this therapy because the presence of the CPAP is felt to be uncomfortable and prevents the patient from falling asleep. Ultimately, one of several surgical procedures involving the semoval of tissue fiom the naso- and hypopherynx may be performed. Behavioral modification (choice A) has no role in the management of OSA. Tis the most effective therapy fr insorania, Imipramine (choice B) may be used along with tehavior modification in children with noctumal enuresis, The most common cause of obstructive sleep apnea in children is adeno tonsilar hypertrophy, and the treatment of choice is adenctonsilectomy (choice C) Triazolam (alcian) (choice D) may be used in combination with behavioral modification for insomnia 2/25:2014 2:19:35 PM “Mark this question €& => of 30 Question “What is the earliest acid-base abnormality that occurs in most patients with salicylate intoxication? a) Respiratory alkalosis, ) Metabolic alkalosis, ©) Respiratory acidosis. 4) Metabolic acidosis, €) Mixed respiratory acidosis and metabolic alkalosis. Question Explanation: Respiratory alkalosis is the earliest acid-base abnormality seen in salicylate intoxication. This occurs because of the stimulatory effect of salicylates on the respiratory center in the brain. The other choices are not correct. 2/25:2014 2:19:35 PM “Mark this question €& => of 30 Question “What is the earliest acid-base abnormality that occurs in most patients with salicylate intoxication? Y © a) Respiratory alkalosis ) Metabolic alkalosis, ©) Respiratory acidosis. 4) Metabolic acidosis, €) Mixed respiratory acidosis and metabolic alkalosis. Question Explanation: Respiratory alkalosis is the earliest acid-base abnormality seen in salicylate intoxication. This occurs because of the stimulatory effect of salicylates on the respiratory center in the brain. The other choices are not correct. 2125/2014 2:19:19 PM “Mark this question => Question Id : 94262, Question 23 of 30 In apatient with mild persistent asthma, treatment with a low dose inhled corticosteroid alone would be the preferred treatment for a patient whose asthina symptoms occur 8) Btimes aweek b) Once daily ©) Daily ) Contimousty ¢) 2 times a week, but only with exercise Question Explanation: Low dose inhaled corticosteroids are preferred for patients with mild persistent asthma, defined as the occurence of symptoms more than twice a week but less than once a day. Patients with severe persistent asthma, defined as continual symptoms, should be treated with high dose inhaled corticasteraids and long acting inhaled Reta2 agorists. Maderate asthma, defined as daily symptoms, should tbe treated with low- to mediurn-dose inhaled corticosteroids and long-acting inhaled Beta2-agonists. Mild intermittent asthma, with symptoms 1 of 2 days per week, dees not requite daily medication, jon Report An Error 2125/2014 2:19:19 PM “Mark this question => Question Id : 94262, Question 23 of 30 Jn apatient with mild persistent asthuna, weatment with a low dose inhaled corticosteroid alone would be the preferred treatment for a patient whose astama symptoms occur SY © a) 3 times aweek ) Once daily ©) Daily 6) Continuously ¢) 2 times a week, but only with exercise Question Explanation: Low dose inhaled corticosteroids are preferred for patients with mild persistent asthma, defined as the occurence of symptoms more than twice a week but less than once a day. Patients with severe persistent asthma, defined as continual symptoms, should be treated with high dose inhaled corticasteraids and long acting inhaled Reta2 agorists. Maderate asthma, defined as daily symptoms, should tbe treated with low- to mediurn-dose inhaled corticosteroids and long-acting inhaled Beta2-agonists. Mild intermittent asthma, with symptoms 1 of 2 days per week, dees not requite daily medication, jon Report An Error 2125/2014 2:20:04 PM Mark this question eo= Question Id : 95432, Question 24 of 30 ‘A patients is brought to the emergency room after a seizure, There is no history of head trauma. Serum electrolyte stucies demonstrate serum sodium of 128mEq/L. The urine osmolarity is higher than the serum osmolarity. CXR shows a hing mass. Head ‘MRI does not reveal any atmermalty. The above mentioned electralyte imbalance is caused by which of the fallowing cancer type? a) Adrenocortical tamor ») Pheorhromocytomna ©) Large cell carcinoma & Srrall cell carcinoma ©) Squamous cell carcinoma Question Explanation: ‘The pationt has SLADHT (syndrome of inappropriate antidiuretic hormone secretion) which can be caused by ectopic ADHT secretion bby sinall cel carcinomas ofthe lung CNS disorders, chronic pulmonary disease, and certain crugs feature (when severe), and serum hypo osmolarity with urine osmolarity greater than serum osmolarity. Adrenocortical tunors may secrete adrenocortical hormones: ahicocorticoids (orimerily cortsol) mineralocotticoids (primarily aldosterone, and androgens (primarily dehydroepiandrosterone and androstenedione), However, it does net secret ADH, which is the cause of hyponatremia in this patient. Hyperaldosteronism causes hypernatremia and hypokalemia. Pheochromocyroma is a catecholamine secreting mmor of chromaffin cells, which are found within the adrenal medulla or paraganghoma found outside the adrenal gland, Pheochromocyroma usually secretes a combination of noradrenaline and adrenaline. and patients present with hypertension, headache palpitetions. and tachycardia. Large cell carcinoma is an aggressive, undifferentiated lung neoplasm. Squamous cells carcinoma is bronchogenic orign and has a strong association with smoking, and hypercalcemia. 2125/2014 2:20:04 PM Mark this question eo= Question Id : 95432, Question 24 of 30 ‘A patients is brought to the emergency room after a seizure, There is no history of head trauma. Serum electrolyte stucies demonstrate serum sodium of 128mEq/L. The urine osmolarity is higher than the serum osmolarity. CXR shows a hing mass. Head ‘MRI does not reveal any atmermalty. The above mentioned electralyte imbalance is caused by which of the fallowing cancer type? a) Adrenocortical tamor ») Pheorhromocytomna ©) Large cell carcinoma Y © & Srrall cell carcinoma ©) Squamous cell carcinoma Question Explanation: ‘The pationt has SLADHT (syndrome of inappropriate antidiuretic hormone secretion) which can be caused by ectopic ADHT secretion bby sinall cel carcinomas ofthe lung CNS disorders, chronic pulmonary disease, and certain crugs feature (when severe), and serum hypo osmolarity with urine osmolarity greater than serum osmolarity. Adrenocortical tunors may secrete adrenocortical hormones: ahicocorticoids (orimerily cortsol) mineralocotticoids (primarily aldosterone, and androgens (primarily dehydroepiandrosterone and androstenedione), However, it does net secret ADH, which is the cause of hyponatremia in this patient. Hyperaldosteronism causes hypernatremia and hypokalemia. Pheochromocyroma is a catecholamine secreting mmor of chromaffin cells, which are found within the adrenal medulla or paraganghoma found outside the adrenal gland, Pheochromocyroma usually secretes a combination of noradrenaline and adrenaline. and patients present with hypertension, headache palpitetions. and tachycardia. Large cell carcinoma is an aggressive, undifferentiated lung neoplasm. Squamous cells carcinoma is bronchogenic orign and has a strong association with smoking, and hypercalcemia. 2i25'2014 2:20:17 PM Mark this question & => Question 25 of 30 ‘The incidence of hospitalizations for asthma is compared with the level of selected airbome pollutants in five counties. The population demographics and climates of the countries are similar. This is an example of which of the following? 2) A cross sectional study. b) A cohort study. ©) An ecological study. d) A randomized clinical inal. ©) Community surveillance Question Explanation: By definition, comparisons of summary statistics for exposnres and diseases are ecologic. If sach information is collectediin specific individuals, the stady is either cross sectional, case control, or cohortin type. A randomized clinical tral is one type of cchort study. ‘The cross sectional study involves simultaneous collection of exposure and disease data on a sample representative of the population. Case contrel snudies compare exposures in a group of indviduals with 2 specific condition with a group of control subjects drawn fiom the same population Clinical trials study responses in a group of subjects randomized to different treatment exposures. Community surveillance is not a specific type of stdy, but rellects efforis to tack disease activity in a population. 2i25'2014 2:20:17 PM Mark this question & => Question 25 of 30 ‘The incidence of hospitalizations for asthma is compared with the level of selected airbome pollutants in five counties. The population demographics and climates of the countries are similar. This is an example of which of the following? 2) A cross sectional study. b) A cohort study. Y © 0) Anecological study. d) A randomized clinical inal. ©) Community surveillance Question Explanation: By definition, comparisons of summary statistics for exposnres and diseases are ecologic. If sach information is collectediin specific individuals, the stady is either cross sectional, case control, or cohortin type. A randomized clinical tral is one type of cchort study. ‘The cross sectional study involves simultaneous collection of exposure and disease data on a sample representative of the population. Case contrel snudies compare exposures in a group of indviduals with 2 specific condition with a group of control subjects drawn fiom the same population Clinical trials study responses in a group of subjects randomized to different treatment exposures. Community surveillance is not a specific type of stdy, but rellects efforis to tack disease activity in a population. 2:25/2014 2:20:30 PM ‘Mark this question = => Question Id : 109540. Question 26 of 30 ‘Which of the following does NOT produce aspiration related injuries? a) Hydrocarbons. ) Lipids c) Water. 4) Food. e) Surfactant. Question Explanation: Surfactant creates a uniformly distributed phospholipids monolayer over the aqueous sub-phase lining the alveolar surface, thereby decreasing the surface tension. Hydrocarbon aspration causes pneumonitis, especially in children, Lipids, such as mineral oi, produce a chronic inflammatory response, especially following chronic aspiration. Aspiration of inert materials, such as water. produce injury through asphysation Small food particles, in the absence of gastric acid, can produce fibrotic, granulomatous pulmonary lesions. Larger food particles may cause death by asphyxiation, 2:25/2014 2:20:30 PM ‘Mark this question = => Question Id : 109540. Question 26 of 30 ‘Which of the following does NOT produce aspiration related injuries? a) Hydrocarbons. ) Lipids ©) Water. ) Food. Y © e) Surfactant. Question Explanation: Surfactant creates a uniformly distributed phospholipids monolayer over the aqueous sub-phase lining the alveolar surface, thereby decreasing the surface tension. Hydrocarbon aspration causes pneumonitis, especially in children, Lipids, such as mineral oi, produce a chronic inflammatory response, especially following chronic aspiration. Aspiration of inert materials, such as water. produce injury through asphysation Small food particles, in the absence of gastric acid, can produce fibrotic, granulomatous pulmonary lesions. Larger food particles may cause death by asphyxiation, 2:25/2014 2:20:41 PM Mark this question & => Question Td : 126135 Question 27 of 30 A 42 year old construction worker who lives in Arizona presents with fever, weightloss, cough, and productive sputum, Examination shows lest leg bone tenderness and pacatracheal adenopethy on X-ray Mild eosinophilia was also prosert. PPD sleia test ard sputum for acid fast bacill were negative. The most likely diagnosis is which one of the following? 2) Aspergillosis ) Actinomycosis ©) Coccidicidomycosis 4) Tuberculosis ©) Muconmycosis Question Explanation: ‘This fungal infection is endemic in the southwestem states and infection occurs by inhalation of arthrospores. Patients can present with filer adenopathy and mild eosinophilia Seroconversion may not occur for up to eight weeks. Skin test converts between the third and twenty Bist day. Aspergillosis occurs by inhalation of spores in immunocompromised patients. A cavity or fingel ball may be seen on the X-ray. Fleeting pulmonary infiltrates may also be scea, as well as eosinophilia, Actinomycosis is caused by infection with an anaerobic Gram-positive branching bacteria by a break in the rmcosa. Cervicofacial, thoracic, pelvic, and abdominal organs are involved. Tuberculosis is unlikely inthis patient, who is PPD negative. Mucormycosis isa fingal infection which mainly occurs in diabetics and seversly inmunccompromised patients Report An Error 2:25/2014 2:20:41 PM Mark this question & => Question Td : 126135 Question 27 of 30 A 42 year old construction worker who lives in Arizora presents wth fever, weight loss, cough, and productive spunam, Examination shows lest leg bone tenderness and pacatracheal adenopethy on X-ray Mild eosinophilia was also prosert. PPD sleia test ard sputum for acid fast bacill were negative. The most likely diagnosis is which one of the following? 2) Aspergillosis ) Actinomycosis Y © ¢) Coccidicidomycosis & Tuberculosis ©) Muconmycosis Question Explanation: ‘This fungal infection is endemic in the southwestem states and infection occurs by inhalation of arthrospores. Patients can present with filer adenopathy and mild eosinophilia Seroconversion may not occur for up to eight weeks. Skin test converts between the third and twenty Bist day. Aspergillosis occurs by inhalation of spores in immunocompromised patients. A cavity or fingel ball may be seen on the X-ray. Fleeting pulmonary infiltrates may also be scea, as well as eosinophilia, Actinomycosis is caused by infection with an anaerobic Gram-positive branching bacteria by a break in the rmcosa. Cervicofacial, thoracic, pelvic, and abdominal organs are involved. Tuberculosis is unlikely inthis patient, who is PPD negative. Mucormycosis isa fingal infection which mainly occurs in diabetics and seversly inmunccompromised patients Report An Error Mark this question >= Question Td : 161323 Question 28 of 30 ‘A. 22 year old man has fever, headache, sore throat, malaise, and myalgia, CAR reveals the presence of lower lobe bilateral infilrates After drawing blood for investigations, an extra heparinized tube of bloodis taken and placed onice. Upon cooling, the red thlood cells are noted to have agghutinated. The mechanism of action of the appropriate drug prescribed is a) Competitive inhibition of para-amainobenzois acid ) Inhibition of bacterial cell wall synthesis ©) Inhibition of DNA gyrase 4) Iiveversible binding to the 30S subunit of bacterial ribosomes «) Reversible binding to the 50S subunit of bacterial rivosomes Anower (EEIRISHON) Other User's Explanas Question Explanation: Jin this question, the student raust, make the correct diagnosis. Select the mest appropriate treatment, and then determine the mechanism of action of the treatment of chcice. The patient described in this questica has an atypical type of pneumonia that is characterized by acute inflammation within the alveoli ofthe lungs. Typical symptoms inclade fever, cough, bronchitis, sore tarot, headache, and tiredness. A common result of Mycoplasma infection is pneumonia (sometimes called "walking pneumonia" since i often does not require hospitalization). Otitis media may also result in patients as well. Symptoms may persist for a few days to more than a month, From alaboratory perspective, infection with Mycoplacma pneumoniae rerults in the production of nonapeoific cold JgM antibody. This antibody generally reacts with red cells having the "T’ antigen, which is often found in most adult red blood cells “When the patient's blood is cooled, it will aggutinate, The macrolides are most commonly used in the treatment of mycoplasmal pneumonia infections. These agents, such as atithromycin and erythromycin, reversibly bind to the 508 subunit of bacterial sibascmes, suppressing bacterial protein synthesis. Sulfonamie antbiotics, such as suamethesazale-trimethoprim, act through the competitive inhibiticn of para-amincbenzoic acid (PABA), thereby inhibiting folic acid biosynthesis required for bacterial growrh. Penicilins and cephalosporins are believed to act via inhibition of bacterial cell wall synthesis. Both antibacterial classes bind to one or more of the peniilln binding proteins located on the cell walls of susceptble organisis, thus resulting m the inkibtion of the third and final stage of bacterial cell wall synthesis. Quinolone antibiotics, such as ciprofloxacin, inhibit DIVA gyrase which is an enzyme necessary for bacterial TVA replication and repair, Aminoglycosides, sush ac gentamicin, irreversibly bind to the 30S subunit of ‘bactetial ribosomes which inhibits bacicrial protein synthesis n Report An Error Mark this question >= Question Td : 161323 Question 28 of 30 ‘A. 22 year old man has fever, headache, sore throat, malaise, and myalgia, CAR reveals the presence of lower lobe bilateral infilrates After drawing blood for investigations, an extra heparinized tube of bloodis taken and placed onice. Upon cooling, the red thlood cells are noted to have agghutinated. The mechanism of action of the appropriate drug prescribed is a) Competitive inhibition of para-amainobenzois acid ) Inhibition of bacterial cell wall synthesis ©) Inhibition of DNA gyrase 4) Iiveversible binding to the 30S subunit of bacterial ribosomes Y © 6) Reversible binding to the 50S subunit of bacterial ribosomes Anower (EEIRISHON) Other User's Explanas Question Explanation: Jin this question, the student raust, make the correct diagnosis. Select the mest appropriate treatment, and then determine the mechanism of action of the treatment of chcice. The patient described in this questica has an atypical type of pneumonia that is characterized by acute inflammation within the alveoli ofthe lungs. Typical symptoms inclade fever, cough, bronchitis, sore tarot, headache, and tiredness. A common result of Mycoplasma infection is pneumonia (sometimes called "walking pneumonia" since i often does not require hospitalization). Otitis media may also result in patients as well. Symptoms may persist for a few days to more than a month, From alaboratory perspective, infection with Mycoplacma pneumoniae rerults in the production of nonapeoific cold JgM antibody. This antibody generally reacts with red cells having the "T’ antigen, which is often found in most adult red blood cells “When the patient's blood is cooled, it will aggutinate, The macrolides are most commonly used in the treatment of mycoplasmal pneumonia infections. These agents, such as atithromycin and erythromycin, reversibly bind to the 508 subunit of bacterial sibascmes, suppressing bacterial protein synthesis. Sulfonamie antbiotics, such as suamethesazale-trimethoprim, act through the competitive inhibiticn of para-amincbenzoic acid (PABA), thereby inhibiting folic acid biosynthesis required for bacterial growrh. Penicilins and cephalosporins are believed to act via inhibition of bacterial cell wall synthesis. Both antibacterial classes bind to one or more of the peniilln binding proteins located on the cell walls of susceptble organisis, thus resulting m the inkibtion of the third and final stage of bacterial cell wall synthesis. Quinolone antibiotics, such as ciprofloxacin, inhibit DIVA gyrase which is an enzyme necessary for bacterial TVA replication and repair, Aminoglycosides, sush ac gentamicin, irreversibly bind to the 30S subunit of ‘bactetial ribosomes which inhibits bacicrial protein synthesis n Report An Error 2125/2014 2:21:15 PM “Mark this question & => Question 29 of 30 A 68 year old male with emphysema is on home oxygen therapy and has no new complaints. He smoked one pack of cigarettes a day for 40 years but quit 7 years ago when his emphysema started to ‘slow’ hin down. ‘Which fiding is likely present in him and is related to the increased compliance caused by his dsease? @) Barrel chest ) Chronic cough ©) Excessive mucus production 6) Long, slow, deep breathing pattern ©) Pink face Question Explanation: ‘A barrel chest with increased antetior/posterior diameter is commonly observed in patients with longstanding. severe emphysema This change in chest shape occurs because these patients, who have high compliance of the lung proper, tend to function with their Jungs to some degree ‘over-inflated! compared with people who have normal lung compliance. This over-inflation limits their abilty to take further deep breaths. (The "balloon" of emphysematous lung remains compliant, but the "box" of the chest wallis not very compliant and limits the volume of air that can be inhaled) Patients with moderately severe emphysema are able to maintain an adequate hing ventilation by taking many short breaths; this physiology is sometimes expressed by describing these patients as "pinke paffers", Chronic cough in emphysema patients is not directly related to the change in compliance. Excessive mucus production is more cheracteristic of chronic bronchitis then of emphysema Report An Error 2125/2014 2:21:15 PM “Mark this question & => Question 29 of 30 A.68 year old male with emphysema is on home oxygen iherapy and has no new complaints. He smoked one pack of cigarettes a day for 40 years but quit 7 years ago when his empiysema started to ‘slow" him down, Which fiding is likely present in him and is related to the increased complance caused by his dsease? Y © &) Barrel chest +) Chronic cough 6) Excessive mucus production 4) Long, slow, deep breathing pattern 6) Pink face Question Explanation: ‘A barrel chest with increased antetior/posterior diameter is commonly observed in patients with longstanding. severe emphysema This change in chest shape occurs because these patients, who have high compliance of the lung proper, tend to function with their Jungs to some degree ‘over-inflated! compared with people who have normal lung compliance. This over-inflation limits their abilty to take further deep breaths. (The "balloon" of emphysematous lung remains compliant, but the "box" of the chest wallis not very compliant and limits the volume of air that can be inhaled) Patients with moderately severe emphysema are able to maintain an adequate hing ventilation by taking many short breaths; this physiology is sometimes expressed by describing these patients as "pinke paffers", Chronic cough in emphysema patients is not directly related to the change in compliance. Excessive mucus production is more cheracteristic of chronic bronchitis then of emphysema Report An Error ‘Mark this question & Question Td : 212569 Question 30 of 30. A.B year old woman has easy bruisabiliy, cutancous striae and excessive hair growth on her face. She has been smoking 1 pack of cigarette a day for 8 years. Her BP is 150/90 mmlg. She has elevated cortisol levels, which are not suppressed when she is aver. high dose dexamethasone. ACTH levels are high After confirmation of the diegnosis, which one is the preferred reatment? a) Acrenalectomy ') General support only ) Mitotane Pnsumonectomy 2) Radiction and chemotherapy direct at lung cancer Answer | Explanation Other User's Explanation Report An Error Question Explanation: The endocrine workup is indicative of ectopic ACTH production, and the most Hely ste is stall cell carcinoma of the lung, Ifthe high dose dexamethasone suppression test had suppressed 50% or more of the ACTH, then the most likely cause would be a primary pituitary adenoma (Cushing disease). In this patient however, the ACTH was not suppressed, therefore, this is suspicious for an ectopic ACTH producing tumor, ether in the lung or the adrenals. The nest step in diagnosis is to check the ACTH levels; flower then normal, the most likely cause for the ectopia would be an adrenal neoplasm. But £ the ACTH is bigher than nomal (asin this case), the mozt lely cause for the ectopia would be lng neoplasia (specifically small cell carcinoma). The iung cancer is what is goingto kill this woman, not the endocrine manifestations of the tumor Smell cell carcinoma is considered to be inoperable because it is a repidly growing tumor that metastasizes early and has an overall poor prognesis. Longer survival can be obtained with radiation and chemotherapy. Adrenalectomy (A) would address the endocrine problem by depsiving the ectopic ACTH ofits target land. But the lethal disease here is the lung cancer thet is causing the parancoplastic syndrome, not an adrenal adenoma, Adrenalectomies are quite useful, and offen curative in patients who have an established unilateral adrenal adenoma, Bilateral adrenalectomy is required for bilateral micronodular and most patients with macronodular adrenal hyperplasia. General support only (B) would lead to death in about 2 months. Radiation and chemotherapy can prolong survival by approximetely 2 years. Mitotane (C) is incorrect, Adrenal carcinomas producing ectopic ACTH almost invariably recur and usually do not respond to either irradiation or chemotherapy Mitotane offers suck patients the only hope of cure. In patients with inoperable, residual, or recurrent adrenal neoplastic disease, its usually palliative and does not prolong life. In patienis with Cushingoid features, alter the high dexamethasone fails to suppress ACTH, the ACTH levels should be taken to differentiate lung cancer versus adrenal adenomas as the cause of the ectopic ACTH. This patient has an increased ACTH level, which indicates that the likely origin of the ectopia is a fang neoplasia, not an adrenal one. Mitotane is not used for ACTH producing hing neoplasia. Pnenmonectomy (D) is not the ‘rcatment for small cell carcinoma of the lung, Its the appropriate treatment for resectable and potentially cureble non small cell cancers of the lung. ‘Mark this question & Question Td : 212569 Question 30 of 30. A.B year old woman has easy bruisabiliy, cutancous striae and excessive hair growth on her face. She has been smoking 1 pack of cigarette a day for 8 years. Her BP is 150/90 mmlg. She has elevated cortisol levels, which are not suppressed when she is aver. high dose dexamethasone. ACTH levels are high After confirmation of the diegnosis, which one is the preferred reatment? a) Acrenalectomy ') General support only ) Mitotane Pnsumonectomy Y © 2) Radistion and chemotherapy direct atlung cancer Answer | Explanation Other User's Explanation Report An Error Question Explanation: The endocrine workup is indicative of ectopic ACTH production, and the most Hely ste is stall cell carcinoma of the lung, Ifthe high dose dexamethasone suppression test had suppressed 50% or more of the ACTH, then the most likely cause would be a primary pituitary adenoma (Cushing disease). In this patient however, the ACTH was not suppressed, therefore, this is suspicious for an ectopic ACTH producing tumor, ether in the lung or the adrenals. The nest step in diagnosis is to check the ACTH levels; flower then normal, the most likely cause for the ectopia would be an adrenal neoplasm. But £ the ACTH is bigher than nomal (asin this case), the mozt lely cause for the ectopia would be lng neoplasia (specifically small cell carcinoma). The iung cancer is what is goingto kill this woman, not the endocrine manifestations of the tumor Smell cell carcinoma is considered to be inoperable because it is a repidly growing tumor that metastasizes early and has an overall poor prognesis. Longer survival can be obtained with radiation and chemotherapy. Adrenalectomy (A) would address the endocrine problem by depsiving the ectopic ACTH ofits target land. But the lethal disease here is the lung cancer thet is causing the parancoplastic syndrome, not an adrenal adenoma, Adrenalectomies are quite useful, and offen curative in patients who have an established unilateral adrenal adenoma, Bilateral adrenalectomy is required for bilateral micronodular and most patients with macronodular adrenal hyperplasia. General support only (B) would lead to death in about 2 months. Radiation and chemotherapy can prolong survival by approximetely 2 years. Mitotane (C) is incorrect, Adrenal carcinomas producing ectopic ACTH almost invariably recur and usually do not respond to either irradiation or chemotherapy Mitotane offers suck patients the only hope of cure. In patients with inoperable, residual, or recurrent adrenal neoplastic disease, its usually palliative and does not prolong life. In patienis with Cushingoid features, alter the high dexamethasone fails to suppress ACTH, the ACTH levels should be taken to differentiate lung cancer versus adrenal adenomas as the cause of the ectopic ACTH. This patient has an increased ACTH level, which indicates that the likely origin of the ectopia is a fang neoplasia, not an adrenal one. Mitotane is not used for ACTH producing hing neoplasia. Pnenmonectomy (D) is not the ‘rcatment for small cell carcinoma of the lung, Its the appropriate treatment for resectable and potentially cureble non small cell cancers of the lung. Mark this question => Question Td : 29014 Question 1 of 30 ‘Which ofthe following statements conceming industria lung disorders is correct? a) Pheumocotiosis is on chest x-ray abnormalities ) Byssinosis is caused by cane-fber ©) Silo-Filler’s disease is cansed by allergy to arain 6) Widespread crepitations are typically heard in extrinsic allergic alveotiis £) Symptoms occur withm minutes £ exposure to mouldy hay in Farmer's hung Question Explanation: Pheumoconiosis is an X-ray diagnosis. Itis due to deposition of coal dust in parenchyina and reaction to its presence. The types - simple complicated - are diagnosed on X-ray appearance. It also occurs more frequently in smokers, Silo filers disease is pulmonary edema caused by inhalation of oxides of nitrogen generated by fresh silage. Byssinosis is caused by exposure to cotton dust. Examination in EAA usually reveals inspiratory crepitations which tend to be basal rather than widespread, sometimes squeaks, but wheeze is not typical Symptoms usually occur within hours. Report An Error Mark this question => Question Td : 29014 Question 1 of 30 ‘Which of the following statements conceming indastial lung disorders is comect? Y © 2) Pneumoconiosis is on chest x-ray abnormalities +b) Byssinosis is caused by cane-dber ©) Silo-Filler’s disease is cansed by allergy to grain 4) Widespread crepitations are typically heard in extrinsic allergic alveolitis ©) Symptoms occur within mirutes # exposure to mouldy hay in Farmer's hing Question Explanation: Pheumoconiosis is an X-ray diagnosis. Itis due to deposition of coal dust in parenchyina and reaction to its presence. The types - simple complicated - are diagnosed on X-ray appearance. It also occurs more frequently in smokers, Silo filers disease is pulmonary edema caused by inhalation of oxides of nitrogen generated by fresh silage. Byssinosis is caused by exposure to cotton dust. Examination in EAA usually reveals inspiratory crepitations which tend to be basal rather than widespread, sometimes squeaks, but wheeze is not typical Symptoms usually occur within hours. Report An Error Mark this question & => Question Td : 29064 Question 2 of 30 A 57-year-old has six months old deteriorating nonproductive cough and exertional dyspaea. She is cyanosed with finger clubbing and bilateral basal crackles. CKR shows bilateral basal shadowing and pulmonary investigations show: PaO? (on air) [3.5 kPa (11.5-12.5) FEV, EVC ratio fase ‘Which of the following investigations is most ikely to establish the diagnosis? a) IgE Titers +) Diffusion Capacity studies ¢) Blood picture 4) Chest CT scan e) Serum ACE level newer [EEIRIIRHAM) othe: Users Explanation Repost An Evor Question Explanation: ‘This patient has restrictive lung disease, most licely Cryptogenic fibrosing alveclitis, the cardinal features being breathlessness end cyanosis, clubbing occurs in two-thirds of cases. She is hypoxic on air, has a restrictive ventilatory defect, and a high resctution CT scan of the chest will show typical changes. Mark this question & => Question Td : 29064 Question 2 of 30 A 57-year-old has six months old deteriorating nonproductive cough and exertional dyspaea. She is cyanosed with finger clubbing and bilateral basal crackles. CKR shows bilateral basal shadowing and pulmonary investigations show: PaO? (on air) [3.5 kPa (11.5-12.5) FEV, EVC ratio fase ‘Which of the following investigations is most ikely to establish the diagnosis? a) IgE Titers +) Diffusion Capacity studies c) Blood picture VM © & Chest CT scan e) Serum ACE level newer [EEIRIIRHAM) othe: Users Explanation Repost An Evor Question Explanation: ‘This patient has restrictive lung disease, most licely Cryptogenic fibrosing alveclitis, the cardinal features being breathlessness end cyanosis, clubbing occurs in two-thirds of cases. She is hypoxic on air, has a restrictive ventilatory defect, and a high resctution CT scan of the chest will show typical changes. “Mark this question & => Question 3 of 30 A 73-year-old chronic smoker has dyspnea on exertion and a chronic, nonproductive cough. He is thin, breathing with pursed lips, respiratory rate 25/nin, with mild wheezing on chest auscultation, Investigations show FEV, ost Eve Le [pz p35 [pac [45 mmmiiz [paOz 55 mmEg “What is the predomiant mechanism of the airflour limitation in this gentleman? a) Bronchospasm b) Loss of elastic recol ©) Foreign body obstruction Increased airways resistence €) Mucus piugaing in the small airways Question Explanation: This patient has typical features of COPD with a predominant emphysematous clement, The patholegy is centrilobular or penacinar ‘with loss of elastic tissue. on Report An Error “Mark this question & => Question 3 of 30 A 73-year-old chronic smoker has dyspnea on exertion and a chronic, nonproductive cough. He is thin, breathing with pursed lips, respiratory rate 25/nin, with mild wheezing on chest auscultation, Investigations show FEV, ost Eve Le [pz p35 [pac [45 mmmiiz [paOz 55 mmEg “What is the predominant mechanism of the airflow limitation in this gentleman? a) Bronchospasm Y © b) Loss of elastic reco c) Foreign body obstruction 4) Increased airways resistance e) Mucus plugging in the small airways Question Explanation: This patient has typical features of COPD with a predominant emphysematous clement, The patholegy is centrilobular or penacinar ‘with loss of elastic tissue. on Report An Error “Mark this question => Question Id : 29573 Question 4 of 30 A 41-year-old worker presents with wheezing and breathlessness which seem to improve over weekends and holiday periods when he is not working What is he most likely to be exposed to at work? 2) Avian bloom b) Aspergilis clavaras ©) Silicon exposure 46) Platinurn salts ¢) Exposure to spores of Actinomyces Question Explanation: Disinfectants and preservations incinde ghutaraldeyde, chlorhexidine and formaldehyde and can cause occupational asthma, Metal causing occupational asthma include isocyanates. cobalt, duminium. chrome, manganese, nickel, zinc and platinum, Exposure to “Actinomyces (farmer's lung, maskroom workers ling) avian bloom (bird fanciers lung) and aspergilus clavatus (matt workers jung) cause extrinsic allergic alveolitis. Silicone exposure causes silicosis which does not present like this. “Mark this question => Question Id : 29573 Question 4 of 30 A4l-year-old worker presents with wheezing and breathlessness which seem to improve over weekends and holiday peticds waen he is not working What is he most lkely to be exposed to at work? 2) Avian bleor +) Aspergillas clavatus 6) Silicon exposure Y¥ © 4) Platinum salts ¢) Exposure to spores of Actinomyces Question Explanation: Disinfectants and preservations incinde ghutaraldeyde, chlorhexidine and formaldehyde and can cause occupational asthma, Metal causing occupational asthma include isocyanates. cobalt, duminium. chrome, manganese, nickel, zinc and platinum, Exposure to “Actinomyces (farmer's lung, maskroom workers ling) avian bloom (bird fanciers lung) and aspergilus clavatus (matt workers jung) cause extrinsic allergic alveolitis. Silicone exposure causes silicosis which does not present like this. 2:25/2014 2:23:17 PM “Mark this question => Question Id : 29673 Question S of 30 A 39-year-old painter has episodic wheese and non-productive cough which occurs particularly at night. Which of the following, suggests a dhagnosis of occupational lung disease? a) Improved symptomatology when on holiday ) Absent family history of asthma ©) Commencement of syinptoms on his first day in this employment o) Elevated serum IgE concentration ) Increased bronchial reactivity Answer | Berianation | Other User's Explanation Report An Error Question Explanation: Episocic cough and wheeze with nocturnal symptoms are classical of asthma, Occupational asthma is the commonest indastrial lung disease with over 400 causes and accourts for up to 10% of adult onset asthma. ‘The commonest occupations affected are spray painters, Bakers, Chemical processors. plastics workers and welders and soldering, Patients are characteristically better when on holiday. The diagnosis is confirmed by serial PEFR measurements at home and at work. Recordings should be performed 2 hourly for 4 weeks or fthis is not possible methacholne/histamine challenges can be undertaken after days at work and away from work. Following objective confirmation of the diagnosis the underlying cause should be identified 2:25/2014 2:23:17 PM “Mark this question => Question Id : 29673 Question S of 30 A 39-year-old painter has episodic wheeze and non-productive cough which occurs particularly at night. Which of the Following, suggests a ckagnosis of occupational lung disease? JV © a) Improved symptomatology when on holiday +) Absent family history of asthma ©) Commencement of syinptoms on his first day in this employment ) Elevated serum IgE concentration o) Increased bronchial reactivity Answer | Berianation | Other User's Explanation Report An Error Question Explanation: Episocic cough and wheeze with nocturnal symptoms are classical of asthma, Occupational asthma is the commonest indastrial lung disease with over 400 causes and accourts for up to 10% of adult onset asthma. ‘The commonest occupations affected are spray painters, Bakers, Chemical processors. plastics workers and welders and soldering, Patients are characteristically better when on holiday. The diagnosis is confirmed by serial PEFR measurements at home and at work. Recordings should be performed 2 hourly for 4 weeks or fthis is not possible methacholne/histamine challenges can be undertaken after days at work and away from work. Following objective confirmation of the diagnosis the underlying cause should be identified 2/25/2014 2:23:34 PM “Mark this question => Question Id : 29815, Question 6 of 30 Arman aged 46 years presents with shortness of breath. He has cisrhosis secondary to hepatitis C infection, He gives a chronic history of progressive shortness of breath on exertion, and now gets short of breath walxing up steps. He further says that he is more short of breath whilst sitting up, preferring to sleep with no pillows. The blood pressure is 110/70 mmHlg, heart rate 85 beats per minute, and pilse oximetry, breathing room air, shows saturations of 95% lying flat. Which is appropriate to confirm the diagnosis? a) High resolution CT chest 6) MRI chest ©) Contrast echocardiography 4) Pulmonary angography ©) VQ sean Anewor [UERRNEHER) otner sors Explanation Repert An ixor Question Explanation: This patient has hepatopulmonary syndrome. Platypnoea and orthodeoxia are typical of this syndrome. A greater than 5% desaturation on sitting up is very suggestive of this condition which complicates liver cirrhosis, and is characterized by pulmonary arteriovenous malformations. Conirast echo is the diggnostic tool of choice. Visualisation of late-appearing bubbles in the lef atrium following the injection of agitated saline is strongly suggestive of a pulmonary arteriovenous shunt. 2/25/2014 2:23:34 PM “Mark this question => Question Id : 29815, Question 6 of 30 Arman aged 46 years presents with shortness of breath. He has cisrhosis secondary to hepatitis C infection, He gives a chronic history of progressive shortness of breath on exertion, and now gets short of breath walxing up steps. He further says that he is more short of breath whilst sitting up, preferring to sleep with no pillows. The blood pressure is 110/70 mmHlg, heart rate 85 beats per minute, and pilse oximetry, breathing room air, shows saturations of 95% lying flat. Which is appropriate to confirm the diagnosis? a) High resolution CT chest b) MRI chest Y © ©) Contrast echocardiography 4) Pulmonary angography ©) VQ sean Anewor [UERRNEHER) otner sors Explanation Repert An ixor Question Explanation: This patient has hepatopulmonary syndrome. Platypnoea and orthodeoxia are typical of this syndrome. A greater than 5% desaturation on sitting up is very suggestive of this condition which complicates liver cirrhosis, and is characterized by pulmonary arteriovenous malformations. Conirast echo is the diggnostic tool of choice. Visualisation of late-appearing bubbles in the lef atrium following the injection of agitated saline is strongly suggestive of a pulmonary arteriovenous shunt. Mark this question —& => Question Id : 29876 Question 7 of 30 AT year old men who was a coal miner by occupation 16 years ago presents with deteriorating dyspnoea. Investigations show: FEV1 1.4L @redicted 2.5) FVC 281 Gredicted 3.0) “Vihat is the mest likely diagnosis? a) Cryptogenic frosing alveoiitis +) Extrinsic allergic alveclitis ©) Silicosis 6) Chronic obstructive pulmonary disease ©) Simple pneumoconiosis Answer | Explanation Other User's Explanation Report An Error Question Explanati Both this man’s FEV1 and FVC are reduced. The tatio of FEVVEVC is also reduced at 50%, This indicates airways obstruction. Chronic obstructive pulmonary disease (COPD) is the only condition above that result in airways obstruction. Although no smoking history is given this may be expected and his occupation is a risk Eactor for the development of COPD, Mark this question —& => Question Id : 29876 Question 7 of 30 AT year old men who was a coal miner by occupation 16 years ago presents with deteriorating dyspnoea. Investigations show: FEV1 1.4L @redicted 2.5) FVC 281 Gredicted 3.0) “Vihat is the mest likely diagnosis? a) Cryptogenic frosing alveoiitis +) Extrinsic allergic alveclitis ©) Silicosis ¥ © & Chronic obstructive pulmonary disease ¢) Simple paeumoconiosis Answer | Explanation Other User's Explanation Report An Error Question Explanati Both this man’s FEV1 and FVC are reduced. The tatio of FEVVEVC is also reduced at 50%, This indicates airways obstruction. Chronic obstructive pulmonary disease (COPD) is the only condition above that result in airways obstruction. Although no smoking history is given this may be expected and his occupation is a risk Eactor for the development of COPD, Mark this question = => Question Td : 42701 Question 8 of 30 ‘The occurrence of malignant mesothelioma has been correlated with industrial exposure to which of the following agents? 2) Beryllium 6) Coal dust ©) Asbestos 6) Nitrogen dioxide ©) Silica, Question Explanation: “Asbestos-related disorders are caused by inhalation ofasbestos fibers, The disorders include asbestosis, lung carcinoma, ‘nonmalignant pleural plaque formation and thickening: benign pleural effusions and melignant mesotheloma. Asbestosis and mesotheloma both cause progressive dyspnea. Diagnosis is based on history and chest x-ray or CT and, in the case of malignancy, tissue biopsy. Treatment is supportive, except for malignancies, which may requite surgery and/or chemotherapy. Report An Error Mark this question = => Question Td : 42701 Question 8 of 30 ‘The occurrence of malignant mesothelioma has been correlated with industrial exposure to which of the following agents? 2) Beryllium ) Coal dust Y © o Asbestos ) Nitrogen dioxide ©) Silica Question Explanation: “Asbestos-related disorders are caused by inhalation ofasbestos fibers, The disorders include asbestosis, lung carcinoma, ‘nonmalignant pleural plaque formation and thickening: benign pleural effusions and melignant mesotheloma. Asbestosis and mesotheloma both cause progressive dyspnea. Diagnosis is based on history and chest x-ray or CT and, in the case of malignancy, tissue biopsy. Treatment is supportive, except for malignancies, which may requite surgery and/or chemotherapy. Report An Error Mark this question = => Question Td : 45863 Question 9 of 30 A woman develops tachycardia, dyspnea and hypozia one day after an orthopedte surgery to repair a fracture The mostlikely agnosis is a) DVT b) Fatembotismn ©) Post operative fewer ©) Post operative infection Question Explanation: Fat embolism can cause acute respiratory failure after long-bone fractures. Itis thought to be caused by deposition of embolic fat within the pulmonary capillaries, resulting in a capillary leak within the ing. The source of the embolic fat apoears to be marrow fat “Alterations in circulating free fatty acids, increases in fibrin split products, and increases in platelet adhesiveness have been observed to occur in this syndrome and probably play an important part in the production of the pulmonary lesion. The signs and symptoms of the syndrome are hypoxemia, tachypnea, petechiae, feves, altered sensorium, and chest x-rays similar to signs of the adult respiratory distress syndrome (ARDS), Early recognition is facilitated by monitoring of arterial blood gas levels for hypoxemia Treatment is directed at ARDS using graduated oxygen therapy and steroids. Mark this question = => Question Td : 45863 Question 9 of 30 A woman develops tachycardia, dyspnea and hypozia one day after an orthopedte surgery to repair a fracture The mostlikely diagnosis is a) DVT v¥ © ») Fatembolism ©) Post operative fewer 6) Post operative infection Question Explanation: Fat embolism can cause acute respiratory failure after long-bone fractures. Itis thought to be caused by deposition of embolic fat within the pulmonary capillaries, resulting in a capillary leak within the ing. The source of the embolic fat apoears to be marrow fat “Alterations in circulating free fatty acids, increases in fibrin split products, and increases in platelet adhesiveness have been observed to occur in this syndrome and probably play an important part in the production of the pulmonary lesion. The signs and symptoms of the syndrome are hypoxemia, tachypnea, petechiae, feves, altered sensorium, and chest x-rays similar to signs of the adult respiratory distress syndrome (ARDS), Early recognition is facilitated by monitoring of arterial blood gas levels for hypoxemia Treatment is directed at ARDS using graduated oxygen therapy and steroids. 2125/2014 2:24:24 PM ‘Mark this question & => Question Td : 49013 Question 10 of 30 “After a long history mitiple myeloma, a 66 year old man is noted to have abundant acellular eosinophile deposits around the pulmonary microvasculature at autopsy. A Congo sed special stain demonstrates apple green breltingence. The condition thatis the most licely explanation ie 2) Sarcoidosis b) Histoplasmosis ¢) Mesothelioma d) Amyloidosis €) Silicosis Answer | Exsianaion Other User's Explanation Report An Error Question Explanation: Amyloidosis is any of a group of disparete conditions characterized by extracellular deposition of various insoluble proteins, These proteins may accumulate locally, causing relatively few symptoms, or widely, involving multiple organs and producing severe multiorgan failure, Amyloidosis can be primary or be secondary to various infectious, inflammatory, or malignant conditions, Rarely, it results from any of several inheried metabolic defects. Diagnosis is by biopsy of affected tissue Amyloid deposits may be formed from at least 18 different protems including immunoglobulin fregments. Amyloid deposits are metabolically inert but interfere physically with organ structare and function. All stain positive with Congo-red dye, stain, and pink with hematoxylin and eosin, and have apole- green birefiingence under-polarized light after Congo red staining, Amyloid deposits have a fibrillar, usually rigid, and nonbranchlag ultrastructure. 2125/2014 2:24:24 PM ‘Mark this question & => Question Td : 49013 Question 10 of 30 After a long history multiole myeloma, a 66 year old man is noted to have abundant acellular eosinophilic deposits around the pulmonary microvasculature at autopsy. A Congo red special stain demonstrates apple green birefringence. The condition thatis the moct lkely explanation ic 2) Sarcoidosis +) Hictoplacmosis ©) Mesothelioma Y © & Amyloidosis €) Silicosis Answer | Exsianaion Other User's Explanation Report An Error Question Explanation: Amyloidosis is any of a group of disparete conditions characterized by extracellular deposition of various insoluble proteins, These proteins may accumulate locally, causing relatively few symptoms, or widely, involving multiple organs and producing severe multiorgan failure, Amyloidosis can be primary or be secondary to various infectious, inflammatory, or malignant conditions, Rarely, it results from any of several inheried metabolic defects. Diagnosis is by biopsy of affected tissue Amyloid deposits may be formed from at least 18 different protems including immunoglobulin fregments. Amyloid deposits are metabolically inert but interfere physically with organ structare and function. All stain positive with Congo-red dye, stain, and pink with hematoxylin and eosin, and have apole- green birefiingence under-polarized light after Congo red staining, Amyloid deposits have a fibrillar, usually rigid, and nonbranchlag ultrastructure. 2/25/2014 2:24:38 PM ‘Mark this question & => Question Id : 50720 Question 11 of 30 Diagnosis of acute symptomatic pulmonary embolism can be excl.ded when which one of the folowing is normal? a) Chestacray +) Ventilation-perfision lang scan ©) Bilateral leg venograms 4) PaOg and A-a Op gradient Question Explanation Pulmonary embolism is the occlusion of one or more pulmonary arteries by thrombi that originate elsewhere, typically in the large ‘reins of the lower extremities or pelvis. Risk factors arc conditions that impair venous retum, conditions thet cause endothelial injury or dysfunction, and underlying hypercoagulable states. Symptoms are nonspecific and include dyspnea, pleuritic chest pain, cough, and, in severe cases, syncope or cardiorespiratory arrest. Signs are also nonspecific and may include tachypnea, tachvcerdia, hypotension, and aloud pulmonic component of the 2nd heart sound, Diagnosis is based on 2 CT angiogram, ventiation/perfision scan, ora pulmonary arteriogram. 2/25/2014 2:24:38 PM ‘Mark this question & => Question Id : 50720 Question 11 of 30 Diagnosis of acute symptomatic pulmonary embolism can be excl.ded when which one of the folowing is normal? a) Chestacray Y © 6) Ventilation-perfusion lang scan ©) Bilateral leg venograms 4) PaOg and A-a Op gradient Question Explanation Pulmonary embolism is the occlusion of one or more pulmonary arteries by thrombi that originate elsewhere, typically in the large ‘reins of the lower extremities or pelvis. Risk factors arc conditions that impair venous retum, conditions thet cause endothelial injury or dysfunction, and underlying hypercoagulable states. Symptoms are nonspecific and include dyspnea, pleuritic chest pain, cough, and, in severe cases, syncope or cardiorespiratory arrest. Signs are also nonspecific and may include tachypnea, tachvcerdia, hypotension, and aloud pulmonic component of the 2nd heart sound, Diagnosis is based on 2 CT angiogram, ventiation/perfision scan, ora pulmonary arteriogram. Mark this question & => Question Td : 54479 Question 12 of 30 A. 26 year old IV drug abuser present with cough, fever. Chest x-ray shows hilar ard mediastinal adenopathy and bilateral lower lobe infilrates. His intermediate Mentous: skin test chows a greater then Sram induration. The most icely underlying conditien associated ‘with this ness is 2) Sickle cell disease ) Alcoholism c) Drug abuse 4) Hodgkin's disease ¢) Humen imrnuno deficiency virus (HIV) infection Anower [UEQIRIREN) omer users Explanation Report An Erot Question Explanation: Tuberculosis and HIV have been closely linked since the emergence of AIDS. HIV infection has contributed to a significantincrease in the worldwide incidence of uberculosis, By producing a progressive deciine in cell-mediated immunity, HIV alers the pathogenesis of tuberculosis, greatly increasing the risk of developing disease in co-infected individuals and leading to more frequent extrapulmonary involvement and atypical radiographic manifestatons. ‘The clinical presentation of pulmonary tuberculosis can vary widely in both immunocompetent and immunocompromised hosts, In general, the presentation in the HIV-infected patient is similar to that seen in HIV-uninfected patients although the signs and symuptoms (such as fevers. weight loss, and malaise) may be attributed to HIV itself and the possibility of tuberculosis overlooked. Symptoms are usuclly present for weeks to months, and the acute onset of fever and cough is more suggestive of a noamycobacterial pulmonary process, Ifthere is no response to antimicrobial therapy, however, the possibilty of teberculosis should be considered. In HIV-infected patients, clinical manifestations of pulinonary tuberculosis reflect different levels of immunosuppression, ‘The chest radiograph is the comerstone of diagnosis for pulmonary tuberculosis. Upper lobe infiltrates and cavities are the typical incings in reactivation tuberculosis, whereas intrathoracic lymphadenopathy and lower lobe disease are scen in primary tuberculosis. Mark this question & => Question Td : 54479 Question 12 of 30 A. 26 year old IV drug abuser present with cough, fever. Chest x-ray shows hilar ard mediastinal adenopathy and bilateral lower lobe infilrates. His intermediate Mentous: skin test chows a greater then Sram induration. The most icely underlying conditien associated ‘with this ness is 2) Sickle cell disease ) Alcoholism c) Drug abuse 4) Hodgkin's disease Y © 0) Human immuno deficiency virus GIIV) infection Anower [UEQIRIREN) omer users Explanation Report An Erot Question Explanation: Tuberculosis and HIV have been closely linked since the emergence of AIDS. HIV infection has contributed to a significantincrease in the worldwide incidence of uberculosis, By producing a progressive deciine in cell-mediated immunity, HIV alers the pathogenesis of tuberculosis, greatly increasing the risk of developing disease in co-infected individuals and leading to more frequent extrapulmonary involvement and atypical radiographic manifestatons. ‘The clinical presentation of pulmonary tuberculosis can vary widely in both immunocompetent and immunocompromised hosts, In general, the presentation in the HIV-infected patient is similar to that seen in HIV-uninfected patients although the signs and symuptoms (such as fevers. weight loss, and malaise) may be attributed to HIV itself and the possibility of tuberculosis overlooked. Symptoms are usuclly present for weeks to months, and the acute onset of fever and cough is more suggestive of a noamycobacterial pulmonary process, Ifthere is no response to antimicrobial therapy, however, the possibilty of teberculosis should be considered. In HIV-infected patients, clinical manifestations of pulinonary tuberculosis reflect different levels of immunosuppression, ‘The chest radiograph is the comerstone of diagnosis for pulmonary tuberculosis. Upper lobe infiltrates and cavities are the typical incings in reactivation tuberculosis, whereas intrathoracic lymphadenopathy and lower lobe disease are scen in primary tuberculosis. Mak ths question =r Question 13 of 30 4.24 year old men, who lived all his lift in Southern Ontario and Central Canada, developed fever, night sweats and a chronic, progressive cough resistant 10 antibiotic treatment. X-ray shows 2 coin-tike lesion in the upper lobe of the right lung. The TE skin test ‘was negative twice. The most likely cause is 2) Legionellosis b) Tuberculoss 6) Silicosis ¢) Ditnotphe mycosis ©) Atypical mycobacteria Question Explanation: Slicosis is caused by inhalation of crystaline fiee silica dust and is characterized by nodular pulmonary fibrosis. Chronic silicosis initially causes no symptoms or only mid dyspnea but over years can advance to involve most ofthe fing and cause dyspnea, hypoxemia, pulmonary hypertension, and respiratory impairment. Diagnosis ic bazed on history and chest x-ray. No effective teament exists except supportive care and, for severe cases, lung wansplantation, Report An Error Mak ths question =r Question 13 of 30 A 24 year old man, who lived allhis lift in Southern Ontario and Central Canada, developed fever, night sweats and a chronic progressive cough resistant to antibiotic treatment. X-ray shows 2 coin-like lesion in the upper lobe of the right ung. The TE skin test was negative twice. The most ikely cause is 8) Legionellosis +) Tuberculosis V © 29) Silicosis 6) Dimorphic mycosis 6) Atypical mycchacteria Question Explanation: Slicosis is caused by inhalation of crystaline fiee silica dust and is characterized by nodular pulmonary fibrosis. Chronic silicosis initially causes no symptoms or only mid dyspnea but over years can advance to involve most ofthe fing and cause dyspnea, hypoxemia, pulmonary hypertension, and respiratory impairment. Diagnosis ic bazed on history and chest x-ray. No effective teament exists except supportive care and, for severe cases, lung wansplantation, Report An Error 2252014 2:26:45 PM ‘Mark this question & => Question Td : 55412 Question 14 of 30 ‘Which pulmonary function test reliably discriminates ‘pure’ chronic bronchitis from emphyserra? a) Total lang capacity ) Functional residual capacity ©) Residual volume 6) Flow at 50% vital capacity €) Single breath diffusing capacity Question Explanation: “The single breath diffusing capacity, or DLCO, is a measure of the ability of the lungs to diffuse oxygen into, and carbon dioxide fiom, the bloodstream. Measurement of lffusing- capacity of the hung for carbon monoxide (DLCO) should be considered a routine initial testin evaluation of chronic airflow obstruction, particularly in, advanced disease. DLCO has been established as sensitive in detection of emphysema that is associated with loss of alveolar surface area and pulmonary circulation. 2252014 2:26:45 PM ‘Mark this question & => Question Td : 55412 Question 14 of 30 ‘Which pulmonary fiction test relably discriminates ‘pure’ chronic bronchtis from emphysema? @) Total mg capacity ') Functional residual capacity ©) Residual voume 6) Flow at 50% vital capacity V © 6) Single breath ciffusing capacity Question Explanation: “The single breath diffusing capacity, or DLCO, is a measure of the ability of the lungs to diffuse oxygen into, and carbon dioxide fiom, the bloodstream. Measurement of lffusing- capacity of the hung for carbon monoxide (DLCO) should be considered a routine initial testin evaluation of chronic airflow obstruction, particularly in, advanced disease. DLCO has been established as sensitive in detection of emphysema that is associated with loss of alveolar surface area and pulmonary circulation. 2125/2014 2:27:00 PM ‘Mark this question & => Question Td : $5743 Question 15 of 30 An merease in the alveolar arterial pO, difference may occur in which of the following? a) Asthma ') Acate hypoventilation ©) Anemia 4) Mild exercise ©) High altitude Anower [UERRENBNEH) otner Users Explanation Report An Error Question Fxplanation: ‘The difference between the calculated PaO, and the measured PaO, is called the Alveoler-arterial pO) dlftrence or A-a Gradient" for short, The A-a gradient answers the important queston: Are the lungs transferring oxygen propealy from the atmosphere to the pulnonary circulation? Ifthe A-a gradient is elevated, the answer is no. Ifthe A-a gradient is normal the answer is ves Hyperventilation (e g., at high altitude) cauces an increace in the alveclar-avterial pO, difference. 2125/2014 2:27:00 PM ‘Mark this question & => Question Td : $5743 Question 15 of 30 An merease in the alveolar arterial pO, difference may occur in which of the following? 2) Asthma b) Acute hypoventilation ) Anexnia &) Mild exercise © ©) High altitude Anower [UEXRENBNEH) otner Users Explanation Report An Error Question Fxplanation: ‘The difference between the calculated PaO, and the measured PaO, is called the Alveoler-arterial pO) dlftrence or A-a Gradient" for short, The A-a gradient answers the important queston: Are the lungs transferring oxygen propealy from the atmosphere to the pulnonary circulation? Ifthe A-a gradient is elevated, the answer is no. Ifthe A-a gradient is normal the answer is ves Hyperventilation (e g., at high altitude) cauces an increace in the alveclar-avterial pO, difference. 2:25/2014 2:27:13 PM Mark this question & => Question 16 of 30 A.72 year old man with a history of coronary heart disease and Lypertension presents with creasing shortness of breath for 3 weeks, A chest X-ray shows a moderate sized right pleural effusion, Thoracertesis shows the protein in the pleuralserum ratio to be 70.5, consistent with a transudate, These results are compatible with a pleural effusion caused by which one of the following? a) Nephrotic syndrome ) Congestive hear failure ©) Adenocarcinoma of the tung 4) Hepatic ven thrombosis €) Cirrhosis of the liver Anower [FEIRIRIRHBNY) Other Users Explanation Report An E01 Question Explanation: Pleural effusions are accumulations of fd within the pleural space. They have multiple causes and usvally are classified as ansudates or exudates, Detection is by physical examination and chest x-ray; thoracentesis and pleural Tuid analysis are often required to determine cause. Asymptomatic transudates requite no treatment. Symptomatic transudates and almost all exudates require thoracentesis, chest tube drainage, plenredesis, andlor pleurectomy ‘Transudative effusions are caused by some combination of increased hydrostatic pressure and decreased oncotic pressure in the pulmonary or systemic circulation, Heart falure is the most common cause, followed by cirthosis with ascites and hypoalbuminemia, usually from the nephrotic syndrome. Exudative efftsions are caused by local processes leading to increased capillary penmeability resulting in exudation of fuid, protein, cells, and other serum constitients, Causes are sumerous, the most common beng preumenia, malignancy, pulmonary embolism, viral infection, and TB. 2:25/2014 2:27:13 PM Mark this question & => Question 16 of 30 A.72 year old man with a history of coronary heart disease and Lypertension presents with creasing shortness of breath for 3 weeks, A chest X-ray shows a moderate sized right pleural effusion, Thoracertesis shows the protein in the pleuralserum ratio to be 70.5, consistent with a transudate, These results are compatible with a pleural effusion caused by which one of the following? a) Nephrotic syndrome Y © b) Congestive hear feiture ©) Adenocarcinoma of the tung 4) Hepatic ven thrombosis €) Cirrhosis of the liver Anower [FEIRIRIRHBNY) Other Users Explanation Report An E01 Question Explanation: Pleural effusions are accumulations of fd within the pleural space. They have multiple causes and usvally are classified as ansudates or exudates, Detection is by physical examination and chest x-ray; thoracentesis and pleural Tuid analysis are often required to determine cause. Asymptomatic transudates requite no treatment. Symptomatic transudates and almost all exudates require thoracentesis, chest tube drainage, plenredesis, andlor pleurectomy ‘Transudative effusions are caused by some combination of increased hydrostatic pressure and decreased oncotic pressure in the pulmonary or systemic circulation, Heart falure is the most common cause, followed by cirthosis with ascites and hypoalbuminemia, usually from the nephrotic syndrome. Exudative efftsions are caused by local processes leading to increased capillary penmeability resulting in exudation of fuid, protein, cells, and other serum constitients, Causes are sumerous, the most common beng preumenia, malignancy, pulmonary embolism, viral infection, and TB. ‘Mauke this question & => Question Td : 84514 Question 17 of 30 A 67-year-old man undergoes lng transplant alter giving informed concent: The operation proves difficut and leaves him in coma and on mechanical ventilation. His son brings his will that states that should the situation arise the patient would net wish to be kept alive by mechanical means Based on this willthe son asks to terminate ventilation immeciately. His wife asks life support to be maintained indefindely. The case would be resolved by which one of the folowing? a) A court ruling to adjudicate the validity of the living will +b) A muling by the hospital ethics committee as to the proper ethical course ofaction ©) Consultation among family members to achiove a family consensus as to how to proceed 4) Consultation with the patient's primary care physician ©) Maintaining the petieat on life support based on the patient's consent until such time as itis determined that the chance of recovery is unlikely £) Maintaining the patient on life support indefinitely 2) Maintaining the patient on life support based on the patient's wife's request unil such time as she changes her mind +) Terminating ie support as the living will requests Answer | Boplanaton Other User's Explanation Report An Error Question Explanation: ‘The subtle, but important point, inthis question is that the life support procedures are part of the standard postoperative recovery procedures to which the patient consented when he agreed to the transplant. In other words this is not a case of the patient's life being prolonged by lie support. Rather, the life supports key partto the treaiment Note that life support will not be maintained indefinitely. Should prognosis change so that recovery is unlikely, then the living will comes into play ard life support would be temninated. The patient has already consented to the treatment Court examination (choice A) of the living will would add nothire Given the consent and the chance of recovery, this is not a situation in which the living wil is germane. The ethics board (cheice E) can add nothing here There is etnple information in the physician to make an on-the spot decision, In general, going to court or to the ethics committee will not be correct answers on the Step 1 exam. The patient, not a committee of family members (choice C), has the tightto decide what should be done. The patient's consentto the surgery is the governing principle for new. IFrecovery should seem urlilely, then the living will comes into play. Ta neither circumstance is a joint decision of family members relevant Tf the patient's consent were not already obtained ard there wes no clear statement of the patient's wishes as in the living wil, then consulting with the patient's primary care physician (choice D) might make sense. The primary care physician may have (should have) discussed end-of-life care issues with this patient end may be a window into the unconscious patient's true wishes. In this case, with other information avadable, the consuitaton with the primary care physician will yield nothing new. It at some point recovery seems wnlikely, then the life support is no longer a past postoperative procedure but is in fact the sort of life-sustaining effort that the patient asked to avoid in his living will Life support should aot be indefinte (choice F) but should be termineted when the prognosis changes for the worse, The issue here is not the wile’s wishes (choice G), butthe patient's consent to mechanical suppor: is a pact of postoperative recovery Remember that spouses are only relevant in so far as they represent the wishes ol their partners. ‘We act, not on her wishes, but only on his. Choice His incorrect The mechanical ventilaiion is a standard part of postoperative recovery to which the patient hhas consented. Once begun, the process must be completed. Remember patients do not consent to each actual movement of the scalpel, but to the operation itself It is the physician's job to know the technical steps needed to bring the treatment to a successful lonchieon: ‘Mauke this question & => Question Td : 84514 Question 17 of 30 A 67-year-old man undergoes lng transplant alter giving informed concent: The operation proves difficut and leaves him in coma and on mechanical ventilation. His son brings his will that states that should the situation arise the patient would net wish to be kept alive by mechanical means Based on this willthe son asks to terminate ventilation immeciately. His wife asks life support to be maintained indefindely. The case would be resolved by which one of the folowing? a) A court ruling to adjudicate the validity of the living will +b) A muling by the hospital ethics committee as to the proper ethical course ofaction ©) Consultation among family members to achiove a family consensus as to how to proceed 4) Consultation with the patient's primary care physician Y © ©) Maintaining the petieat on life support based on the patient's consent until such time as itis determined that the chance of recovery is unlikely £) Maintaining the patient on life support indefinitely 2) Maintaining the patient on life support based on the patient's wife's request unil such time as she changes her mind +) Terminating ie support as the living will requests Answer | Boplanaton Other User's Explanation Report An Error Question Explanation: ‘The subtle, but important point, inthis question is that the life support procedures are part of the standard postoperative recovery procedures to which the patient consented when he agreed to the transplant. In other words this is not a case of the patient's life being prolonged by lie support. Rather, the life supports key partto the treaiment Note that life support will not be maintained indefinitely. Should prognosis change so that recovery is unlikely, then the living will comes into play ard life support would be temninated. The patient has already consented to the treatment Court examination (choice A) of the living will would add nothire Given the consent and the chance of recovery, this is not a situation in which the living wil is germane. The ethics board (cheice E) can add nothing here There is etnple information in the physician to make an on-the spot decision, In general, going to court or to the ethics committee will not be correct answers on the Step 1 exam. The patient, not a committee of family members (choice C), has the tightto decide what should be done. The patient's consentto the surgery is the governing principle for new. IFrecovery should seem urlilely, then the living will comes into play. Ta neither circumstance is a joint decision of family members relevant Tf the patient's consent were not already obtained ard there wes no clear statement of the patient's wishes as in the living wil, then consulting with the patient's primary care physician (choice D) might make sense. The primary care physician may have (should have) discussed end-of-life care issues with this patient end may be a window into the unconscious patient's true wishes. In this case, with other information avadable, the consuitaton with the primary care physician will yield nothing new. It at some point recovery seems wnlikely, then the life support is no longer a past postoperative procedure but is in fact the sort of life-sustaining effort that the patient asked to avoid in his living will Life support should aot be indefinte (choice F) but should be termineted when the prognosis changes for the worse, The issue here is not the wile’s wishes (choice G), butthe patient's consent to mechanical suppor: is a pact of postoperative recovery Remember that spouses are only relevant in so far as they represent the wishes ol their partners. ‘We act, not on her wishes, but only on his. Choice His incorrect The mechanical ventilaiion is a standard part of postoperative recovery to which the patient hhas consented. Once begun, the process must be completed. Remember patients do not consent to each actual movement of the scalpel, but to the operation itself It is the physician's job to know the technical steps needed to bring the treatment to a successful lonchieon: 2252014 2:27:51 PM Mark this question €&=> Question 18 of 30 A.58 year cld male visits his physician, complaning of recent difficulty with his vision, According to him approximately 2 months ago the vision in his right eye deteriorated. and it can be improved only when he holds the eyelid open with his hand. He also has some dificuity seeing thinge in the davk. Right eye examination shows drooping of right eyelid and right cided miosic, Tae patient does not have any other complain but the physician notes hoarseness of he man’s voice. The clinical condition that is most likely t> result in hoarseness during speech is 2) Brenchogenic carcinoma ofthe apical segment oftthe right lung ) Mitral valve prolapse ) Pulnonacy throb oomeb lism 4) Retroesophageal right subclavian artery ) Thymoma Anower [UERPIRNSHER) thor Urovs Explanation Report An Error Question Explanation: “A. tumor of the apical segment ofthe upper lobe may impinge on the recurrert laryageal nerve since it accends from the superior rnediastinum to the root of the neck in a groove between the trachea and esophagus. The recurrent laryngeal nerve supplies all intrnsic muscles of the larynx except the cricothyroid. Additionally, an apical tumor, also called a Pancoast tumor after the pathologist who described it, can invade the sympathetic plesus, which causes varying degrees of ptosis, miosis, and anhidrosis (Homer's eyndroms). Mitral valve prolapse is not a cause of hoarseness, although mitral valve insufficiency and stenosis may sometimes cause compression of the recurrent laryngeal nerve by the enlarged left atrium. Pulmonary embolus causes an obstruction to arterial blood flow of the lung resulting in infarction of the affected segment. Itmay also cause pleursy, resuting in pain conveyed by intercostal nerves. When the right subclavian artery arises from the arch of the aorta, zather than rom the brachiocephalic trunk (en aberrant suclavian artery), it passes posterior to the esophagus to reach the right upper extremity. This may cause compression of the esophagus, resulting in dysphagia ‘Thymoma or tumor ofthe thymus gland may cause dyspnea (difficulty breathing) due to pressure on the trachea. It may also cause engorgement of deep and superficial veins of the necle due to pressure om the superior vena cava. Norrnal thymic tissue will never compress adjacent structures in the mediastinum, no matter how large, since itis soft and pliable. A thymoma classically will indent and exert mass effect upon structures such as the trachea. 2252014 2:27:51 PM Mark this question €&=> Question 18 of 30 A.58 year cld male visits his physician, complaning of recent difficulty with his vision, According to him approximately 2 months ago the vision in his right eye deteriorated. and it can be improved only when he holds the eyelid open with his hand. He also has some dificuity seeing thinge in the davk. Right eye examination shows drooping of right eyelid and right cided miosic, Tae patient does not have any other complain but the physician notes hoarseness of he man’s voice. The clinical condition that is most likely t> result in hoarseness during speech is Y © 2) Bronchogenic carcinoma of the apical segment ofthe right hung ) Mitral valve prolapse ) Pulnonacy throb oomeb lism 4) Retroesophageal right subclavian artery ) Thymoma Anower [UERPIRNSHER) thor Urovs Explanation Report An Error Question Explanation: “A. tumor of the apical segment ofthe upper lobe may impinge on the recurrert laryageal nerve since it accends from the superior rnediastinum to the root of the neck in a groove between the trachea and esophagus. The recurrent laryngeal nerve supplies all intrnsic muscles of the larynx except the cricothyroid. Additionally, an apical tumor, also called a Pancoast tumor after the pathologist who described it, can invade the sympathetic plesus, which causes varying degrees of ptosis, miosis, and anhidrosis (Homer's eyndroms). Mitral valve prolapse is not a cause of hoarseness, although mitral valve insufficiency and stenosis may sometimes cause compression of the recurrent laryngeal nerve by the enlarged left atrium. Pulmonary embolus causes an obstruction to arterial blood flow of the lung resulting in infarction of the affected segment. Itmay also cause pleursy, resuting in pain conveyed by intercostal nerves. When the right subclavian artery arises from the arch of the aorta, zather than rom the brachiocephalic trunk (en aberrant suclavian artery), it passes posterior to the esophagus to reach the right upper extremity. This may cause compression of the esophagus, resulting in dysphagia ‘Thymoma or tumor ofthe thymus gland may cause dyspnea (difficulty breathing) due to pressure on the trachea. It may also cause engorgement of deep and superficial veins of the necle due to pressure om the superior vena cava. Norrnal thymic tissue will never compress adjacent structures in the mediastinum, no matter how large, since itis soft and pliable. A thymoma classically will indent and exert mass effect upon structures such as the trachea. 2/25/2014 2:28:04 PM “Mark this question «=> Question Id : 91334 Question 19 of 30 “What is the initial treatment for a pulmonary embolus? 2) Warfarin bb) Aspirin. c) Touprofen, 6) IV heparin. ©) Acute embolectomy. Question Explanation: TV heparin is the inital treatment of pulmonary embolus “Warfarin takes 36-48 hours to reach a therapeutic effect, and its role is mainly in the maintenance of anticoagulation after heparinization. Aspirin and ibuprofen have no role in the treatment of pulmonary embolism, and acute embolectomy is rarely useful because of high mortality. Report An Error 2/25/2014 2:28:04 PM “Mark this question «=> Question Id : 91334 Question 19 of 30 “Vihat is the inital treatment for a pulmonary embolus? 2) Waeficin +) Aspirin ) Ibuprofen. Y © 4) IV heparin, ©) Acute embolectomy. Question Explanation: TV heparin is the inital treatment of pulmonary embolus “Warfarin takes 36-48 hours to reach a therapeutic effect, and its role is mainly in the maintenance of anticoagulation after heparinization. Aspirin and ibuprofen have no role in the treatment of pulmonary embolism, and acute embolectomy is rarely useful because of high mortality. Report An Error Mark this question ea => Question Td : 93232 Question 20 of 30 Familial emphysema is caused by which one of the following? a) Diabetes mellinas ») ol antitrypsin deficiency ©) Ehlers-Danlos syncrome 4) Marfan's syndrome ©) Hereditary hemorrhagic telangiectasia Anower [BIIRIGENY) otrorUeorsExplana Question Explanation: This condition is associated with emphysema and chronic liver disease and is inherited. None of the other conditions is associated ‘with an increased incidence of emphysema. Report An Error Mark this question ea => Question Td : 93232 Question 20 of 30 Familial emphysema is caused by which one of the following? a) Diabetes mellinas Y © b) ol antitrypsin deficiency ©) Ehlers-Danlos syncrome 4) Marfan's syndrome ©) Hereditary hemorrhagic telangiectasia Anower [BIIRIGENY) otrorUeorsExplana Question Explanation: This condition is associated with emphysema and chronic liver disease and is inherited. None of the other conditions is associated ‘with an increased incidence of emphysema. Report An Error 2:25/2014 2:28:28 PM ‘Mark this question & => Question Td : 94342 Question 21 of 30 A 25 year old male developed hemoptysis and dyspnea over the course of months. His physician suspected tuberculosis and started him on triple therapy with isoniacid, rélampin, and ethambutol. The petient complained of pins and neadies sensations in his feet after 2 inonths treatment. Neurologic examination was otherwise untemarkable, Strength was good in all imbs, He is suffering from which of the folowing deficiency? a) Vitamin B6 (pyridoxine) deficiency b) Vitamin B12 deficiency c) Vitamin E (alphatocopherol) deficiency d) Vitamin D deficiency ¢) Vitamin B| (thiamine) deficiency Question Explanation: ‘There ate many drugs that interfere with the metabolism of vitamin BS, Isoniazid, which is used to treat tuberculosis, alters the activity of vitamnin B6 Acute |soniazid toxicity can result in coma and seimires that are reversed by vtamnin B6. It is recommend to take a supplement that provides B6 when Isoniazid is prescribed, which is usually enough to prevent symptoms of vitamin B6 deficiency. 2:25/2014 2:28:28 PM ‘Mark this question & => Question Td : 94342 Question 21 of 30 A.25 year old male developed hemoptysis and dyspnea over the course of 3months. His physician suspected tuberculosis and started hise on triple therapy with isoniazid, rifampin, and ethambutol. Tae patient complained of pine and nesdies sensations in his feet after 2 months treatment, Neurologic examination was otherwise unremarkable, Strength was good in alllimbs. He is suffering from waich of the folowing deficiency? Y © a) Vitamin B6 (pyridoxine) deficiency b) Vitamin B12 deficiency ) Vitamin E (alphatocepherol) deficiency 4) Vitamin D deficiency ) Vitamin 51 (lhiame) deficiency Question Explanation: ‘There ate many drugs that interfere with the metabolism of vitamin BS, Isoniazid, which is used to treat tuberculosis, alters the activity of vitamnin B6 Acute |soniazid toxicity can result in coma and seimires that are reversed by vtamnin B6. It is recommend to take a supplement that provides B6 when Isoniazid is prescribed, which is usually enough to prevent symptoms of vitamin B6 deficiency. 2/25/2014 2:28:40 PM ‘Mark this question & => Question Td : 113439) Question 22 of 30 Al ofthe following occur in primary pulmonary hypestension, EXCEPT 8) Pulmonary fianction is usually normal ) Hypoxemia is common. c) Pulmonary artery pressure is insreased, 6) Pulmonary arteries only respond to vasodilators in the late stages of the disease ©) Pulmonary capillary wedge pressure remains normal unil late stages in the disease. Question Explanation: Ihitially the arteries may respond to vasodlators, but with discase progression, the clevated pulmonary vascular resistance becomes fixed, The underlying hemodynamic abnormality in primary pulmonary hypertension is an increased resistance to pulmonary blood flow. Early in the disease, there is a marked elevation of pulmonary artery pressure. The pulmonary capillary wedge pressure remains normal until late stage disease, when it increases in response to impaired diastolic filing. Pulmonary fiction is typically normal. Hypoxemia probably results from the ventilation perfusion mismatch, augmented by alow cardiac output 2/25/2014 2:28:40 PM ‘Mark this question & => Question Td : 113439) Question 22 of 30 All of the folowing occur in primary puimonery hypestension, EXCEPT a) Pulmonary fanction is usually normal. ) Hypoxemiais common. c) Pulmonary artery pressure is increased. ¥ © @)Bulmonary arteries only respond to vasodilators in the late stages of the disease. ©) Pulmonary capillary wedge pressure remains normal until late stages in the discase. Question Explanation: Ihitially the arteries may respond to vasodlators, but with discase progression, the clevated pulmonary vascular resistance becomes fixed, The underlying hemodynamic abnormality in primary pulmonary hypertension is an increased resistance to pulmonary blood flow. Early in the disease, there is a marked elevation of pulmonary artery pressure. The pulmonary capillary wedge pressure remains normal until late stage disease, when it increases in response to impaired diastolic filing. Pulmonary fiction is typically normal. Hypoxemia probably results from the ventilation perfusion mismatch, augmented by alow cardiac output 2125/2014 2:28:54 PM “Mark this question €&=> Question 23 of 30 A metastatic nodular lesion is desected in the hangs on a chest X-ray. Which of the following does NOT represent likely candidates for the primary tumor? 2) Renal cell carsinoma. +) Colorectal cancer. 6) Sarcoma. 6) Breast cancer, 6) Thyroid cancer. Question Explanatio ‘Metastatic lesions in the lung that are aodular in appearance typically occur fiom cancers that metastasize via the hematogenous route, These include renal cell carcinoma, colorectal cancer, sarcoma, and thyroid cancer. Breast cancer migrates through the lymphatics and tends to produce a lymphargilis picture of metastasis, not a nodular one. 2125/2014 2:28:54 PM “Mark this question €&=> Question 23 of 30 A metastatic nodular lesion is desected in the hangs on a chest X-ray. Which of the following does NOT represent likely candidates for the primary tumor? 2) Renal cell carsinoma. +) Colorectal cancer. 6) Sarcoma. YM © 4) Breast cancer. 6) Thyroid cancer. Question Explanatio ‘Metastatic lesions in the lung that are aodular in appearance typically occur fiom cancers that metastasize via the hematogenous route, These include renal cell carcinoma, colorectal cancer, sarcoma, and thyroid cancer. Breast cancer migrates through the lymphatics and tends to produce a lymphargilis picture of metastasis, not a nodular one. ‘Matic this question <= => Question Ti : 139609 Question 24 of 30 CER of a45 year old woman who presents with gradual onset of shortness of breath, chest pain, and fatigue shows bilateral hilar adenopathy and pulmonary, infltrates. Biopsy of one of these lesions shows non-necrotiing granulomas. Stains for fungi and mycobacteria ate negative and has no history of occupational exposure to airbome minerals or organic dusts. The most likely diagnosis is a) Asbestosis b) Berglliosis ©) Byssinosis, 4d) Sarcoidosis €) Tuberculosis Question Explanation: The diagnosis of sarcoidosis is usually made by exclusion. This discase is characterized by aon-necrotiring granulomas developing ‘most frequently in the lungs, iymph nodes, retina, heart, spleen, skin, and liver. Non-necrotizing granulomas may be seen in a number of other conditions, however, such as infections and certain forms of pneumoconiosis, which must be suled out before making a diagnosis of sarcoidosis The etiopatkogenesis of sarcoidosis is obscure, Crs mediated mechanisms are thought to be involved in an imeaune responce to as yet usidentiied antigens. About two-thirds of patients with sarcoidesis recover without residual functional deficits, 20% have permanent pulmonary cr visual damage, and 1.0% die of pulmonary or cardiac involvement. Asbestosis, beryliosis, and byssinosis are al img diseases caused by inhelation of aibbome dusts. Asbestosis is due to chronic inhalation of asbestos fibers, which may affect workers involved in installation and removal of insulation. Asbestosis leads to diffuse fibrosis of the Junge. In this cate, granulomas ere absent or scenty, the most characterictic sign of asbestos exposure is the arbestos body, which is an elongated; beaded rod composed of asbestos fbers coated by proteins. Chronic exposure to beryllium in mining and fabrication leads to beryliosis, a granulomatous condition of the lungs thet is morphologically indistinguishable from sarcoidosis. Clinical history is therefore essential in excluding this condition Byccinocis resus from occupational exposure to cotton fibers. This condition belongs to a diversified category of diseases mediated by abyperseasiiviy seaction against inhaled organic dusts. Farmer's lung, pigeon breeder's lung, ar conditioner lung, and byssinosis are typical examples of these concltions, which lead to 2 granulomatous reacton that may progress to diffise pulmonary fibrosis. Again, clinical history is crucial in the identification of this etiology. Tuberculosis is usually associated with necrotizing eramalommas. This type ofaecrosic is alco decenbed ac caseating because the necrotic material resembles cheese on gross examination. The absence of acid fast organisms in the biopsy material belps rule out this diagnosis. ‘Matic this question <= => Question Ti : 139609 Question 24 of 30 CER of a 45 year old woman who presents with gradual onset of shortness of breath, chest pain, and fatigue shows bilateral hilar adenopathy and pulmonary, infiltrates. Biopsy of one of these lesions shows non-necrotizing granulomas. Stains for fungi and mycobacteria are negative and has nc history of occupational exposure to airbome minerals or organic dusts. The most licely diagnosis is a) Asbestosis ) Bergliosis c) Byssinosis Y © A) Sarcoidosis e) Tuberculosis Question Explanation: The diagnosis of sarcoidosis is usually made by exclusion. This discase is characterized by aon-necrotiring granulomas developing ‘most frequently in the lungs, iymph nodes, retina, heart, spleen, skin, and liver. Non-necrotizing granulomas may be seen in a number of other conditions, however, such as infections and certain forms of pneumoconiosis, which must be suled out before making a diagnosis of sarcoidosis The etiopatkogenesis of sarcoidosis is obscure, Crs mediated mechanisms are thought to be involved in an imeaune responce to as yet usidentiied antigens. About two-thirds of patients with sarcoidesis recover without residual functional deficits, 20% have permanent pulmonary cr visual damage, and 1.0% die of pulmonary or cardiac involvement. Asbestosis, beryliosis, and byssinosis are al img diseases caused by inhelation of aibbome dusts. Asbestosis is due to chronic inhalation of asbestos fibers, which may affect workers involved in installation and removal of insulation. Asbestosis leads to diffuse fibrosis of the Junge. In this cate, granulomas ere absent or scenty, the most characterictic sign of asbestos exposure is the arbestos body, which is an elongated; beaded rod composed of asbestos fbers coated by proteins. Chronic exposure to beryllium in mining and fabrication leads to beryliosis, a granulomatous condition of the lungs thet is morphologically indistinguishable from sarcoidosis. Clinical history is therefore essential in excluding this condition Byccinocis resus from occupational exposure to cotton fibers. This condition belongs to a diversified category of diseases mediated by abyperseasiiviy seaction against inhaled organic dusts. Farmer's lung, pigeon breeder's lung, ar conditioner lung, and byssinosis are typical examples of these concltions, which lead to 2 granulomatous reacton that may progress to diffise pulmonary fibrosis. Again, clinical history is crucial in the identification of this etiology. Tuberculosis is usually associated with necrotizing eramalommas. This type ofaecrosic is alco decenbed ac caseating because the necrotic material resembles cheese on gross examination. The absence of acid fast organisms in the biopsy material belps rule out this diagnosis. Mark this question & => Question Td : 141429 £30 Question 25 An HIV positive male develops symptoms of pneumonia. His fingertips and lips are dusky blue and his blood oxygen saturation is 23%. The test that would be most helpful in confirming the most likely causative agent is 2) CD4 cell count +) Direct fluorescent antibody test ©) Enayme-inked immmosorbent assay 4) Indirect fnorescent antibody tests ©) Western blot Question Explanation: ‘The most commen atypical pneumonia in AIDS is caused by the atypical fungus Pacumocystis jroveci (formerly corind). Patients with Pneumocystis pneumonia have great dificulty producing spunum, so bronchial lavage or transmural biopsy is generally required. The satrple is then subrnitted to the direct flaorescent antbody test or stained with Gomori’s methenamine silver stain to visualize the organisms. The direct fluorescent antibody testis a technique to demonstrate the presence of antigen from a pathogen in a patient “Antigen detecting tests will remain usefil throughout the life of an ATDS patient because they do not depend on detection of the ppatient’s immune response to the pathogen (which maybe absent at the end of AIDS) When stained with methenamine siver, Pneumocystis cysts have a characteristic cup or boat shape; the trophozoites are diffiouk to demonstrate without electron tnicroscopy. CD4 cell count determination is used to measure the progression of infection with the human immunodeficiency virus (EID) Since the virus infects and destroys these cells, the number remaining in the circulation becomes a good predictor ofthe progression of disease. Enzyme linked immunosorbent assay is used as the screening test to diagnose HIV infection. Iris a serologic test (Identifies antibodies against the virus) and once the patient progresses into full blowm AIDS, antibody levels other than IgM will ecline: so in general, serologic tests in late HIV infection tend to be less useful than tests that detect entigens from a pathogen, Tadirect fuorescent antibody tests are techniques that identity antibody production in a patient. They are commonly used to identity autoimmune antibodies, Westem blot is the confirmatory serologic test that is used in the diagnosis of HIV infection Mark this question & => Question Td : 141429 £30 Question 25 An HIV positive male develops symptoms of paeumonia, His fingertips and lips are dasky blue and his blood oxygen saturation is 23%. The test that would be most helpful in confirming the most likely causative agent is a) CD4 cell count Y © b) Direct fluorescent antibody test c) Enzyme-linked iramunosorbent assay 4) Indrect fncrescent antibody tests £) Western blot Question Explanation: ‘The most commen atypical pneumonia in AIDS is caused by the atypical fungus Pacumocystis jroveci (formerly corind). Patients with Pneumocystis pneumonia have great dificulty producing spunum, so bronchial lavage or transmural biopsy is generally required. The satrple is then subrnitted to the direct flaorescent antbody test or stained with Gomori’s methenamine silver stain to visualize the organisms. The direct fluorescent antibody testis a technique to demonstrate the presence of antigen from a pathogen in a patient “Antigen detecting tests will remain usefil throughout the life of an ATDS patient because they do not depend on detection of the ppatient’s immune response to the pathogen (which maybe absent at the end of AIDS) When stained with methenamine siver, Pneumocystis cysts have a characteristic cup or boat shape; the trophozoites are diffiouk to demonstrate without electron tnicroscopy. CD4 cell count determination is used to measure the progression of infection with the human immunodeficiency virus (EID) Since the virus infects and destroys these cells, the number remaining in the circulation becomes a good predictor ofthe progression of disease. Enzyme linked immunosorbent assay is used as the screening test to diagnose HIV infection. Iris a serologic test (Identifies antibodies against the virus) and once the patient progresses into full blowm AIDS, antibody levels other than IgM will ecline: so in general, serologic tests in late HIV infection tend to be less useful than tests that detect entigens from a pathogen, Tadirect fuorescent antibody tests are techniques that identity antibody production in a patient. They are commonly used to identity autoimmune antibodies, Westem blot is the confirmatory serologic test that is used in the diagnosis of HIV infection “Mark this question & => Question 26 of 30 A 59 year old man who is being treated for COPD now presents with an upper respitatory tract infection. He is currently taking theophylline. The man presents to the emergency department 5 days later complaining of persistent tachycardia, insomnia, and agitation, What antibiotic was most licely prescribed for the treatment of his infection? a) Amoxicillin b) Cefaclor ) Chloramphenicol D Doxycycline e) Erythromycin Answer | Explanation Other User's Explanation Report An Error Question Explanation: ‘Theophyline is a xanthine derivative thet relaxes smooth muscle by a direct action, The smooth muscle in the bronchi and pulmonary vessels are particularly affected This agent also stimulates the respiratory center Theophylline is often used for prophylazis and symptomatic relief of bronchial asthma and chronic obstructive pulmonary discase (COED). This agent is extensively metabolized by the CYP450 system in the liver, therefore, any agent that inhibits the iver’ ability to metabolize theopiyline would increase its level in the blood, This would subsequently potentiate the effects, as well as the adverse effecis of theophyline, Erythromycin is a macrolide antibiotic commonly used in the treatment of upper respiratory and skin/subcutanecus infections. Iris a relatively potent hepatic microsomal enzyme inhibitor. Since erythromycin is likely to potentiate the effects of theophylline, it could account for this patient's symptoms. The other agents do not interact with theophylline, Amoxicilin is a penicilin antibiotic used to treat a variety of infections caused by many dfferent organisms. Cefaclor is a second generation cephalosporin used commonly to treat upper and lower respiratory infections. Chloramphenicdl is an antibiotic used to treat severe infections when less tozic agents cannot be used. Doxycycline is a tetracycline antibiotic corunonly used to teat acne and various sexually transmitted diseases “Mark this question & => Question 26 of 30 A 59 year old man who is being treated for COPD now presents with an upper respitatory tract infection. He is currently taking theophylline. The man presents to the emergency department 5 days later complaining of persistent tachycardia, insomnia, and agitation, What antibiotic was most licely prescribed for the treatment of his infection? a) Amoxicillin b) Cefaclor ) Chloramphenicol D Doxycycline ¥ © ¢) Erythromycin Answer | Explanation Other User's Explanation Report An Error Question Explanation: ‘Theophyline is a xanthine derivative thet relaxes smooth muscle by a direct action, The smooth muscle in the bronchi and pulmonary vessels are particularly affected This agent also stimulates the respiratory center Theophylline is often used for prophylazis and symptomatic relief of bronchial asthma and chronic obstructive pulmonary discase (COED). This agent is extensively metabolized by the CYP450 system in the liver, therefore, any agent that inhibits the iver’ ability to metabolize theopiyline would increase its level in the blood, This would subsequently potentiate the effects, as well as the adverse effecis of theophyline, Erythromycin is a macrolide antibiotic commonly used in the treatment of upper respiratory and skin/subcutanecus infections. Iris a relatively potent hepatic microsomal enzyme inhibitor. Since erythromycin is likely to potentiate the effects of theophylline, it could account for this patient's symptoms. The other agents do not interact with theophylline, Amoxicilin is a penicilin antibiotic used to treat a variety of infections caused by many dfferent organisms. Cefaclor is a second generation cephalosporin used commonly to treat upper and lower respiratory infections. Chloramphenicdl is an antibiotic used to treat severe infections when less tozic agents cannot be used. Doxycycline is a tetracycline antibiotic corunonly used to teat acne and various sexually transmitted diseases ‘Mark this question & => Question Td : 177768 Question 27 of 30 A.27 year old male is admitied via emergency department to the hospital for a heroin overdose. His heart rate is 45beats(min, and his blood pressure is 75/40 min Hg. Which one of the following best depicts the results from an arterial blood sample? » [Ph [PaCO2 (@amHg)|HCO3- (mEq/L) [7.22166 26 » [Ph [PaCO2 G@amHg)|HCO3- (nF q/L) "7.3429 15, 9 [Ph [PaCO2 (mmHg)|[HCO3- @nEq1)) \/7.4|}40. 24 4) ‘Ph |PaCO2 (mmHg) |HCO3- (mEq/L) "7.4720 14 2 (Ph |PaCO2 (mmHg) |HCO3- (mEq/L) /7.49}48 [35 Answer | Explanation | Other User's Explanation _Report An Error Question Explanation: ‘This man has a respiratory acidosis. Overdose with drugs that suppress ventilation (e.g., herom, morphine, barbiturates, inethaqualone, and ‘sleeping pills’) often causes hypercapaia Tn patients with an intact renal response, the respiratory acidosis causes a compensatory rise in plasma HCO3- which lessens the fallin pH. However, the reral response requires several days te develop filly. The plasma HCO3- of 26 mEq/L (normal: 22- 28 mEq/L) for this man is typical of acute respiratory acidosis with litle ot no renal compensation. Option B reflects metabolic acidosis. Option Cis normal, Option D reflects respiratory alkalosis. Option E-reflects metabolic, allealosis, ‘Mark this question & => Question Td : 177768 Question 27 of 30 A.D7 year old male is admitted via emergency department to the hospital for a heroin overdose, His heast rate is 45beatsfmin, and his blood pressure is 75/40 mm Hg, Which one ofthe fellowing best depicts the results fom an arterial blood sample? Vea [Ph [PaCO2 (@amHg)|HCO3- (mEq/L) [7.22166 26 » [Ph [PaCO2 G@amHg)|HCO3- (nF q/L) "7.3429 15, 9 [Ph [PaCO2 (mmHg)|[HCO3- @nEq1)) \/7.4|}40. 24 4) ‘Ph |PaCO2 (mmHg) |HCO3- (mEq/L) "7.4720 14 2 (Ph |PaCO2 (mmHg) |HCO3- (mEq/L) /7.49}48 [35 Answer | Explanation | Other User's Explanation _Report An Error Question Explanation: ‘This man has a respiratory acidosis. Overdose with drugs that suppress ventilation (e.g., herom, morphine, barbiturates, inethaqualone, and ‘sleeping pills’) often causes hypercapaia Tn patients with an intact renal response, the respiratory acidosis causes a compensatory rise in plasma HCO3- which lessens the fallin pH. However, the reral response requires several days te develop filly. The plasma HCO3- of 26 mEq/L (normal: 22- 28 mEq/L) for this man is typical of acute respiratory acidosis with litle ot no renal compensation. Option B reflects metabolic acidosis. Option Cis normal, Option D reflects respiratory alkalosis. Option E-reflects metabolic, allealosis, ‘Marke this question = => Question Td : 213679 Question 28 of 30 A.A9-year-old diabetic with a 2-week old lower-extrenity cellulitis has mental status changes and shoriness of breath. He becomes hypotensive, is intubated and admitted to ICU. Temperature =39.4 °C (102.9 °F), pulse= 14min, BP= 85/45 mm Hg, and SPO, = 90% on 100% oxyaen Decreased breath sounds and bilateral crackles exist ABG's show pH-7 18, PCO2—10 mmlg, PO2-60 mmHg and HCO3-20mEqL. CBC shows WBC=18,300/nm?, Hb=12.8 g/dL, Ho=38% and Platelets=225,000smnmn? Cardiac enzymes and ECG are normal. CVP is 8c H20 and PCWP is 14. CXR is as follows The diagnosis ARDS is suspected. * f ome “Which ofthe following features is most suggestive of a diagnosis of ARDS? a) Central venous pressure higher than 12 om H20 b) Positive blood cultures with evidence of sepsis c) Presence of bilateral pleural effusions 4) Pulmonary capillary wedge pressure greater than 18mm Hyg @) Ratio of PaO2/FiO2 of 200 mm Hg of less Answer | Bepianaton | Other User's Explanation Report An Error Question Explanatic This patient has developed acute respiratory distress syndrome (ARDS), a syndrome of acute and persistent lung inlammation with increased vascular permeability. It is characterized by three main features: + Bilateral radiographic infiltrates + A ratio of the partial pressure of arterial oxygen to the traction of inspired oxygen PaO2/FiOZ of 200 mm Hg or less + No clinical evidence for an elevated left atrial pressure or heart felure. The chest x-ray shows bilateral patchy infiltrates. Furthermore, the patient's PaO? is 601 mm Hg on 100% oxygen; thus, the Pa2/PiO2 ratio is less than 200. Finally, the patient has a pulmonary capillary wedge pressure of less than 18. (Central venous pressure that is higher than 12 cm HO is abnormal, Iris not only indicative of ARDS but also may be present as a result of volume overload, right-sided heart failure, and pulmonary hypertension, making ita very nonspecific finding A postive blood ° culture with evidence of sepsis does not necessarily indicate the presence of ARDS. Athough sepsis is the most common cause of ARDS, not every patient with sepsis develops ARDS. The presence of bilateral pleural effusions is not characteristic for ARDS. Pulmonary capilary wedge pressure greater than 18 mm Fide ce ania Seda eed Saar aae co Sag ce cay ne pee ‘Marke this question = => Question Td : 213679 Question 28 of 30 A.A9-year-old diabetic with a 2-week old lower-extrenity cellulitis has mental status changes and shoriness of breath. He becomes hypotensive, is intubated and admitted to ICU. Temperature =39.4 °C (102.9 °F), pulse= 14min, BP= 85/45 mm Hg, and SPO, = 90% on 100% oxyaen Decreased breath sounds and bilateral crackles exist ABG's show pH-7 18, PCO2—10 mmlg, PO2-60 mmHg and HCO3-20mEqL. CBC shows WBC=18,300/nm?, Hb=12.8 g/dL, Ho=38% and Platelets=225,000smnmn? Cardiac enzymes and ECG are normal. CVP is 8c H20 and PCWP is 14. CXR is as follows The diagnosis ARDS is suspected. * f ome “Which ofthe following features is most suggestive of a diagnosis of ARDS? a) Central venous pressure higher than 12 om H20 b) Positive blood cultures with evidence of sepsis c) Presence of bilateral pleural effusions 4) Pulmonary capillary wedge pressure greater than 18mm Hyg Y © @) Ratio of PaO2/FiO2 of 200 mm Hg or less Answer | Bepianaton | Other User's Explanation Report An Error Question Explanatic This patient has developed acute respiratory distress syndrome (ARDS), a syndrome of acute and persistent lung inlammation with increased vascular permeability. It is characterized by three main features: + Bilateral radiographic infiltrates + A ratio of the partial pressure of arterial oxygen to the traction of inspired oxygen PaO2/FiOZ of 200 mm Hg or less + No clinical evidence for an elevated left atrial pressure or heart felure. The chest x-ray shows bilateral patchy infiltrates. Furthermore, the patient's PaO? is 601 mm Hg on 100% oxygen; thus, the Pa2/PiO2 ratio is less than 200. Finally, the patient has a pulmonary capillary wedge pressure of less than 18. (Central venous pressure that is higher than 12 cm HO is abnormal, Iris not only indicative of ARDS but also may be present as a result of volume overload, right-sided heart failure, and pulmonary hypertension, making ita very nonspecific finding A postive blood ° culture with evidence of sepsis does not necessarily indicate the presence of ARDS. Athough sepsis is the most common cause of ARDS, not every patient with sepsis develops ARDS. The presence of bilateral pleural effusions is not characteristic for ARDS. Pulmonary capilary wedge pressure greater than 18 mm Fide ce ania Seda eed Saar aae co Sag ce cay ne pee ‘Mark this question & => Question Td : 213702 Question 29 of 30 ‘A 60-year-old man with history of COPD diagnosed 6 months back by PFTs has productive cough for the last 3 to 4 weeks. He is smoking 1 to 1.5 packs of cigarettes! day for *35 years. He has decreased breath sounds in right upper lobe and diffuse wheezing bilaterally. CHR reveals an upper right-sided pulmonary nodule, CT scan of the chest is shown below: “Which is the nex: step in management of this patient? a) Bronchoscopy ) CT-guided lung biopsy ©) Itraconazole & Pummonery fnction testing ¢) Thoracoscopy Pleural biopsy Answer | Brotanaton | Other User's Explanation Report An Error Question Explanation: ‘This patient has a tumor in his right upper lobe that is most likely cancer. Bronchoscopy with biopsy is the best intial step in eveluating a suspected Ing cancer that is centrally located. There may alsa be astaciated upper lobe collapse secondary ta complete obstruction of the broach. Bronchoscopy is the best test to evaluate the CT scan finding ofa soltary lung mess CT-guided lung biopsy is usually performed when bronchoscopy-guided biopsy is nondiagnostic, or when the lesion is too peripheral for bronchoscopic evaluation. Trraconazale is indicated in the treatment of mild pulmonary aspergilosis. Pulmonary aspergillosis typically presents ia inrunosuppressed or neutropenic patients, and preseats with "Fingal ball" on chest x-ray. This patient is at high risk for hung cancer, and aspergilosis is not the most ikely cause of his condition. Pulmonary fiction testing assists in the evaluation of chronic obstructive pulmonary disease, but it has no diagnostic uility in suspected lang tumors. Thoracoscopy is usefil in the evaluation of pleural pathologies. In this case, the patient hes a solitary lung mess withia the larg parenchyma and the best diagnostic test for this mass is bronchoscopy. ‘A pleural biopsy is a procedure to remove a sample of the tissue lining the lungs and the inside of the chest wall. Ibis indicated in all ‘natente with 2 mieural effieen Even ifeutable it will nat he an initial tect ‘Mark this question & => Question Td : 213702 Question 29 of 30 ‘A 60-year-old man with history of COPD diagnosed 6 months back by PFTs has productive cough for the last 3 to 4 weeks. He is smoking 1 to 1.5 packs of cigarettes! day for *35 years. He has decreased breath sounds in right upper lobe and diffuse wheezing bilaterally. CHR reveals an upper right-sided pulmonary nodule, CT scan of the chest is shown below: “Which is the next step in management of this patient? ¥ © a) Bronchoscopy ) CT-guided lung biopsy ©) Itraconazole & Pummonery fnction testing ¢) Thoracoscopy Pleural biopsy Answer | Brotanaton | Other User's Explanation Report An Error Question Explanation: ‘This patient has a tumor in his right upper lobe that is most likely cancer. Bronchoscopy with biopsy is the best intial step in eveluating a suspected Ing cancer that is centrally located. There may alsa be astaciated upper lobe collapse secondary ta complete obstruction of the broach. Bronchoscopy is the best test to evaluate the CT scan finding ofa soltary lung mess CT-guided lung biopsy is usually performed when bronchoscopy-guided biopsy is nondiagnostic, or when the lesion is too peripheral for bronchoscopic evaluation. Trraconazale is indicated in the treatment of mild pulmonary aspergilosis. Pulmonary aspergillosis typically presents ia inrunosuppressed or neutropenic patients, and preseats with "Fingal ball" on chest x-ray. This patient is at high risk for hung cancer, and aspergilosis is not the most ikely cause of his condition. Pulmonary fiction testing assists in the evaluation of chronic obstructive pulmonary disease, but it has no diagnostic uility in suspected lang tumors. Thoracoscopy is usefil in the evaluation of pleural pathologies. In this case, the patient hes a solitary lung mess withia the larg parenchyma and the best diagnostic test for this mass is bronchoscopy. ‘A pleural biopsy is a procedure to remove a sample of the tissue lining the lungs and the inside of the chest wall. Ibis indicated in all ‘natente with 2 mieural effieen Even ifeutable it will nat he an initial tect ‘Mark this question <= ‘Question Ta : 213888 Question 30 of 30 A 67-year-old alcoholic and chronic smelcer (60 pack years) has shoriness of breath and productive cough. He has had fevers and yellow-green spurum for past several weeks. Being febrile, tachycardic and tachypneic, he maintains SPO = 85% on toom ait ‘There are bilateral decreased breath sounds, occasional wheezing, and scme crackles in the left uoper lobe. Arterial blood ges (ABG) E736 C02: [49mm Poe: [51 amis [CO3'[24 mm He) Complete tlood count [WBC: |]14,200/mn2 fb f4a gat feict 3% Platelets||256,000'mun? CXR shows “What is the most likely diagnosis? a) Acate respiratory distress syndrome (ARDS) ) Chronic obstructive pulmonary disease (COPD) ) Lobar pneumonia 4) Pneumocystis jiroveci pneumonia €) Pulmonary edema 1) Tuberculosis Anowor [EEIRRRHGNY) Other Users Explanation — Repost An Evor Question Explanation An alcoholic patient who presents with productive cough and fever with 2 cavitary lesion in the left upper lobe should be suspected of having reactivational (or secondary) tuberculosis. Respiratory isolation and treatment should be established. “Although acute respiratory distress syndrome (ARDS), chronic obstructive pulmonery disease, lobar pneumonia, Pneumocystis jiroveci pneumonia and pulmonary edema could give a similar clinical presentation and laboratory findings as those of this patient they ra ne ee ee a a ae ae ‘Mark this question <= ‘Question Ta : 213888 Question 30 of 30 A 67-year-old alcoholic and chronic smelcer (60 pack years) has shoriness of breath and productive cough. He has had fevers and yellow-green spurum for past several weeks. Being febrile, tachycardic and tachypneic, he maintains SPO = 85% on toom ait ‘There are bilateral decreased breath sounds, occasional wheezing, and scme crackles in the left uoper lobe. Arterial blood ges (ABG) E736 C02: [49mm Poe: [51 amis [CO3'[24 mm He) Complete tlood count [WBC: |]14,200/mn2 fb f4a gat feict 3% Platelets||256,000'mun? CXR shows “What is the most likely diagnosis? a) Acate respiratory distress syndrome (ARDS) ) Chronic obstructive pulmonary disease (COPD) ) Lobar pneumonia 4) Pneumocystis jiroveci pneumonia ) Pulmonery edema Y © # Tuberculosis Anower [EEIIRRRHBRY) Other Users Explanation — Repost An vor Question Explanation An alcoholic patient who presents with productive cough and fever with 2 cavitary lesion in the left upper lobe should be suspected of having reactivational (or secondary) tuberculosis. Respiratory isolation and treatment should be established. “Although acute respiratory distress syndrome (ARDS), chronic obstructive pulmonery disease, lobar pneumonia, Pneumocystis jiroveci pneumonia and pulmonary edema could give a similar clinical presentation and laboratory findings as those of this patient they ra ne ee ee a a ae ae 2:25/2014 21346 PM Mark this question => Question Td : 29004 Question 1 of 30 AAG-year-old male with Type 2 diabetes presented to the clinic as his wife complained that he snored excessively. Which of the following would suggest a diagnosis of obstructive sleep apnea? a) Nasal polyps ) Noctumal cough ¢) Poor memory 4) Daytime somnolence 8) Stridor Question Explanation: The typical problem associated with sleep apnea syndrome is excessive daytime somnolence n Report An Error “Associated with: - Obesity - Acromegaly - Hypothyroidism and - Cushing's smdvome Eis fet that the somnolence is duc to the interruption of rapid eye movement (REM) sleep by frequent episodes of welking due to apneic episodes. 2:25/2014 21346 PM Mark this question => Question Td : 29004 Question 1 of 30 A.A6-year-old male with Type 2 diabetes presented to the olinic as his wife ccmplained that he saored excessively. Which of the following would suggest a diagnosis of obstructive sleep apnea? a) Nasal polyps ) Nocturnal cough ¢) Poor memory oY © 4d Daytime somnolence €) Stridor Question Explanation: The typical problem associated with sleep apnea syndrome is excessive daytime somnolence n Report An Error “Associated with: - Obesity - Acromegaly - Hypothyroidism and - Cushing's smdvome Eis fet that the somnolence is duc to the interruption of rapid eye movement (REM) sleep by frequent episodes of welking due to apneic episodes. 2/25/2014 2:44:03 PM Mark this question => Question 2 of 30 A 48-year-old woman with breatilessness has following arterial blood gases results pO 87 kPa co, [44 kPa bu [746 ICO? 24 mEq/L ‘Which of the followingiis the most lkely diagnosis? 8) Hyperveatilation syndrome ') Emphysema ) Eyphescoliosis 4) Acute severe asthma ©) Opiate overdose Answer (Explanation) Other User's Explanation Report An Error Question Explanation: ‘This patient has an acute respiratory alkalosis with associcted hypoxia This is consistent with an acute asthmatic attack. A normal or rising CO3 is an ominous sign indicative of a life threatening attack and the need to consider ventilatory support, Patients with hyperventilation syndrome do show respiratory alkalosis tut this is not associated with hypoxia. 2/25/2014 2:44:03 PM Mark this question => Question 2 of 30 A 48-year-old woman with breatilessness has following arterial blood gases results pO 87 kPa co, [44 kPa bu [746 ICO? 24 mEq/L ‘Which of the followingiis the most lkely diagnosis? 8) Hyperveatilation syndrome ') Emphysema ) Eyphescoliosis Y © d) Acute severe asthma ©) Opiate overdose Answer (Explanation) Other User's Explanation Report An Error Question Explanation: ‘This patient has an acute respiratory alkalosis with associcted hypoxia This is consistent with an acute asthmatic attack. A normal or rising CO3 is an ominous sign indicative of a life threatening attack and the need to consider ventilatory support, Patients with hyperventilation syndrome do show respiratory alkalosis tut this is not associated with hypoxia. 2/25/2014 2:14:25 PM “Mark this question <=> Question Id: 29114 Question 3 of 30 A Al-year-old man is undergoing investigation for Acromegaly. MRI of the pitutary is normal, but a routine chest x-ray reveals @ large centraly based mass. The patientis a non-smoker. What is the most likely type of this lung tumcur? a) Squamous cell ') Sunall cell c) Large cell 4) Carcinoid ©) Adenocarcinoma Question Explanation ‘A central besed mass in anon-smoker showing clinical evidence of nsuroendocrine cell origin is consistent with a carcinoid and surgery offers a very high chance of cure. 2/25/2014 2:14:25 PM “Mark this question <=> Question Id: 29114 Question 3 of 30 A Al-year-old man is undergoing investigation for Acromegaly. MRI of the pitutary is normal, but a routine chest x-ray reveals @ large centraly based mass. The patientis a non-smoker. What is the most likely type of this lung tumcur? a) Squamous cell ') Sunall cell c) Large cell VW © & Carcinoid ©) Adenocarcinoma Question Explanation ‘A central besed mass in anon-smoker showing clinical evidence of nsuroendocrine cell origin is consistent with a carcinoid and surgery offers a very high chance of cure. 2252014 2:44:41 PM ‘Mark this question & => Question Id : 29124 Question 4 of 30 A 66-year-old with known COPD has 6 week old gradually increasing shortness of breath. treated with inhalers. He was afebrile. sildy confused with a respiratory rate =27 bpm and no CXR changes Investigations revealed [pO2 |? kPa (9-12.6) sco, fa5ePa (47-60) [pe [73 (7-36-7.44) ‘What isthe most appropriate immediate management? a) High flow oxygen therapy ) Nebulized salbutamol and ipretropiumn bromide o) Intravenous aminophylline 4) Intravenous hydrocortisone @) Intubation and mechanical ventilation Question Explanatior ‘The patient normally uses inhalers and therefore is likely to respond to nebulized bronchodilator, which should be nebulized with air and not high flow oxygen. Corticosteroids have been shown to reduce length of stay in hospital and are usually given in acute exacerbations of COPD. Non-invasive ventilation chould be tried fst in severe casec before progressive to mecharical ventilation if appropriate. IV aminophylline is recommended as a second to third line therapy of COPD with reversible airways 2252014 2:44:41 PM ‘Mark this question & => Question Id : 29124 Question 4 of 30 A 66-year-old with known COPD has 6 week old gradually increasing shortness of breath. treated with inhalers. He was afebrile. sildy confused with a respiratory rate =27 bpm and no CXR changes Investigations revealed [pO2 |? kPa (9-12.6) sco, fa5ePa (47-60) [pe [73 (7-36-7.44) “What is the most appropriate immediate management? a) High flow oxygen therapy ¥ © b) Nebulized salbutamol and ipratropium bromide c) Intravenous aminophylline 4) Intravenous hydrocortisone ) Intubation and mechanical vervilation Question Explanatior ‘The patient normally uses inhalers and therefore is likely to respond to nebulized bronchodilator, which should be nebulized with air and not high flow oxygen. Corticosteroids have been shown to reduce length of stay in hospital and are usually given in acute exacerbations of COPD. Non-invasive ventilation chould be tried fst in severe casec before progressive to mecharical ventilation if appropriate. IV aminophylline is recommended as a second to third line therapy of COPD with reversible airways ‘Mark this question & => Question Td : 29643 Question 5 of 30 Regarding the radiological appearance ofa chest x-ray, which of the fellowing statements is not true? a) Consolidation of the right middle lobe will obliterate the nght atrial shadow in the PA view +b) Consclidation offthe left lower lobe wil elevate the left hemidiaplragm c) Consolidation of the right apical segment will extend to the horizontal fissure in the PA view: 4) Consolidation of the right anterior segment of the right middle lobe will extend to the right transverse fissure and the right hilum in BA view ©) Consolidation of the lingular lobe well obliterate the aortic knuckle and pulmonary trank in the PA view Answer | Biplanation | Other User's Explanation Report An Error Question Explanation: Consolidation n leftlower lobe obliterates the diaphragm whilst ingular consolidation will obliterate the left heart border. Oblique fissure runs obliquely at 45° from T4 or TS vertebra to anterior costophrenic angle on lateral chest film. The horizontal fissure rans fiom the hilum anteriorly to anterior chest wall. The area above the horizontal fissure is upper lobe, below the horizontal fesure is the middle lobe and below the oblique Lssure is the lower lobe. ‘Mark this question & => Question Td : 29643 Question 5 of 30 Regarding the radiological appearance ofa chest x-ray, which of the fellowing statements is not true? a) Consolidation of the right middle lobe will obliterate the nght atrial shadow in the PA view Y © ¥) Consolidation of the left lower lobe will elevate the left hemidiaphragm c) Consolidation of the right apical segment will extend to the horizontal fissure in the PA view: 4) Consolidation of the right anterior segment of the right middle lobe will extend to the right transverse fissure and the right hilum in BA view ©) Consolidation of the lingular lobe well obliterate the aortic knuckle and pulmonary trank in the PA view Answer | Biplanation | Other User's Explanation Report An Error Question Explanation: Consolidation n leftlower lobe obliterates the diaphragm whilst ingular consolidation will obliterate the left heart border. Oblique fissure runs obliquely at 45° from T4 or TS vertebra to anterior costophrenic angle on lateral chest film. The horizontal fissure rans fiom the hilum anteriorly to anterior chest wall. The area above the horizontal fissure is upper lobe, below the horizontal fesure is the middle lobe and below the oblique Lssure is the lower lobe. 2i25/2014 2:45:13 PM Merk this question & => Question Td : 29703 Question 6 of 30 “Which of the following is a recognized treatment for complications of cystic fisrosis? a) DINAase to assist in reinflating collapsed Iung segments ) Hypotoric saline drinks for hypernatremic dehydration ©) Pancreatic transplant for diabetes melitus ¢) Nebulized tobramycin for pseudomonas colonization of the lower respiratory tract 8) Rectal pull-through and anastorrosis for rectal profapse Question Explanatio Euman recombinant DNAse given at a single daily aerosol seems to improve pukmoncry finction, decrease the frequency of chest exaverbations and promotes a sense of well-being in patients with mild to moderate disease with purulent secretions ‘This may be because in the inflamed airways, the nuclei om dead cells account for rmuch of the wiscidity of secretions Rectal prolapse is usually idiopathic, occurring between one and five years. Intestinal parasites, malnutrition, acute diarvhea, ulcerative coliis, pertussis, Ehler’s Danlos Syndrome, meningocele, cystic ftrosis and chronic constipation can also predispose to it, Following defecation the prolapse usually resolves spontaneously or through manual reinsertion by the pakent or parent. Netulised tobramycin or gentamicin may be given when airway pathogens are resistant to oral antibiotics, or where infection is difficub to conivol at home. Hypemnatremic dehydration should be treated in the usual way 2i25/2014 2:45:13 PM Merk this question & => Question Td : 29703 Question 6 of 30 “Which ofthe following is a recognized treatment for complications of cystic rosie? a) DINAase to assist in reinflaing collapsed hing segments +b) Hypotoric saline chinks for hypematromic dehydration ©) Pancreatic transplant for diabetes melitus V © & Nebulzed tobramycin for pseudomonas colonization of the lower respiratory tract 8) Rectal pull-through and anastorrosis for rectal profapse Question Explanatio Euman recombinant DNAse given at a single daily aerosol seems to improve pukmoncry finction, decrease the frequency of chest exaverbations and promotes a sense of well-being in patients with mild to moderate disease with purulent secretions ‘This may be because in the inflamed airways, the nuclei om dead cells account for rmuch of the wiscidity of secretions Rectal prolapse is usually idiopathic, occurring between one and five years. Intestinal parasites, malnutrition, acute diarvhea, ulcerative coliis, pertussis, Ehler’s Danlos Syndrome, meningocele, cystic ftrosis and chronic constipation can also predispose to it, Following defecation the prolapse usually resolves spontaneously or through manual reinsertion by the pakent or parent. Netulised tobramycin or gentamicin may be given when airway pathogens are resistant to oral antibiotics, or where infection is difficub to conivol at home. Hypemnatremic dehydration should be treated in the usual way 2/25/2014 2:45:26 PM “Mark this question => Question Id : 29733 Question 7 of 30 A 21-year-old female with cystic bros presents in early pregnancy wanting advice. Her partner is a cartier of the cystic fibrosis gene. Which of the following percentages best represents the chance of er child having cystic fibrosis? a) 15% 6) 30% ) 50% ) 7% ) 90% Avewor [UBIPIRNSRIR) otnorUeor® Explanation Report An Error Question Explanation: ‘The patient is homozygous for CP (CF/CF) and the father is heterozygous for CF (CF/M). Thus there is a 50% chance that her child will be homozygous for CF and a 50% chance that the child will be a carrier. 2/25/2014 2:45:26 PM “Mark this question => Question Id : 29733 Question 7 of 30 A 21-year-old female with cystic Sbrosis presents in early pregnancy wanting advice. Her partner is a cavrier of the cystic fibrosis gene. Which of the following percentages best represents the chance of er child having cystic fibrosis? a) 15% 6) 30% Y 2) 50% ) 7% ©) 90% Avewor [UBIPIRNSRIR) otnorUeor® Explanation Report An Error Question Explanation: ‘The patient is homozygous for CP (CF/CF) and the father is heterozygous for CF (CF/M). Thus there is a 50% chance that her child will be homozygous for CF and a 50% chance that the child will be a carrier. “Mark this question => Question Id : 29774 Question 8 of 30 A61 year old male presents with inspiratory stiidor. Findings of chest X-ray reveals compression of the trachea by a retvostemal goitre. The severity of his airway obstruction can be assessed by which one of the following? ) forced expiratory vohume ) forved vital capacity ©) flow/volume loop 4) peak expiratory low rate 6) residual volume Question Explanation: A flow volume loop is the most appropriate investigation to assess severity of the obstruction. “Mark this question => Question Id : 29774 Question 8 of 30 A61 year old male presents with inspiratory stiidor. Findings of chest X-ray reveals compression of the trachea by a retvostemal goitre. The severity of his airway obstruction can be assessed by which one of the following? 2) forced expiratory volume ) forced wital capacity Y © 0) flow/volume loop ¢) peak expiratory flow rate ¢) residual volume Question Explanation: A flow volume loop is the most appropriate investigation to assess severity of the obstruction. 2/25/2014 2:45:55 PM ‘Mark this question & => Question Td : 29844 Question 9 of 30 A patientis suffering from sarcoidosis. Which of the following would be the least lly finding in this patient? a) Hepatic granulomas +b) X bodies on bronchoalveolar lavage (EAL) duid ©) Restrictive pulmonary function tests 6) Skin lesions ©) Uveitis Question Explanation: Sarcoidosis is associated with Uveitis, Arthritis, Pulmonary fibrosis, Lymphadenopathy and Skin change- lupus perniof Erythema nodosum Tis charachterised histologically by the presence of non-caseating granulomas which may occur anywhere. These granulomas have the capacity to produce 1, 25 vitamin D explaining the associated hypercalcaemia, Pentalaminar X{ bodies (Birbeck granules) found on BAL are considered diagnostic of pulmonary histiocytosis X and so would not be expected with sarcoidosis, 2/25/2014 2:45:55 PM ‘Mark this question & => Question Td : 29844 Question 9 of 30 A patientis suffering from sarcoidosis. Which of the following would be the least lly finding in this patient? a) Hepatic granulomas Y © b) X bodies on bronchoalveolar lavage (EAL) uid ©) Restrictive pulmonary function tests 6) Skin lesions ©) Uveitis Question Explanation: Sarcoidosis is associated with Uveitis, Arthritis, Pulmonary fibrosis, Lymphadenopathy and Skin change- lupus perniof Erythema nodosum Tis charachterised histologically by the presence of non-caseating granulomas which may occur anywhere. These granulomas have the capacity to produce 1, 25 vitamin D explaining the associated hypercalcaemia, Pentalaminar X{ bodies (Birbeck granules) found on BAL are considered diagnostic of pulmonary histiocytosis X and so would not be expected with sarcoidosis, 2:25/2014 2:46:09 PM ‘Mark this question & => Question Td : 46870 Question 10 of 30 “Which of the following could best diagnose tensicn pneumothorax’? a) Stat CT scan ») Clinical exam c) Chest x-ray. ) Watch and wait Answer (Explanation) Other User's Explanation Report An Error Question Explanation: ‘Tension peumotorast is a pneumothorax causing progressive rise in intrapleural pressure to levels that become positive throughout the respiratery cycle and collapse the lung, shift the mediastinum, and inpair venous retum to the heart. Air continues to getinto the pleural space but cannot exit, Without proper treatment, the impaired venous return can cause systemic hypotension and respiratory and cardiac arrest within minutes ‘Tension pneumotorast most commonly occurs in patients receiving positive pressure mechanical ventlation (particularly during resuscitation), Ibis rarely a complication of traumatic pneumothorax, when a chest wound acts as a one way valve that traps increasing volumes of air in the pleural space with inspiration. Diagnosis is based on physical examination and chest x-ray. Most paeumothoraces require needle thoracentesis or tube thoracostomy. 2:25/2014 2:46:09 PM ‘Mark this question & => Question Td : 46870 Question 10 of 30 “Which of the following could best diagnose tensicn pneumothorax’? a) Stat CT sean Y © b) Clinical exam c) Chest x-ray ) Watch and wait Answer (Explanation) Other User's Explanation Report An Error Question Explanation: ‘Tension peumotorast is a pneumothorax causing progressive rise in intrapleural pressure to levels that become positive throughout the respiratery cycle and collapse the lung, shift the mediastinum, and inpair venous retum to the heart. Air continues to getinto the pleural space but cannot exit, Without proper treatment, the impaired venous return can cause systemic hypotension and respiratory and cardiac arrest within minutes ‘Tension pneumotorast most commonly occurs in patients receiving positive pressure mechanical ventlation (particularly during resuscitation), Ibis rarely a complication of traumatic pneumothorax, when a chest wound acts as a one way valve that traps increasing volumes of air in the pleural space with inspiration. Diagnosis is based on physical examination and chest x-ray. Most paeumothoraces require needle thoracentesis or tube thoracostomy. 2/25/2014 2:46:22 PM Mark this question & => Question Td : 54189 Question 11 of 30 A.66 year cld man develops hemoptysis, weight loss and chest pain, His initial chest X-ray reveals a mass whichis further confirmed by CT ofthe chest. Biopsy confirms malignancy. All ofthe fellowing could be the cause ofhis hing cancer, except 2) Asbestos +) Coal miring ©) Marijuana 4) Nickel mining, €) Tobacco Quostion Explanation: Lung cancer is now the number one cause of cancer deaths in both men and women, Initial symptoms include hemoptysis, chest pain and weight loss. Imaging studies such as chest x-ray and chest CT are done. A bronchoscopy guided biopsy is needed to obtain a tissue sample which will be looked at by the pathologist to determine ifthe mass is malignant. Smoking causes 87% of all lung cancers, Other cauces include exposure to the following: asbestos, radon, arcenic, chromium, coal dust, uranium, sickel, aromatic hydrocarbons and ethers. Marijuana, unlike tobacco and alcohol, does no: appear to cause head, neck, or lung cancer. 2/25/2014 2:46:22 PM Mark this question & => Question Td : 54189 Question 11 of 30 A.66 year cld man develops hemoptysis, weight loss and chest pain, His initial chest X-ray reveals a mass whichis further confirmed by CT ofthe chest. Biopsy confitms malignancy. All ofthe following could be the cause ofhis ing cancer, except 2) Asbestos 6) Coal mining ¥ © ©) Marijuana 4) Nickel mining, ) Tobacco Quostion Explanation: Lung cancer is now the number one cause of cancer deaths in both men and women, Initial symptoms include hemoptysis, chest pain and weight loss. Imaging studies such as chest x-ray and chest CT are done. A bronchoscopy guided biopsy is needed to obtain a tissue sample which will be looked at by the pathologist to determine ifthe mass is malignant. Smoking causes 87% of all lung cancers, Other cauces include exposure to the following: asbestos, radon, arcenic, chromium, coal dust, uranium, sickel, aromatic hydrocarbons and ethers. Marijuana, unlike tobacco and alcohol, does no: appear to cause head, neck, or lung cancer. 2252014 2:46:34 PM “Mark this question €& => Question 12 of 30 Tn an acute attack of asthma, which of the following is not given? 8) Sodiura cromolya ') Antibiotic ©) Theopiylline 6) Salmeterol «) Steroid newer [JERVIRNSHAN) otner Users Exptan Question Explanation: n Report An Error ‘Asthma is a disease of diffuse airway inflammation caused by a vasiety of triggering stimuli resulting in pavtally or compleiely reversible bronchoconstriction. Symnptoms and signs include dyspnea, chest tightness, and wheezing, ‘The diagnosis is based on history. physical examination, and pulmonary function tests, Treatment involves controlling tiggering factors and drug therapy, most commonly with inhaled beta 2 agonists and ithaled corticosteroids. Cromolynis used more in the prophylaxis of asthuna, 2252014 2:46:34 PM “Mark this question €& => Question 12 of 30 In an acute attack of asthma, which of the folowing is not given? Y © 2) Sodium cromolyn ') Antibiotic ©) Theopiylline ©) Salinetero! «) Steroid newer [JERVIRNSHAN) otner Users Exptan Question Explanation: n Report An Error ‘Asthma is a disease of diffuse airway inflammation caused by a vasiety of triggering stimuli resulting in pavtally or compleiely reversible bronchoconstriction. Symnptoms and signs include dyspnea, chest tightness, and wheezing, ‘The diagnosis is based on history. physical examination, and pulmonary function tests, Treatment involves controlling tiggering factors and drug therapy, most commonly with inhaled beta 2 agonists and ithaled corticosteroids. Cromolynis used more in the prophylaxis of asthuna, ‘Mark this question = => Question 13 of 30 A.21 years old boy was stabbed in the chest in a bar fight and arrived at the emergency room within 30 minutes. You noticed thet the trachea is deviated away from the side of the chest that suffered the puncture, Upon physical exemination of the traumatized side what will you find? a) Increased fremitus b) Increased breath sounds ©) Dullnsss to percussion ) Wheezing and stridor ¢) Hyper-resonant percussion Anowor (UBRNBREN) omer Users Explanation Report An or Question Explanation: A pneumothorax refers to a collection of gas in the pleural space resulting in collapse of the lung on the aifected side. A tension pneumothorax is a life-threatening condition caused by air within the pleural space that is under pressure; displacing mediastinal structures and compromising cardiopulmonary finction. A iraumatic pneumothorax results from blunt or penetrating injury that disrupts the parietal or visceral pleura Physical findings classically consist of absent tactile fremibus, kyper-resonance to percussion, and decrsased breath sounds on the side with the pneumothorax. If the pneumothorax is large, the side with the pneumothorax may be enlarged with the trachea visibly shiited to the opposite side. Pneumothorax sat /, Stoo Actas ten Tot ofatr tend ibcimg nde pasison be ingand Testa of ls 7 amnion a ome” Diagnosis is made with an upright inspiratory chest x-ray, Treatmert invelves needle thoracentesis to release the trapped air followed [ee ecene er achaor eee ‘Mark this question = => Question 13 of 30 A.21 years old boy was stabbed in the chest in a bar fight and arrived at the emergency room within 30 minutes. You noticed thet the trachea is deviated away from the side of the chest that suffered the puncture, Upon physical exemination of the traumatized side what will you find? a) Increased fremitus b) Increased breath sounds ©) Dullnsss to percussion ) Wheezing and stridor Y © c)Hyper-resonant percussion Anowor [UBQHRNBRENE omnes Users Explanation Report An or Question Explanation: A pneumothorax refers to a collection of gas in the pleural space resulting in collapse of the lung on the aifected side. A tension pneumothorax is a life-threatening condition caused by air within the pleural space that is under pressure; displacing mediastinal structures and compromising cardiopulmonary finction. A iraumatic pneumothorax results from blunt or penetrating injury that disrupts the parietal or visceral pleura Physical findings classically consist of absent tactile fremibus, kyper-resonance to percussion, and decrsased breath sounds on the side with the pneumothorax. If the pneumothorax is large, the side with the pneumothorax may be enlarged with the trachea visibly shiited to the opposite side. Pneumothorax sat /, Stoo Actas ten Tot ofatr tend ibcimg nde pasison be ingand Testa of ls 7 amnion a ome” Diagnosis is made with an upright inspiratory chest x-ray, Treatmert invelves needle thoracentesis to release the trapped air followed [ee ecene er achaor eee Mark this question e => Question Id : 55702 Question 14 of 30 An asthmatic patient presents to the ER with dyspnea, He used his inhaled salbutamol with no improvement. Paysical exam shows éyspnea with thonchi in the upper chest. The most appropriate management for this patient is a) IV duids and Aminophyline +) Inhaled Corticosteroids c) Inhaled Salbutamol, 4) IV methiprednisolone Question Explanation: Jn the treatment of an asthma exacerbation systemic corticosteroids (prednisone, predaisolone, methylprednisolone) should be given for all acute exacerbation; they are unnecessary for patients whose PEFR normalizes after 1 or 2 bronchodilator doses. IV methylprednicolone oan be given an IV line is already in place and can be ewitched to ceal dosing whenever necessary o ‘convenient Mark this question e => Question Id : 55702 Question 14 of 30 An asthmatic patient presents to the ER with dyspnea. He used his inhaled salbutamol with no improvement. Paysical exam shows éyspnea with thonchi in the upper chest. The most appropriate management for this patient is a) TV Quids and Aminophyline Y © b) Inhaled Corticosteroids c) Inhaled Salbutamol, 4) IV methiprednisolone Question Explanation: Jn the treatment of an asthma exacerbation systemic corticosteroids (prednisone, predaisolone, methylprednisolone) should be given for all acute exacerbation; they are unnecessary for patients whose PEFR normalizes after 1 or 2 bronchodilator doses. IV methylprednicolone oan be given an IV line is already in place and can be ewitched to ceal dosing whenever necessary o ‘convenient 2i25/2014 2:47:21 PM ‘Mark this question & => Question 15 of 30 “Which one of'the following measures of pulmonary function is characteristically increased in elderly? a) Toval lung capacity ) Vital capacity ) Resting PaO, artenal tension 4) Functional residual capacity 6) One-second forced expiratory vaume (FEV1) Anower (FEIGIRRRHONY) Other Users Explanation Report An E01 Question Explanation: Although vital capacity decines with age, total lung capacity remains constant, The reduction in vital capacity results from an increase in residual volume, This increase in residual volume and functional residual capacity results from the collapse of stall aways that occurs at higher lung volumes as age increases. Residual volume increases nearly 50% between early adulthood and age 70. In addition, standard spirometric measurements of lang finction ((e,, forced expiratory volume in 1 second, peak expiratory flow rate, and maximal expiratory low volume) have been shown to decline with age. Asterial oxygen tension also slowly decines with age. 2i25/2014 2:47:21 PM ‘Mark this question & => Question 15 of 30 “Which one of'the following measures of pulmonary function is characteristically increased in elderly? a) Total hing capacity ') Vital capacity ¢) Resting PaO artenal tension Y © 4) Functional residual capacity 6) One-second forced expiratory volume (FEV1) Anower (FEIGIRRRHONY) Other Users Explanation Report An E01 Question Explanation: Although vital capacity decines with age, total lung capacity remains constant, The reduction in vital capacity results from an increase in residual volume, This increase in residual volume and functional residual capacity results from the collapse of stall aways that occurs at higher lung volumes as age increases. Residual volume increases nearly 50% between early adulthood and age 70. In addition, standard spirometric measurements of lang finction ((e,, forced expiratory volume in 1 second, peak expiratory flow rate, and maximal expiratory low volume) have been shown to decline with age. Asterial oxygen tension also slowly decines with age. Mark this question e-_> Question Td : 60350 Question 16 of 30 “Which one of the following is the LEAST commen tumor to be found in the anterior mediastinumn? 2) Thymoma &) Lymphoma ) Mesothelioma 4) Teratomna ©) Germ cell mmor Question Explanation: Mesothelioma: are less licely to be found in the anterior mediastinum than a thymoma, Lymphoma, teratoma, and germ call tumor (€.g,, seminoma, teratocarcnoma), n Report An Error Mark this question e-_> Question Td : 60350 Question 16 of 30 “Which one of the following is the LEAST commen tumor to be found in the anterior mediastinumn? 2) Thymoma &) Lymphoma Y © 0) Mesothelioma 4) Teratomna ©) Germ cell mmor Question Explanation: Mesothelioma: are less licely to be found in the anterior mediastinum than a thymoma, Lymphoma, teratoma, and germ call tumor (€.g,, seminoma, teratocarcnoma), n Report An Error 2125/2014 2:47:48 PM “Mark this question => Question Id : 67225 Question 17 of 30 A.27 year old man with chest pan presents to the ER with increased respiratory rate On physical exam he has decteased breath sounds on the right side and his trachea is shattly deviated to the left. The most appropriate initial management of this patient is a) Needle thoracentesis ) Chest tube ©) Albuterol ) Epinephrine Answer | Bibianaton | Other User's Explanation Report An Error Question Explanation Paeumothoraxis air in the pleural space causing partial or complete hing collapse. Preumothorax can occur spontaneously or from underlying puimonary disease, trauma, or medical procedures. Symptoms include dyspnea, pleuriic chest pain, and anxiety. Dyspnea may be sudden or gradual in onsot depending on the rate of development and size ofthe pneumothorast. Pain can simulate cardiac ischemia, musculoskeletal injury (when referred to the shoulder), or an Tntra-abdominal process (when referred to the abdomen), Physical findings classically consist of absent tactile fremius, hyperresorance to percussion, and decreased breath sounds on the side with the pneumothorax. IFthe pneumothorax is large, the side with the pneumothorax may be enlarged with the trachea visibly shifted to the opposite side, Tension pneumothorax is a medical emergency. It should te treated immedictely with needle thoracentocis, followed by placement of chest mube. 2125/2014 2:47:48 PM “Mark this question => Question Id : 67225 Question 17 of 30 A.27 year old man with chest pan presents to the ER with increased respiratory rate On physical exam he has decteased breath sounds on the right side and his trachea is shattly deviated to the left. The most appropriate initial management of this patient is Y © aj Needle thoracentesis ) Chest tube ©) Albuterol ) Epinephrine Answer | Bibianaton | Other User's Explanation Report An Error Question Explanation Paeumothoraxis air in the pleural space causing partial or complete hing collapse. Preumothorax can occur spontaneously or from underlying puimonary disease, trauma, or medical procedures. Symptoms include dyspnea, pleuriic chest pain, and anxiety. Dyspnea may be sudden or gradual in onsot depending on the rate of development and size ofthe pneumothorast. Pain can simulate cardiac ischemia, musculoskeletal injury (when referred to the shoulder), or an Tntra-abdominal process (when referred to the abdomen), Physical findings classically consist of absent tactile fremius, hyperresorance to percussion, and decreased breath sounds on the side with the pneumothorax. IFthe pneumothorax is large, the side with the pneumothorax may be enlarged with the trachea visibly shifted to the opposite side, Tension pneumothorax is a medical emergency. It should te treated immedictely with needle thoracentocis, followed by placement of chest mube. ‘Marle this question & => Question Td : 73994 Question 18 of 30 A 6-year-old man who has recently moved to the Ohio River Valley develops cough, fever, pneumonitis and fatigue. & likely finding on radiographic evaluation is a) Congestive heart falure. ) Enlarged heart. ©) Aortic calcification ©) Splenic caleificaton ©) Elevated hemidiaphragm. Answer [BEIBIBERNBN) other Users Explanation Repost An Exor Question Explanatic This patient lely has Histoplasmosis which is associated with splenic califications on radiography. None of the other radiographic findings are typical of Histoplasmosis, ‘Marle this question & => Question Td : 73994 Question 18 of 30 A 46-year-old man who has recently maved ta the Okio River Valley develope cough, fever, pneumonitis and fatigue A likely finding on radiographic evaluation is a) Congestive heart falure. ) Enlarged heart ©) Aortic calcification V © @ Splenic calcification. 6) Elevated hemidiaphragra, Answer [BEIBIBERNBN) other Users Explanation Repost An Exor Question Explanatic This patient lely has Histoplasmosis which is associated with splenic califications on radiography. None of the other radiographic findings are typical of Histoplasmosis, 2125/2014 2:50:56 PM “Mark this question €&c> Question 19 of 30 All of the following are true of cough EXCEPT a) Tthas both afferent and efferent pathways +b) Regurgitation of acidic gastric contents into the tracheobronchial tree may cause 2 ©) By short-circuiting glottic closure, a tracheostomy increases the effectiveness of the cough mechanism, ) Edema and hyperemia of respiratory mucous membranes stimulate cough receptors, ©) Airway compression by an aortic aneurysm prochices it by stimulating mechanical receptors Question Explanation: By short-circuiting glottic closure, both tracheostomy and endotracheal tubes decrease the effectiveness of the cough mechanism. ‘Cough is an explosive expiration that clears the tracheobronchial tree of secretions and foreign bodies. Ac a defensive reflex, ithas both afferent and efferent pathways, Inflammatory, mechanical, chemical and thermal stimulation of cough receptors produces a cough, Regurgtation of gastric contents into the tracheobronchial tree during sleep stirmulates these receptors. Inflammatory stimuli ace nnitated by edema and byperemia of the respiratory mucous membranes in concitions such as bacterial or viral bronchitis, ‘Mechanical stimuli are initiated by dust particles cr lesions associated with airway compression, such as aortic aneurysms, mediastinal tumors, or endobronchial granulomas 2125/2014 2:50:56 PM “Mark this question €&c> Question 19 of 30 All ofthe following are true of cough EXCEPT a) Tthas both afferent and efferent pathways +b) Regurgitation of acidic gastric contents into the tracheobronchial tree may cause i VY © o) By short-citcuiting glottic closure, a tracheostomy increases the effectiveness of the cough mechanism. 4) Edema and hyperemia of respiratory mucous membranes stimulate cough receptors @) Airway compression by an aortic aneurysm prochces it by stimulating mechanical receptors Question Explanation: By short-circuiting glottic closure, both tracheostomy and endotracheal tubes decrease the effectiveness of the cough mechanism. ‘Cough is an explosive expiration that clears the tracheobronchial tree of secretions and foreign bodies. Ac a defensive reflex, ithas both afferent and efferent pathways, Inflammatory, mechanical, chemical and thermal stimulation of cough receptors produces a cough, Regurgtation of gastric contents into the tracheobronchial tree during sleep stirmulates these receptors. Inflammatory stimuli ace nnitated by edema and byperemia of the respiratory mucous membranes in concitions such as bacterial or viral bronchitis, ‘Mechanical stimuli are initiated by dust particles cr lesions associated with airway compression, such as aortic aneurysms, mediastinal tumors, or endobronchial granulomas 2/25/2014 2:51:09 PM 280218 Mark this question & => Question Td Question 20 of 30 Cigarette smoking is associeted with all of the following EXCEPT 2) Emphysema ') Chronic bronchits. ©) Exacerbation of asthma. 6) Tnfinenza ©) Squamous Cell Carcinoma of Lung. ‘newer [UBQIRNBHER) otter Usor's Explanation Report An Error Question Explanation: Tafluenza is not associated with cigarette smoking. All of the other conditions are associated with cigarettes. 2/25/2014 2:51:09 PM 280218 Mark this question & => Question Td Question 20 of 30 Cigarette smoking is associeted with all ofthe following EXCEPT a) Emphysema. +) Chronic bronchitis ¢) Execerbation of asthma. Y¥ © & Infnenza ¢) Squamous Cell Carcinoma of Lung, ‘newer [UBQIRNBHER) otter Usor's Explanation Report An Error Question Explanation: Tafluenza is not associated with cigarette smoking. All of the other conditions are associated with cigarettes. 2:25/2014 2:51:31 PM Mark this question & => Question Td : 91151 Question 21 of 30 A.47 year cld male recently got a job as a construction worker excavating a sie for a new building in Indianapolis. He presents with graduel onset of shortness of breath, fevers, and fatigue. His chest radiogragh chews hilar adenopathy. Ho is likely to be suffering fiom which one of the following? 2) Blastomycesie. +b) Coctidicidomycosis ©) Mycosis fimgoides. 6) Histoplasmosis. «) Tuberculosis Question Explanation: Histoplastnosis is endemic to Indianapolis and the rest of the Ohio River Valley andiis often associated with exposure of new residents to construction site excavations. In such individuals, constiutional and pulmonary involvement is not unusual, Blastomycosis and coccidisidomycosis do not occur in this part oF the country. Mycosis fingoides is a malgnant condition, Tuberculosis would nat produce this constellaton of symptoms in this setiing 2:25/2014 2:51:31 PM Mark this question & => Question Td : 91151 Question 21 of 30 A.47 year cld male recently got a job as a construction worker excavating a sie for a new building in Indianapolis. He presents with graduel onset of shortness of breath, fevers, and fatigue. His chest radiogragh chews hilar adenopathy. Ho is likely to be suffering fiom which one of the following? 2) Blastomycesie. +b) Coctidicidomycosis ©) Mycosis fimgoides. Y © 6) Histoplasmosis. «) Tuberculosis Question Explanation: Histoplastnosis is endemic to Indianapolis and the rest of the Ohio River Valley andiis often associated with exposure of new residents to construction site excavations. In such individuals, constiutional and pulmonary involvement is not unusual, Blastomycosis and coccidisidomycosis do not occur in this part oF the country. Mycosis fingoides is a malgnant condition, Tuberculosis would nat produce this constellaton of symptoms in this setiing 2/25/2014 2:52:09 PM ‘Mark this question & => Question Td : 91283 Question 22 of 30 ‘Which one of the following statement is FALSE regarding bronchial irritants? 8) Cigarettes are the most frequent irtants in simple chronic, bronchitis. ') They stimulate mucus secretion in the airways. ©) They impair mucus clearance by interfering with ciliary action, ©) They disturb alveolar macrophage fiction, thereby lowering resistance to infection. ©) They produce a cough by stimulating bronchial chemoreceptors Question Explanation: "Simple chronic bronchitis" is used to describe a productive cough that occurs at least three months per year for two consecutive years. Cigarettes are the most frequent cause. All bronchial iritants produce chronic bronchits by stimulating mucus secretion in the airways. These itrtants also impair mucus secretion by interfering with ciliary activity. They lower resistance to infection of the lurgs and bronchi by disturbing alveolar macrophage finction Cough results from an attempt to clear the airways of accurnulated secretions. n Report An Error 2/25/2014 2:52:09 PM ‘Mark this question & => Question Td : 91283 Question 22 of 30 ‘Which one of the following statement is FALSE regarding bronchial irritants? 8) Cigarettes are the most frequent irtants in simple chronic, bronchitis. ') They stimulate mucus secretion in the airways. ©) They impair mucus clearance by interfering with ciliary action, 6) They disturb alveolar macrophage finetion, thereby lowering resistance to infection, © & They produce a cough by stimulating bronchial chemoreceptors. Question Explanation: "Simple chronic bronchitis" is used to describe a productive cough that occurs at least three months per year for two consecutive years. Cigarettes are the most frequent cause. All bronchial iritants produce chronic bronchits by stimulating mucus secretion in the airways. These itrtants also impair mucus secretion by interfering with ciliary activity. They lower resistance to infection of the lurgs and bronchi by disturbing alveolar macrophage finction Cough results from an attempt to clear the airways of accurnulated secretions. n Report An Error 2/25/2014 2:52:23 PM Mark this question => Question Id : 91416 Question 23 of 30 A 32 year old man has a fever of 102° F, shortness of breath, cty cough, and oral thrush, CXR shows bilateral intersttial infiltrates, He has a history of HIV cisease, He is founc'to be hypoxic with an elevated arterial alveolar gradient. Diamosis of pneumocystis carinii pneumonia can be macle by all of the following, EXCEPT a) Transbronchial biopsy. ) Bronchoalveolar lavage ©) Elevation of lactic dehydrogenase 4) gallinm 67 scanning @) Blood cuitures Question Explanation: Blood cutmtes would not grow peumocystis. PCP is mainly presentin the ings. Blood cultures would grow mycobacterium avium if this is disseminated in the bloodstream. Transbronchial biopsy and bronchoalveolar lavage are very specific to diagnose pneumocystis catinii Both make the diagnosis 90-97% of the time. An elevation of the lactate dehydrogenase level is very non specific, but itis elevated in the correct clinical setting, then the diagnosis of PCP is easier. Gallum-§7 scanning is anon specific nuclear imaging modality. Whea a gallium scan “lights-up" in the lungs, then it tells us that there is inflammation in the Imgs and, possibly active PCR 2/25/2014 2:52:23 PM Mark this question => Question Id : 91416 Question 23 of 30 A 32 year old man has a fever of 102°F, shortness of breath, cry cough, and oral thrush. CAR shows bilateral interstitial infiltrates. He has a history of HIV cisease, He is founc'to be hypoxic with an elevated arterial alveolar gradient. Diamosis of pneumocystis arin pneumonia can be made by all ofthe following, EXCEPT a) Transtronchial biopsy. +) Bronchoalveolar lavage ©) Hlevation of lactic dehydrogenase ©) gallium 67 scanning VW © 6) Blood cultures Question Explanation: Blood cutmtes would not grow peumocystis. PCP is mainly presentin the ings. Blood cultures would grow mycobacterium avium if this is disseminated in the bloodstream. Transbronchial biopsy and bronchoalveolar lavage are very specific to diagnose pneumocystis catinii Both make the diagnosis 90-97% of the time. An elevation of the lactate dehydrogenase level is very non specific, but itis elevated in the correct clinical setting, then the diagnosis of PCP is easier. Gallum-§7 scanning is anon specific nuclear imaging modality. Whea a gallium scan “lights-up" in the lungs, then it tells us that there is inflammation in the Imgs and, possibly active PCR 2:25/2014 2:52:37 PM “Mark this question €&c> Question 24 of 30 A person develops respiratory faire and blincness after an overdose of ethylene glycol, Whatis the antidote of choice? a) Ethanol b) Glucagon, ©) N-acetyleysteine 4) Methytene blue #) There is no known antidote Question Explanation: N-acetylcysteine regenerates alutathione and prevents tozic metabolism of acetaminophen, Ethanol is an antidete for ethylene glycol and methanol toxicity. Glucagon is an antidote for beta-blocker toxicity. Methylene blue is an antidote for methemoglobinemia, which can occur after the ingestion of nitrites, nitroprusside, dapsone, or antimalarials 2:25/2014 2:52:37 PM “Mark this question €&c> Question 24 of 30 A person develops respiratory failure and blindness after an overdose of ettylene glycol. Whats the antidote of cheice? Y © a) Fthandl. +b) Ghucagon. ©) N-acetyleysteine 4) Methytene blue #) There is no known antidote Question Explanation: N-acetylcysteine regenerates alutathione and prevents tozic metabolism of acetaminophen, Ethanol is an antidete for ethylene glycol and methanol toxicity. Glucagon is an antidote for beta-blocker toxicity. Methylene blue is an antidote for methemoglobinemia, which can occur after the ingestion of nitrites, nitroprusside, dapsone, or antimalarials 2/25/2014 2:52:57 PM ‘Mark this question & => Question Id : 114699 Question 25 of 30 #66 year old female has noted epistaxis and tinnitus for several months She has had several bouts of what was diagnosed as sinusitis and was treated with antbiotics with minimal response. Now she presents because of a persistent cough and fatigne. She is most likely having which one of the following? 2) Positive rheumatoid factor. +) Elevated angiotensin-coaverting enzyme. ©) Antineutrophil cytoplasmic antibodies. 4) Anti-SSA antibodies ©) Ant-DNA antibodies. Anower (UERNSNEEN) ner voers Explanation Report An Error Question Explanatio. This patient most probably has Wegener's granulomatosis, which is associated with antineutrophil cytoplasmric antibodies (ANCA). None of the other choices are associated with Wegener's. 2/25/2014 2:52:57 PM ‘Mark this question & => Question Id : 114699 Question 25 of 30 A. 65 year old female has noted epistais and tinnitus for several months. She has had several bouts of what wes diagnosed as sinusitis and was treated with antbiotics with minimal response. Now she presents because of a persistent cough and fatigne. She is most likely having which one of the following? 2) Positive rheumatoid factor. ) Elevated angiotensin-coaverting enzyme. ¥ © © Antineutrophil cytoplasmic antibodies, 6) Ani-SSA antibodies ©) Anti-DINA antibodies. Anower (UERNSNEEN) ner voers Explanation Report An Error Question Explanatio. This patient most probably has Wegener's granulomatosis, which is associated with antineutrophil cytoplasmric antibodies (ANCA). None of the other choices are associated with Wegener's. Mark this question & => Question Td : 118867 Question 26 of 30 A patientis brought to the emergency room in acute pulmonary edema The respiratory rate is 4S/min and arterial blood gas reveals a 02 saturation of 69%. ECG is normal. Whet is the most appropriate management at this time? a) Place the patient in a siting position and stast the patient on 100% ©2 No drug therapy is usvally needed. 'b) Emergent clalysis c) Start 100% O2 end administer 10-20 mg furosemide and 2 mg momphine IV. 4) Start 100% O2 and bolus patient with 2L of lactated ringers e) Start 100% O2 and load the patient with digitalis Question Explanation Thitial therapy for acute pulmonary edema includes placing the patient in the sitting position to decrease venous remum and the work of breathing Oxygen is used to increase the arterial pO2, Furosemide and morphine are the standard first line medications in the management of acute pulmonary edema, In this patient, with a rapid respiratory rate and low ©2 sanuration, these mecications will be required to rapidly o decrease pulmonary congestion Gurosemide) and decrease venous return (momphine). Dialysis is reserved for cases that fal intial pharmacologic intervention. A fluid bolus will exacerbate the pulmonary congestion. Digtals is not acutely indicated in the absence of arhytiunia. Mark this question & => Question Td : 118867 Question 26 of 30 A patientis brought to the emergency room in acute pulmonary edema The respiratory rate is 4S/min and arterial blood gas reveals a 02 saturation of 69%. ECG is normal. Whet is the most appropriate management at this time? a) Place the patient in a siting position and stast the patient on 100% ©2 No drug therapy is usvally needed. 'b) Emergent clalysis V © 6) Start 100% O2 and administer 10-20 mg furosemide and 2.mg morphine TV. 4) Start 100% O2 and bolus patient with 2L of lactated ringers e) Start 100% O2 and load the patient with digitalis Question Explanation Thitial therapy for acute pulmonary edema includes placing the patient in the sitting position to decrease venous remum and the work of breathing Oxygen is used to increase the arterial pO2, Furosemide and morphine are the standard first line medications in the management of acute pulmonary edema, In this patient, with a rapid respiratory rate and low ©2 sanuration, these mecications will be required to rapidly o decrease pulmonary congestion Gurosemide) and decrease venous return (momphine). Dialysis is reserved for cases that fal intial pharmacologic intervention. A fluid bolus will exacerbate the pulmonary congestion. Digtals is not acutely indicated in the absence of arhytiunia. 2125/2014 2:53:27 PM “Mark this question €&=> Question Id : 119664 Question 27 of 30 A.57 year old man undergoes a thoracentesis for a lest pleural effusion. The pH of the fuid is 7.43 and the pleural uid glucose is 30 mg/dL. The diagnosis consistent with this pleura effusion is which one of the folowing? 2) Eepyoma ') Congestive heart failure ) Rheumatoid arthritis 4) Pulmonary embolism «) Circhosie Question Explanation: Rheumatoid arthritis, tsberculosis, and empyema can cause a pleural effusion with a low fluid ghicose. However. in an empyema. the pH ofthe fluid is ually less than 7.0, so only rheumatoid arthritis is consistent with a pH of 7.45 and pleural fluid ghacose less than 30 myfdL. The other choices do not lower the pleural id glacose. 2125/2014 2:53:27 PM “Mark this question €&=> Question Id : 119664 Question 27 of 30 A.57 year old man undergoes a thoracentesis for a lest pleural effusion. The pH of the fuid is 7.43 and the pleural uid glucose is 30 mg/dL. The diagnosis consistent with this pleura effusion is which one of the folowing? 2) Eepyoma ') Congestive heart failure JY © 0) Rheumatoid arthritis 4) Pulmonary embolism «) Circhosie Question Explanation: Rheumatoid arthritis, tsberculosis, and empyema can cause a pleural effusion with a low fluid ghicose. However. in an empyema. the pH ofthe fluid is ually less than 7.0, so only rheumatoid arthritis is consistent with a pH of 7.45 and pleural fluid ghacose less than 30 myfdL. The other choices do not lower the pleural id glacose. “Maatke this question & => Question 28 of 30 A college stident aged 20 years complains of a cry cough and fever for almost one week. She has had no recent exposure to sick roommates, pets, or any recent travel. CXR reveals a bilateral diffuse interstitial infiltrate, The most likely diagnosis is 2) Legionella pneumophila +) Psittacosis ©) Klebsiella pneamonia 8) CMY peemnonitis ©) Mycoplasma pneumonia Question Explanation Mycoplasma pneumonia is more common in young people, and itis a comrmurity-acquired pneumonia The chest X-ray infitrare appears worse than findings on physical exam of the patient. A dry, hacking cough, fever, and myalgias are common. Bullous myringtis of the ears and positive serum for celd agglutinins are common, Legionella pneumophila is also a community-acquired pneumonia, but a procuctive cough is more common. Diagnosis usually depends on antibody titer in serum (rather than sputam) by indirect immunoflourescence assay. Psitacosis is associated with exposure to birds. This patient has no recent exposure to animals Rlebsislla pneumonia preseats usually as a lobar pneumonia and is common in alcoholics. Itis associated with a Gram-negative organism and the sputumis very productive CMV pneumonia also presents as bilateral interstitial infiltrates, but patents are usually jmmunocompromised, which this patient is not “Maatke this question & => Question 28 of 30 A college stident aged 20 years complains of a cry cough and fever for almost one week. She has had no recent exposure to sick roommates, pets, or any recent travel. CXR reveals a bilateral diffuse interstitial infiltrate, The most likely diagnosis is 2) Legionella pneumophila b) Paittacosis, ©) Klebsiella pneamonia 8) CMY peemnonitis Y © &) Mycoplasma pneumonia Question Explanation Mycoplasma pneumonia is more common in young people, and itis a comrmurity-acquired pneumonia The chest X-ray infitrare appears worse than findings on physical exam of the patient. A dry, hacking cough, fever, and myalgias are common. Bullous myringtis of the ears and positive serum for celd agglutinins are common, Legionella pneumophila is also a community-acquired pneumonia, but a procuctive cough is more common. Diagnosis usually depends on antibody titer in serum (rather than sputam) by indirect immunoflourescence assay. Psitacosis is associated with exposure to birds. This patient has no recent exposure to animals Rlebsislla pneumonia preseats usually as a lobar pneumonia and is common in alcoholics. Itis associated with a Gram-negative organism and the sputumis very productive CMV pneumonia also presents as bilateral interstitial infiltrates, but patents are usually jmmunocompromised, which this patient is not Mark this question <= => Question Td : 213570 Question 29 of 30 A 69-year-old hypertensive and diabetic has chest pain for 3 hows that is left-sided and radkating to this left arm along diephoresis. Tk began at rest, Blood pressure is 137/80 mmElg and heart rate is 8beats/min and SPO2 = 97% on room air Bilateral carotid bruts ‘without jugular venous distension exist. No other findings exist. A lateral CKR is shown below: ‘An acute MI is red out, and his pan is relieved with sublingual nitroglycerin, What is the next step in management of this patient? 2) Contrast enhanced computed tomography of the chest +) lhitiate beta-biocker therapy ) Magnetic resonance imaging of the thoracic spine ©} Open lang biopsy ©) Percutancous lung biopsy Question Explanation: This patient has an incidental finding of a middle mediastinal mass on chest radiograph. His intial presentation is concerning for an acute coronary syndrome. Thus, ruling himn out for myocardial infarction is paramount at first which has been done. The middle ‘mediastinal mass has a differential diagnosis thet includes an aortic aneurysm, pulmonary artery aneurysm, massive lymphadenopathy, and other entities The appearance on this radiograph is most conceming for an aortic aneurysm. Regardless, computed tomography of the chest will cifferentiate between all these entities. Initiating beta-blocker therapy is premature given that the diagnosis is not yet clear. Ifthe mass is found to be predominantly a descending aortic aneurysm, then medical management with beta blockacte is indicated However. ifthe middle mediastinal mass has an ascending’ component then surgery would be indicated. ‘Magnetic resonance imaging of the thoracic spine would be incicated for a posterior mediastinal mass. Computed tomography of the chest will help with localization of the lesion and, in any case, will evaluate the middle meciastinum. Piowceeniisal Bodies Geena terete arid danenanas. execeiiy aruensin Ginly cuseeessc fea an aeries -anmrmnas: Report An Error Mark this question <= => Question Td : 213570 Question 29 of 30 A 69-year-old hypertensive and diabetic has chest pain for 3 hows that is left-sided and radkating to this left arm along diephoresis. Tk began at rest, Blood pressure is 137/80 mmElg and heart rate is 8beats/min and SPO2 = 97% on room air Bilateral carotid bruts ‘without jugular venous distension exist. No other findings exist. A lateral CKR is shown below: ‘An acute MI is ruled out, and his pan is relieved with sublingual nitroglycerin, What is the next step in management of this patieat? Y © a) Contrast enhanced computed tomography of the chest +) lhitiate beta-biocker therapy ©) Magnetic resonance imaging of the thoracic spine ©) Open tung biopsy «) Fercutancous lung biopsy Question Explanation: This patient has an incidental finding of a middle mediastinal mass on chest radiograph. His intial presentation is concerning for an acute coronary syndrome. Thus, ruling himn out for myocardial infarction is paramount at first which has been done. The middle ‘mediastinal mass has a differential diagnosis thet includes an aortic aneurysm, pulmonary artery aneurysm, massive lymphadenopathy, and other entities The appearance on this radiograph is most conceming for an aortic aneurysm. Regardless, computed tomography of the chest will cifferentiate between all these entities. Initiating beta-blocker therapy is premature given that the diagnosis is not yet clear. Ifthe mass is found to be predominantly a descending aortic aneurysm, then medical management with beta blockacte is indicated However. ifthe middle mediastinal mass has an ascending’ component then surgery would be indicated. ‘Magnetic resonance imaging of the thoracic spine would be incicated for a posterior mediastinal mass. Computed tomography of the chest will help with localization of the lesion and, in any case, will evaluate the middle meciastinum. Piowceeniisal Bodies Geena terete arid danenanas. execeiiy aruensin Ginly cuseeessc fea an aeries -anmrmnas: Report An Error ‘Mark: this question & Question Td : 213811 Question 30 of 30 A S1-year-old alecholic has fewer and productive cough with bloody sputum. Temperature =38 8 SC (101.8 °F), BP= 130/70 mmElg, pulse=120imin, and respirations 24/min, there are decreased breath sounde and crackles in the right upper lobe. CVS examination reveals tachycardic pulse with nomnal $1 and $2. CXR is as fellows: ae sia ses ay coal pink bacteria Which organism is the cause? a) Anaerobic palymicrobial pneumonia ) Heemophinis infenzae «) Klebsiella pneumoniae 4) Moraxella catarhalis 2) Mycoplasma pneumenice Pseudomonas aeruginosa 2) Streptococcus pneumoniae i) Staphylococcus aureus Answer | Bxntanaion Other User's Explanation Report An Error Question Explanation: ‘The patient has Klebsiela pneumonia This is a classic scenario: alcoholic (or debilitated) patient who has bloody sputum (or currant jelly) and bulging interlobar fissures on chest radiograph. The most important factor in answering this question cosrectiy is the presence of gram-negative (pink) bacteria on Gram stain Remember that regardless of the risk factors, Streptocaccus pneumoniae is. by far the most common cause of commanity- acquired pneumonia (up to 50%) and would be the correct answer ifthe Grem stain 1was not specified. Anaerobic polymicrobial paeumonia is common cause of aspiration pneumonia in alcoholic patients. The clinical presentation, Gram stain, and rad.ograph findings, however. suggest Klebsiella pneumonia as the most Hkely causative organist Haemophilus infuenzae is a common cause of community- acquired pneumonia, but the clirical presertation and radiograph findings suggest Klebsiela pneumonia as the most likely organism, Moraxella catarrhalis can be seen in patients who hiave chronic obstructive pulnonary disease, diabetes mellitus, or cancer, or in patients on corticosteroids. Mycoplasma pneumonize is often sea in young individuals as a cause of atypical pneumonia. In the classic case, the chest radiograph ss usually worse then what clinical presentation suggests. Pseudomonas aeruginosa is common cause of pneumonia in hospitalized patients and patienis who have cystic Gibrosis, and itis the most common infection in patients on mechanical ventilation Staphylococcus aureus is often seen in patients who are IV drug abusers or who have cystic fibrosis ot chronic granulomatous disease, those who ate postinfluenza, and in hospitalized patients. The sputumis salmon-colored. There is a high rate of necrosis lrponmee and manitare Ieeiane’ and the mnrtaite ie ac hich ac AOL ‘Mark: this question & Question Td : 213811 Question 30 of 30 A S1-year-old alecholic has fewer and productive cough with bloody sputum. Temperature =38 8 SC (101.8 °F), BP= 130/70 mmElg, pulse=120imin, and respirations 24/min, there are decreased breath sounde and crackles in the right upper lobe. CVS examination reveals tachycardic pulse with nomnal $1 and $2. CXR is as fellows: ae sia ses ay coal pink bacteria Which organism is the cause? a) Anaerobic palymicrobial pneumonia ) Heemophinis infenzae Y © ¢) Klebsiella pneumoniae 4) Moraxella catarhalis 2) Mycoplasma pneuncniae 6) Pseudomonas aeruginosa ® Sueplococeus pneumoniae i) Staphylococcus aureus Answer | Bxntanaion Other User's Explanation Report An Error Question Explanation: ‘The patient has Klebsiela pneumonia This is a classic scenario: alcoholic (or debilitated) patient who has bloody sputum (or currant jelly) and bulging interlobar fissures on chest radiograph. The most important factor in answering this question cosrectiy is the presence of gram-negative (pink) bacteria on Gram stain Remember that regardless of the risk factors, Streptocaccus pneumoniae is. by far the most common cause of commanity- acquired pneumonia (up to 50%) and would be the correct answer ifthe Grem stain 1was not specified. Anaerobic polymicrobial paeumonia is common cause of aspiration pneumonia in alcoholic patients. The clinical presentation, Gram stain, and rad.ograph findings, however. suggest Klebsiella pneumonia as the most Hkely causative organist Haemophilus infuenzae is a common cause of community- acquired pneumonia, but the clirical presertation and radiograph findings suggest Klebsiela pneumonia as the most likely organism, Moraxella catarrhalis can be seen in patients who hiave chronic obstructive pulnonary disease, diabetes mellitus, or cancer, or in patients on corticosteroids. Mycoplasma pneumonize is often sea in young individuals as a cause of atypical pneumonia. In the classic case, the chest radiograph ss usually worse then what clinical presentation suggests. Pseudomonas aeruginosa is common cause of pneumonia in hospitalized patients and patienis who have cystic Gibrosis, and itis the most common infection in patients on mechanical ventilation Staphylococcus aureus is often seen in patients who are IV drug abusers or who have cystic fibrosis ot chronic granulomatous disease, those who ate postinfluenza, and in hospitalized patients. The sputumis salmon-colored. There is a high rate of necrosis lrponmee and manitare Ieeiane’ and the mnrtaite ie ac hich ac AOL ‘Mark this question => Question Td : 28652 Question 1 of 30 Progressive massive fibrosis (PME) is most likely to be found in which of the following? a) Pulmonary Sequestration b) Lobar pneumonia ©} Sarcoidosis, @ ¢} Simple coal workers pneumoconiosis Question Explanation: Progressive massive fibrosis is diagnosed by chest x-ray as round masses, several Centimeters in diameter usually in the upper lobes, ‘They may have necrotic centres In slicosis amore accurate term is ‘conglomerate nodules’, Iris due to exposure to dust of high silicon content and hence PMF is more likely with higher silton exposure than in simple coal workers" lung Complicated silicosis ‘Mark this question => Question Td : 28652 Question 1 of 30 Progressive massive fibrosis (PME) is most likely to be found in which of the following? a) Pulmonary Sequestration ) Lobar pneumenia ©} Sarcoidosis, Y © d) Complicated silicosis ¢) Simple coal workers pneumoconiosis Question Explanation: Progressive massive fibrosis is diagnosed by chest x-ray as round masses, several Centimeters in diameter usually in the upper lobes, ‘They may have necrotic centres In slicosis amore accurate term is ‘conglomerate nodules’, Iris due to exposure to dust of high silicon content and hence PMF is more likely with higher silton exposure than in simple coal workers" lung ‘Mark this question & => Question Td : 28924 Question 2 of 30 A 17-year-old male presents to his GP with exertional breathlessness. The chest x-ray reveals a lesioniin the anterior mediastinum, ‘Which one of the folowing is the mastlikely cause for such an appearance? a) Ascending aorta ') Thyrnus gland ) Hila Igmmph nodes 4) Left atrium ©) Eeophagus Question Explanation: Abnormalities of the anterior/superior mediastinum may relate to the thymus, thyroid Inferior or middle mediastinal masses are related the aorta, lunge, hilar lymph nodes, esophagus and heart Posterior mediastinal masses may relate to the nerves and vertebrae. ‘Mark this question & => Question Td : 28924 Question 2 of 30 A 17-year-old male presents to his GP with exertional breathlessness. The chest x-ray reveals a lesion in the anterior mediastinum. “Which one of the followirg is the most likely cause for such an appearance? a) Ascending aorta YM © b) Thymus gland c) Hilar lymph nedes 6) Left atrium e) Esophagus Question Explanation: Abnormalities of the anterior/superior mediastinum may relate to the thymus, thyroid Inferior or middle mediastinal masses are related the aorta, lunge, hilar lymph nodes, esophagus and heart Posterior mediastinal masses may relate to the nerves and vertebrae. ‘Marke this question & => Question Td : 29533 Question 3 of 30 A 29-year-cldis wreated for pulmonary tuberculosis with Fifampicin, isoniazid, pyrazinamide and Ethambutol for four weeks Pre-treatment LFTs were normaal but recent investigations reveal (Serum total biirubin 5.73 meld. [Serum alanine aminotransferase S25TU/ Serum aspartate aminotransferase [455 TT A [Serum alkaine phosphatase [72510 0 “Which ofthe fellowingis the most appropdate next stop? a) Stop Ethambutol only +) Stop isoniazid and Rifempicin ©) Stop all treatment 4) Stop pyrazinamide and Bthanbutol ) Stop rifampicin Answer | Explanation Other User's Explanation Report An Error Question Explanatio: ‘All tuberculosis patients should have pretreatment LFT, should be supervised by a chest physician and should be informed of possible side-effects of treatment. Tfthere is no pre-existing liver disease, LFT are only repeated (and treatment stopped) iffever, malaise, vomiting, jaundice or unexplained deterioration occurs during treatment. Regular LET should be performed in patients with previously known chronic liver disease. IF ASTJALT levels rise by 5 times normnalbilimbin level rises then rifampicin/isoniazicl pyrazinamide should be stopped. TF the patients is not unwell and! or has non-infectious TB, no treatment untl LFT retums to normal Telnically unwell or sputum smezr positive within 2 weeks of starting treatment, consider streptomycin and ethambutol until LET retums to normal (Once LET tack to normel, challenge dosages can be reintroduced sequentialy in order of isoniazid, rifampicin and pyrazinamide with daily moritoring of patients‘c condition and LFT. Iefirther reaction occurs, offending drug is excluded and a suitable abernative reginen used. ‘Marke this question & => Question Td : 29533 Question 3 of 30 A 29-year-cldis wreated for pulmonary tuberculosis with Fifampicin, isoniazid, pyrazinamide and Ethambutol for four weeks Pre-treatment LFTs were normaal but recent investigations reveal (Serum total biirubin 5.73 meld. [Serum alanine aminotransferase S25TU/ Serum aspartate aminotransferase [455 TT A [Serum alkaine phosphatase [72510 0 “Which ofthe fellowingis the most appropdate next stop? a) Stop Ethambutol only +) Stop isoniazid and Rifempicin ¥ © o) Stop all teatment 4) Stop pyrazinamide and Bthanbutol ) Stop rifampicin Answer | Explanation Other User's Explanation Report An Error Question Explanatio: ‘All tuberculosis patients should have pretreatment LFT, should be supervised by a chest physician and should be informed of possible side-effects of treatment. Tfthere is no pre-existing liver disease, LFT are only repeated (and treatment stopped) iffever, malaise, vomiting, jaundice or unexplained deterioration occurs during treatment. Regular LET should be performed in patients with previously known chronic liver disease. IF ASTJALT levels rise by 5 times normnalbilimbin level rises then rifampicin/isoniazicl pyrazinamide should be stopped. TF the patients is not unwell and! or has non-infectious TB, no treatment untl LFT retums to normal Telnically unwell or sputum smezr positive within 2 weeks of starting treatment, consider streptomycin and ethambutol until LET retums to normal (Once LET tack to normel, challenge dosages can be reintroduced sequentialy in order of isoniazid, rifampicin and pyrazinamide with daily moritoring of patients‘c condition and LFT. Iefirther reaction occurs, offending drug is excluded and a suitable abernative reginen used. 2:25/2014 2:56:49 PM Mark this question & => Question Td : 29543 Question 4 of 30 “Which of the following is a recognized cause of phrenic nerve palsy? a) Dermoid ) Ganglioneuroma ©) Peticarchal cyst 4) Adjacent tumor €) Sarvoidesis Answer | Explanation Other User's Explanation Report An Error Question Explanation: ‘The diaphragm is imervated by the phrenic nerve (C3,4,5). Palsy is a recognized complication of thoracic surgery infection, Guillain- Barre of invasion by an adjacent tumour, Ie may also be stretched by an aortic aneurysm. 2:25/2014 2:56:49 PM Mark this question & => Question Td : 29543 Question 4 of 30 “Which of the following is a recognized cause of phrenic nerve palsy? 2) Demoid b) Gangloneuroma ©) Pericardial cyst Y © 4d) Adjacent tumor «) Sarcoidasis Answer | Explanation Other User's Explanation Report An Error Question Explanation: ‘The diaphragm is imervated by the phrenic nerve (C3,4,5). Palsy is a recognized complication of thoracic surgery infection, Guillain- Barre of invasion by an adjacent tumour, Ie may also be stretched by an aortic aneurysm. 2:25/2014 2:57:00 PM Mark this question => Question 5 of 30 ‘Medizstinal lymph node tuberculosis was diagnosed in a 29 year old two weeks ago and he was taking rifampicin, isoniazid and pyrazinamide, He now has increasing dyspaea and stridor. CXR. showed compression of both mein bronchi by carinal lymph node enlargement. Whatis the next step in management? a) Mediastnoscopy and biopsy ) Refer for stent insertiontracheostomy ©) Start preduisolone &) Increase the regimen dose «) The addition of streptomycin Anower [UEQRERGREN) tier Users Explanation Report An Error Question Explanation: The treatment of TB medistinal lymphadentis is the same as pulmonary TB. The nodes may enlarge curing or after treatment as a result of hypersensitivity. Corticosteroids are efficiive in reducing the enlargement and hence will help the stridor and breathlessness, 2:25/2014 2:57:00 PM Mark this question => Question 5 of 30 ‘Medizstinal lymph node tuberculosis was diagnosed in a 29 year old two weeks ago and he was taking rifampicin, isoniazid and pyrazinamide, He now has increasing dyspaea and stridor. CXR. showed compression of both mein bronchi by carinal lymph node enlargement. Whatis the next step in management? a) Mediastinoscopy and biopsy +) Refer for stent insertiontracheostomy Y © ¢) Stat prednisolone 4) Increase the regimen dose «) The addition of streptomycin Anower [UEQRERGREN) tier Users Explanation Report An Error Question Explanation: The treatment of TB medistinal lymphadentis is the same as pulmonary TB. The nodes may enlarge curing or after treatment as a result of hypersensitivity. Corticosteroids are efficiive in reducing the enlargement and hence will help the stridor and breathlessness, 2i25/2014 2:57:12 PM Mark this question & => Question 6 of 30 Recagnized associations. Whish of the following is correct? 2) Paeumoconiosis and clubbing +) Beenchopulmonary aspergillosis and wheezing o) Lung carcinoids and pleural effusion <4) Pulncnery embolus and left bundle branch block ©) Pulmonary fibrosis and typercapnia Question Explanation: Question Td : 29693 Pulmonary fbrosis associated with type 1 respiratory faiure which is associated with ventlaton/perfision mismatch. Allergic bronchopulnonary aspergillosis caused by Aspergilus fimigetus, which can present with asthma and eosinophilia 2i25/2014 2:57:12 PM Mark this question & => Question 6 of 30 Recagnized associations. Whish of the following is correct? 2) Paeumoconiosis and clubbing Y © b) Bronchopulmonary aspergillosis and wheezing o) Lung carcinoids and pleural effusion <4) Pulncnery embolus and left bundle branch block ©) Pulmonary fibrosis and typercapnia Question Explanation: Question Td : 29693 Pulmonary fbrosis associated with type 1 respiratory faiure which is associated with ventlaton/perfision mismatch. Allergic bronchopulnonary aspergillosis caused by Aspergilus fimigetus, which can present with asthma and eosinophilia ‘Mark this question & => Question Id : 29713 Question 7 of 30 A 28-year-old emaciated man with a history of alcohol and drug abuse was admitted with a 14 day history offever, cough and fatigue. Temperature = 39.4°C. Cervical and auillery lymphadenopathy was present, CAR showed bilateral areas of pultaonary shadowing Which of the following is the most likely diagnosis? a) Alcoholic cardiomyopathy b) Pheumscoceal pneumenia Pulmonary tuberculosis 4) €) Tricuspid endocarditis Question Explanation: Pacumocystis carinii is he most commen opportunistic infection in AIDS. ‘This patient s at risk of HIV with the history of drug abuse. Persistent generalized lymphadenopathy may develop in HIV before the patient falls the criteria of AIDS. ‘As the disease progresses, there is atrophy of the lymph nodes. Less lilely is pulmonary tuberculosis which can also cause any abnormality on chest x ray and involve peripheral lymph nodes. The other are unlikely to cause lymphadenopathy. c) i) Pneumocystis pneumonia ‘Mark this question & => Question Id : 29713 Question 7 of 30 A 28-year-old emaciated man with a history of alcohol and drug abuse was admitted with a 14 day history offever, cough and fatigue. Temperature = 39.4°C. Cervical and auillery lymphadenopathy was present, CAR showed bilateral areas of pultaonary shadowing, Which of the following is the most Hely diagnosis? a) Alcoholic cardiomyopathy 'b) Pneumococcal preumcnia ) Pulmonary tubercalesis Y © 4) Pneumocystis pneumonia @) Tricuspid endocarditis Question Explanation: Pacumocystis carinii is he most commen opportunistic infection in AIDS. ‘This patient s at risk of HIV with the history of drug abuse. Persistent generalized lymphadenopathy may develop in HIV before the patient falls the criteria of AIDS. ‘As the disease progresses, there is atrophy of the lymph nodes. Less lilely is pulmonary tuberculosis which can also cause any abnormality on chest x ray and involve peripheral lymph nodes. The other are unlikely to cause lymphadenopathy. 2/25:2014 2:57:36 PM Mark this question e => Question Id : 29835 Question 8 of 30 Tn an ongoing study of a new drug for asthma, a researcher wishes to compare average serum drug concentrations in volunteers four hours after taking the rug: A. in the fasting state then B, after ameal “Which is the most appropriate statistical test to use? a) Chi-squared test ) Pearson's correlation coefficient c) Student's unpaired t test 4) Wileoxon test 6) Student's paired t test Question Explanation: Tn this scenario we are dealing with use of the drug in the same volunteers with the intervention being the effect of feeding on drug concentrations. ‘Thus you will be comparing means in the same subjects and the paired test would be the most appropriate test. Further reading Choosing statistical tests GraptiPad.com 2/25:2014 2:57:36 PM Mark this question e => Question Id : 29835 Question 8 of 30 In an ongoing study ofa new ceug for asthma, a researcher wishes to compare average serum drug concentrations in volunteers four hours after taking the rug: A. in the fasting state then B, after ameal “Whichis the most appropriate statistical test to use? 2) Chi-squared test ) Pearson's correlation coefficient c) Student's unpaired t test 4) Wilcoxon test Y © ©) Student's paired t test Question Explanation: Tn this scenario we are dealing with use of the drug in the same volunteers with the intervention being the effect of feeding on drug concentrations. ‘Thus you will be comparing means in the same subjects and the paired test would be the most appropriate test. Further reading Choosing statistical tests GraptiPad.com ‘Marke this question = => Question Td : 29898 Question of 30 The characteristic association of Obstructive sleep apnoea is usually with a) Polyphaga ) Insomnia c) Hypersomnolence 4) Macrognathia @) Impotence Answer [Botanaton | Other User's Explanation Report An Ertor Question Explanation: ‘The dominant symptom is hypersommolence (sleepiness). Other most common symptoms include apparent personality changes, witnessed apnoeas and tre nocturnal polyuria, Reduced libido is less common, Sleep apnoea may be associated with acromegaly, myxoedema, obesity aud micrognathial retrognathia ‘Marke this question = => Question Td : 29898 Question of 30 The characteristic association of Obstructive sleep apnoea is usually with: a) Polyphaga b) Insomnia Y © c) Hypersomnolence d) Macrognathia ¢) Impotence Answer [Botanaton | Other User's Explanation Report An Ertor Question Explanation: ‘The dominant symptom is hypersommolence (sleepiness). Other most common symptoms include apparent personality changes, witnessed apnoeas and tre nocturnal polyuria, Reduced libido is less common, Sleep apnoea may be associated with acromegaly, myxoedema, obesity aud micrognathial retrognathia 2/25/2014 2:58:04 PM ‘Mark this question & => Question Td : 29909 Question 10 of 30 A.47 year old seaman presents with a history of cough and fever. Cavitating lesions are visualized on a chest X-ray. Which of the followings the mast likely cause? a) Syphilis +) Sarcoidosis ©) Histoplasmosis 6) Amnoebiacis ©) Erucelosis Question Explanation: Histoplasmosis normally evelves slowly over as long as 20 years but may follow a more rapid course in the inmmmnocempromised (Seamen may be more prone to sexually transmitted diseases such as HIV). Amoebic abscesses can develop in the right lower lobe following trans diaphragmatic spread from amoebic liver abscess (tender hepatomegaly, malaise, spiking temperature). Amoebiasis is also fiesh water pathogen 2/25/2014 2:58:04 PM ‘Mark this question & => Question Td : 29909 Question 10 of 30 A.47 year old seaman presents with a history of cough and fever. Cavitating lesions are visualized on a chest X-ray. Which of the followings the mast likely cause? a) Syphilis +) Sarcoidosis Y © ¢) Histoplasmosis 6) Amnoebiacis ©) Erucelosis Question Explanation: Histoplasmosis normally evelves slowly over as long as 20 years but may follow a more rapid course in the inmmmnocempromised (Seamen may be more prone to sexually transmitted diseases such as HIV). Amoebic abscesses can develop in the right lower lobe following trans diaphragmatic spread from amoebic liver abscess (tender hepatomegaly, malaise, spiking temperature). Amoebiasis is also fiesh water pathogen 2i25:2014 2:58:16 PM ‘Mark this question & => Question Id : 29996 Question 11 of 30 A.40 year old man with history ofnasal congestion, breattlessness, cough and wheeze presents with a left foot drop. Which ofthe followings the mast likely ciagnosis? a) Diabetes melas 16) Wegener's Gramilomatasis c) Pulmonary eosinophilia 4) Polyasteritsie Nodosa ©) Churg Strauss Syndrome Question Explanation: (Churg Strauss syndrome is an eosinophilic vasculitis involving small and medium sized arteries and veins. Ithas three progressive phases; the first prodromal phase is of asthma and rhinitis, the second of tissue end peripheral blood eosinophilia and the final phase of systemic vasculitis, Astama may precede the onset of vasculitis by many years although all three phases can occur together. Peripheral nervous sysiem involvement usualy takes the form of a mononeuriis multiplex or amixed sensory and motor polyneutits. 2i25:2014 2:58:16 PM ‘Mark this question & => Question Id : 29996 Question 11 of 30 A.40 year old man with history ofnasal congestion, breattlessness, cough and wheeze presents with a left foot drop. Which ofthe followings the mast likely ciagnosis? a) Diabetes melas 16) Wegener's Gramilomatasis c) Pulmonary eosinophilia 4) Polyasteritsie Nodosa Y © e&) Churg Strauss Syndrome Question Explanation: (Churg Strauss syndrome is an eosinophilic vasculitis involving small and medium sized arteries and veins. Ithas three progressive phases; the first prodromal phase is of asthma and rhinitis, the second of tissue end peripheral blood eosinophilia and the final phase of systemic vasculitis, Astama may precede the onset of vasculitis by many years although all three phases can occur together. Peripheral nervous sysiem involvement usualy takes the form of a mononeuriis multiplex or amixed sensory and motor polyneutits. “Mark this question & => Question 12 of 30 An 83 year old femele with advanced Alzheimer’s dementia has inhaled her heating aid. The respiratory rate is 35 breaths per minute A bronchoscopy is proposed, Regarding obtaiting consent which of the folowing applies? a) Inthe mini mental score is above 20/30, consent can be taken from patient ) The procedure does not require a written consent c) Psychogeriatric opinion is needed before consent can be obtained ) Relativesinext of kin wil hawe to consent for the patient €) The doctor wil be able to consent for the patient Question Explanation Tn this case, which is an emergency informed written consent is not essential as the procedure is potentially ife-seving and the doctor would be seen to be acting in the patient‘s best interest. The process of consent for a procedure relies upon written and informed, consert which the patient has impaired mental capacity; the doctors may be uncertain whether the patient is competent to give informed consent. Under English law no other person can consent to treatment on betalf of an adult, hough itis desirable that next of kin are consulted before treating an adult without consent. Recent legislation gives legal authority to people appointed by the paliert, or by the state, or arelative or carer, to consent (or refuse) on behalf of the patient A mini-mental score may not adequetely identify those unable to give consent “Mark this question & => Question 12 of 30 An 83 year old femele with advanced Alzheimer’s dementia has inhaled her heating aid. The respiratory rate is 35 breaths per minute A bronchoscopy is proposed, Regarding obtaiting consent which of the folowing applies? a) Inthe mini mental score is above 20/30, consent can be taken from patient Y © 6) The procedure does not require a written consent c) Psychogeriatric opinion is needed before consent can be obtained ) Relativesinext of kin wil hawe to consent for the patient €) The doctor wil be able to consent for the patient Question Explanation Tn this case, which is an emergency informed written consent is not essential as the procedure is potentially ife-seving and the doctor would be seen to be acting in the patient‘s best interest. The process of consent for a procedure relies upon written and informed, consert which the patient has impaired mental capacity; the doctors may be uncertain whether the patient is competent to give informed consent. Under English law no other person can consent to treatment on betalf of an adult, hough itis desirable that next of kin are consulted before treating an adult without consent. Recent legislation gives legal authority to people appointed by the paliert, or by the state, or arelative or carer, to consent (or refuse) on behalf of the patient A mini-mental score may not adequetely identify those unable to give consent 22512014 2:58:42 PM Mark this question €& => Question 13 of 30 A.4S year old man complains of chronic fatigue for 1 year. His wife states he snores loudly and at times she thinks he seems to stop bbeeathing The most likely diagnosis is which of the fallowing? 2) Narcolepsy syndrome ) Catalepsy symdrome ©) Sleep apnea syndrome 4) Eleine Levin syndrome ©) Hypersomma disorder Question Explanation: Obstructive sleep apnea consists of episodes of partial or complete closure of the upper airway thet occur during sleep and lead to breathing cessation (defined as a period of apnea > 10 sec). Symptoms include restlessness, snoring, recurrent awekeenirg, morning, headache, and excessive daytime sleepiness. Diagnosis is based on sleep history, and polysomnography. Treatment is with nesal continuous postive airway pressure, oral appliances, and, in reffactory cases, surgery. Prognosis is good with treatment. Most cases remain undiagnosed and untreated and are often associated with hypertension, heart fahite, and injury or death from motor vehicle crashes and other accidents resulting fiom hypersomnolence. n Report An Error 22512014 2:58:42 PM Mark this question €& => Question 13 of 30 A.4S year old man complains of chronic fatigue for 1 year. His wife states he snores loudly and at times she thinks he seems to stop bbeeathing The most likely diagnosis is which of the fallowing? 2) Narcolepsy syndrome ) Catalepsy symdrome Y © ©) Sleep apnea syndrome 4) Eleine Levin syndrome ©) Hypersomma disorder Question Explanation: Obstructive sleep apnea consists of episodes of partial or complete closure of the upper airway thet occur during sleep and lead to breathing cessation (defined as a period of apnea > 10 sec). Symptoms include restlessness, snoring, recurrent awekeenirg, morning, headache, and excessive daytime sleepiness. Diagnosis is based on sleep history, and polysomnography. Treatment is with nesal continuous postive airway pressure, oral appliances, and, in reffactory cases, surgery. Prognosis is good with treatment. Most cases remain undiagnosed and untreated and are often associated with hypertension, heart fahite, and injury or death from motor vehicle crashes and other accidents resulting fiom hypersomnolence. n Report An Error “Marke this question & => Question 14 of 30 In apost- operative patient whet is the most common cause of development of a lung abscess? 2) Aspiration ) Bacterial discrimination from operative site ©) Atelectasis 6) Pneumonia ©) Septic emboli Question Explanation: Lung abscess is @ necrotizing infection characterized by localized pus. Ibis almost always caused by aspiration of oral secretions by patients who have impaired consciousness. Symptoms are persistent cough, fever, sweats, and weight loss, Diagnosis is based on history, physical examination, and chest x-ray. Treatment usvally is with clindamycin or combination B-lactawp-lactamase inhibitors “Marke this question & => Question 14 of 30 In apost-operative patient whet is the most common cause of development of a lung abscess? SY © & Aspiration +) Bacterial discrimination from operative site 6) Atelectasis 6) Pneumonia 6) Septic embati Question Explanation: Lung abscess is @ necrotizing infection characterized by localized pus. Ibis almost always caused by aspiration of oral secretions by patients who have impaired consciousness. Symptoms are persistent cough, fever, sweats, and weight loss, Diagnosis is based on history, physical examination, and chest x-ray. Treatment usvally is with clindamycin or combination B-lactawp-lactamase inhibitors 2/25/2014 2:59:09 PM Mark this question & => Question Id : 72607 Question 15 of 30 A paraneoplastic syndrome may often be the first evidence of pulmonary neoplasm. All of the following are examples of paraneoplastic syndromes EXCEPT 2) Syndrome of inappropriate secretion of antidiuretic hormone (SLADE). ) Hypercalcemia. ©) Hypertrophic pulmonary osteoarthrepathy. 4) Eosinephila ©) Ectopic ACTH production. Question Explanation: Eosinoptilia is not usually afiliated with lung cancer and its paraneoplastic syndromes. SLADH and ectopic ACTH production is usually associated with oat-cell hing cancer. Hypercalcemia is most common with squamous-cell lung cancer. Periosteal new-bone formation in the long bones associated with clubbingis called hypertrophic pulmonary osteoarthropathy, and it is most commonly associated with adenocarcinoma of the lung. 2/25/2014 2:59:09 PM Mark this question & => Question Id : 72607 Question 15 of 30 A paraneoplastic syndrome may often be the first evidence of pulmonary neoplasm. All of the following are examples of paraneoplastic syndromes EXCEPT 2) Syndrome of inappropriate secretion of antidiuretic hormone (SLADE). ) Hypercalcemia. ©) Hypertrophic pulmonary osteoarthrepathy. V © 4) Eosinophilia, ©) Ectopic ACTH production. Question Explanation: Eosinoptilia is not usually afiliated with lung cancer and its paraneoplastic syndromes. SLADH and ectopic ACTH production is usually associated with oat-cell hing cancer. Hypercalcemia is most common with squamous-cell lung cancer. Periosteal new-bone formation in the long bones associated with clubbingis called hypertrophic pulmonary osteoarthropathy, and it is most commonly associated with adenocarcinoma of the lung. 2:25/2014 2:59:21 PM Mark this question & => Question Td : 75941 Question 16 of 30 “Which one of the following is NOT true of pneumothorax? @) Spontaneous pneumothorax usually resubs from rupture of basilar subpleural blebs. ) Tension pneumothorax results from increased positive pressure through a "ball-valve" air leak ©) Secondary pneumothoras usually resuits from widespread emphysema, 49) Tension pneumothorax causes decreased venous retum to the heart €) Tension pneumothorax causes decreased cardiac output Question Explanation: Peumoathoras is the presence of gas in the pleural space. A spontaneous pneumothorax usually results from rupture of apical subpleural blebs. The most common cause of a secondary pneumothorax: is widespread emphysema, although other underlying pulmonary diseases, such as asthma and interstitial ng disease, may also produce it, A tension pneumothorax usualy occurs dunng ‘mechanical ventilation or ducing resuscitation efforts Tkis caused by increased positive pressure through a "ball-valve" air leek. The postive pressure is transmitted to the mediastinum, sesultingin decreased venous return to the heart and decreased cardiac output 2:25/2014 2:59:21 PM Mark this question & => Question Td : 75941 Question 16 of 30 “Which one of the following is NOT true of pneumothorax? Y © 2) Spontaneous pneumothorax usually resus from rupture of basilar subpleural blebs. ) Tension pneumothorax results from increased positive pressure through a "ball-valve" air leak ©) Secondary pneumothoras usually resuits from widespread emphysema, 49) Tension pneumothorax causes decreased venous retum to the heart €) Tension pneumothorax causes decreased cardiac output Question Explanation: Peumoathoras is the presence of gas in the pleural space. A spontaneous pneumothorax usually results from rupture of apical subpleural blebs. The most common cause of a secondary pneumothorax: is widespread emphysema, although other underlying pulmonary diseases, such as asthma and interstitial ng disease, may also produce it, A tension pneumothorax usualy occurs dunng ‘mechanical ventilation or ducing resuscitation efforts Tkis caused by increased positive pressure through a "ball-valve" air leek. The postive pressure is transmitted to the mediastinum, sesultingin decreased venous return to the heart and decreased cardiac output “Mark this question & => Question 17 of 30 A 53 year old Caucasian male presents with sinus pain and drainage, bloody nasal discharge and nasal mucosal ulceration, On laborztory examination, the man is found to have proteinuria, hematuria, and red blood cell cast. A biopsy of the upper airway tissue reveals gramulomatous indlammation with necrosis. Renal bicpsy confirms the existence of glomerulonephuitis. Blood tests show the presence of antinewrophil cytoplasmic antibodies (c-ANCA). The most ikely diagnosis is a) Allergic angiitis ») Goodpasture syndrome ©) Non-infection granulomatous disease & Tumors of the upper airway ©) Wegener granulomatosis Question Explanation: Wegener granulomatosis has the triad of necrotizing granilomas of the upper airways. vasculitis, and renal disease. Th addition to the triad, antineutrophil cytoplasmic antibodies yielding a cytoplasmic immunofluorescence pattern (c-ANCA) are found in over 90% of patients with Wegener. The classic clinicopathologic complex of Wegener granulomatosis usually provides ready differentiation from other disorders in which c-ANCA ss positive. C-ANCA is not considered pathognomonic, but has a very strong association with ‘Wegener andis aot found in any ofthe other diseases listed c-ANCA is generaly negative in patients with allerac angitis (choice A) non-infectious granulomatous disease (choice C) and tumors of the upper airway (choice D) c-ANCA is typically negative in Goodpasture syndrome (choice B) but p-ANCA have been detected in some patients with Goodpasture. “Mark this question & => Question 17 of 30 A 53 year old Caucasian male presents with sinus pain and drainage, bloody nasal discharge and nasal mucosal ulceration, On laborztory examination, the man is found to have proteinuria, hematuria, and red blood cell cast. A biopsy of the upper airway tissue reveals gramulomatous indlammation with necrosis. Renal bicpsy confirms the existence of glomerulonephuitis. Blood tests show the presence of antinewrophil cytoplasmic antibodies (c-ANCA). The most ikely diagnosis is a) Allergic angiitis ») Goodpasture syndrome ©) Non-infection granulomatous disease & Tumors of the upper airway Y © 6) Wegener gramalornatosis Question Explanation: Wegener granulomatosis has the triad of necrotizing granilomas of the upper airways. vasculitis, and renal disease. Th addition to the triad, antineutrophil cytoplasmic antibodies yielding a cytoplasmic immunofluorescence pattern (c-ANCA) are found in over 90% of patients with Wegener. The classic clinicopathologic complex of Wegener granulomatosis usually provides ready differentiation from other disorders in which c-ANCA ss positive. C-ANCA is not considered pathognomonic, but has a very strong association with ‘Wegener andis aot found in any ofthe other diseases listed c-ANCA is generaly negative in patients with allerac angitis (choice A) non-infectious granulomatous disease (choice C) and tumors of the upper airway (choice D) c-ANCA is typically negative in Goodpasture syndrome (choice B) but p-ANCA have been detected in some patients with Goodpasture. 2125/2014 2:59:47 PM “Mark this question €—& => Question Id : 85799 Question 18 of 30 A665 year old smoker presents to the physician with weight loss, cough shortness of breata, and hemoptysis. He appears cachectic, and he admits to "not having much of an appetite." Laboratory studies are remarkable for a serum sodium of 134 mEq/L. The most appropriate reason for this laboratory abnormality is 2) Central diabetes insipidus 1) Central inappropriate articuretic hormone (ADH) secretion ©) Excessive water intake 4) Parathyroid hormone (PTE) aephtogenic diabetes insipidus ©) Paraneoplastic ADH secretion Paraneoplastic parethyrcid hormone (PTE) secretion, Answer | Explanation Other User's Explanation Report An Error Question Explanation: ‘This patientis hyponatremic which has along differential diagnosis. Cf the choices listed, central ADH secretion can occur after head trauma or due to an intracranial neoplasia, not the case presented in this question stem; excessive water intake and paraneoplastic ADH secretion cause hyponatremia. Given the patient's history and presentation, a broachogerio carcinoma producing ADH ir the most lkely diagnosis, Diabetes insipidus resulis in hyponatremia PTH causes hypercalcemia. 2125/2014 2:59:47 PM “Mark this question €—& => Question Id : 85799 Question 18 of 30 A665 year old smoker presents to the physician with weight loss, cough shortness of breata, and hemoptysis. He appears cachectic, and he admits to "not having much of an appetite." Laboratory studies are remarkable for a serum sodium of 134 mEq/L. The most appropriate reason for this laboratory abnormality is 2) Central diabetes insipidus +) Central inappropriate anticiuretic hormone (ADHD secretion. ©) Excessive water intake 4) Parathyroid hormone (PTE) aephtogenic diabetes insipidus © © Paraneoplastic ADH secretion Paraneoplastic parethyrcid hormone (PTE) secretion, Answer | Explanation Other User's Explanation Report An Error Question Explanation: ‘This patientis hyponatremic which has along differential diagnosis. Cf the choices listed, central ADH secretion can occur after head trauma or due to an intracranial neoplasia, not the case presented in this question stem; excessive water intake and paraneoplastic ADH secretion cause hyponatremia. Given the patient's history and presentation, a broachogerio carcinoma producing ADH ir the most lkely diagnosis, Diabetes insipidus resulis in hyponatremia PTH causes hypercalcemia. 2125/2014 2:59:59 PM “Mark this question => Question 19 of 30 A.44 year old man with cirthosis due to a-l-antiypsin deficiency receives a liver transplant. He is currently in good health but he is at increased risk of developing which type of emphysema? a) Centriacinar ») Compensatory ) Interstitial ® Panacinar ©) Paraseptal Anowor [UERERBRERT) ter Uoors Explanation Report An Error Question Explanation: ‘There are two main morphologic forms of exephysema, centviaciner end panacinay. The panccinar verientis related to al-artitrypsin deficiency; the entire acinus is enlarged, from the respiratory bronchiole to the distal alveoi. Centriacinar emphysema is characterized by enlargement of the central portions of the acinus the sespiratory bronchiole and its pathogenesis is related to exposure to tobacco products and coal dust Interstitial emphysema is not a true form of emphysema. Tt resuits from penetration of air into the pulmonary fnterstitiu. This may occur when alveolar toars develop becaute of a combination of coughing and airway cbstruction (e g., children with whooping cough) or a chest wound that injures the underlying inag parenchyma (e.g. a fractured rib). Compensatory emphysema and paraseptal emphysema are associated with scarring Both are frequent but usually clinically silent. Paraseptal emphysema, however may lead to spontaneous pneumathorax in young patients. In fact this form is more severe in areas adjacent to the pleura, whore leege cystlike structares may develop and rapture into the pleural cavity. 2125/2014 2:59:59 PM “Mark this question => Question 19 of 30 A.44 year old man with cirthosis due to a-l-antiypsin deficiency receives a liver transplant. He is currently in good health but he is at increased risk of developing which type of emphysema? a) Centriacinar ») Compensatory ) Interstitial Y¥ © @) Panacinar ©) Paraseptal Anowor [UERERBRERT) ter Uoors Explanation Report An Error Question Explanation: ‘There are two main morphologic forms of exephysema, centviaciner end panacinay. The panccinar verientis related to al-artitrypsin deficiency; the entire acinus is enlarged, from the respiratory bronchiole to the distal alveoi. Centriacinar emphysema is characterized by enlargement of the central portions of the acinus the sespiratory bronchiole and its pathogenesis is related to exposure to tobacco products and coal dust Interstitial emphysema is not a true form of emphysema. Tt resuits from penetration of air into the pulmonary fnterstitiu. This may occur when alveolar toars develop becaute of a combination of coughing and airway cbstruction (e g., children with whooping cough) or a chest wound that injures the underlying inag parenchyma (e.g. a fractured rib). Compensatory emphysema and paraseptal emphysema are associated with scarring Both are frequent but usually clinically silent. Paraseptal emphysema, however may lead to spontaneous pneumathorax in young patients. In fact this form is more severe in areas adjacent to the pleura, whore leege cystlike structares may develop and rapture into the pleural cavity. 2:25/2014 3:00:13 PM ‘Mark this question => Question Id : 87028 Question 20 of 30 A.76 year old smoker with history of hemoptysis and weight loss undergoes a leftupper lobectomy for squamous cell carcinoma. Destruction of the alveolar septae around the respiratory bronchioles, with marked enlargement of the air spaces is seen in the uninvolved Inng tismue. Anthracatic pigmert is deposited heavily in the walls of these tissues. These findings are most likely associated with which of the following? a) Bronchiectasis 'b) Chrenie bronchitic c) Emphysema 4) Pulmonary hypertension €) Silicosis Question Explanation: Emphysema is a pulmonary disease characterized by enlargement of the alveolar air spaces due to destruction of the septae without consequent fibrosis. The gross appearance of emphysematous hings is characteristic: alveoli are sufficiently dilated to alow visualization with the naked eye, and destruction of structural support to lymphatic vessels produces heavy pigment deposition in the tissue, Microscopic findings that confirm the diagnosis inclade enlarged, round air spaces with club lice ends of broken septae sticking into the alveoli, Bronchiectasis is an abnormal dilatation of proximal and mediuin sized bronchi, This condition may be congenital or acquired (commonly secondary to an infectious process). Symptoms include shortness of breath, cough with daily production of mucopurulent sputum, and hemoptysis CT scan characteristically shows bronchial wall thickening and furninal dilatation, Chronic bronchitis produces marked hypersecretion of mucus in the large airways, and can be identiied by hypertrophy of mucous glands in the bronchi and goblet cell hyperplasia in the smaller airways. Pulmonary hypertension effects neither the airways nor the alveol, Itis characterized by thickening ofthe arterial smooth muscle wath intimal hyperplasia and fibrosis, Atherosclerotic changes in the normally plaque-free larger pulmonary arteries may be seen. Silicosis, one of the forms of pneumoconiosis, is an interstitial fbrosing disease that produces thick: pleural scars and dense noduies of collagen that may calcfy. The silica particles may be visualized within the nodules by means of polarized ight. 2:25/2014 3:00:13 PM ‘Mark this question => Question Id : 87028 Question 20 of 30 A.76 year old smoker with history of hemoptysis and weight loss undergoes a leftupper lobectomy for squamous cell carcinoma. Destruction of the alveolar septae around the respiratory bronchioles, with marked enlargement of the air spaces is seen in the uninvolved Inng tismue. Anthracatic pigmert is deposited heavily in the walls of these tissues. These findings are most likely associated with which of the following? a) Bronchiectasis 'b) Chrenie bronchitic Y © 0) Emphysema 4) Pulmonary hypertension €) Silicosis Question Explanation: Emphysema is a pulmonary disease characterized by enlargement of the alveolar air spaces due to destruction of the septae without consequent fibrosis. The gross appearance of emphysematous hings is characteristic: alveoli are sufficiently dilated to alow visualization with the naked eye, and destruction of structural support to lymphatic vessels produces heavy pigment deposition in the tissue, Microscopic findings that confirm the diagnosis inclade enlarged, round air spaces with club lice ends of broken septae sticking into the alveoli, Bronchiectasis is an abnormal dilatation of proximal and mediuin sized bronchi, This condition may be congenital or acquired (commonly secondary to an infectious process). Symptoms include shortness of breath, cough with daily production of mucopurulent sputum, and hemoptysis CT scan characteristically shows bronchial wall thickening and furninal dilatation, Chronic bronchitis produces marked hypersecretion of mucus in the large airways, and can be identiied by hypertrophy of mucous glands in the bronchi and goblet cell hyperplasia in the smaller airways. Pulmonary hypertension effects neither the airways nor the alveol, Itis characterized by thickening ofthe arterial smooth muscle wath intimal hyperplasia and fibrosis, Atherosclerotic changes in the normally plaque-free larger pulmonary arteries may be seen. Silicosis, one of the forms of pneumoconiosis, is an interstitial fbrosing disease that produces thick: pleural scars and dense noduies of collagen that may calcfy. The silica particles may be visualized within the nodules by means of polarized ight.

You might also like